127479134 MA12 Matematica Discreta Ed 2012

362

Transcript of 127479134 MA12 Matematica Discreta Ed 2012

1

1

Números Naturais

Sumário

1.1 Introdução . . . . . . . . . . . . . . . . . . . . . . . 2

1.2 O Conjunto dos Números Naturais . . . . . . . . . . 3

1.3 O Axioma da Indução . . . . . . . . . . . . . . . . . 4

1.4 As Duas Operações: Adição e Multiplicação . . . . 5

1.5 A Ordenação nos Números Naturais . . . . . . . . . 6

1.6 Exercícios Recomendados . . . . . . . . . . . . . . . 8

1.7 Textos Complementares . . . . . . . . . . . . . . . . 10

1.1 Introdução

Deus criou os números naturais. O resto é obra dos homens.

Leopold Kronecker

Enquanto os conjuntos constituem um meio auxiliar, os números são um dos dois

objetos principais de que se ocupa a Matemática. O outro objeto é o espaço,

juntamente com as guras geométricas nele contidas. Os números são objetos

abstratos que foram desenvolvidos pelo homem para servir como modelos que

permitem contar e medir e, portanto avaliar as diferentes quantidades de uma

grandeza.

Os compêndios tradicionais dizem o seguinte:

Número é o resultado da comparação entre uma grandeza e a unidade. Se

a grandeza é discreta, essa comparação chama-se uma contagem e o resultado

é um número inteiro; se a grandeza é contínua, a comparação chama-se uma

medição e o resultado é um número real.

Nos padrões atuais de rigor matemático, o trecho acima não pode ser consi-

derado como uma denição matemática, pois faz uso de idéias (como grandeza,

unidade, discreta, contínua) e processos (como comparação) de signicado não

estabelecido. Entretanto, todas as palavras que nela aparecem possuem um

sentido bastante claro na linguagem do dia-a-dia. Por isso, embora não sirva

para demonstrar teoremas a partir dela, a denição tradicional tem o grande

mérito de nos revelar para que servem e por qual motivo foram inventados os

números. Isto é muito mais do que se pode dizer sobre a denição que encon-

tramos no nosso dicionário mais conhecido e festejado, conforme reproduzimos

a seguir.

Número. [Do lat. numeru.] S.m. 1. Mat. O conjunto de todos os conjuntos

equivalentes a um conjunto dado.

Discutiremos este ponto logo mais, quando tratarmos de números cardinais.

+ Para Saber Mais - Comentários sobre Denições e axiomas - Cli-

que para ler

Unidade 1Números Naturais

+ Na Sala de Aula - Reexões sobre a sala de aula - Clique para ler

1.2 O Conjunto dos Números Naturais

Lentamente, à medida em que se civilizava, a humanidade apoderou-se desse

modelo abstrato de contagem (um, dois, três, quatro, ...) que são os números

naturais. Foi uma evolução demorada. As tribos mais rudimentares contam

apenas um, dois, muitos. A língua inglesa ainda guarda um resquício desse

estágio na palavra thrice, que tanto pode signicar três vezes como muito

ou extremamente.

As necessidades provocadas por um sistema social cada vez mais complexo

e as longas reexçõs, possíveis graças à disponibilidade de tempo trazida pelo

progresso econômico, conduziram, através dos séculos, ao aperfeiçoamento do

extraordinário instrumento de avaliação que é o conjunto dos números naturais.

Decorridos muitos milênios, podemos hoje descrever concisa e precisamente

o conjunto N dos números naturais, valendo-nos da notável síntese feita pelo

matemático italiano Giuseppe Peano no limiar do século 20.

N é um conjunto, cujos elementos são chamados números naturais. A

essência da caracterização de N reside na palavra sucessor. Intuitivamente,

quando n, n′ ∈ N, dizer que n′ é o sucessor de n signica que n′ vem logo depois

de n, não havendo outros números naturais entre n e n′. Evidentemente, esta

explicação apenas substitui sucessor por logo depois, portanto não é uma

denição. O termo primitivo sucessor não é denido explicitamente. Seu uso

e suas propriedades são regidos por algumas regras, abaixo enumeradas:

a) Todo número natural tem um único sucessor;

b) Números naturais diferentes têm sucessores diferentes;

c) Existe um único número natural, chamado um e representado pelo símbolo

1, que não é sucessor de nenhum outro;

d) Seja X um conjunto de números naturais (isto é, X ⊂ N). Se 1 ∈ X e se,

além disso, o sucessor de todo elemento de X ainda pertence a X, então

X = N.

3

Unidade 1 O Axioma da Indução

As armações (a), (b), (c) e (d) acima são conhecidas como os axiomas de

Peano. Tudo o que se sabe sobre os números naturais pode ser demonstrado

como consequência desses axiomas.

+ Para Saber Mais - Sobre o sistema de numeração - Clique para ler

+ Para Saber Mais - Um comentário gramatical - Clique para ler

+ Na Sala de Aula - Uma recomendação - Clique para ler

1.3 O Axioma da Indução

O último dos axiomas de Peano é conhecido como o axioma da indução. Ele

é a base de um eciente método de demonstração de proposições referentes a

números naturais (demonstrações por indução, ou por recorrência). Enunciado

sob a forma de propriedades em vez de conjuntos, ele se formula assim:

Seja P (n) uma propriedade relativa ao número natural n. Suponhamos que

i) P (1) é válida;

ii) Para todo n ∈ N, a validez de P (n) implica a validez de P (n′), onde n′ é o

sucessor de n.

Então P (n) é válida qualquer que seja o número natural n.

Com efeito, se chamarmos de X o conjunto dos números naturais n para

os quais P (n) é válida, veremos que:

1 ∈ X em virtude de (i); e que

n ∈ X ⇒ n′ ∈ X em virtude de (ii).

Logo, pelo axioma da indução, concluímos que X = N.

Definição 1 Esta formulação do Axioma da Indução é chamada de Princípio de Indução

Matemática

4

Unidade 1Números Naturais

+ Para Saber Mais - Cuidado! - Clique para ler

1.4 As Duas Operações: Adição e Multipli-

cação

Entre os números naturais estão denidas duas operações fundamentais:

a adição, que aos números n, p ∈ N faz corresponder a soma n + p e a

multiplicação, que lhes associa o produto np.

A soma n + p é o número natural que se obtém a partir de n aplicando-se

p vezes seguidas a operação de tomar o sucessor. Em particular, n + 1 é o

sucessor de n, n + 2 é o sucessor do sucessor de n, etc. Por exemplo, tem-se

2 + 2 = 4 simplesmente porque 4 é o sucessor do sucessor de 2.

De agora em diante, o sucessor do número natural n será designado por

n+ 1.

Quanto ao produto, põe-se n · 1 = n por denição e, quando p 6= 1, np é a

soma de p parcelas iguais a n.

Em última análise, a soma n+ p e o produto np têm mesmo os signicados

que lhes são atribuídos pelas explicações dadas acima. Entretanto, até que

saibamos utilizar os números naturais para efetuar contagens, não tem sentido

falar em p vezes e p parcelas. Por isso, as operações fundamentais devem

ser denidas por indução, como se segue.

Adição: n + 1 = sucessor de n e n + (p + 1) = (n + p) + 1 . Esta última

igualdade diz que se sabemos somar p a todos os números naturais n, sabemos

também somar p+1: a soma n+(p+1) é simplesmente o sucessor (n+p)+1

de n + p . O axioma da indução garante que a soma n + p está denida para

quaisquer n, p ∈ N.

Multiplicação: n·1 = n e n(p+1) = np+n. Ou seja: multiplicar um número

n por 1 não o altera. E se sabemos multiplicar todos os números naturais n por

p, sabemos também multiplicá-los por p+1: basta tomar n(p+1) = np+n. Por

indução, sabemos multiplicar todo n por qualquer p. Estas operações gozam das

conhecidas propriedades de associatividade, comutatividade e distributividade.

As demonstrações são feitas por indução. (Voltaremos ao assunto na Unidade

5 de MA12, onde mais detalhes serão apresentados.)

5

Unidade 1 A Ordenação nos Números Naturais

1.5 A Ordenação nos Números Naturais

Nossa breve descrição do conjunto N dos números naturais termina com a

relação de ordem m < n.

Dados m, n ∈ N, diz-se que m é menor do que n, e escreve-se m < n,

para signicar que existe algum p ∈ N tal que n = m + p. (Isto quer dizer

que n é o sucessor do sucessor... do sucessor de m, o ato de tomar o sucessor

sendo iterado p vezes.)

A relação m < n tem as seguintes propriedades:

Transitividade: Se m < n e n < p então m < p.

Tricotomia: Dados m, n ∈ N, vale uma, e somente uma, das alternativas:

m = n, m < n ou n < m.

Monotonicidade: Se m < n então, para qualquer p ∈ N, tem-se m+p < n+p

e mp < np.

Boa-ordenação: Todo subconjunto não-vazio X ⊂ N possui um menor ele-

mento. Isto signica que existe um elemento m0 ∈ X que é menor do que

todos os demais elementos de X. A boa-ordenação pode muitas vezes

substituir com vantagem a indução como método de prova de resultados

referentes a números naturais.

São muito raros e pouco interessantes os exemplos de demonstração por

indução que podem ser dados sem usar as operações fundamentais e as desi-

gualdades. Por isso, somente agora apresentamos um deles, seguido de uma

demonstração por boa-ordenação.

Exemplo 1. Queremos provar a validez, para todo número natural n, da

igualdade

P (n) : 1 + 3 + 5 + . . .+ (2n− 1) = n2

Usaremos indução. Para n = 1, P (1) se resume a armar que 1 = 1. Supondo

P (n) verdadeira para um certo valor de n, somamos 2n+1 a ambos os membros

da igualdade acima, obtendo

1 + 3 + 5 + . . .+ (2n− 1) + (2n+ 1) = n2 + 2n+ 1,

6

Unidade 1Números Naturais

ou seja:

1 + 3 + 5 + . . .+ [2(n+ 1)− 1] = (n+ 1)2.

Mas esta última igualdade é P (n+1). Logo P (n) ⇒ P (n+1). Assim, P (n)

vale para todo n ∈ N. Podemos então armar que a soma dos n primeiros

números ímpares é igual ao quadrado de n.

Exemplo 2. (Usando boa-ordenação.) Lembremos que um número natural

p chama-se primo quando não pode ser expresso como produto p = mn de dois

números naturais, a menos que um deles seja igual a 1 (e o outro igual a p);

isto equivale a dizer que os fatores m, n não podem ser ambos menores do que

p. Um resultado fundamental em Aritmética diz que todo número natural é

primo ou é um produto de fatores primos. Provaremos isto por boa ordenação.

Usaremos a linguagem de conjuntos. Seja X o conjunto dos números naturais

que são primos ou produtos de fatores primos. Observemos que se m e n

pertencem a X então o produto mn pertence a X. Seja Y o complementar

de X. Assim, Y é o conjunto dos números naturais que não são primos nem

são produtos de fatores primos. Queremos provar que Y é vazio. Isto será

feito por redução ao absurdo (como sempre se dá nas demonstrações por boa-

ordenação). Com efeito, se Y não fosse vazio, haveria um menor elemento

a ∈ Y . Então todos os números menores do que a pertenceriam a X. Como a

não é primo, ter-se-ia a = m · n, com m < a e n < a, logo m ∈ X e n ∈ X.

Sendo assim, mn ∈ X. Mas mn = a, o que daria a ∈ X, uma contradição.

Segue-se que Y = ∅ , concluindo a demonstração.

7

Unidade 1 Exercícios Recomendados

1.6 Exercícios Recomendados

1. Dado o número natural a, seja Y ⊂ N um conjunto com as seguintes

propriedades:

(1) a ∈ Y ;

(2) n ∈ Y ⇒ n+ 1 ∈ Y .

Prove que Y contém todos os números naturais maiores do que ou iguais

a a.

(Sugestão: considere o conjunto X = Ia ∪ Y , onde Ia é o conjunto dos números

naturais 6 a, e prove, por indução, que X = N.)

2. Use o exercício anterior para provar que 2n+ 1 6 2n para todo n > 2 e,

em seguida, que n2 < 2n para todo n > 5.

3. Complete os detalhes da seguinte demonstração do Princípio de Boa

Ordenação: Seja A ⊂ N um conjunto que não possui um menor ele-

mento. Considere o conjunto X formado pelos números naturais n tais

que 1, 2, ..., n não pertencem a A. Observe que 1 ∈ X e, além disso, se

n ∈ X então todos os elementos de A são > n + 1. Como n + 1 não

pode ser o menor elemento de A, conclua que n + 1 ∈ X. Logo, por

indução, segue-se que X = N. Portanto A é vazio.

4. Prove, por indução, que (n+ 1

n

)n

6 n

para todo n > 3 e conclua daí que a sequência

1,√2,

3√3,

4√4 . . .

é decrescente a partir do terceiro termo.

5. Prove, por indução, que

1 + 22 + 32 + · · ·+ n2 =n(n+ 1)(2n+ 1)

6.

6. Critique a seguinte argumentação: Quer-se provar que todo número natu-

ral é pequeno. Evidentemente, 1 é um número pequeno. Além disso, se n

for pequeno, n+ 1 também o será, pois não se torna grande um número

pequeno simplesmente somando-lhe uma unidade. Logo, por indução,

todo número natural é pequeno.

8

Unidade 1Números Naturais

7. Use a distributividade para calcular (m + n)(1 + 1) de duas maneiras

diferentes e em seguida use a lei do corte para concluir que m+n = n+m.

8. Seja X ⊂ N um conjunto não-vazio, com a seguinte propriedade: para

qualquer n ∈ N, se todos os números naturais menores do que n perten-

cem a X então n ∈ X. Prove que X = N. (Sugestão: boa ordenação.)

9. Seja P (n) uma propriedade relativa ao número natural n. Suponha que

P (1), P (2) são verdadeiras e que, para qualquer n ∈ N, a verdade de

P (n) e P (n + 1) implica a verdade de P (n + 2). Prove que P (n) é

verdadeira para todo n ∈ N.

10. Use indução para provar que

13 + 23 + 33 + · · ·+ n3 =1

4n2(n+ 1)2.

9

Unidade 1 Textos Complementares

1.7 Textos Complementares

Na Sala de Aula Reexões sobre a sala de aula

Do ponto de vista do ensino em nível do ensino médio, não tem cabimento

expor a Matemática sob forma axiomática. Mas é necessário que o professor

saiba que ela pode ser organizada sob a forma acima delineada. Uma linha de

equilíbrio a ser seguida na sala de aula deve basear-se nos seguintes preceitos:

1. Nunca dar explicações falsas sob o pretexto de que os alunos ainda não têm

maturidade para entender a verdade. (Isto seria como dizer a uma criança que

os bebês são trazidos pela cegonha.) Exemplo: innito é um número muito

grande. Para outro exemplo, vide RPM 29, págs. 13-19.

2. Não insistir em detalhes formais para justicar armações que, além de

verdadeiras, são intuitivamente óbvias e aceitas por todos sem discussão nem

dúvidas. Exemplo: o segmento de reta que une um ponto interior a um ponto

exterior de uma circunferência tem exatamente um ponto em comum com essa

circunferência.

Em contraposição, fatos importantes cuja veracidade não é evidente, como

o Teorema de Pitágoras ou a Fórmula de Euler para poliedros convexos, devem

ser demonstrados (até mesmo de várias formas diferentes).

Excetuam-se, naturalmente, demonstrações longas, elaboradas ou que fa-

çam uso de noções e resultados acima do alcance dos estudantes desse nível

(como o Teorema Fundamental da Algebra, por exemplo).

Provar o óbvio transmite a falsa impressão de que a Matemática é inútil. Por

outro lado, usar argumentos elegantes e convincentes para demonstrar resulta-

dos inesperados é uma maneira de exibir sua força e sua beleza. As demons-

trações, quando objetivas e bem apresentadas, contribuem para desenvolver o

raciocínio, o espírito crítico, a maturidade e ajudam a entender o encadeamento

lógico das proposições matemáticas.

3. Ter sempre em mente que, embora a Matemática possa ser cultivada por

si mesma, como um todo coerente, de elevado padrão intelectual, formado por

conceitos e proposições de natureza abstrata, sua presença no currículo escolar

não se deve apenas ao valor dos seus métodos para a formação mental dos

jovens.

A importância social da Matemática provém de que ela fornece modelos

10

Unidade 1Números Naturais

para analisar situações da vida real. Assim, por exemplo, conjuntos são o

modelo para disciplinar o raciocínio lógico, números naturais são o modelo para

contagem e números reais são o modelo para medida; funções ans servem de

modelo para situações, como o movimento uniforme, em que os acréscimos da

função são proporcionais aos acréscimos da variável independente. E assim por

diante.

11

Unidade 1 Textos Complementares

Na Sala de Aula Uma recomendação

Não se deve dar muita importância à eterna questão de saber se 0 (zero) deve

ou não ser incluído entre os números naturais. (Vide Meu Professor de Ma-

temática, pág. 150.) Praticamente todos os livros de Matemática usados nas

escolas brasileiras consideram 0 como o primeiro número natural (consequente-

mente 1 é o segundo, 2 é o terceiro, etc). Como se viu acima, não adotamos

esse ponto-de-vista. Trata-se, evidentemente, de uma questão de preferência.

Deve-se lembrar que o símbolo 0 (sob diferentes formas grácas) foi empregado

inicialmente pelos maias, posteriormente pelos hindus, difundido pelos árabes e

adotado no ocidente, não como um número e sim como um algarismo, com o

utilíssimo objetivo de preencher uma casa decimal vazia. (No caso dos maias, a

base do sistema de numeração era 20, e não 10.) De resto, a opção do número

natural para iniciar a sequência não se limita a escolher entre 0 e 1. Frequente-

mente esquecemos que, do mesmo modo que conhecemos e usamos o zero mas

começamos os números naturais com 1, a Matemática grega, segundo apre-

sentada por Euclides, não considerava 1 como um número. Nos Elementos,

encontramos as seguintes denições:

Unidade é aquilo pelo qual cada objeto é um. Número é uma multitude de

unidades.

12

Unidade 1Números Naturais

Para Saber MaisComentários sobre Denições e axiomas

Uma denição matemática é uma convenção que consiste usar um nome, ou

uma breve sentença, para designar um objeto ou uma propriedade, cuja descrição

normalmente exigiria o emprego de uma sentença mais longa. Vejamos algumas

denições, como exemplo.

• Ângulo é a gura formada por duas semirretas que têm a mesma origem.

• Primos entre si são dois ou mais números naturais cujo único divisor

comum é a unidade.

Mas nem sempre foi assim. Euclides, por exemplo, começa os Elementos

com uma série de denições, das quais selecionamos as seguintes:

• Linha é um comprimento sem largura.

• Superfície é o que possui comprimento e largura somente.

• Quando uma reta corta outra formando ângulos adjacentes iguais, cada

um desses ângulos chama-se reto e as retas se dizem perpendiculares.

As denições de ângulo e de números primos entre si, dadas acima, bem

como as denições de ângulo reto e retas perpendiculares dadas por Euclides,

são corretas. Elas atendem aos padrões atuais de precisão e objetividade. Por

outro lado, nas denições de linha e superície, Euclides visa apenas oferecer ao

seu leitor uma imagem intuitiva desses conceitos. Elas podem servir para ilustrar

o pensamento geométrico mas não são utilizáveis nos raciocínios matemáticos

porque são formuladas em termos vagos e imprecisos.

Na apresentação de uma teoria matemática, toda denição faz uso de termos

especícos, os quais foram denidos usando outros termos, e assim sucessiva-

mente. Este processo iterativo leva a três possibilidades:

a) Continua indenidamente, cada denição dependendo de outras anteriores,

sem nunca chegar ao m.

b) Conduz a uma circularidade, como nos dicionários. (Onde se vê, por exemplo:

compreender → perceber, perceber → entender e entender → compreender.)

13

Unidade 1 Textos Complementares

c) Termina numa palavra, ou num conjunto de palavras (de preferência dotadas

de conotações intuitivas simples) que não são denidas, isto é, que são tomadas

como representativas de conceitos primitivos. Exemplos: ponto, reta, conjunto.

Evidentemente, as alternativas (a) e (b) acima citadas não convêm à Ma-

temática. A alternativa (c) é a adotada. Se prestarmos atenção, veremos que

foi assim que aprendemos a falar. Numerosas palavras nos foram apresentadas

sem denição e permanecem até hoje em nosso vocabulário como conceitos

primitivos, que aprendemos a usar por imitação e experiência.

Para poder empregar os conceitos primitivos adequadamente, é necessário

dispor de um conjunto de princípios ou regras que disciplinem sua utilização

e estabeleçam suas propriedades. Tais princípios são chamados axiomas ou

postulados. Assim como os conceitos primitivos são objetos que não se denem,

os axiomas são proposições que não se demonstram.

Uma vez feita a lista dos conceitos primitivos e enunciados os axiomas

de uma teoria matemática, todas as demais noções devem ser denidas e as

armações seguintes devem ser demonstradas.

Nisto consiste o chamado método axiomático. As proposições a serem

demonstradas chamam-se teoremas e suas consequências imediatas são

denominadas corolários. Uma proposição auxiliar, usada na demonstração de

um teorema, é chamada um lema.

Ser um axioma ou ser um teorema não é uma característica intrínseca de

uma proposição. Dependendo da preferência de quem organiza a apresentação

da teoria, uma determinada proposição pode ser adotada como axioma ou então

provada como teorema, a partir de outra proposição que a substituiu na lista

dos axiomas.

A seguir veremos um resumo da teoria matemática dos números naturais,

onde os conceitos primitivos são número natural e sucessor e os axiomas são

os de Peano.

14

Unidade 1Números Naturais

Para Saber MaisSobre o sistema de numeração

Um engenhoso processo, chamado sistema de numeração decimal, permite

representar todos os números naturais com o auxílio dos símbolos 0, 1, 2, 3, 4,

5, 6, 7, 8 e 9. Além disso, os primeiros números naturais têm nomes: o sucessor

do número um chama se dois, o sucessor de dois chama-se três, etc. A partir

de um certo ponto, esses nomes tornam-se muito complicados, sendo preferível

abrir mão deles e designar os grandes números por sua representação decimal.

(Na realidade, os números muito grandes não possuem nomes. Por exemplo,

como se chamaria o número 101000?).

Deve car claro que o conjunto N = 1, 2, 3, . . . dos números naturais é

uma sequência de objetos abstratos que, em princípio, são vazios de signicado.

Cada um desses objetos (um número natural) possui apenas um lugar determi-

nado nesta sequência. Nenhuma outra propriedade lhe serve de denição. Todo

número tem um sucessor (único) e, com exceção de 1, tem também um único

antecessor (número do qual é sucessor).

Vistos desta maneira, podemos dizer que os números naturais são números

ordinais : 1 é o primeiro, 2 é o segundo, etc.

15

Unidade 1 Textos Complementares

Para Saber Mais Um comentário gramatical

Quando dizemos o número um, o número dois ou o número três, as

palavras um, dois e três são substantivos, pois são nomes de objetos. Isto

contrasta com o uso destas palavras em frases como um ano, dois meses e

três dias, onde elas aparecem para dar a ideia de número cardinal, isto é, como

resultados de contagens. Nesta frase, um, dois e três não são substanti-

vos. Pertencem a uma categoria gramatical que, noutras línguas (como francês,

inglês e alemão, por exemplo) é chamada adjetivo numeral e que os gramáticos

brasileiros e portugueses, há um par de décadas, resolveram chamar de numeral

apenas. Este comentário visa salientar a diferença entre os números naturais,

olhados como elementos do conjunto N, e o seu emprego como números cardi-

nais. Este segundo aspecto será abordado no capítulo seguinte.

16

Unidade 1Números Naturais

Para Saber MaisCuidado!

O axioma da indução é uma forma sagaz e operacional de dizer que qualquer

número natural n pode ser alcançado se partirmos de 1 e repetirmos suciente-

mente a operação de tomar o sucessor de um número. Ele está presente (pelo

menos de forma implícita) sempre que, ao armarmos a veracidade de uma pro-

posição referente aos números naturais, vericamos que ela é verdadeira para

n = 1, n = 2, n = 3 e dizemos e assim por diante.... Mas é preciso ter

cuidado com esta última frase. Ela pressupõe que P (n) ⇒ P (n′) para todo

n ∈ N. No nal deste capítulo, apresentamos como exercícios algumas propo-

sições demonstráveis por recorrência, bem como alguns curiosos paradoxos que

resultam do uso inadequado do axioma da indução.

17

Unidade 1 Textos Complementares

18

2

1

Números Cardinais

Sumário

2.1 Introdução . . . . . . . . . . . . . . . . . . . . . . . 2

2.2 Funções . . . . . . . . . . . . . . . . . . . . . . . . . 2

2.3 A Noção de Número Cardinal . . . . . . . . . . . . 4

2.4 Conjuntos Finitos . . . . . . . . . . . . . . . . . . . 7

2.5 Exercícios Recomendados . . . . . . . . . . . . . . . 10

2.6 Exercícios Suplementares . . . . . . . . . . . . . . . 10

Unidade 2 Introdução

2.1 Introdução

A importância dos números naturais provém do fato de que eles constituem

o modelo matemático que torna possível o processo de contagem. Noutras

palavras, eles respondem a perguntas do tipo: Quantos elementos tem este

conjunto?

Para contar os elementos de um conjunto é necessário usar a noção de cor-

respondência biunívoca, ou bijeção. Trata-se de um caso particular do conceito

de função, abordado aqui de forma breve, que será desenvolvido com maiores

detalhes na Unidade 3 de MA11.

2.2 Funções

Definição 1 Dados os conjuntos X, Y , uma função f : X → Y (lê-se uma função de

X em Y ) é uma regra (ou conjunto de instruções) que diz como associar a

cada elemento x ∈ X um elemento y = f(x) ∈ Y .

O conjunto X chama-se o domínio e Y é o contra-domínio da função f .

Para cada x ∈ X, o elemento f(x) ∈ Y chama-se a imagem de x pela função

f , ou o valor assumido pela função f no ponto x ∈ X. Escreve-se x 7→ f(x)

para indicar que f transforma (ou leva) x em f(x).

Exemplos particularmente simples de funções são a função identidade f :

X → X, denida por f(x) = x para todo x ∈ X e as funções constantes

f : X → Y , onde se toma um elemento c ∈ Y e se põe f(x) = c para todo

x ∈ X.

+ Para Saber Mais - Recomendações - Clique para ler

Exemplo 1 Sejam X o conjunto dos triângulos do plano Π e R o conjunto dos números

reais (que abordaremos logo mais). Se, a cada t ∈ X, zermos corresponder o

número real f(t) = área do triângulo t, obteremos uma função f : X → R.

2

Unidade 2Números Cardinais

Exemplo 2Sejam S o conjunto dos segmentos de reta do plano Π e ∆ o conjunto das

retas desse mesmo plano. A regra que associa a cada segmento AB ∈ S sua

mediatriz g(AB) dene uma função g : S → ∆.

Exemplo 3A correspondência que associa a cada número natural n seu sucessor n+ 1

dene uma função s : N→ N, com s(n) = n + 1.

Definição 2Uma função f : X → Y chama-se injetiva quando elementos diferentes

em X são transformados por f em elementos diferentes em Y . Ou seja, f é

injetiva quando x 6= x′ em X ⇒ f(x) 6= f(x′).

Esta condição pode também ser expressa em sua forma contrapositiva:

f(x) = f(x′) ⇒ x = x′.

Nos três exemplos dados acima, apenas o terceiro é de uma função injetiva.

(Dois triângulos diferentes podem ter a mesma área e dois segmentos distintos

podem ter a mesma mediatriz mas números naturais diferentes têm sucessores

diferentes.)

Definição 3Diz-se que uma função f : X → Y é sobrejetiva quando, para qualquer

elemento y ∈ Y , pode-se encontrar (pelo menos) um elemento x ∈ X tal que

f(x) = y.

Nos três exemplos dados acima, apenas o segundo apresenta uma função

sobrejetiva. (Toda reta do plano é mediatriz de algum segmento mas apenas

os números reais positivos podem ser áreas de triângulos e o número 1 não é

sucessor de número natural algum.)

Definição 4Chama-se imagem do subconjunto A ⊂ X pela função f : X → Y ao

subconjunto f(A) ⊂ Y formado pelos elementos f(x), com x ∈ A.

3

Unidade 2 A Noção de Número Cardinal

Portanto, uma função f : X → Y é sobrejetiva quando f(X) = Y . O

conjunto f(X), imagem do domínio X pela função f chama-se também a

imagem da função f .

Nos Exemplos 1, 2 e 3, a imagem da função f é o conjunto dos números

reais positivos, a imagem de g é todo o conjunto ∆ e a imagem de s é o

conjunto dos números naturais ≥ 2.

Dada a função f : X → Y , para saber se um certo elemento b ∈ Y pertence

ou não à imagem f(X), escrevemos a equação f(x) = b e procuramos

achar algum x ∈ X que a satisfaça. Consequentemente, para mostrar que f é

sobrejetiva deve-se provar que a equação f(x) = y possui uma solução x ∈ X,

seja qual for o y ∈ Y dado.

+ Para Saber Mais - Recomendação - Clique para ler

Exemplo 4 Considere a tentativa de denir uma função f : N → N, estipulando que,

para todo n ∈ N, o número natural p = f(n) deve ser tal que p2 + 3 = n. O

número p = f(n) só pode ser encontrado se n for igual a 4, 7, 12, 19, ... pois

nem todos os números naturais são da forma p2 + 3. Assim, esta regra não

dene uma função com domínio N, porque tem exceções.

Exemplo 5 Indiquemos com X o conjunto dos números reais positivos e com Y o

conjunto dos triângulos do plano. Para cada x ∈ X, ponhamos f(x) = t caso t

seja um triângulo cuja área é x. Esta regra não dene uma função f : X → Y

porque é ambígua: dado o número x > 0, existe uma innidade de triângulos

diferentes com área x.

2.3 A Noção de Número Cardinal

A conceito de número cadinal se estabelece por meio da noção de bijeção.

Definição 5 Uma função f : X → Y chama-se uma bijeção, ou uma correspondência

biunívoca entre X e Y quando é ao mesmo tempo injetiva e sobrejetiva.

4

Unidade 2Números Cardinais

Exemplo 6Sejam X = 1, 2, 3, 4, 5 e Y = 2, 4, 6, 8, 10. Denindo f : X → Y

pela regra f(n) = 2n, temos uma correspondência biunívoca, onde f(1) = 2,

f(2) = 4, f(3) = 6, f(4) = 8 e f(5) = 10.

Exemplo 7Um exemplo particularmente curioso de correspondência biunívoca, que

estende o exemplo anterior, foi descoberto pelo físico Galileu Galilei, que viveu

há quatrocentos anos. Seja P o conjunto dos números naturais pares:

P = 2, 4, 6, . . . , 2n, . . ..

Obtém-se uma correspondência biunívoca f : N→ P pondo-se f(n) = 2n para

todo n ∈ N. O interessante deste exemplo é que P é um subconjunto próprio

de N.

Exemplo 8Sejam Y a base de um triângulo e X um segmento paralelo a Y , unindo

os outros dois lados desse triângulo. Seja ainda P o vértice oposto à base

Y . Obtém-se uma correspondência biunívoca f : X → Y associando a cada

x ∈ X o ponto f(x) onde a semirreta Px intersecta a base Y .

5

Unidade 2 A Noção de Número Cardinal

Figura 2.1: Correspondência biunívoca entre dois segmentos

Exemplo 9 Neste exemplo, X = C \ P é o conjunto obtido retirando da circun-

ferência o ponto P e Y é uma reta perpendicular ao diâmetro que não passa

por P . Dena a correspondência biunívoca f : X → Y pondo, para cada

x ∈ X, f(x) = intersecção da semirreta Px com a reta Y .

Figura 2.2: O círculo sem um ponto e a reta

Definição 6 Diz-se que dois conjuntos X e Y tem o mesmo número cardinal quando

se pode denir uma correspondência biunívoca f : X → Y .

Cada um dos quatro exemplos acima exibe um par de conjuntos X, Y com

o mesmo cardinal.

6

Unidade 2Números Cardinais

Exemplo 10Sejam X = 1 e Y = 1, 2. Evidentemente não pode existir uma

correspondência biunívoca f : X → Y , portanto X e Y não têm o mesmo

número cardinal.

+ Para Saber Mais - A palavra número no dicionário - Clique para ler

2.4 Conjuntos Finitos

Dado n ∈ N, indiquemos com a notação In o conjunto dos números naturais

de 1 até n. Assim, I1 = 1, I2 = 1, 2, I3 = 1, 2, 3 e, mais geralmente,

um número natural k pertence a In se, e somente se, 1 ≤ k ≤ n.

Definição 7Seja X um conjunto. Diz-se que X é nito, e que X tem n elementos

quando se pode estabelecer uma correspondência biunívoca f : In → X.

O número natural n chama-se então o número cardinal do conjunto X ou,

simplesmente, o número de elementos de X. A correspondência f : In → X

chama-se uma contagem dos elementos de X. Pondo f(1) = x1, f(2) =

x2, . . . , f(n) = xn, podemos escrever X = x1, x2, . . . , xn. Para todo n, o

conjunto In é nito e seu número cardinal é n. Assim, todo número natural n

é o número cardinal de algum conjunto nito.

A m de evitar exceções, admite-se ainda incluir o conjunto vazio ∅ entre

os conjuntos nitos e diz-se que ∅ tem zero elementos. Assim, por denição,

zero é o número cardinal do conjunto vazio.

Diz-se que um conjunto X é innito quando ele não é nito. Isto quer

dizer que X não é vazio e que, não importa qual seja n ∈ N , não existe

correspondência biunívoca f : In → X.

No Exemplo 6 acima, temos X = I5 e f : X → Y é uma contagem

dos elementos de Y . Assim, Y é um conjunto nito, com 5 elementos. O

conjunto N dos números naturais é innito. Com efeito, dada qualquer função

f : In → N , não importa qual n se xou, pomos k = f(1) + f(2) + · · ·+ f(n)

e vemos que, para todo x ∈ In, tem-se f(x) < k, logo não existe x ∈ In tal

7

Unidade 2 Conjuntos Finitos

que f(x) = k. Assim, é impossível cumprir a condição de sobrejetividade na

denição de correspondência biunívoca.

O número cardinal de um conjunto nito X, que indicaremos com a notação

n(X), goza de algumas propriedades básicas, entre as quais destacaremos as

seguintes:

1. O número de elementos de um conjunto nito é o mesmo, seja qual for

a contagem que se adote. Isto signica que se f : Im → X e g : In → X

são correspondências biunívocas então m = n.

2. Todo subconjunto Y de um conjunto nito X é nito e n(Y ) ≤ n(X).

Tem-se n(Y ) = n(X) somente quando Y = X.

3. Se X e Y são nitos então X ∪Y é nito e tem-se n(X ∪Y ) = n(X) +

n(Y )− n(X ∩ Y ) .

4. Sejam X, Y conjuntos nitos. Se n(X) > n(Y ), nenhuma função f :

X → Y é injetiva e nenhuma função g : Y → X é sobrejetiva.

As demonstrações destes fatos se fazem por induçãoo ou por boa-ordenação.

(Veja, por exemplo, [Lima]: Curso de Análise, vol. 1, págs. 33-38.) A primeira

parte do item 4. acima é conhecida como o princípio das casas de pombos : se

há mais pombos do que casas num pombal, qualquer modo de alojar os pombos

deverá colocar pelo menos dois deles na mesma casa. As vezes, o mesmo fato

é chamado o princípio das gavetas : se m > n, qualquer maneira de distribuir

m objetos em n gavetas deverá por ao menos dois desses objetos na mesma

gaveta. (Na referência [Lima] citada, este é o Corolário 1 na página 35.)

O princípio das casas de pombos, com toda sua simplicidade, possui inte-

ressantes aplicações. Vejamos duas delas.

Exemplo 11 Tomemos um número natural de 1 a 9. Para xar as ideias, seja 3 esse

número. Vamos provar que todo número natural m possui um múltiplo cuja

representação decimal contém apenas os algarismos 3 ou 0. Para isso, conside-

remos o conjunto X = 3, 33, . . . , 33 . . . 3, cujos elementos são os m primeiros

números naturais representados somente por algarismos iguais a 3. Se algum

dos elementos de X for múltiplo de m, nosso trabalho acabou. Caso contrário,

8

Unidade 2Números Cardinais

formamos o conjunto Y = 1, 2, . . . ,m− 1 e denimos a função f : X → Y

pondo, para cada x ∈ X,

f(x) = resto da divisão de x por m.

Como X tem mais elementos do que Y , o princípio das casas de pombos

assegura que existem elementos x1 < x2 no conjunto X tais que f(x1) = f(x2).

Isto signica que x1 e x2 , quando divididos por m, deixam o mesmo resto. Logo

x2−x1 é múltiplo de m. Mas é claro que se x1 tem p algarismos e x2 tem p+q

algarismos então a representação decimal de x2 − x1 consiste em q algarismos

iguais a 3 seguidos de p algarismos iguais a 0.

Exemplo 12Vamos usar o princípio das gavetas para provar que, numa reunião com n

pessoas (n ≥ 2), há sempre duas pessoas (pelo menos) que têm o mesmo nú-

mero de amigos naquele grupo. Para ver isto, imaginemos n caixas, numeradas

com 0, 1, . . . , n − 1. A cada uma das n pessoas entregamos um cartão que

pedimos para depositar na caixa correspondente ao número de amigos que ela

tem naquele grupo. As caixas de números 0 e n− 1 não podem ambas receber

cartões pois se houver alguém que não tem amigos ali, nenhum dos presentes

pode ser amigo de todos, e vice-versa. Portanto temos, na realidade, n cartões

para serem depositados em n−1 caixas. Pelo princípio das gavetas, pelo menos

uma das caixas vai receber dois ou mais cartões. Isto signica que duas ou mais

pessoas ali têm o mesmo número de amigos entre os presentes.

+ Para Saber Mais - Sobre Conjuntos Innitos - Clique para ler

+ Para Saber Mais - Fantasia Matemática - Clique para ler

+ Para Saber Mais - Cuidado! - Clique para ler

9

Unidade 2 Exercícios Recomendados

2.5 Exercícios Recomendados

1. Prove, por indução, que se X é um conjunto nito com n elementos

então existem n! bijeções f : X → X.

2. Prove, por indução, que um conjunto com n elementos possui 2n subcon-

juntos.

3. Sejam X e Y dois conjuntos nitos, com m e n elementos, respectiva-

mente. Mostre que existem nm funções f : X → Y . Você seria capaz

de resolver diretamente o Exercício 2, utilizando este resultado?

2.6 Exercícios Suplementares

1. Dena uma função sobrejetiva f : N → N tal que, para todo n ∈ N, aequação f(x) = n possui uma innidade de raízes x ∈ N.

Sugestão: Todo número natural se escreve, de modo único sob a forma

2a · b, onde a, b ∈ N e b é ímpar.

2. Dados n (n > 2) objetos de pesos distintos, prove que é possível deter-

minar qual o mais leve e qual o mais pesado fazendo 2n − 3 pesagens

em uma balança de pratos. É esse o número mínimo de pesagens que

permitem determinar o mais leve e o mais pesado?

3. Prove que, dado um conjunto com n elementos, é possível fazer uma la

com seus subconjuntos de tal modo que cada subconjunto da la pode

ser obtido a partir do anterior pelo acréscimo ou pela supressão de um

único elemento.

4. Todos os quartos do Hotel Georg Cantor estão ocupados, quando chegam

os trens T1, T2, . . . , (em quantidade innita), cada um deles com innitos

passageiros. Que deve fazer o gerente para hospedar todos?

10

Referências Bibliográcas

[Lima] Lima, Elon Lages. Curso de Análise, Vol. 1. Projeto Euclides, IMPA,

Rio de Janeiro, 1976. 8, 15

11

Unidade 2 Textos Complementares

2.7 Textos Complementares

Para Saber Mais Recomendações

1. É importante ressaltar que f(x) é a imagem do elemento x ∈ X pela

função f , ou o valor da função f no ponto x ∈ X. Os livros antigos, bem

como alguns atuais, principalmente os de Cálculo, costumam dizer a função

f(x) quando deveriam dizer a função f . Algumas vezes essa linguagem

inexata torna a comunicação mais rápida e ca difícil resistir à tentação de

usá-la. Mas é indispensável a cada momento ter a noção precisa do que se está

fazendo.

Na prática, há algumas funções com as quais é simples e natural lidar usando

a terminologia correta. Por exemplo, é fácil acostumar-se a escrever as funções

sen : R → R e log : R+ → R, guardando as notações senx e log x para os

números reais que são os valores destas funções num dado ponto x. Por outro

lado, quando se trata de uma função polinomial, o bom-senso nos leva a dizer

a função x2 − 5x + 6

em vez da forma mais correta e mais pedante a função p : R→ R tal que

p(x) = x2 − 5x + 6

para todo x ∈ R . Caso análogo se dá com a função exponencial ex, embora

recentemente se tenha tornado cada vez mais frequente escrever exp(x) = ex

e assim poder falar da função exp : R→ R.2. Deve-se ainda recordar que uma função consta de três ingredientes: domínio,

contra-domínio e a lei de correspondência x 7→ f(x). Mesmo quando dizemos

simplesmente a função f , cam subentendidos seu domínio X e seu contra-

domínio Y . Sem que eles sejam especicados, não existe a função. Assim

sendo, uma pergunta do tipo Qual é o domínio da função f(x) = 1/x ?,

estritamente falando, não faz sentido. A pergunta correta seria: Qual é o

maior subconjunto X ⊂ R tal que a fórmula f(x) = 1/x dene uma função

f : X → R ? Novamente, a pergunta incorreta é mais simples de formular.

Se for feita assim, é preciso saber seu signicado.

Segue-se do que foi dito acima que as funções f : X → Y e g : X ′ → Y ′

são iguais se, e somente se, X = X ′, Y = Y ′ e f(x) = g(x) para todo x ∈ X.

12

Unidade 2REFERÊNCIAS BIBLIOGRÁFICAS

Para Saber MaisRecomendação

3. Em muitos exemplos de funções f : X → Y , principalmente na Matemática

Elementar, X e Y são conjuntos numéricos e a regra x 7→ f(x) exprime o valor

f(x) por meio de uma fórmula que envolve x. Mas em geral não precisa ser

assim. A natureza da regra que ensina como obter f(x) quando é dado x é

inteiramente arbitrária, sendo sujeita apenas a duas condições:

a) Não deve haver exceções: a m de que a função f tenha o conjunto X

como domínio, a regra deve fornecer f(x), seja qual for x ∈ X dado.

b) Não pode haver ambiguidades: a cada x ∈ X, a regra deve fazer corres-

ponder um único f(x) em Y . Os exemplos a seguir ilustram essas exigências.

13

Unidade 2 Textos Complementares

Para Saber Mais A palavra número no dicionário

As vezes se diz que os conjuntos X e Y são (numericamente) equivalentes

quando é possível estabelecer uma correspondência biunívoca f : X → Y , ou

seja, quando X e Y têm o mesmo número cardinal.

Isto explica (embora não justique) a denição dada no dicionário mais

vendido do país. Em algumas situações, ocorrem em Matemática denições

do tipo seguinte: um vetor é o conjunto de todos os segmentos de reta do

plano que são equipolentes a um segmento dado. (Denição por abstração.)

Nessa mesma veia, poder-se-ia tentar dizer: número cardinal de um conjunto

é o conjunto de todos os conjuntos equivalentes a esse conjunto. No caso

do dicionário, há um conjunto de defeitos naquela denição, com um número

cardinal razoavelmente elevado. Os três mais graves são:

1. Um dicionário não é um compêndio de Matemática, e muito menos de Ló-

gica. Deve conter explicações acessíveis ao leigo (de preferência, corretas). As

primeiras acepções da palavra número num dicionário deveriam ser quanti-

dade e resultado de uma contagem ou de uma medida.

2. A denição em causa só se aplica a números cardinais, mas a ideia de número

deveria abranger os racionais e, pelo menos, os reais.

3. O conjunto de todos os conjuntos equivalentes a um conjunto dado é um

conceito matematicamente incorreto. A noção de conjunto não pode ser usada

indiscriminadamente, sem submeter-se a regras determinadas, sob pena de con-

duzir a paradoxos, ou contradições. Uma dessas regras proíbe que se forme

conjuntos a não ser que seus elementos pertençam a, ou sejam subconjuntos

de, um determinado conjunto-universo. Um exemplo de paradoxo que resulta

da desatenção a essa regra é o conjunto X de todos os conjuntos que não

são elementos de si mesmos. Pergunta-se: X é ou não é um elemento de si

mesmo? Qualquer que seja a resposta, chega-se a uma contradição.

14

Unidade 2REFERÊNCIAS BIBLIOGRÁFICAS

Para Saber MaisSobre Conjuntos Innitos

Para encerrar estas considerações a respeito de números cardinais, faremos

alguns comentários sobre conjuntos innitos.

Em primeiro lugar, convém esclarecer que a maior contribuição de Cantor

não foi a adoção da linguagem e da notação dos conjuntos e sim suas desco-

bertas sobre os números cardinais de conjuntos innitos. Ele foi o primeiro a

descobrir que existem conjuntos innitos com diferentes cardinalidades ao pro-

var que não pode haver uma correspondência biunívoca entre N e o conjunto

R dos números reais e que nenhum conjunto X pode estar em correspondência

biunívoca com o conjunto P(X) cujos elementos são os subconjuntos de X.

Além disso, ele mostrou que a reta, o plano e o espaço tri-dimensional (ou

mesmo espaços com dimensão superior a três) têm o mesmo número cardinal.

Estes fatos, que atualmente são considerados corriqueiros entre os matemáticos,

causaram forte impacto na época (meados do século dezenove).

A segunda observação diz respeito a funções f : X → X de um conjunto em

si mesmo. Quando X é nito, f é injetiva se, e somente se, é sobrejetiva (veja

a referência [Lima].) Mas isto não é verdadeiro para X innito. Por exemplo,

se denirmos a função f : N → N pondo, para cada n ∈ N, f(n) = número

de fatores primos distintos que ocorrem na decomposição de n, veremos que f

é sobrejetiva mas não é injetiva. (Para cada b ∈ N existe uma innidade de

números n tais que f(n) = b.) Além disso, as funções f : N→ N, g : N→ N,h : N→ N e ϕ : N→ N, denidas por

f(n) = n + 1,

g(n) = n + 30,

h(n) = 2n e

ϕ(n) = 3n

(2.1)

são injetivas mas não são sobrejetivas. Estas quatro funções são protagonistas

da historinha seguinte que fecha a unidade.

15

Unidade 2 Textos Complementares

Para Saber Mais Fantasia Matemática

O Grande Hotel Georg Cantor tinha uma innidade de quartos, numera-

dos consecutivamente, um para cada número natural. Todos eram igualmente

confortáveis. Num m-de-semana prolongado, o hotel estava com seus quartos

todos ocupados, quando chega um viajante. A recepcionista vai logo dizendo:

Sinto muito, mas não há vagas.

Ouvindo isto, o gerente interveio:

Podemos abrigar o cavalheiro, sim senhora.

E ordena:

Transra o hóspede do quarto 1 para o quarto 2, passe o do quarto 2

para o quarto 3 e assim em diante. Quem estiver no quarto n, mude para o

quarto n + 1. Isto manterá todos alojados e deixará disponível o quarto 1 para

o recém-chegado.

Logo depois chegou um ônibus com 30 passageiros, todos querendo hospe-

dagem. A recepcionista, tendo aprendido a lição, removeu o hóspede de cada

quarto n para o quarto n+ 30 e acolheu assim todos os passageiros do ônibus.

Mas cou sem saber o que fazer quando, horas depois, chegou um trem com

uma innidade de passageiros. Desesperada, apelou para o gerente que pron-

tamente resolveu o problema dizendo:

Passe cada hóspede do quarto n para o quarto 2n. Isto deixará vagos todos

os apartamentos de número ímpar, nos quais poremos os novos hóspedes.

Pensando melhor: mude quem está no quarto n para o quarto 3n. Os novos

hóspedes, ponha-os nos quartos de número 3n+2. Deixaremos vagos os quartos

de número 3n + 1. Assim, sobrarão ainda innitos quartos vazios e eu poderei

ter sossego por algum tempo.

16

Unidade 2REFERÊNCIAS BIBLIOGRÁFICAS

Para Saber MaisCuidado!

Não confunda conjunto innito com aquele que tem um número muito grande

(porém nito) de elementos. Quando, na linguagem comum, se diz algo como -

Já ouvi isto uma innidade de vezes, trata-se de uma mera força de expressão.

Não há distâncias innitas (mesmo entre duas galáxias bem afastadas) e até

o número de átomos do universo é nito. (O físico Arthur Eddington estimou

o número de prótons do universo em 136 × 2256. O número de átomos é

certamente menor pois todo átomo contém ao menos um próton.) E importante

ter sempre em mente que nenhum número natural n é maior do que todos os

demais: tem-se sempre n < n + 1.

17

Unidade 2 Textos Complementares

18

3

1

O Princípio de InduçãoMatemática

Sumário

3.1 Introdução . . . . . . . . . . . . . . . . . . . . . . . 2

3.2 O Poder do Método de Indução . . . . . . . . . . . 2

3.3 Exercícios Recomendados . . . . . . . . . . . . . . . 12

3.4 Exercícios Suplementares . . . . . . . . . . . . . . . 13

3.5 Textos Complementares . . . . . . . . . . . . . . . . 14

Unidade 3 Introdução

3.1 Introdução

Nesta unidade e na próxima, mostraremos como utilizar o Axioma de Indução

para denir com rigor objetos matemáticos e também como utilizá-lo como

poderoso instrumento para demonstrar os mais variados resultados envolvendo

números naturais. Algumas das noções introduzidas nesta e na próxima unidade

serão retomadas de modo mais sistemático nas Unidades 5 a 8.

3.2 O Poder do Método de Indução

Comecemos com a pergunta:

O que signicam expressões do tipo 1 + 2 + · · ·+ n e 1 · 2 · · · · n?Note que as operações de adição e de multiplicação nos números naturais (ou em

qualquer sistema numérico) são binárias, isto é, elas relacionam dois elementos

de cada vez. Apesar disso, temos uma ideia bastante intuitiva do signicado

das expressões acima, até mesmo no que diz respeito aos pontinhos que nelas

aparecem. Existe, contudo, um modo de tornar mais rigorosas denições desse

tipo por meio do Princípio de Indução Matemática, como veremos mais adiante.

Antes, porém, recordemos este princípio que demonstramos na Unidade 1.

Princípio de Indução Matemática Se P (n) é uma propriedade relativa ao

número natural n, tal que

i) P (1) é válida;

ii) Para todo n ∈ N, a validez de P (n) implica a validez de P (n+ 1).

Então P (n) é válida qualquer que seja o número natural n.

+ Para Saber Mais - Comentário - Clique para ler

+ Para Saber Mais - Indução Empírica vs Indução Matemática - Cli-

que para ler

2

Unidade 3O Princípio de Indução Matemática

Para denir uma expressão En, para todo número natural n, basta denirmos

E1 e mostrar, para todo n ∈ N, como obter sem ambiguidade En+1 a partir de

En.

Nesse caso, dizemos que En foi denido por recorrência.

Vejamos como intervém o Princípio de Indução Matemática para justicar

este tipo de denição. Seja X o subconjunto de N, determinado pela condição:

n ∈ X ⇐⇒ En está denido.

Pela caracterização do conjunto X, temos que 1 ∈ X e, para todo n ∈ N,n ∈ X ⇒ n+ 1 ∈ X. Portanto, X = N.

Denições por recorrência podem ser utilizadas para dar um signicado a

expressões como no início da unidade.

Exemplo 1Denimos S1 = 1. Em seguida, supondo Sn denido, pomos

Sn+1 = Sn + (n+ 1).

Damos assim, um sentido matemático preciso à expressão:

Sn = 1 + 2 + · · ·+ n.

Por outro lado, denindo 1! = 1 e pondo (n+1)! = n!(n+1), supondo que

n! esteja denido, damos também, neste caso, um sentido matemático para a

expressão:

n! = 1 · 2 · · ·n.

Para generalizar os exemplos acima, vamos introduzir a noção de sequência.

Teremos oportunidade de comprovar, ao longo do curso, o quanto é central este

conceito.

Definição 1Uma sequência de elementos de um conjunto A é uma função x : N→ A.

3

Unidade 3 O Poder do Método de Indução

Tendo em vista que uma função conhecida quando se sabe qual é a ima-

gem de todo elemento de seu domínio, uma sequência x : N → A pode ser

representada como

x(1), x(2), . . . , x(n), . . . ,

ou ainda, denotando x(n) por xn, podemos representá-la concisamente por

(xn).

Por motivo de economia, quando dissermos que um conjunto A possui uma

adição ou uma multiplicação satisfazendo às leis básicas da aritmética, esta-

remos supondo que em A estão denidas duas operações com propriedades

semelhantes às correspondentes operações nos naturais.

Seja agora (xn) uma sequência de elementos de um conjunto A que possui

duas operações, de adição e de multiplicação, satisfazendo às leis básicas da

aritmética.

Definição 2 Denem-se Sn e Pn em A, como se segue: S1 = P1 = x1 e

Sn+1 = Sn + xn+1 e Pn+1 = Pn · xn+1.

Isto dá sentido às seguintes expressões:

Sn = x1 + x2 + · · ·+ xn e Pn = x1 · x2 · · ·xn.

Somas e produtos, como acima, serão também escritos com as notações de

somatórios e produtórios:

Sn =n∑

i=1

xi e Pn =n∏

i=1

xi,

que se leem como somatório quando i varia de 1 até n de xi e produto

quando i varia de 1 até n de xi, respectivamente.

Note que a partir de uma sequência dada (xn), pudemos denir de modo

natural duas outras sequências, a saber: (Sn) e (Pn).

Dada uma sequência constante, x(n) = a, para todo n ∈ N, onde a ∈ A,os termos da sequência Pn a ela associada são por denição as potências de a.

Pela sua importância, destacamos essa denição a seguir.

4

Unidade 3O Princípio de Indução Matemática

Definição 3Seja a um elemento de um conjunto A munido de uma multiplicação sujeita

às leis básicas da aritmética. As potências an de a, com n ∈ N, são denidas

por recorrência como segue: a1 = a e an+1 = an · a.

Quando a 6= 0, convenciona-se denir a0 = 1. Isto será especialmente con-

veniente quando estendermos as potências para expoentes não necessariamente

números naturais. Isto se tornará bem mais claro na Unidade 13 de MA11.

Exemplo 2Neste exemplo, queremos determinar uma fórmula para a soma dos n pri-

meiros números naturais: Sn = 1 + 2 + · · ·+ n.

Conta-se a seguinte história sobre o matemático alemão Carl Friedrich Gauss

(1777-1855), considerado um dos maiores gênios da matemática de todos os

tempos, quando ainda garoto. Na escola, o professor, para aquietar a turma de

Gauss, mandou os alunos calcularem a soma de todos os números naturais de

1 a 100. Qual não foi a sua surpresa quando, instantes depois, o menino deu

a resposta: 5050. Indagado sobre como tinha descoberto tão rapidamente o

resultado, Gauss, então, descreveu o método a seguir.

Sendo

Sn = 1 + 2 + · · ·+ n,

o objetivo é encontrar uma fórmula fechada1 para Sn.

Somando a igualdade acima, membro a membro, com ela mesma, porém

com as parcelas do segundo membro em ordem invertida, temos que

Sn = 1 + 2 + · · · + n

Sn = n + (n− 1) + · · · + 1

2Sn = (n+ 1) + (n+ 1) + · · · + (n+ 1)

Daí segue-se que 2Sn = n(n+ 1) e, portanto,

Sn =n(n+ 1)

2.

1Uma fórmula fechada para Sn, a grosso modo, é uma função de n que permite calcular

diretamente os valores de Sn fazendo um pequeno número de cálculos.

5

Unidade 3 O Poder do Método de Indução

Vamos ser críticos com relação à prova acima. Para a maioria das pessoas,

essa prova parece impecável, mas se alguém nos perguntasse o que está escon-

dido atrás dos pontinhos, talvez nos sentíssemos embaraçados. Também, como

ter absoluta certeza de que nada acontece fora do nosso controle, exatamente

na imensa região coberta pelos pontinhos?

Para não pairar nenhuma dúvida sobre o nosso resultado, vamos provar a

fórmula utilizando o Princípio de Indução Matemática.

Considere a sentença sobre os naturais:

P (n) : 1 + 2 + · · ·+ n =n(n+ 1)

2. (3.1)

Note que

P (1) : 1 =1(1 + 1)

2

é verdadeira.

Observe também que

P (n+ 1): 1 + 2 + · · ·+ n+ (n+ 1) =(n+ 1)(n+ 2)

2.

Agora, suponhamos que para algum n ∈ N, tenhamos P (n) verdadeira, isto

é, a fórmula (1.1) é válida para tal valor de n. Somando n+1 a ambos os lados

dessa igualdade, temos que é verdadeira a igualdade

1 + 2 + · · ·+ n+ (n+ 1) =n(n+ 1)

2+ n+ 1 =

n(n+ 1) + 2(n+ 1)

2=

(n+ 1)(n+ 2)

2,

o que estabelece a veracidade de P (n+ 1).

Pelo Princípio de Indução, tem-se que a fórmula P (n) é verdadeira para

todo n ∈ N.

+ Na Sala de Aula - Considerações sobre o Rigor - Clique para ler

Exemplo 3 Queremos validar a fórmula

P (n) : 12 + 22 + · · ·+ n2 =n(n+ 1)(2n+ 1)

6. (3.2)

6

Unidade 3O Princípio de Indução Matemática

Note que

P (1) : 12 =1(1 + 1)(2 + 1)

6

é verdadeira.

Suponha que, para algum n ∈ N, se tenha que P (n) é verdadeira, isto é,

(1.2) é válida. Somando (n + 1)2 a ambos os lados da igualdade (1.2), temos

que

12 + 22 + · · ·+ n2 + (n+ 1)2 =n(n+ 1)(2n+ 1)

6+ (n+ 1)2 =

n(n+ 1)(2n+ 1) + 6(n+ 1)2

6=

(n+ 1)[n(2n+ 1) + 6(n+ 1)]

6=

(n+ 1)[(n+ 1) + 1][2(n+ 1) + 1]

6,

estabelecendo assim a veracidade de P (n+ 1).

Portanto, a fórmula é válida para todo n ∈ N.

Exemplo 4Vamos provar que é verdadeira, para todo n ∈ N, a fórmula:

P (n) :1

1.2+

1

2.3+ · · ·+ 1

n(n+ 1)=

n

n+ 1. (3.3)

Observemos inicialmente que

P (1) :1

1.2=

1

1 + 1

é verdadeira.

Suponhamos que, para algum n, tem-se que P (n) é verdadeira, ou seja,

que a fórmula (1.3) seja verdadeira para esse valor de n. Somando a ambos os

lados dessa igualdade1

(n+ 1)(n+ 2), temos que

1

1.2+

1

2.3+ · · ·+ 1

n(n+ 1)+

1

(n+ 1)(n+ 2)=

n

n+ 1+

1

(n+ 1)(n+ 2)=n+ 1

n+ 2,

mostrando, assim, que P (n+ 1) é verdadeira.

7

Unidade 3 O Poder do Método de Indução

Portanto, pelo Princípio de Indução Matemática, temos que a fórmula vale

para todo n ∈ N.

A seguir, vamos estabelecer, por meio de indução, as propriedades usuais

das potências.

Proposição 4 Sejam a, b ∈ A e m,n ∈ N. Então,

i) am · an = an+m.

ii) (am)n = amn.

iii) (a · b)n = an · bn.

Demonstração Provaremos (i), deixando o restante como exercício.

Fixemos a ∈ A e m ∈ N, arbitrariamente. Demonstremos a propriedade

por indução sobre n.

Para n = 1, a propriedade é válida, pois, pelas denições,

am · a1 = am · a = am+1.

Por outro lado, supondo que am · an = am+n, temos que

am · an+1 = am · (an · a) = (am · an) · a = am+n · a = am+n+1.

Isso, pelo Princípio de Indução Matemática, prova a nossa propriedade.

Exemplo 5 Utilizando a noção de potência e de suas propriedades, mostraremos que 3

divide 5n + 2 · 11n nos inteiros, para todo n ∈ N.De fato, para n = 1, temos que 3 divide 51 + 2 · 111 = 27.

Suponha, agora, que, para algum n ≥ 1, saibamos que 3 divide 5n+2 ·11n.Logo, existe um número inteiro a tal que

5n + 2 · 11n = 3a.

Mutiplicando por 5 ambos os lados da igualdade acima, temos

5 · 3a = 5n+1 + 5 · 2 · 11n = 5n+1 + 2 · 11 · 11n − 12 · 11n.

8

Unidade 3O Princípio de Indução Matemática

Daí segue-se a igualdade

5n+1 + 2 · 11n+1 = 5 · 3a+ 12 · 11n,

cujo segundo membro é divisível por 3, por ser igual a 3(5a+ 4 · 11n).Assim, provamos que 3 divide 5n+1 + 2 · 11n+1, o que, pelo Princípio de

Indução Matemática, acarreta que 3 divide 5n + 2 · 11n, para todo número

natural n.

Pode ocorrer que uma determinada propriedade seja válida para todos os

números naturais a partir de um determinado valor a, mas não necessariamente

para valores menores. Como proceder nesses casos? Por exemplo, como provar

que a desigualdade 2n > n2 é verdadeira para todo valor de n natural maior ou

igual do que 5? Fazemos isso baseados na seguinte pequena generalização do

Princípio de Indução Matemática:

Teorema 1Seja P (n) uma sentença sobre N, e seja a ∈ N. Suponha que:

(i) P (a) é verdadeira, e

(ii) qualquer que seja n ∈ N, com n ≥ a, sempre que P (n) é verdadeira,

segue-se que P (n+ 1) é verdadeira.

Então, P (n) é verdadeira para todo número natural n ≥ a.

DemonstraçãoDena o conjunto

S = m ∈ N; P (m+ a− 1) .

Por (i) temos que 1 ∈ S. Por outro lado, se m ∈ S, temos que P (m + a −1) é verdadeira. Logo, por (ii), P (m + 1 + a − 1) é verdadeira. Portanto,

m+ 1 ∈ S. Em vista do Princípio de Indução Matemática, temos que S = N.Consequentemente, P (n) é verdadeira para todo n ≥ a.

Exemplo 6Vamos mostrar que a desigualdade na sentença P (n) : 2n > n2 é verdadeira,

para todo número natural n ≥ 5.

9

Unidade 3 O Poder do Método de Indução

Note que P (1) : 21 > 12 é verdadeira, P (2) : 22 > 22 é falsa, P (3) : 23 > 32

é falsa e P (4) : 24 > 42 é falsa. Tudo isso não importa, pois queremos vericar

a veracidade dessa desigualdade para n ≥ 5.

De fato, temos que P (5) : 25 > 52 é verdadeira. Seja n ≥ 5 tal que

2n > n2. Multiplicando ambos os lados da desigualdade acima por 2, obtemos

2n+1 > 2n2. Note que 2n2 > (n + 1)2, se n ≥ 3, pois tal desigualdade é

equivalente a n(n − 2) > 1. Daí, deduzimos que 2n+1 > (n + 1)2, o que

signica que P (n + 1) é verdadeira, estabelecendo o resultado em vista do

Teorema 1.

Exemplo 7 Um banco tem um suprimento ilimitado de notas de 3 e de 5 (unidades de

moeda). Mostre que ele pode pagar qualquer quantia (de unidades de moeda)

maior do que 7.

Para isto, basta mostrar que a sentença:

P (n) : A equação 3x+ 5y = n tem solução em (N ∪ 0)× (N ∪ 0),

é verdadeira para todo n ≥ 8.

De fato, ela é verdadeira para n = 8, pois a equação 3x+ 5y = 8 admite a

solução (x, y) = (1, 1).

Suponha agora que a equação 3x + 5y = n tenha uma solução (a, b) para

algum n ≥ 8; isto é, 3a + 5b = n. Note que, para qualquer solução (a, b),

devemos ter a ≥ 1 ou b ≥ 1.

Se b ≥ 1, observando que 3× 2− 5× 1 = 1, segue que

3(a+ 2) + 5(b− 1) = 3a+ 5b+ 3× 2− 5× 1 = 3a+ 5b+ 1 = n+ 1,

o que mostra que a equação 3x + 5y = n + 1 admite a solução (a + 2, b− 1)

em (N ∪ 0)× (N ∪ 0).Se, por acaso, b = 0, então, a ≥ 3; usando a igualdade −3× 3+5× 2 = 1,

temos

3(a− 3) + 5× 2 = 3a− 3× 3 + 5× 2 = 3a+ 5b+ 1 = n+ 1,

o que mostra que a equação 3x + 5y = n + 1 admite a solução (a− 3, b + 2)

em (N ∪ 0)× (N ∪ 0).

10

Unidade 3O Princípio de Indução Matemática

Mostramos assim, que, em qualquer caso, a equação 3x+5y = n+1 admite

solução, sempre que a equação 3x+5y = n, para algum n ≥ 8, tenha solução.

Como o resultado vale para n = 8, segue a conclusão desejada pelo Teorema 1.

Note que n0 = 8 é o menor valor de n para o qual a equação tem solução

para todo n ≥ n0.

11

Unidade 3 Exercícios Recomendados

3.3 Exercícios Recomendados

1. Mostre, por indução, a validez das seguintes fórmulas:

(a) 1.20 + 2.21 + 3.22 + · · ·+ n.2n−1 = 1 + (n− 1)2n;

(b)

(1 +

1

1

)(1 +

1

2

)2

· · ·(1 +

1

n− 1

)n−1

=nn−1

(n− 1)!,

(c) 1.1! + 2.2! + 3.3! + · · ·+ n.n! = (n+ 1)!− 1.

2. Sejam a e b números reais distintos. Mostre que, para todo n ∈ N, valea igualdade:

bn + abn−1 + a2bn−2 + · · ·+ an−1b+ an =bn+1 − an+1

b− a.

3. Se senα 6= 0, mostre que, para todo n ∈ N, vale a igualdade:

cosα · cos 2α · cos 22α · · · cos 2nα =sen 2n+1α

2n+1 senα.

Sugestão: Use a fórmula sen 2β = 2 sen β cos β.

4. Para todo n ∈ N, mostre que, nos inteiros,

(a) 80 divide 34n − 1;

(b) 9 divide 4n + 6n− 1;

(c) 8 divide 32n + 7;

(d) 9 divide n4n+1 − (n+ 1)4n + 1.

5. Mostre que

(a) n! > 2n, se n ≥ 4;

(b) n! > 3n,se n ≥ 7;

(c) n! > 4n, se n ≥ 9.

6. Prove que, para todo n natural, vale a desigualdade:

1

2· 34· 56· · · 2n− 1

2n≤ 1√

3n+ 1.

12

Unidade 3O Princípio de Indução Matemática

7. Mostre que o número de diagonais de um polígono convexo de n lados é

dado por

dn =n(n− 3)

2.

3.4 Exercícios Suplementares

1. Mostre que n0 = 32 é o menor valor para o qual a equação 5x+ 9y = n

possui solução em (N ∪ 0)2 para todo n ≥ n0.

2. Prove que, para qualquer número natural n:

a) n3 + (n+ 1)3 + (n+ 2)3 é divisível por 9;

b) 32n+2 + 8n− 9 é divisível por 16;

c) 4n + 15n− 1 é divisível por 9;

d) 11n+2 + 122n+1 é divisível por 133;

e) 23n+ 1 é divisível por 3n+1.

3. Prove que:

a) 2n > n, onde n é um número natural arbitrário;

b)1 · 3 · 5 · · · (2n− 1)

2 · 4 · 6 · · · 2n≤ 1√

2n+ 1, para qualquer n ∈ N;

c)1

n+ 1+

1

n+ 2+ · · ·+ 1

2n>

13

24, se n ∈ N e n ≥ 2.;

d) 2n > 1 + n√2n−1, se n ∈ N e n ≥ 2.

4. Suponha que x+ 1xseja um número natural. Prove que xn+ 1

xn é também

um número natural, qualquer que seja o número natural n.

5. Mostre que o número 111 . . . 1 (formado por 3n algarismos iguais a 1) é

divisível por 3n.

Sugestão: Para o passo indutivo, divida o número escrito com 3n+1

algarismos iguais a 1 pelo número formado por 3n algarismos iguais a 1

e verique que o resultado é um número divisível por 3.

13

Unidade 3 Textos Complementares

3.5 Textos Complementares

Para Saber Mais Comentário

Note que, na argumentação acima, poderia parecer que estamos usando

o fato de P (n) ser verdadeira para deduzir que P (n + 1) é verdadeira para

em seguida concluir que P (n) é verdadeira. O que está ocorrendo? Estamos

usando a tese para provar o resultado?

A resposta é não! Preste bem atenção, pois essa é a parte mais delicada de

toda a trama.

Dado um número natural n, temos duas possibilidades:

(a) P (n) é verdadeira, ou (b) P (n) é falsa.

A hipótese (ii) do Princípio não exige em absoluto que assumamos P (n)

verdadeira para todo n ∈ N, podendo eventualmente ser falsa para algum valor

de n, ou mesmo para todos os valores de n. O que a hipótese (ii) exige é

que sempre que algum n pertença à categoria (a), acima, então n+1 também

pertença a essa mesma categoria; não exigindo nada quando n pertencer à

categoria (b).

Por exemplo, a sentença P (n) : n = n+1 satisfaz (por vacuidade) a hipótese

(ii) do Princípio, já que nenhum n ∈ N pertence à categoria (a). O que falha

para que o Princípio de Indução nos garanta que P (n) é verdadeira para todo

n é que a hipótese (i) não é vericada, pois P (1) : 1 = 2 é falsa!

14

Unidade 3O Princípio de Indução Matemática

Para Saber MaisIndução Empírica vs Indução Matemática

É preciso ter clareza que a Indução Matemática é diferente da indução

empírica das ciências naturais, em que é comum, após um certo número de

experimentos, necessariamente nito, enunciar leis gerais que governam o fenô-

meno em estudo. Essas leis são tidas como verdades, até prova em contrário.

Na matemática, não há lugar para armações verdadeiras até prova em con-

trário. A Prova por Indução Matemática trata de estabelecer que determinada

sentença sobre os naturais é sempre verdadeira.

A indução empírica foi batizada, de modo irônico, pelo matemático, lósofo

e grande humanista inglês do século passado, Bertrand Russel (1872-1970), de

indução galinácea, com base no seguinte conto:

Havia uma galinha nova no quintal de uma velha senhora. Diariamente,

ao entardecer, a boa senhora levava milho às galinhas. No primeiro dia, a

galinha, desconada, esperou que a senhora se retirasse para se alimentar. No

segundo dia, a galinha, prudentemente, foi se alimentando enquanto a senhora

se retirava. No nonagésimo dia, a galinha, cheia de intimidade, já não fazia

caso da velha senhora. No centésimo dia, ao se aproximar a senhora, a galinha,

por indução, foi ao encontro dela para reclamar o seu milho. Qual não foi a

sua surpresa quando a senhora pegou-a pelo pescoço com a intenção de pô-la

na panela.

15

Unidade 3 Textos Complementares

Na Sala de Aula Considerações sobre o Rigor

Neste curso, o nosso objetivo é mostrar como se pode estabelecer um maior

padrão de rigor no tratamento de problemas matemáticos que ocorrem no En-

sino Médio, mas isso não deve ser tomado ao pé da letra na sua prática docente.

Certos argumentos informais, quando acompanhados de um raciocínio correto,

são corriqueiramente aceitos. Por exemplo, o argumento utilizado por Gauss

para somar os n primeiros números naturais é perfeitamente aceitável. Por-

tanto, um conselho: use o formalismo para ajudar e não para atrapalhar e

nunca o deixe se sobrepor à criatividade, pois, via de regra, primeiro vem a

descoberta para depois vir a formalização. Procure estimular sempre os seus

alunos a serem criativos. Num primeiro momento, deixe as ideias uirem, só

depois preocupe-se com a sua organização e formalização.

16

4

1

Aplicações do Princípio deIndução Matemática

Sumário

4.1 Exercícios Recomendados . . . . . . . . . . . . . . . 9

4.2 Exercícios Suplementares . . . . . . . . . . . . . . . 9

4.3 Textos Complementares . . . . . . . . . . . . . . . . 11

Unidade 4

Apresentaremos nesta unidade algumas aplicações lúdicas do Princípio de

Indução Matemática ao mundo material.

Exemplo 1 [A Torre de Hanói]

Você provavelmente já conhece esse jogo bastante popular e que pode ser

facilmente fabricado ou ainda encontrado em lojas de brinquedos de madeira.

O jogo é formado por n discos de diâmetros distintos com um furo no

seu centro e uma base onde estão ncadas três hastes. Numa das hastes,

estão enados os discos, de modo que nenhum disco esteja sobre um outro de

diâmetro menor (veja gura abaixo).

Figura 1

O jogo consiste em transferir a pilha de discos para uma outra haste, des-

locando um disco de cada vez, de modo que, a cada passo, a regra acima seja

observada.

As perguntas naturais que surgem são as seguintes:

1. O jogo tem solução para cada n ∈ N?

2. Em caso armativo, qual é o número mínimo jn de movimentos para

resolver o problema com n discos?

Usando Indução Matemática, vamos ver que a resposta à primeira pergunta

é armativa, qualquer que seja o valor de n. Em seguida, deduziremos uma

fórmula que nos fornecerá o número jn.

Considere a sentença

P (n) : O jogo com n discos tem solução.

2

Unidade 4Aplicações do Princípio de Indução Matemática

Obviamente, P (1) é verdade. Suponha que P (n) seja verdadeiro, para

algum n; ou seja, que o jogo com n discos tem solução. Vamos provar que o

jogo com n+ 1 discos tem solução.

Para ver isso, resolva inicialmente o problema para os n discos superiores da

pilha, transferindo-os para uma das hastes livre (isso é possível, pois estamos

admitindo que o problema com n discos possua solução):

Figura 2

Em seguida, transra o disco que restou na pilha original (o maior dos

discos) para a haste vazia:

Figura 3

Feito isto, resolva novamente o problema para os n discos que estão juntos,

transferindo-os para a haste que contém o maior dos discos:

Figura 4

Isso mostra que o problema com n + 1 discos também possui solução, e,

portanto, por Indução Matemática, que P (n) é verdadeira para todo n ∈ N.

3

Unidade 4

Para determinar uma fórmula para jn, veja que, para resolver o problema

para n + 1 discos com o menor número de passos, temos, necessariamente,

que passar duas vezes pela solução mínima do problema com n discos. Temos,

então, que

jn+1 = 2jn + 1.

Obtemos, assim, uma sequência (jn) denida recorrentemente. Pode-se

mostrar, sem diculdade, por indução, que seu termo geral é dado por

jn = 2n − 1.

(Este tipo de sequências, as recorrências, será estudado de modo sistemático

nas Unidades U7 e U8.)

+ Para Saber Mais - Origem da Torre de Hanói - Clique para ler

Exemplo 2 [Os Coelhos de Fibonacci]

Trata-se do seguinte problema proposto e resolvido pelo matemático italiano

Leonardo de Pisa em seu livro Liber Abacci, de 1202:

Quot paria coniculorum in uno anno ex uno pario germinentur.

Como não se ensina mais latim nas escolas, aí vai uma tradução:

Quantos casais de coelhos descendem de um casal em um ano.

Leonardo passa a explicar o seu problema e a sua solução, como segue (com

adaptação nossa):

Um casal de coelhos recém-nascidos foi posto num lugar cercado. Deter-

minar quantos casais de coelhos ter-se-ão após um ano, supondo que, a cada

mês, um casal de coelhos produz outro casal e que um casal começa a procriar

dois meses após o seu nascimento.

Vamos organizar a nossa contagem na tabela a seguir.

4

Unidade 4Aplicações do Princípio de Indução Matemática

mêsnúmero de casais

do mês anterior

número de casais

recém-nascidostotal

1o 0 1 1

2o 1 0 1

3o 1 1 2

4o 2 1 3

5o 3 2 5

6o 5 3 8

7o 8 5 13

8o 13 8 21

9o 21 13 34

10o 34 21 55

11o 55 34 89

12o 89 55 144

Portanto, o número de casais de coelhos em um determinado mês é igual ao

número total de casais do mês anterior acrescido do número de casais nascidos

no mês em curso, que é igual ao número total de casais do mês anterior ao

anterior.

Se denotarmos o número de coelhos existentes no n-ésimo mês por un,

temos, então, que

un = un−1 + un−2, u1 = u2 = 1.

Essas relações denem, por recorrência, uma sequência de números naturais,

chamada de sequência de Fibonacci, cujos elementos, chamados de números de

Fibonacci, possuem propriedades aritméticas notáveis, que ainda hoje são objeto

de investigação.

+ Para Saber Mais - O que é uma Recorrência? - Clique para ler

+ Para Saber Mais - Leonardo de Pisa - Fibonacci - Clique para ler

5

Unidade 4

Exemplo 3 [O Enigma do Cavalo de Alexandre]

Num mosaico romano, Bucéfalo, o cavalo de Alexandre, o Grande, é repre-

sentado como um fogoso corcel cor de bronze. Nesse exemplo, vamos provar

que isso é uma falácia (uma grande mentira).

Inicialmente, provaremos que todos os cavalos têm mesma cor. De fato,

considere a sentença:

P (n) : Num conjunto com n cavalos, todos têm a mesma cor.

Note que P (1) é obviamente verdadeira. Agora, suponha o resultado válido

para conjuntos contendo n cavalos. Considere um conjunto

C = C1, C2, . . . , Cn, Cn+1

com n+ 1 cavalos. Decompomos o conjunto C numa união de dois conjuntos:

C = C ′ ∪ C ′′ = C1, . . . , Cn ∪ C2, . . . , Cn+1,

cada um dos quais contém n cavalos.

Pela hipótese indutiva, segue-se que os cavalos em C ′ têm mesma cor, ocor-

rendo o mesmo para os cavalos em C ′′. Como

C2 ∈ C ′ ∩ C ′′,

segue-se que os cavalos de C ′ têm a mesma cor dos cavalos de C ′′, permitindo

assim concluir que todos os cavalos em C têm a mesma cor.

Assim, a nossa demonstração por indução está terminada, provando que

P (n) é verdadeira para todo n ∈ N.

Agora, todo mundo sabe (você sabia?) que Marengo, o famoso cavalo de

Napoleão, era branco. Logo, Bucéfalo deveria ser branco.

Onde está o erro nessa prova?

Sugestão: Para achá-lo, sugerimos que você tente provar que, se P (1) é

verdadeira, então P (2) é verdadeira.

Esse problema foi inventado pelo matemático húngaro George Polya (1887-

1985).

6

Unidade 4Aplicações do Princípio de Indução Matemática

Exemplo 4[Descobrindo a Moeda Falsa]

Têm-se 3n moedas de ouro, sendo uma delas falsa, com peso menor do

que as demais. Dispõe-se de uma balança de dois pratos, sem nenhum peso.

Vamos mostrar, por indução sobre n, que é possível achar a moeda falsa com

n pesagens.

Para n = 1, isso é fácil de ver, pois, dadas as três moedas, basta pôr uma

moeda em cada prato da balança e descobre-se imediatamente qual é a moeda

falsa.

Suponha, agora, que o resultado seja válido para algum valor de n e que se

tenha que achar a moeda falsa dentre 3n+1 moedas dadas. Separemos as 3n+1

moedas em 3 grupos de 3n moedas cada. Coloca-se um grupo de 3n moedas

em cada prato da balança. Assim, poderemos descobrir em que grupo de 3n

moedas encontra-se a moeda falsa. Agora, pela hipótese de indução, descobre-

se a moeda falsa com n pesagens, que, junto com a pesagem já efetuada,

perfazem o total de n+ 1 pesagens.

Exemplo 5[A Pizza de Steiner]

O grande geômetra alemão Jacob Steiner (1796-1863) propôs e resolveu,

em 1826, o seguinte problema:

Qual é o maior número de partes em que se pode dividir o plano com n

cortes retos?

Pensando o plano como se fosse uma grande pizza, temos uma explicação

para o nome do problema.

Denotando o número máximo de pedaços com n cortes por pn, vamos provar

por indução a fórmula:

pn =n(n+ 1)

2+ 1.

Para n = 1, ou seja, com apenas um corte, é claro que só podemos obter

dois pedaços. Portanto, a fórmula está correta, pois

p1 =1(1 + 1)

2+ 1 = 2.

Admitamos agora que, para algum valor de n, a fórmula para pn esteja

correta. Vamos mostrar que a fórmula para pn+1 também está correta.

7

Unidade 4

Suponhamos que, com n cortes, obtivemos o número máximo n(n+1)/2+1

de pedaços e queremos fazer mais um corte, de modo a obter o maior número

possível de pedaços.

Vamos conseguir isso se o (n + 1)-ésimo corte encontrar cada um dos n

cortes anteriores em pontos que não são de interseção de dois cortes (faça um

desenho para se convencer disso).

Por outro lado, se o (n+1)-ésimo corte encontra todos os n cortes anteriores,

ele produz n + 1 novos pedaços: o corte começa em um determinado pedaço

e, ao encontrar o primeiro corte, ele separa em dois o pedaço em que está,

entrando em outro pedaço. Ao encontar o segundo corte, ele separa em dois o

pedaço em que está, entrando em outro pedaço, e assim sucessivamente, até

encontrar o n-ésimo corte separando o último pedaço em que entrar em dois.

Assim, são obtidos n+ 1 pedaços a mais dos que já existiam; logo,

pn+1 = pn + n+ 1 =n(n+ 1)

2+ 1 + n+ 1 =

(n+ 1)(n+ 2)

2+ 1,

mostrando que a fórmula está correta para n + 1 cortes. O resultado segue

então do Princípio de Indução Matemática.

8

Unidade 4Aplicações do Princípio de Indução Matemática

4.1 Exercícios Recomendados

1. Prove que, qualquer que seja o número natural n maior do que 3, existe

um polígono convexo com n lados e exatamente 3 ângulos agudos.

2. Um plano está dividido em regiões por várias retas. Prove que é pos-

sível colorir essas regiões com duas cores de modo que quaiquer duas

regiões adjacentes tenham cores diferentes (dizemos que duas regiões são

adjacentes se elas tiverem pelo menos um segmento de reta em comum).

3. A sequência (an) é denida pelos dados: a1 = 1, a2 = 2, an+1 = an−an−1se n > 2. Prove que an+6 = an para todos os números naturais n.

Descreva todos os termos dessa sequência.

4. A sequência a1, a2, . . . , an, . . . de números é tal que a1 = 3, a2 = 5 e

an+1 = 3an − 2an−1 para n > 2. Prove que an = 2n + 1 para todos os

números naturais n.

4.2 Exercícios Suplementares

1. Ache o erro na prova do seguinte

Teorema Todos os números naturais são iguais.

Vamos provar o resultado mostrando que, para todo n ∈ N, é verdadeira

a sentença

P (n) : : dado n ∈ N, todos os número naturais menores ou iguais do

que n são iguais.

(i) P (1) é claramente verdadeira.

(ii) Suponha que P (n) seja verdadeira, logo n − 1 = n. Somando 1 a

ambos os lados dessa igualdade, obtemos n = n + 1. Como n era igual

a todos os naturais anteriores, segue que P (n+ 1) é verdadeira.

Portanto, P (n) 'e vedadeira para todo n ∈ N .

2. (O queijo de Steiner) Para fazer a sua pizza, Steiner teve que cortar,

primeiro, o queijo. Imaginando que o espaço é um enorme queijo, você

9

Unidade 4 Exercícios Suplementares

seria capaz de achar uma fórmula para o número máximo de pedaços que

poderíamos obter ao cortá-lo por n planos?

3. Mostre que a sequência de Fibonacci satisfaz às seguintes identidades:

(a) u1 + u2 + · · ·+ un = un+2 − 1.

(b) u1 + u3 + · · ·+ u2n−1 = u2n.

(c) u2 + u4 + · · ·+ u2n = u2n+1 − 1.

(d) u21 + u2

2 + · · ·+ u2n = unun+1.

4. Sabendo que q =1 +√5

2é raiz da equação x2 = x + 1, mostre que

qn = unq + un−1.

5. Prove que

u3 + u6 + u9 + · · ·+ u3n =u3n+2 − 1

2.

6. Dada a recorrência an+2 = 2an+1 + an, com a1 = 1 e a2 = 3, ache uma

fórmula para an.

10

Unidade 4Aplicações do Princípio de Indução Matemática

4.3 Textos Complementares

Para Saber MaisOrigem da Torre de Hanói

Esse jogo foi idealizado e publicado pelo matemático francês Edouard Lucas,

em 1882, que, para dar mais sabor à sua criação, inventou a seguinte lenda:

Na origem do tempo, num templo oriental, uma Divindade colocou 64 discos

perfurados de ouro puro ao redor de uma de três colunas de diamante e ordenou

a um grupo de sacerdotes que movessem os discos de uma coluna para outra,

respeitando as regras acima explicadas. A Divindade sentenciou que, quando

todos os 64 discos fossem transferidos para uma outra coluna, o mundo acabaria.

Você não deve se preocupar com a iminência do m do mundo, pois, se,

a cada segundo, um sacerdote movesse um disco, o tempo mínimo para que

ocorresse a fatalidade seria de 264− 1 segundos e isto daria, aproximadamente,

um bilhão de séculos!

11

Unidade 4 Textos Complementares

Para Saber Mais O que é uma Recorrência?

Uma recorrência é uma fórmula que dene um elemento de uma sequência

a partir de termos anteriores.

Uma recorrência do tipo:

xn = xn−1 + xn−2, (4.1)

só permite determinar o elemento xn se conhecermos os elementos anteriores

xn−1 e xn−2, que, para serem calculados, necessitam do conhecimento dos dois

elementos anteriores, e assim por diante. Fica, portanto, univocamente denida

a sequência quando são dados x1 e x2. A sequência de Fibonacci corresponde

à recorrência (4.1), onde x1 = x2 = 1.

Quando é dada uma recorrência, um problema importante é determinar uma

fórmula fechada para o termo geral da sequência, isto é, uma fórmula que não

recorre aos termos anteriores. No caso da sequência de Fibonacci, existe uma

tal fórmula, chamada fórmula de Binet, que apresentamos a seguir e que será

demonstrada em um contexto mais geral na Unidade 8.

Para todo n ∈ N, tem-se que

un =

(1+√5

2

)n

−(

1−√5

2

)n

√5

É notável que seja necessário recorrer a fórmulas envolvendo números ir-

racionais para representar os elementos da sequência de Fibonacci, que são

números naturais. Mais notável, ainda, é que o número ϕ =1 +√5

2seja a

proporção áurea que aparece nas artes, e que1−√5

2= −ϕ−1 seja o simétrico

de seu inverso. Intrigante essa inesperada relação entre criar coelhos e a divina

proporção, não?

12

Unidade 4Aplicações do Princípio de Indução Matemática

Para Saber MaisLeonardo de Pisa - Fibonacci

Leonardo de Pisa (1170-1250), lho de Bonacci, e por isso apelidado Fi-

bonacci, teve um papel fundamental no desenvolvimento da Matemática no

Ocidente. Em 1202, publicou o livro Liber Abacci, que continha grande parte

do conhecimento sobre números e álgebra da época. Esta obra foi responsável

pela introdução na Europa do sistema de numeração indo-arábico e pelo pos-

terior desenvolvimento da álgebra e da aritmética no mundo ocidental.

13

Unidade 4 Textos Complementares

14

5

1

Progressões Aritméticas

Sumário

5.1 Introdução . . . . . . . . . . . . . . . . . . . . . . . 2

5.2 Primeiros Exemplos . . . . . . . . . . . . . . . . . . 2

5.3 Soma dos Termos de uma PA . . . . . . . . . . . . 6

5.4 Somas Polinomiais . . . . . . . . . . . . . . . . . . . 9

5.5 Exercícios Recomendados . . . . . . . . . . . . . . . 14

5.6 Exercícios Suplementares . . . . . . . . . . . . . . . 16

5.7 Textos Complementares . . . . . . . . . . . . . . . . 20

Unidade 5 Introdução

5.1 Introdução

As Progressões Aritméticas (PA) constituem-se na família mais simples de

sequências denidas recorrentemente. Elas são comuns na vida real e sempre

aparecem quando se apresentam grandezas que sofrem variações iguais em in-

tervalos de tempos iguais como, por exemplo, no cálculo de juros simples, ou

desvalorização de um bem ao longo do tempo.

Nessa unidade, você encontrará também a fórmula que fornece a soma dos

n primeiros termos de uma PA, fórmula que generaliza a que foi descoberta por

Gauss, quando menino, conforme vimos na Unidade 3.

Em seguida, são denidas generalizações do conceito de PA, introduzindo

as PAs de segunda ordem, terceira ordem, etc. Esse tópico, em geral, não

é explorado no Ensino Médio, mas coloca à disposição do professor métodos

poderosos para calcular somas.

5.2 Primeiros Exemplos

São comuns na vida real, grandezas que sofrem variações iguais em inter-

valos de tempos iguais. Vejamos algumas situações concretas.

Exemplo 1 Uma fábrica de automóveis produziu 400 veículos em janeiro e aumen-

tou mensalmente sua produção de 30 veículos. Quantos veículos produziu em

junho?

Solução Os valores da produção mensal, a partir de janeiro, são 400, 430, 490,

520, 550, . . . . Em junho, a fábrica produziu 550 veículos.

Poderíamos ter evitado escrever a produção mês a mês, racionando do modo

a seguir. Se a produção aumenta de 30 veículos por mês, em 5 meses ela

aumenta 5×30 = 150 veículos. Em junho, a fábrica produziu 400 + 150 = 550

veículos.

Progressões aritméticas são sequências nas quais o aumento de cada termo

para o seguinte é sempre o mesmo.

A sequência (400, 430, 460, 490, 520, 550, . . .) é um exemplo de uma pro-

gressão aritmética.

2

Unidade 5Progressões Aritméticas

O aumento constante de cada termo para o seguinte é chamado de razão

de progressão. A razão da progressão acima é igual a 30.

Vamos à denição formal.

Definição 1Uma progressão aritmética é uma sequência na qual a diferença entre cada

termo e o termo anterior é constante. Essa diferença constante é chamada de

razão da progressão e representada pela letra r.

Exemplo 2As sequências (5, 8, 11, 14, . . .) e (7, 5, 3, 1, . . .) são progressões aritméticas

cujas razões valem respectivamente 3 e −2.

Em uma progressão aritmética (a1, a2, a3, . . .), para avançar um termo,

basta somar a razão; para avançar dois termos, basta somar duas vezes a razão,

e assim por diante. Assim, por exemplo, a13 = a5 + 8r, pois, ao passar de a5

para a13, avançamos 8 termos; a12 = a7 + 5r, pois avançamos 5 termos ao

passar de a7 para a12; a4 = a17 − 13r, pois retrocedemos 13 termos ao passar

de a17 para a4 e, de modo geral,

an = a1 + (n− 1)r,

pois, ao passar de a1 para an, avançamos n− 1 termos.

Exemplo 3Em uma progressão aritmética, o quinto termo vale 30 e o vigésimo termo

vale 50. Quanto vale o oitavo termo dessa progressão?

Solução a20 = a5 + 15r, pois ao passar do quinto termo para o vigésimo,

avançamos 15 termos. Logo, 50 = 30 + 15r e r =4

3. Analogamente, a8 =

a5 + 3r = 30 + 3.4

3= 34. O oitavo termo vale 34.

Exemplo 4Qual é a razão da progressão aritmética que se obtém inserindo 10 termos

entre os números 3 e 25?

Solução. Temos a1 = 3 e a12 = 25. Como a12 = a1+11r, temos 25 = 3+11r.

Daí, r = 2.

3

Unidade 5 Primeiros Exemplos

Exemplo 5 O cometa Halley visita a Terra a cada 76 anos. Sua última passagem por

aqui foi em 1986. Quantas vezes ele visitou a Terra desde o nascimento de

Cristo? Em que ano foi sua primeira passagem na era cristã?

Solução Os anos de passagem do cometa foram 1986, 1910, 1834,... e formam

uma progressão aritmética de razão −76. O termo de ordem n dessa progressão

é an = a1 + (n − 1)r, isto é, an = 1986 − 76(n − 1) = 2062 − 76n. Temos

an > 0 quando n <2062

76= 27, 13 . . . . Portanto, os termos positivos dessa

progressão são os 27 primeiros, a1, a2, a3, . . . , a27. Logo, ele nos visitou 27

vezes na era cristã e sua primeira passagem na era cristão foi no ano a27 =

2062− 76× 27 = 10.

Poderíamos também ter resolvido o problema aproveitando o fato dos termos

dessa progressão serem inteiros.

Em uma progressão aritmética de termos inteiros e razão não-nula, todos os

termos dão o mesmo resto quando divididos pelo módulo da razão. Como 1986

dividido por 76 dá resto 10, todos os anos em que o cometa por aqui passou

dão resto 10 quando divididos por 76. A primeira visita ocorreu entre os anos

1 e 76, inclusive. Entre esses anos, o único que dividido por 76 dá resto 10 é o

ano 10. Para descobrir a ordem desse termo, usamos an = a1 + (n− 1)r, isto

é, 10 = 1986− 76(n− 1). Daí,

n =2062

76= 27.

Muitas vezes é conveniente enumerar os termos de uma progressão aritmé-

tica a partir de zero, conforme mostra o exemplo a seguir.

Exemplo 6O preço de um carro novo é de R$ 15 000,00 e diminui de R$1 000,00 a

cada ano de uso. Qual será o preço com 4 anos de uso?

Solução Chamando o preço com n anos de uso de an, temos a0 = 15000 e

queremos calcular a4. Como a desvalorização anual é constante, (an) é uma

progressão aritmética. Logo, a4 = a0 + 4r = 15000 + 4 × (−1000) = 11000.

O preço será de R$11 000,00.

4

Unidade 5Progressões Aritméticas

Exemplo 7Os lados de um triângulo retângulo formam uma progressão aritmética

crescente. Mostre que a razão dessa progressão é igual ao raio do círculo

inscrito.

Solução. Chamemos os lados do triângulo de x− r, x, x + r. Esse é um bom

truque para facilitar as contas; ao representar uma progressão aritmética com

um número ímpar de termos, começar pelo termo central.

Como a progressão é crescente, a hipotenusa é o último termo. Pelo Teo-

rema de Pitágoras, (x + r)2 = (x − r)2 + x2. Daí, x2 = 4rx e, já que x 6= 0

pois x é um dos catetos, x = 4r. Os lados são então 3r, 4r e 5r. O perímetro

é 2p = 3r + 4r + 5r = 12r e a área éS

p=

6r2

6r= r.

Exemplo 8Determine 4 números em progressão aritmética crescente, conhecendo sua

soma 8 e a soma de seus quadrados 36.

Solução Um bom truque, para representar progressões aritméticas com um

número par de termos, é chamar os dois termos centrais de x− y e x+ y. Isso

faz com que a razão seja (x + y)− (x− y) = 2y.

A progressão é então x− 3y, x− y, x + y, x + 3y.

Temos (x− 3y) + (x− y) + (x + y) + (x + 3y) = 8

(x− 3y)2 + (x− y)2 + (x + y)2 + (x + 3y)2 = 364x = 8

4x2 + 20y2 = 36x = 2

y = ±1

Como a progressão é crescente, y > 0. Logo, x = 2 e y = 1. Os números são

−1, 1, 3, 5.

Em uma progressão aritmética, o termo geral é dado por um polinômio em

n, an = a1 + (n − 1)r = r . n + (a1 − r). Se r 6= 0, ou seja, se a progressão

não for estacionária (constante), esse polinômio é de grau 1. Se r = 0, isto é,

se a progressão for estacionária, esse polinômio é de grau menor que 1.

5

Unidade 5 Soma dos Termos de uma PA

Por esse motivo, as progressões aritméticas de razão r 6= 0 são chamadas

de progressões aritméticas de primeira ordem.

Reciprocamente, se em uma sequência o termo de ordem n for dado por

um polinômio em n, de grau menor que ou igual a 1, ela será uma progressão

aritmética. Com efeito, se xn = an + b, (xn) é uma progressão aritmética na

qual a = r e b = a1 − r, ou seja, r = a e a1 = a + b.

Como em uma progressão aritmética an = a0 + nr, a função que associa a

cada natural n o valor de an é simplesmente a restrição aos naturais da função

am a(x) = a(0) + rx.

Portanto, pensando em uma progressão aritmética como uma função que

associa a cada número natural n o valor an, o gráco dessa função é formado

por uma sequência de pontos colineares no plano.

Em outras palavras, (an) é uma progressão aritmética se e somente se os

pontos do plano que têm coordenadas (1, a1), (2, a2), (3, a3), etc. estão em

linha reta.

Figura 5.1: Gráco de uma PA

5.3 Soma dos Termos de uma PA

Baseados na ideia de Gauss, usada para calcular a soma 1 + 2 + · · ·+ 100,

podemos calcular a soma dos n primeiros termos de uma progressão aritmética

qualquer.

6

Unidade 5Progressões Aritméticas

Teorema 2A soma dos n primeiros termos da progressão aritmética (a1, a2, a3, ...) é

Sn =(a1 + an)n

2.

DemonstraçãoTemos Sn = a1 + a2 + a3 + · · ·+ an−1 + an e, escrevendo a soma de trás

para frente, Sn = an + an−1 + an−2 + · · ·+ a2 + a1. Daí,

2Sn = (a1 + an) + (a2 + an−1) + (a3 + an−2) + · · ·+ (an−1 + a2) + (an + a1).

Observe que, ao passar de um parêntese para o seguinte, a primeira parcela

aumenta de r e a segunda parcela diminui de r, o que não altera a soma.

Portanto, todos os parênteses são iguais ao primeiro, (a1 + an). Como são n

parênteses, temos

2Sn = (a1 + an) . n e Sn =(a1 + an)n

2.

Exemplo 9Qual é o valor da soma dos 20 primeiros termos da progressão aritmética

2, 6, 10, . . . ?

Solução a20 = a1 + 19r = 2 + 19× 4 = 78.

S20 =(2 + 78)20

2= 800.

Exemplo 10A soma dos n primeiros números inteiros e positivos é

n∑k=1

k = 1 + 2 + 3 + · · ·+ n =n(n + 1)

2.

Observe que Sn, no exemplo anterior, é um polinômio do segundo grau em

n, sem termo independente.

7

Unidade 5 Soma dos Termos de uma PA

Exemplo 11 A soma dos n primeiros números ímpares é

1 + 3 + 5 + · · ·+ (2n− 1) =(1 + 2n− 1)n

2= n2.

Observe que Sn, no exemplo anterior, é também um polinômio do segundo

grau em n, sem termo independente. Isto se generaliza como segue.

A soma dos n primeiros termos de uma progressão aritmética é

Sn =(a1 + an)n

2=

[a1 + a1 + (n− 1)r]n

2=

r

2n2 +

(a1 −

r

2

)n.

Observe que, se r 6= 0, então Sn é um polinômio do segundo grau em n,

desprovido de termo independente. Se r = 0, Sn é um polinômio de grau

menor que 2, sem termo independente.

Reciprocamente, todo polinômio do segundo grau em n, desprovido de termo

independente, é o valor da soma dos n primeiros termos de alguma progressão

aritmética. Com efeito P (n) = an2 + bn é a soma dos n primeiros termos da

progressão aritmética na qualr

2= a e a1−

r

2= b, ou seja, r = 2a e a1 = a+b.

Definição 3 Dene-se para sequências o operador ∆, chamado de operador diferença,

por ∆an = an+1 − an.

Portanto, da denição segue imediatamente que uma sequência (an) é uma

progressão aritmética se e somente se (∆an) = (an+1 − an) é constante.

Definição 4 Uma progressão aritmética de segunda ordem é uma sequência (an) na qual

as diferenças ∆an = an+1 − an, entre cada termo e o termo anterior, formam

uma progressão aritmética não-estacionária.

Exemplo 12 A sequência (an) = (1, 3, 6, 10, 15, 21, . . .) é uma progressão aritmética de

segunda ordem porque a sequência das diferenças entre cada termo e o anterior,

(bn) = (∆an) = (an+1 − an) = (2, 3, 4, 5, 6, . . .)

é uma progressão aritmética não-estacionária.

8

Unidade 5Progressões Aritméticas

+ Para Saber Mais - PAs de Ordem Superior - Clique para ler

5.4 Somas Polinomiais

A pergunta que nos colocamos é como calcular somas do tipo∑n

k=1 P (k),

onde P (k) é um polinômio em k.

Se o polinômio é P (k) = a0 + a1k + a2k2 + · · ·+ amk

m, temos que∑nk=1 P (k) =

∑nk=1 a0 +

∑nk=1 a1k +

∑nk=1 a2k

2 + · · ·+∑n

k=1 amkm

= a0∑n

k=1 1 + a1∑n

k=1 k + a2∑n

k=1 k2 + · · ·+ am

∑nk=1 k

m,

que pode ser calculado desde que saibamos calcular, para p ∈ N, somas do tipo:

n∑k=1

kp = 1p + 2p + · · ·+ np.

Exemplo 13A soma dos quadrados dos n primeiros números inteiros e positivos é

12 + 22 + · · ·+ n2 =n∑

k=1

k2

e pode ser calculada do modo a seguir:

n∑k=1

(k + 1)3 =n∑

k=1

k3 + 3n∑

k=1

k2 + 3n∑

k=1

k +n∑

k=1

1.

Os dois primeiros somatórios têm várias parcelas comuns, pois

n∑k=1

(k + 1)3 = 23 + 33 + · · ·+ n3 + (n + 1)3

en∑

k=1

k3 = 13 + 23 + 33 + · · ·+ n3.

Simplicando as parcelas comuns aos dois membros, obtemos

(n + 1)3 = 13 + 3n∑

k=1

k2 + 3n∑

k=1

k +n∑

k=1

1.

9

Unidade 5 Somas Polinomiais

Comon∑

k=1

k = 1 + 2 + · · ·+ n =n(n + 1)

2

en∑

k=1

1 = 1 + 1 + · · ·+ 1 = n,

temos

(n + 1)3 = 13 + 3n∑

k=1

k2 + 3n(n + 1)

2+ n.

Daí,n∑

k=1

k2 =2n3 + 3n2 + n

6=

n(n + 1)(2n + 1)

6.

Observe que 12 + 22 + · · · + n2 =n∑

k=1

k2 é um polinômio do terceiro grau

em n.

Exemplo 14 Sabendo que

12 + 22 + · · ·+ n2 =n∑

k=1

k2

é um polinômio do terceiro grau em n, poderíamos ter determinado o valor de

p(n) = 12 + 22 + 32 + · · ·+ n2

pondo p(n) = an3 + bn2 + cn + d. Assim, temos

p(1) = 12, p(2) = 12 + 22, p(3) = 12 + 22 + 32 e p(4) = 12 + 22 + 32 + 42.

Obtemos o sistema de equaçõesa + b + c + d = 1

8a + 4b + 2c + d = 5

27a + 9b + 3c + d = 14

64a + 16b + 4c + d = 30

Resolvendo, encontramos a =1

3, b =

1

2, c =

1

6, d = 0. Então

12 + 22 + 32 + · · ·+ n2 =1

3n3 +

1

2n2 +

1

6n =

n(n + 1)(2n + 1)

6.

10

Unidade 5Progressões Aritméticas

Os teoremas a seguir generalizam os últimos exemplos.

Teorema 51p + 2p + 3p + · · ·+ np =n∑

k=1

kp é um polinômio de grau p + 1 em n.

DemonstraçãoVamos proceder por indução sobre p. Para p = 1, o teorema já foi provado

anteriormente.

Suponhamos agora quen∑

k=1

kp seja um polinômio de grau p + 1 em n,

para todo p ∈ 1, 2, . . . , s, Mostraremos que essa armação é verdadeira para

p = s + 1, isto é, mostraremos quen∑

k=1

ks+1 é um polinômio de grau s + 2 em

n. Observe que

(k + 1)s+2 = ks+2 + (s + 2)ks+1 + · · · ,

onde os termos que não foram escritos explicitamente formam um polinômio de

grau s em k. Temos então,

n∑k=1

(k + 1)s+2 =n∑

k=1

ks+2 + (s + 2)n∑

k=1

ks+1 + F (n),

onde F (n) é um polinômio de grau s + 1 em n, pela hipótese da indução.

Simplicando os termos comuns aos dois primeiros somatórios, obtemos

(n + 1)s+2 = 1 + (s + 2)n∑

k=1

ks+1 + F (n).

Daí,n∑

k=1

ks+1 =(n + 1)s+2 − 1− F (n)

s + 2

que é um polinômio de grau s + 2 em n.

11

Unidade 5 Somas Polinomiais

Corolário 6 Se F é um polinômio de grau p entãon∑

k=1

F (k) é um polinômio de grau

p + 1 em n.

Exemplo 15

Vamos calcular Sn =n∑

k=1

k(k+2). Pelo corolário, sabemos que o valor dessa

soma é um polinõmio do terceiro grau em n. Então Sn = an3 + bn2 + cn + d.

Atribuindo a n os valores 1, 2, 3 e 4 obtemos as equaçõesa + b + c + d = 3

8a + 4b + 2c + d = 11

27a + 9b + 3c + d = 26

64a + 16b + 4c + d = 50

Resolvendo, encontramos a =1

3, b =

3

2, c =

7

6, d = 0. Então,

Sn =1

3n3 +

3

2n2 +

7

6n =

2n3 + 9n2 + 7n

6=

n(n + 1)(2n + 7)

6.

+ Para Saber Mais - PA com Termo Geral Polinomial - Clique para ler

O exemplo a seguir é conhecido como Teorema Fundamental da Somação

e fornece uma técnica bastante eciente para o cálculo de somas.

Exemplo 16

Mostre quen∑

k=1

∆ak = an+1 − a1.

Solução

n∑k=1

∆ak = ∆a1 + ∆a2 + ∆a3 + · · ·+ ∆an−1 + ∆an =

(a2 − a1) + (a3 − a2) + (a4 − a3) + · · ·+ (an − an−1) + (an+1 − an) =

an+1 − a1.

12

Unidade 5Progressões Aritméticas

Exemplo 17

Calculen∑

k=1

k(k + 1)

Solução Determinaremos ak tal que ∆ak = k(k + 1) = k2 + k. Como(∆ak) é

uma progressão aritmética de segunda ordem, (ak) é uma progressão aritmética

de terceira ordem. Logo, ak é um polinômio de terceiro grau. Se

ak = ak3 + bk2 + ck + d,

∆ak = ak+1 − ak

= a(k + 1)3 + b(k + 1)2 + c(k + 1) + d− [ak3 + bk2 + ck + d]

= 3ak2 + (3a + 2b)k + (a + b + c) = k2 + k.

Devemos ter 3a = 1, 3a + 2b = 1, a + b + c = 0. Daí, a =1

3, b = 0, c = −1

3

e d é arbitrário. Logo, ak =1

3k3 − 1

3k + d.

n∑k=1

k(k + 1) =n∑

k=1

∆ak = an+1 − a1

=(n + 1)3 − (n + 1)

3+ d− d =

n(n + 1)(n + 2)

3.

13

Unidade 5 Exercícios Recomendados

5.5 Exercícios Recomendados

1. Formam-se n triângulos com palitos, conforme a gura. Qual o número

de palitos usados para construir n triângulos?

Figura 5.2:

2. Calcule a soma de todos os inteiros que divididos por 11 dão resto 7 e

estão compreendidos entre 200 e 400.

3. Quanto vale o produto (a)(aq)(aq2)(aq3) . . . (aqn−1)?

4. Um quadrado mágico de ordem n é uma matriz n × n, cujos elementos

são os inteiros 1, 2, . . . , n2, sem repetir nenhum, tal que todas as linhas

e todas as colunas têm a mesma soma. O valor dessa soma é chamado

de constante mágica. Por exemplo, os quadrados

1 5 9

8 3 4

6 7 2

8 1 6

3 5 7

4 9 2

e

17 24 1 8 15

23 5 7 14 16

4 6 13 20 22

10 12 19 21 3

11 18 25 2 9

são mágicos, com constantes mágicas respectivamente iguais a 15, 15

e 65. Aliás, os dois últimos são hipermágicos, pois as linhas, colunas e

também as diagonais têm a mesma soma. Calcule a constante mágica de

um quadrado mágico de ordem n.

5. Suprimindo um dos elementos do conjunto 1, 2, . . . , n, a média aritmé-

tica dos elementos restantes é 16,1. Determine o valor de n e qual foi o

elemento suprimido.

14

Unidade 5Progressões Aritméticas

6. Um bem, cujo valor hoje é de R$ 8000,00, desvaloriza-se de tal forma

que seu valor daqui a 4 anos será de R$ 2000,00. Supondo que o valor

do bem cai segundo uma linha reta, determine o valor do bem daqui a 3

anos.

7. Prove que a soma de todos os inteiros positivos de n dígitos, n > 2, é

igual ao número 49499...95500...0, no qual há n− 3 dígitos sublinhados

que são iguais a 9 e n− 2 dígitos sublinhados que são iguais a 0.

8. Considere um jogo entre duas pessoas com as seguintes regras:

i) Na primeira jogada, o primeiro jogador escolhe um número no conjunto

A = 1, 2, 3, 4, 5, 6, 7 e diz esse número.

ii) As pessoas jogam alternadamente.

iii) Cada pessoa ao jogar escolhe um elemento de A, soma-o ao número

dito pela pessoa anterior e diz a soma.

iv) Ganha quem disser 63.

Qual dos jogadores tem uma estratégia vencedora e qual é essa estratégia?

9. Na primeira fase do campeonato brasileiro de futebol, que é disputado por

24 clubes, quaisquer dois times jogam entre si uma única vez. Quantos

jogos há?

10. Qual o número máximo de regiões em que n retas podem dividir o plano?

11. Há dois tipos de anos bissextos: os que são múltiplos de 4 mas não de

100 e os que são múltiplos de 400.

(a) Quantos são os anos bissextos entre 1997 e 2401?

(b) Se 1o de janeiro de 1997 foi quarta-feira, que dia será 1o de janeiro

de 2500?

(c) Escolhido um ano ao acaso, qual a probabilidade dele ser bissexto?

12. O número triangular Tn é denido como a soma dos n primeiros ter-

mos da progressão aritmética 1, 2, 3, 4, . . .. O número quadrangular Qn

é denido como a soma dos n primeiros termos da progressão aritmética

15

Unidade 5 Exercícios Suplementares

1, 3, 5, 7, . . .. Analogamente são denidos números pentagonais, hexago-

nais, etc. A gura abaixo justica essa denominação.

Determine o número j-gonal de ordem n.

Figura 5.3:

13. Mostre que ∆ak = ∆bk então ak − bk é constante.

14. Use o teorema fundamental da somação para calcular:

(a)n∑

k=1

3k.

(b)n∑

k=1

k . k!.

(c)n∑

k=1

1

k(k + 1).

5.6 Exercícios Suplementares

1. Os ângulos internos de um pentágono convexo estão em progressão arit-

mética. Determine o ângulo mediano.

2. Se 3 − x, −x,√

9− x, . . . é uma progressão aritmética, determine x e

calcule o quinto termo.

3. Calcule a soma dos termos da progressão aritmética 2, 5, 8, 11,... desde

o 25o até o 41o termo, inclusive.

16

Unidade 5Progressões Aritméticas

4. Quantos são os inteiros, compreendidos entre 100 e 500, que não são

divisíveis nem por 2, nem por 3 e nem por 5? Quanto vale a soma desses

inteiros?

5. Determine o maior valor que pode ter a razão de uma progressão aritmé-

tica que admita os números 32, 227 e 942 como termos da progressão.

6. De quantos modos o número 100 pode ser representado como uma soma

de dois ou mais inteiros consecutivos? E como soma de dois ou mais

naturais consecutivos?

7. Os inteiros de 1 a 1000 são escritos ordenadamente em torno de um

círculo. Partindo de 1, riscamos os números de 15 em 15, isto é, riscamos

1, 16, 31,... O processo continua até se atingir um número já previamente

riscado. Quantos números sobram sem riscos?

8. Podem os números√

2,√

3,√

5 pertencer a uma mesma progressão

aritmética?

9. Um bem, cujo valor hoje é de R$ 8000,00, desvaloriza-se de tal forma

que seu valor daqui a 4 anos será de R$ 2000,00. Supondo constante a

desvalorização anual, qual será o valor do bem daqui a 3 anos?

10. Calcule a soma de todas as frações irredutíveis, da formap

72, que perten-

çam ao intervalo [4,7].

11. Qual a maior potência de 7 que divide 1000!?

12. Calcule o valor das somas dos n primeiros termos das sequências?

(a) 13, 23, 33, . . .

(b) 1 . 4, 3 . 7, 5 . 10, 7 . 13, . . .

13. Representando por bxc a parte inteira do real x, isto é, o maior número

inteiro que é menor que ou igual a x e por x o inteiro mais próximo do

real x, determine:

(a) b√

1c+ b√

2c+ b√

3c+ · · ·+ b√n2 − 1c.

(b) b 3√

1c+ b 3√

2c+ b 3√

3c+ · · ·+ b 3√n3 − 1c.

17

Unidade 5 Exercícios Suplementares

(c)1

1+

1

2+

1

3+ · · ·+ 1

1000.

(d) √

1+ √

2+ √

3+ · · ·+ √

1000.

14. Determine o primeiro termo e a razão da progressão aritmética na qual a

soma dos n primeiros termos é, para todo n:

(a) Sn = 2n2 + n

(b) Sn = n2 + n + 1

15. Determine no quadro abaixo:

1

3 5

7 9 11

13 15 17 19

21 23 25 27 29

. . . . . . . . . . . . . . . . . .

(a) o primeiro elemento da 31a linha.

(b) a soma dos elementos da 31a linha.

16. Refaça o Exercício Recomendado 8 para o caso do vencedor ser quem

disser 64.

17. Refaça o exercício anterior para o conjunto 3, 4, 5, 6.

18. Mostre que no Exercício Recomendado 8, se o conjunto fosse A =

3, 5, 6, 7, o segundo jogador tem a estratégia que impede o primeiro

jogador de ganhar.

19. Uma bobina de papel tem raio interno 5cm, raio externo 10cm e a espes-

sura do papel é 0,01cm. Qual é o comprimento da bobina desenrolada?

20. Dividem-se os números naturais em blocos do modo seguinte:

(1), (2, 3) (4, 5, 6) (7, 8, 9, 10) (11.12.13.14) . . . . Em seguida suprimem-

se os blocos que contêm um número par de elementos, formando-se o

quadro:

18

Unidade 5Progressões Aritméticas

1

4 5 6

11 12 13 14 15

. . . . . . . . . . . . . . . . . . . . .

Determine:

(a) o primeiro elemento da linha k.

(b) o elemento central da linha k.

(c) a soma dos elementos da linha k.

(d) a soma dos elementos das k primeiras linhas.

21. Prove: se an é um polinômio de grau p então ∆an é um polinômio de

grau p− 1.

22. Prove o Corolário 6.

23. Quantos são os termos comuns às progressões aritméticas

(2, 5, 8, 11, . . . , 332) e (7, 12, 17, 22, . . . , 157)?

24. Benjamin começou a colecionar calendários em 1979. Hoje, sua coleção

já tem algumas duplicatas - por exemplo, o calendário de 1985 é igual ao

de 1991 - mas ainda não está completa.

(a) Em que ano Benjamim completará sua coleção?

(b) Quando a coleção estiver completa, quantos calendários diferentes

nela haverá?

25. A razão entre as somas dos n primeiros termos de duas progressões arit-

méticas é2n + 3

4n− 1, para todo valor de n. Quanto vale a razão entre seus

termos de ordem n?

19

Unidade 5 Textos Complementares

5.7 Textos Complementares

Para Saber Mais PAs de Ordem Superior

De modo geral, uma progressão aritmética de ordem k (k > 2) é uma

sequência na qual as diferenças entre cada termo e o termo anterior formam

uma progressão aritmética de ordem k − 1.

Exemplo A tabela abaixo mostra uma sequência (an) = (n3 − n) e suas

diferenças

(∆an), (∆2an) = (∆∆an), (∆3an) = (∆∆2an) etc...

n an ∆an ∆2an ∆3an

0 0 0 6 6

1 0 6 12 6

2 6 18 18 6

3 24 36 24 6

4 60 60 30 2

5 120 90 2

6 210 2

7 2

Se (∆an), como parece, for constante, (∆2an) será uma progressão aritmé-

tica, (∆an) será uma progressão aritmética de segunda ordem e (an) será uma

progressão aritmética de terceira ordem. Isso é verdade, pois

an = n3 − n

∆an = an+1 − an = (n + 1)3 − (n + 1)− [n3 − n] = 3n2 + 3n,

∆2an = 3(n + 1)2 + 3(n + 1)− [3n2 + 3n] = 6n + 6,

∆3an = 6(n + 1) + 6− [6n + 6] = 6

e ∆3an realmente é constante.

Observe que, nesse quadro, a soma de dois elementos lado a lado é igual

ao elemento que está embaixo do primeiro desses elementos. Isso nos permite

calcular os elementos que estão assinalados por 2 na tabela acima. Da direita

para a esquerda, eles são iguais a 6, 30 + 6 = 36, 90 + 36 = 126 e 210 + 126 =

336. Portanto, a7 = 336 e este foi o processo mais exótico que você já viu para

calcular a7 = 73 − 7.

20

Unidade 5Progressões Aritméticas

Proposição Toda sequência na qual o termo de ordem n é um polinômio

em n, do segundo grau, é uma progressão aritmética de segunda ordem e,

reciprocamente, se (an) é uma pregressão aritmética de segunda ordem então

(an) é um polinômio de segundo grau em n.

Demonstração Com efeito, se an = an2 + bn + c, com a 6= 0, temos

∆an = an+1−an = a(n+ 1)2 + b(n+ 1) + c− (an2 + bn+ c) = 2an+ (a+ b),

que é do primeiro grau em n. Pelo que comentamos acima, (∆an) é uma

progressão aritmética não-estacionária.

Por outro lado, se (an) é uma progressão aritmética de segunda ordem,

bn = ∆an = an+1 − an é uma progressão aritmética com razão diferente de

zero e

b1 + b2 + b3 + · · ·+ bn−2 + bn−1 =

(a2 − a1) + (a3 − a2) + (a4 − a3) + · · ·+ (an − an−1) + (an+1 − an) =

an+1 − a1

é um polinômio do segundo grau em n. Em consequência, an também é um

polinômio do segundo grau em n.

Observação O resultado anterior será generalizado mais adiante.

21

Unidade 5 Textos Complementares

Para Saber Mais PA com Termo Geral Polinomial

Com o corolário acima, podemos generalizar o teorema em [Para Saber

Mais: PAs de Ordem Superior], conforme foi prometido lá.

Teorema (an) é uma progressão aritmética de ordem p, (p > 2), se, e somente

se an é um polinômio de grau p em n.

Demonstração Vamos proceder por indução sobre p.

Para p = 2, o teorema foi provado em [Para Saber Mais: PAs de Ordem

Superior, Proposição].

Suponhamos que o teorema seja verdadeiro para todo p ∈ 2, 3, . . . , s.Mostraremos que essa armação é verdadeira para p = s + 1.

Se (an) é uma progressão aritmética de ordem s+1, bn = ∆an = an+1−an

é uma progressão aritmética de ordem s e, pela hipótese da indução, bn é um

polinômio de grau s em n. Entãon∑

k=1

bk = an+1 − a1 é, pelo corolário do

Teorema 5, um polinômio de grau s + 1 em n. Se an é um polinômio de grau

s + 1 em n, ∆an é um polinômio de grau s em n, conforme você facilmente

vericará. Pela hipótese da indução, (∆an) é uma progressão aritmética de

ordem s, ou seja, (an) é uma progressão aritmética de ordem s + 1.

22

6

1

Progressões Geométricas

Sumário

6.1 Introdução . . . . . . . . . . . . . . . . . . . . . . . 2

6.2 Primeiros Exemplos . . . . . . . . . . . . . . . . . . 3

6.3 A Fórmula das Taxas Equivalentes . . . . . . . . . . 8

6.4 A Soma dos Termos de uma Progressão Geométrica 9

6.5 Exercícios Recomendados . . . . . . . . . . . . . . . 11

6.6 Exercícios Suplementares . . . . . . . . . . . . . . . 12

6.7 Textos Complementares . . . . . . . . . . . . . . . . 18

Unidade 6 Introdução

6.1 Introdução

Ao utilizarmos progressões aritméticas para modelar problemas de juros simples,

obtemos uma situação como segue. Considere um capital de R$ 10.000, 00

aplicado a uma taxa de juros mensal de 2%:

Mês Valor Inicial Juros Valor Final

1 10.000 10.000× 2% = 200 10.200

2 10.200 10.000× 2% = 200 10.400

3 10.400 10.000× 2% = 200 10.600

4 10.600 10.000× 2% = 200 10.800

5 10.800 10.000× 2% = 200 11.000

6 11.000 10.000× 2% = 200 11.200

Você já viu alguém aplicar dinheiro dessa forma? Pense na sua caderneta de

poupança. O mais verossímil é que o juro incida sobre juros, pois já no segundo

mes o nosso capital não é mais R$ 10.000,00, mas R$ 10.200,00; logo é esse

capital que deve ser remunerado no segundo mês. Obtemos assim uma nova

tabela (com arredondamento na segunda casa decimal):

Mês Valor Inicial Juros Valor Final

1 10.000 10.000× 2% = 200 10.200

2 10.200 10.200× 2% = 204 10.404

3 10.404 10.404× 2% = 208, 08 10.612, 08

4 10.612, 08 10.612, 08× 2% = 212, 24 10.824, 32

5 10.824, 32 10.824, 32× 2% = 216, 49 11.040, 81

6 11.040, 81 11.040, 81× 2% = 220, 82 11.260, 92

O que se nota nessa tabela é que, a menos das aproximações feitas, o quociente

entre o nosso capital em um mês e o do mês anterior é constante igual a 1, 02.

Isto motiva a seguinte denição:

Definição 1 Uma Progressão Geométrica (PG) é uma sequência numérica na qual a taxa

de crescimento (ou decrescimento) de cada termo para o seguinte é sempre a

mesma.

2

Unidade 6Progressões Geométricas

Portanto, de acordo com o problema acima, as PGs modelam fenômenos

como o aumento de um capital aplicado a uma taxa anual prexada. Da mesma

forma, as PGs modelam o crescimento de uma população a uma taxa anual xa

ou, ainda, o decaimento da radiação emitida por um material radioativo.

Assim, as PGs aparecem muito frequentemente não só nas aplicações, mas

também, em vários contextos matemáticos e por isso, certamente, são muito

mais interessantes do que as progressões aritméticas.

Nessa unidade, você encontrará um grande leque de problemas em cuja solução

intervêm as PGs. Alguns desses problemas são resolvidos usando a muito útil

fórmula das taxas equivalentes, como no Exemplo 10, onde se pede para deter-

minar quanto a população de um país crescerá em n anos se ela cresce a uma

taxa de 2% ao ano.

Em seguida, será deduzida a fórmula da soma dos n primeiros termos de uma

PG e o cálculo do limite da soma dos termos de uma PG decrescente.

A unidade se encerra com mais uma técnica de somação, a fórmula da so-

mação por partes, que aumentará o nosso arsenal de técnicas de somação de

sequências, permitindo, em particular, somar sequências cujos termos são ob-

tidos multiplicando termos de uma PA com termos de uma PG. A semelhança

dessa fórmula com a fórmula de integração por partes do Cálculo Integral não

é mera coincidência, pois a somação corresponde à discretização da integração.

6.2 Primeiros Exemplos

Um problema interessante, que costuma deixar os alunos intrigados e os

professores desconados, é o problema a seguir, adaptado de um problema do

exame nacional da MAA (Mathematical Association of America).

Exemplo 1Uma pessoa, começando com R$ 64,00, faz seis apostas consecutivas, em

cada uma das quais arrisca perder ou ganhar a metade do que possui na ocasião.

Se ela ganha três e perde três dessas apostas, pode-se armar que ela:

A) Ganha dinheiro.

B) Não ganha dinheiro nem perde dinheiro.

C) Perde R$ 27,00.

3

Unidade 6 Primeiros Exemplos

D) Perde R$ 37,00.

E) Ganha ou perde dinheiro, dependendo da ordem em que ocorreram suas

vitórias e derrotas.

Comentário. Em geral os alunos escolhem uma ordem para ver o que acon-

teceu; aliás, essa é até uma boa estratégia. Por exemplo, se ela vence as três

primeiras apostas e perde as últimas três, o seu capital evoluiu de acordo com

o esquema: 64→ 96→ 144→ 216→ 108→ 54→ 27.

Se ela começou com R$ 64,00 e terminou com R$ 27,00, ela perdeu R$

37,00. Já houve um progresso. Sabemos agora que a resposta só poderá ser

(D) ou (E).

Em seguida os alunos costumam experimentar uma outra ordem; por exem-

plo, ganhando e perdendo alternadamente. Obtêm-se: 64→ 96→ 48→ 72→36→ 54→ 27. Nessa ordem a pessoa também perdeu R$ 37,00.

Em seguida, experimentam outra ordem, torcendo para que a pessoa não ter-

mine com R$ 27,00, o que permitiria concluir que a resposta é (E). Infelizmente

encontram que a pessoa novamente termina com R$ 27,00 e permanecem na

dúvida. Alguns se dispõem a tentar todas as ordens possíveis, mas logo desistem

ao perceber que há 20 ordens possíveis.

Solução A melhor maneira de abordar problemas nos quais há uma grandeza

variável, da qual é conhecida a taxa (porcentagem) de variação, é concentrar

a atenção, não na taxa de variação da grandeza, e sim no valor da grandeza

depois da variação.

Neste problema, devemos pensar assim: Cada vez que ganha, o capital

aumenta1

2(ou seja, 50%) e passa a valer 1 +

1

2=

3

2do que valia; cada vez

que perde, o capital diminui de1

2(ou seja, 50%) e passa a valer 1− 1

2=

1

2do

que valia.

Pensando assim, ca claro que se a pessoa vence as três primeiras apostas e

perde as três últimas, a evolução de seu capital se dá de acordo com o esquema:

64→ 64 · 3

2→ 64 · 3

2· 3

2→ 64 · 3

2· 3

2· 3

2→ 64 · 3

2· 3

2· 3

2· 1

2→

→ 64 · 3

2· 3

2· 3

2· 1

2· 1

2→ 64 · 3

2· 3

2· 3

2· 1

2· 1

2· 1

4

Unidade 6Progressões Geométricas

Ela termina com 64 · 3

2· 3

2· 3

2· 1

2· 1

2· 1

2= 27 Reais. Além disso, ca claro

também que se as vitórias e derrotas tivessem ocorrido em outra ordem, isso

apenas mudaria a ordem dos fatores, sem alterar o produto, e a pessoa também

terminaria com R$ 27,00.

Se ela começou com R$ 64,00 e terminou com R$ 27,00 ela perdeu R$

37,00. A resposta é (D).

Exemplo 2Aumentando de 20% o raio da base de um cilindro e diminuindo de 30%

sua altura, de quanto variará seu volume?

O volume é diretamente proporcional ao quadrado do raio e à altura. Por-

tanto, V = kr2h, onde k é a constante de proporcionalidade. Sabemos que

k = π, mas isso é irrelevante para o problema.

Depois da variação, os novos valores de r e de h serão r′ = 1, 2r e h′ = 0, 7h,

pois o aumento de 20% passa a valer 120% = 1, 2 do que valia e o que diminui

de 30% passa a valer 70% = 0, 7 do que valia.

O novo volume será

V ′ = k(1, 2r)20, 7h = 1, 008 k r2h = 100, 8%V.

O volume aumenta de 0, 8%.

O que deve ter cado claro nesses exemplos é que se uma grandeza tem

taxa de crescimento igual a i, cada valor da grandeza é igual a (1 + i) vezes

o valor anterior. Progressões geométricas são sequências nas quais a taxa de

crescimento i de cada termo para o seguinte é sempre a mesma.

Exemplo 3A população de um país é hoje igual a P0 e cresce 2% ao ano. Qual será

a população desse país daqui a n anos?

Se a população cresce 2% ao ano, em cada ano a população é de 102%

da população do ano anterior. Portanto, a cada ano que passa, a população

sofre uma multiplicação de 102% = 1, 02. Depois de n anos, a população será

P0 × 1, 02n.

5

Unidade 6 Primeiros Exemplos

Exemplo 4 A torcida de certo clube é hoje igual a P0 e decresce 5% ao ano. Qual será

a torcida desse clube daqui a n anos?

Se a torcida decresce 5% ao ano, em cada ano a torcida é de 95% da torcida

anterior. Portanto, a cada ano que passa, a torcida sofre uma multiplicação por

95% = 0, 95. Depois de n anos, a torcida será P0 × 0, 95n.

Exemplo 5 A sequência (1, 2, 4, 8, 16, 32, . . .) é um exemplo de uma progressão geo-

métrica. Aqui a taxa de crescimento de cada termo para o seguinte é de 100%,

o que faz com que cada termo seja igual a 200% do termo anterior.

Exemplo 6 A sequência (1000, 800, 640, 512, . . .) é um exemplo de uma progressão

geométrica. Aqui, cada termo é 80% do termo anterior. A taxa de crescimento

de cada termo para o seguinte é de −20%.

É claro então que numa progressão geométrica cada termo é igual ao anterior

multiplicado por 1 + i, onde i é a taxa de crescimento dos termos. Chamamos

1 + i de razão da progressão e representamos a razão por q.

Portanto, uma progressão geométrica é uma sequência na qual é constante

o quociente da divisão de cada termo pelo termo anterior. Esse quociente é

chamado de razão da progressão e é representado pela letra q. A razão q de

uma progressão geométrica é simplesmente o valor de 1 + i, onde i é a taxa de

crescimento constante de cada termo para o seguinte.

Exemplo 7 As sequências (2, 6, 18, 54, . . .) e (128, 32, 8, 2, . . .) são progressões geo-

métricas cujas razões valem respectivamente q1 = 3 e q2 = 14. Suas taxas de

crescimento são respectivamente i1 = 2 = 200% e i2 = −34

= −75%, pois

q = 1 + i.

Em uma progressão geométrica (a1, a2, a3, . . .), para avançar um termo

basta multiplicar pela razão; para avançar dois termos, basta multiplicar duas

vezes pela razão, e assim por diante.

Por exemplo, a13 = a5q8, pois avançamos 8 termos ao passar de a5 para a13;

a12 = a7q5, pois avançamos 5 termos ao passar de a7 para a12; a4 =

a17q13

, pois

6

Unidade 6Progressões Geométricas

ao passar de a17 para a4, retrocedemos 13 termos; de modo geral, an = a1qn−1,

pois, ao passar de a1 para an, avançamos n− 1 termos.

Em muitos casos é mais natural numerar os termos a partir de zero, como

foi feito nos Exemplos 3 e 4; nesse caso, an = a0qn, pois avançamos n termos

ao passar de a0 para an.

Exemplo 8Em uma progressão geométrica, o quinto termo vale 5 e o oitavo termo

vale 135. Quanto vale o sétimo termo dessa progressão?

Temos a8 = a5q3, pois ao passar do quinto termo para o oitavo, avançamos

3 termos. Logo, 135 = 5q3 e q = 3. Analogamente, a7 = a5q2 = 5 . 32 = 45.

O sétimo termo vale 45.

Como em uma progressão geométrica an = a0qn, a função que associa a

cada natural n o valor de an é simplesmente a restrição aos naturais da função

exponencial a(x) = a(0)qx. Portanto, pensando em uma progressão geométrica

como uma função que associa a cada número natural n o valor an, o gráco

dessa função é formado por uma sequência de pontos pertencentes ao gráco

de uma função exponencial.

Figura 6.1: Gráco de uma PG

Exemplo 9Qual é a razão da progressão geométrica que se obtém inserindo 3 termos

entre os números 30 e 480?

Temos a1 = 30 e a5 = 480. Como a5 = a1q4, 480 = 30q4, q4 = 16 e

q = ±3.

7

Unidade 6 A Fórmula das Taxas Equivalentes

Um resultado importante é a fórmula que relaciona taxas de crescimento

referidas a períodos de tempo diversos. É o que abordaremos na próxima seção.

6.3 A Fórmula das Taxas Equivalentes

Lema 2 Se I é a taxa de crescimento de uma grandeza relativamente ao período de

tempo T e i é a taxa de crescimento relativamente ao período t, e se T = nt,

então 1 + I = (1 + i)n.

Demonstração Seja G0 o valor inicial da grandeza. Após um período de tempo T , o

valor da grandeza será G0(1 + I)1. Como um período de tempo T equivale

a n períodos de tempo iguais a t, o valor da grandeza será também igual a

G0(1 + i)n. Logo, G0(1 + i)1 = G0(1 + i)n e 1 + I = (1 + i)n.

Exemplo 10 Se a população de um país cresce 2% ao ano, quanto crescerá em 25 anos?

Temos i = 2% = 0, 02 e n = 25. Daí, 1 + I = (1 + i)n = (1 + 0, 02)25 ∼=1, 6406 e I ∼= 0, 6406 = 64, 06%.

Exemplo 11 Uma bomba de vácuo retira, em cada sucção, 2% do gás existente em certo

recipiente. Depois de 50 sucções, quanto restará do gás inicialmente existente?

Temos i = −2% = −0, 02 e n = 50. Daí, 1+I = (1+i)n = (1−0, 02)50 ∼=0, 3642 e I ∼= −0, 6358 = −63, 58%. A quantidade de gás diminuirá de aproxi-

madamente 63, 58%. Restarão aproximadamente 36, 42% do gás inicialmente

existente.

Outro resultado importante é a fórmula que fornece a soma dos n primeiros

termos de uma progressão geométrica. Isto é fornecido na seção seguinte.

8

Unidade 6Progressões Geométricas

6.4 A Soma dos Termos de uma Progressão

Geométrica

Lema 3A soma nos n primeiros termos de uma progressão geométrica (an) de

razão q 6= 1, é Sn = a11− qn

1− q.

DemonstraçãoSn = a1 + a2 + a3 + · · ·+ an−1 + an. Multiplicando por q, obtemos

qSn = a2 + a3 + a4 + · · ·+ an + an+1.

Subtraindo, temos Sn − qSn = a1 − an+1, isto é, Sn(1 − q) = a1 − a1qn e,

nalmente, Sn = a11− qn

1− q.

Exemplo 12Diz a lenda que o inventor do xadrez pediu como recompensa 1 grão de

trigo pela primeira casa, 2 grãos pela segunda, 4 pela terceira e assim por diante,

sempre dobrando a quantidade a cada casa nova. Como o tabuleiro de xadrez

tem 64 casas, o número de grãos pedidos pelo inventor do jogo é a soma dos

64 primeiros termos da progressão geométrica 1, 2, 4, . . .. O valor dessa soma é

Sn = a11− qn

1− q= 1

1− 264

1− 2= 264 − 1.

Calculando, obtemos um estupendo número de dígitos:

18 446 744 073 709 551 615.

Nas progressões geométricas em que |q| < 1, a soma dos n primeiros termos

tem um limite nito quando n→∞. Como nesse caso limn→∞

qn = 0 temos

limn→∞

Sn = a11− 0

1− q,

isto é,

limn→∞

Sn =a1

1− q.

9

Unidade 6 A Soma dos Termos de uma Progressão Geométrica

Exemplo 13 O limite da soma 0, 3 + 0, 03 + 0, 003 + . . . quando o número de parcelas

tende a innito é igual a0, 3

1− 0, 1=

1

3. O resultado é intuitivo pois somando um

número muito grande de termos da progressão encontraremos aproximadamente

a dízima periódica 0, 33333 · · · = 1

3.

Exemplo 14 Calcule o limite da soma da progressão geométrica

1

2+

1

4+

1

8+

1

16+ . . . .

Temos que limn→∞

Sn =q1

1− q=

12

1− 12

= 1.

O resultado admite uma interessante paráfrase. Suponha que Salvador deva

correr 1 km. Inicialmente ele corre metade dessa distância, isto é, 12km; em

seguida ele corre metade da distância que falta, isto é, 14km; depois, metade

da distância restante, isto é, 18km, e assim por diante.

Depois de n etapas, Salvador terá corrido

1

2+

1

4+

1

8+ · · ·+ 1

2nkm.

Se n for grande, essa soma será aproximadamente igual a 1 km.

O teorema da somação,n∑

k=1

∆ak = an+1−a1, também nos permitiria deter-

minar o valor da soma dos n primeiros termos de uma progressão geométrica.

Supondo q 6= 1 e observando que ∆qk−1 = qk − qk−1 = qk−1(q − 1), temos

a1 + a2 + a3 + · · ·+ an =n∑

k=1

ak =n∑

k=1

a1qk−1 =

a1q − 1

n∑k=1

∆qk−1

=a1q − 1

(qn+1−1 − q0) = a11− qn

1− q.

+ Para Saber Mais - A Fórmula de Somação por Partes - Clique para ler

+ Na Sala de Aula - Sobre o Ensino de Progressões - Clique para ler

10

Unidade 6Progressões Geométricas

6.5 Exercícios Recomendados

1. Aumentos sucessivos de 10% e 20% equivalem a um aumento único de

quanto?

2. Aumentando sua velocidade em 60%, de quanto você diminui o tempo de

viagem?

3. Um decrescimento mensal de 5% gera um decrescimento anual de quanto?

4. Mantida constante a temperatura, a pressão de um gás perfeito é inver-

samente proporcional a seu volume. De quanto aumenta a pressão quando

reduzimos de 20% o volume?

5. Um carro novo custa R$ 18 000,00 e, com 4 anos de uso, vale R$ 12 000,00.

Supondo que o valor decresça a uma taxa anual constante, determine o valor

do carro com 1 ano de uso.

6. Determine três números em progressão geométrica, conhecendo sua soma

19 e a soma de seus quadrados 133.

7. Número perfeito é aquele que é igual a metade da soma dos seus divisores

positivos. Por exemplo, 6 é perfeito pois a soma dos seus divisores é 1 + 2 +

3 + 6 = 12. Prove que, se 2p − 1 é um número primo, então 2p−1 . (2p − 1) é

um número perfeito.

8. Calcule o valor da soma de n parcelas 1 + 11 + · · ·+ 111 . . . 1.

9. Um garrafão contém p litros de vinho. Retira-se um litro de vinho do

garrafão e acrescenta-se um litro de água, obtendo-se uma mistura homogênea;

retira-se, a seguir um litro da mistura e acrescenta-se um litro de água e assim

por diante. Qual a quantidade de vinho que restará no garrafão após n dessas

operações?

10. Larga-se uma bola de uma altura de 5cm. Após cada choque com o solo,

ela recupera apenas 4/9 da altura anterior. Determine:

a) A distância total percorrida pela bola.

b) O tempo gasto pela bola até parar.

11. Uma faculdade recebe todos os anos 300 alunos novos no primeiro semestre

e 200 alunos novos no segundo semestre. 30% dos alunos são reprovados no

11

Unidade 6 Exercícios Suplementares

primeiro período e repetem o período no semestre seguinte. Sendo an e bn,

respectivamente, o número de alunos do primeiro período no primeiro e no

segundo semestres do ano n, calcule lim an e lim bn.

12. Se (an) é uma progressão geométrica de termos positivos, prove que (bn)

denida por bn = log an é uma progressão aritmética.

13. Se (an) é uma progressão aritmética, prove que (bn) denida por bn = ean

é uma progressão geométrica.

14. Seja A =

[1 2

2 4

]. Determine An.

6.6 Exercícios Suplementares

1. Descontos sucessivos de 10% e 20% equivalem a um desconto único de

quanto?

2. Um aumento de 10% seguido de um desconto de 20% equivale a um desconto

único de quanto?

3. O período de um pêndulo simples é diretamente proporcional à raiz qua-

drada do seu comprimento. De quanto devemos aumentar o comprimento para

aumentar de 20% o período?

4. Se a base de um retângulo aumenta de 10% e a altura diminui de 10%, de

quanto aumenta a área?

5. Os lados de um triângulo retângulo formam uma progressão geométrica

crescente. Determine a razão dessa progressão.

6. Os lados de um triângulo estão em progressão geométrica. Entre que valores

pode variar a razão?

7. Qual o quarto termo da progressão geométrica√

2, 3√

2, 6√

2,...?

8. A soma de três números em progressão geométrica é 19. Subtraindo-se 1

ao primeiro, eles passam a formar um progressão aritmética. Calcule-os.

9. Quatro números são tais que os três primeiros formam uma progressão

aritmética de razão 6, os três últimos uma progressão geométrica e o primeiro

número é igual ao quarto. Determine-os.

12

Unidade 6Progressões Geométricas

10. Mostre que o número 444 . . . 488 . . . 89, formado por n dígitos iguais a 4,

n−1 dígitos iguais a 8 e um dígito igual a 9, é um quadrado perfeito. Determine

sua raiz quadrada.

11. A espessura de uma folha de estanho é 0,1mm. Forma-se uma pilha

de folhas colocando-se uma folha na primeira vez e, em cada uma das vezes

seguintes, tantas quantas já houveram sido colocadas anteriormente. Depois

de 33 dessas operações, a altura da pilha será, aproximadamente:

a) a altura de um poste de luz;

b) a altura de um prédio de 40 andares;

c) o comprimento da praia de Copacabana;

d) a distância Rio-São Paulo;

e) o comprimento do equador terrestre.

12. Calcule a soma dos divisores de 12.600 que sejam:

a) positivos;

b) ímpares e positivos.

13. Determine as geratrizes das dízimas periódicas:

a) 0, 141414141 . . . b) 0, 345454545 . . .

c) 0, 999999999 . . . d) 1, 711111111 . . .

14. Determine os limites das somas abaixo:

a) 2 +2

3+

2

9+ · · ·

b)1

7+

2

72+

1

73+

2

74+

1

75+

2

76+ · · ·

c)1

2+

3

4+

5

8+

7

16+

9

32+ · · ·

d) 1 + 2x+ 3x2 + 4x3 + . . ., −1 < x < 1;

e) 1− 1

2− 1

4+

1

8− 1

16− · · ·

15. Na gura abaixo temos uma linha poligonal, de lados ora perpendiculares a

AB, ora perpendiculares a AC. Sendo a e b, respectivamente, os dois primeiros

lados da poligonal, pede-se determinar:

a) o comprimento da mesma;

b) o comprimento do n-ésimo lado da poligonal.

13

Unidade 6 Exercícios Suplementares

Figura 6.2: Poligonal

16. Na gura abaixo temos uma espiral formada por semicírculos cujos centros

pertencem ao eixo das abscissas. Se o raio do primeiro semicírculo é igual a 1

e o raio de cada semicírculo é igual à metade do raio do semicírculo anterior,

determine:

a) o comprimento da espiral;

b) a abscissa do ponto P , ponto assintótico da espiral.

Figura 6.3: Espiral

17. Na gura abaixo temos uma sequência de círculos tangentes a duas retas.

O raio do primeiro círculo é 1 e o raio do segundo é r < 1. cada círculo

tangencia externamente o círculo anterior. Determine a soma dos raios dos n

primeiros círculos.

18. Seja Sn a soma das áreas dos n primeiros quadrados obtidos a partir de

um quadrado Q1 de lado 1 pelo seguinte processo: os vértices do quadrado

Qn+1 são os pontos médios dos lados de Qn. Determine quais das armações

abaixo são verdadeiras:

14

Unidade 6Progressões Geométricas

Figura 6.4: Sequência de círculos

1) é possível escolher Sn de modo que Sn > 1, 9;

2) é possível escolher Sn de modo que Sn > 2;

3) é possível escolher Sn de modo que Sn > 2, 1;

4) é possível escolher Sn de modo que Sn = 2;

5) é possível escolher Sn de modo que Sn = 1, 75.

19. Sendo x e y positivos, calcule:

a)

√x

√x

√x√x . . . b)

√x

√y√x√y . . .

20. Começando com um segmento de tamanho 1, dividimo-lo em três partes

iguais e retiramos o interior da parte central, obtendo dois segmentos de com-

primento 1/3. Repetimos agora essa operação com cada um desses segmentos

e assim por diante. Sendo Sn a soma dos comprimentos dos intervalos que

restaram depois de n dessas operações, determine:

a) O valor de Sn;

b) O valor de limSn;

c) Certo livro, muito citado em aulas de análise de erros de livros didáticos,

arma que, ao nal, o conjunto dos pontos não retirados é vazio. Isso é verdade?

21. O rádio-226 tem meia-vida (período de tempo em que metade da massa

inicialmente presente se desintegra) de 1600 anos. A taxa de variação da massa

é constante. Em quanto tempo a terça parte da massa inicialmente presente se

desintegrará?

15

Unidade 6 Exercícios Suplementares

22. Sejam a = 111 . . . 1 (n dígitos iguais a 1) e b = 100 . . . 05 (n − 1 dígitos

iguais a 0). Prove que ab + 1 é um quadrado perfeito e determine sua raiz

quadrada.

23. A curva de Koch é obtida em estágios pelo processo seguinte:

i) No estágio 0, ela é um triângulo equilátero de lado 1.

ii) O estágio n + 1 é obtido a partir do estágio n, dividindo cada lado em

três partes iguais, construindo externamente sobre a parte central um triângulo

equilátero e suprimindo então a parte central (ver gura abaixo). Sendo Pn e

An respectivamente o perímetro e a área do n-ésimo estágio da curva de Koch,

determine:

a) Pn b) An c) limPn d) limAn.

Figura 6.5: A curva de Koch

24. Pitágoras 1, que estudou a geração dos sons, observou que duas cordas

vibrantes, cujos comprimentos estivessem na razão de 1 para 2, soariam em

uníssono. Hoje sabemos que a razão das frequências dos sons emitidos por

essas cordas seria a razão inversa dos seus comprimentos, isto é, de 2 para 1 e

que duas cordas vibram em uníssono se e só se a razão de seus comprimentos

é uma potência inteira de 2.

A frequência da nota lá-padrão (o lá central do piano) é 440Hz e a frequência

do lá seguinte, mais agudo, é 880Hz (Hz é a abreviatura de hertz, unidade de

frequência que signica ciclo por segundo).

1Pitágoras, matemático de Samos, cerca de cinco séculos e meio antes de Cristo.

16

Unidade 6Progressões Geométricas

A escala musical ocidental (de J.S.Bach pra cá), dita cromática, divide esse

intervalo em doze semitons iguais, isto é, tais que a razão das frequências de

notas consecutivas é constante.

Sabendo que essas notas são

LÁ - LÁ# - SI - DÓ - DÓ# - RÉ - RÉ# - MI - FÁ - FÁ# - SOL - SOL# - LÁ

determine:

a) a frequência desse dó, primeiro dó seguinte ao lá padrão;

b) a frequência do sinal de discar de um telefone, que é o primeiro sol anterior

ao lá padrão;

c) a nota cuja frequência é 186Hz.

25. A lei de Weber (Ernest Heinrich Weber; 1795-1878; siologista alemão),

para resposta de seres humanos a estímulos físicos, declara que diferenças mar-

cantes na resposta a um estímulo ocorrem para variações de intensidade do

estímulo proporcionais ao próprio estíímulo. Por exemplo, um homem, que sai

de um ambiente iluminado para outro, só percebe uma variação da luminosidade

se esta for superior a 2%; só distingue entre soluções salinas se a variação da

salinidade for superior a 25%, etc...

Fechner (Gustav Theodor Fechner; 1801-1887; físico e lósofo alemão) pro-

põe um método de construção de escalas baseado na lei de Weber: propôs que

enquanto os estímulos variassem em progressão geométrica, as medidas das

respostas variassem em progressão aritmética.

a) Mostre que numa escala de Fechner, as medidas da resposta y e do estímulo

x se relacionam por y = a+ b log x.

b) Uma das mais conhecidas escalas de Fechner é a que mede a sensação de

ruído. Ela é denida por L = 120 + 10 log10 l, onde L é a medida da sensação

d e ruído em decibéis (dB) e l é a intensidade sonora, medida em W/m2. Duas

bandas de heavy metal provocam um ruído quantos decibéis acima do ruído

provocado por uma banda?

26. Determine o valor de:

a)∞∑k=1

k2

2kb)

n∑k=1

k . 2k

17

Unidade 6 Textos Complementares

6.7 Textos Complementares

Na Sala de Aula Sobre o Ensino de Progressões

1. Não encha a cabeça de seus alunos com casos particulares desnecessários.

Isso só serve para obscurecer as ideias gerais e acaba dicultando as coisas.

Saber que, numa progressão aritmética, cada termo é a média aritmética entre

seu antecedente e seu consequente não só não substitui, ou pelo menos não

substitui de modo eciente, o conhecimento de que uma progressão aritmética

é uma sequência na qual a diferença entre cada termo e o termo anterior é

constante, como é uma consequência imediata disso. Realmente, se x, y, z,

estão em progressão aritmética, y − x = z − y. Daí, quem se interessar em

calcular y obterá y =x+ z

2.

Do mesmo modo são conhecimentos desnecessários:

• Em uma progressão aritmética com um número ímpar de termos, o termo

central é a média aritmética dos extremos.

• Em uma progressão aritmética, a soma de dois termos equidistantes dos

extremos é igual a soma dos extremos.

• Em uma progressão geométrica cada termo é a média geométrica entre seu an-

tecedente e seu consequente. (Seria isso verdadeiro para a progressão 1,−2, 4?)

• Em uma progressão geométrica com um número ímpar de termos, o termo

central é a média geométrica dos extremos (Seria isso verdadeiro para a pro-

gressão 1,−2, 4?)

• Em uma progressão geométrica, o produto de dois termos equidistantes dos

extremos é igual ao produto dos extremos.

2. Na maioria dos livros se encontram as fórmulas an = a1 + (n − 1)r, para

progressões aritméticas e an = a1qn−1, para progressões geométricas. Nada

contra essas fórmulas, já que usualmente o que se conhece de uma progressão

são o primeiro termo e a razão.

Entretanto é bom lembrar que o conhecimento apenas dessas fórmulas cos-

tuma atrapalhar muitos alunos quando a progressão começa em a0. É cer-

tamente mais eciente saber que para avançar um termo basta somar r ou

multiplicar por q, para avançar dois termos basta somar 2r ou multiplicar q2,

etc... Assim, facilmente se conclui que an = a0 + nr e an = a1 + (n − 1)r,

18

Unidade 6Progressões Geométricas

nas progressões aritméticas, e que an = a0qn e an = a1q

n−1, nas progressões

geométricas.

3. Em alguns livros se encontram, além da fórmula an = a1+(n−1)r, fórmulas

como a1 = an − (n− 1)r, r =an − a1n− 1

, n = 1 +an − a1

r, supostamente para

facilitar o cálculo. Depois nos queixamos que os alunos não sabem resolver

equações do primeiro grau!

Mais cedo ou mais tarde, aparecerá um livro com uma fórmula para o cálculo

do 1: 1 = n− an − a1r

.

4. Alguns livros chegam ao cúmulo de trazerem duas versões da (desnecessária)

fórmula para o cálculo de r: r =an − a1n− 1

e r =an+2 − a1n+ 1

, a segunda para ser

usada quando a progressão tiver n + 2 termos, isto é, dois termos extremos e

mais n termos entre eles, como no Exemplo 4 da Unidade 5.

5. Alguns livros trazem uma fórmula para o cálculo do produto dos n primeiros

termos de uma progressão geométrica, Pn = (√a1an )n.

Em primeiro lugar, essa fórmula está errada. Por ela, o produto dos três

primeiros termos da progressão1,−2, 4, . . . seria (√

1 . 4)3 = 23 = 8.

Em segundo lugar, se corrigirmos essa fórmula obteremos P 2n = (a1an)n e,

nas progressões cujos termos não são todos positivos, teremos algum trabalho

em descobrir se Pn = (a1an)n ou se Pn = −(a1an)n.

Em terceiro lugar, não há o menor interesse, prático ou teórico, em deter-

minar o produto dos termos de uma progressão geométrica.

Em quarto lugar, é muito simples determinar o produto dos termos de uma

progressão geométrica. Com efeito, isso já foi feito no Exercício 3 da Unidade

5.

6. Moderação nos problemas. Problemas em que são dados a soma do 24O

termo com o 47O e é pedida a diferença entre o 36O e o 11O não aparecem na

vida real, não são interessantes e não desenvolvem o raciocínio. Uma pergunta

que devemos sempre nos fazer é a seguinte: Se meu professor de Matemática

tivesse feito estes problemas, eu teria gostado de Matemática?

7. Tenha sempre em mente que uma progressão geométrica é uma sequên-

cia na qual a taxa de crescimento de cada termo para o seguinte é sempre

a mesma e esse instrumento matemático foi criado para descrever grandezas

19

Unidade 6 Textos Complementares

que variam com taxa de crescimento constante. É absurdo, mas infelizmente é

comum, ensinar progressões geométricas e não relacioná-las à idéia de taxa de

crescimento.

8. A melhor maneira de resolver problemas com progressões com um número

pequeno de termos é escrevê-las e esquecer completamente as fórmulas para

calcular termos e somas de termos conforme zemos nos Exemplos 7 e 8 da

Unidade 5.

Entretanto, ao contrário do que ocorria com as progressões aritméticas, não

há nenhuma vantagem, ao escrever progressões geométricas, em começar pelo

termo central. Chamar três números em progressão geométrica dex

q, x, xq

em vez de chamá-los de x, xq, xq2, só serve para criar desnecessariamente

denominadores e complicar as contas.

9. Calculadoras são indispensáveis para a resolução de quase todos os problemas

de progressões geométrica da vida real.

10. Se você ensina exponenciais e logaritmos antes de progressões, não há

grandes diculdades em falar intuitivamente de limite da soma dos termos de

uma progressão geométrica pois, ao fazer os grácos das funções exponenciais e

logarítmicas, você já deve ter comentado quais os limites de ax quando x tende

para +∞ ou para −∞. Se a primeira noção de limite aparece no limite da soma

da progressão geométrica, os Exemplos 13 e 14 de progressões geométricas são

muito bons.

20

Unidade 6Progressões Geométricas

Para Saber MaisA Fórmula de Somação por Partes

Encerramos esta seção com a chamada fórmula de somação por partes.

Temos

∆(akbk) = ak+1bk+1−akbk = ak+1(bk+1−bk)+bk(ak+1−ak) = ak+1∆bk+bk∆ak

Daí resulta

ak+1∆bk = ∆(akbk)− bk∆ak.

Somando, obtemos a fórmula de somação por partes:

n∑k=1

ak+1∆bk = an+1bn+1 − a1b1 −n∑

k=1

ak∆ak.

Exemplo Calculen∑

k=1

k3k.

Temos ∆3k = 3k+1 − 3k = 3k(3− 1) = 2× 3k. Logo, 3k =1

2∆3k e

n∑k=1

k3k =1

2

n∑k=1

k∆3k.

Aplicando a fórmula de somação por partes

n∑k=1

ak+1∆bk = an+1bn+1 − a1b1 −n∑

k=1

bk∆ak.

com ak+1 = k (logo, ak = k − 1 e ∆ak = ak+1 − ak = 1) e bk = 3k, temos

n∑k=1

k3k =1

2

n∑k=1

k∆3k =1

2

[n . 3n+1 − 0−

n∑k=1

3k . 1

].

Masn∑

k=1

3k = 31− 3n

1− 3=

3n+1

2− 3

2

Daí resulta

n∑k=1

k3k =n

23n+1 − 3n+1

4+

3

4=

2n− 1

43n+1 +

3

4.

21

Unidade 6 Textos Complementares

22

7

1

Recorrências Lineares dePrimeira Ordem

Sumário

7.1 Introdução . . . . . . . . . . . . . . . . . . . . . . . 2

7.2 Sequências Denidas Recursivamente . . . . . . . . 3

7.3 Exercícios Recomendados . . . . . . . . . . . . . . . 4

7.4 Exercícios Suplementares . . . . . . . . . . . . . . . 4

7.5 Recorrências Lineares de Primeira Ordem . . . . . . 5

7.6 Exercícios Recomendados . . . . . . . . . . . . . . . 9

7.7 Exercícios Suplementares . . . . . . . . . . . . . . . 9

7.8 Textos Complementares . . . . . . . . . . . . . . . . 10

Unidade 7 Introdução

7.1 Introdução

O assunto dessa unidade é o estudo mais aprofundado das sequências numé-

ricas denidas recursivamente (ou por recorrência) que abordamos nas Unidades

3 e 4.

Conforme vimos anteriormente, uma sequência é denida recursivamente

se ela for dada por uma regra (recorrência) que permite calcular um termo

qualquer por meio de um ou mais termos anteriores. Por exemplo, PAs, PGs,

fatorial, potências com expoentes números naturais e a sequência de Fibonacci

são denidas por recorrência.

Nesta unidade, são estudadas as recorrências lineares de primeira ordem, ou

seja, sequências em que um termo qualquer é denido por uma expressão que

envolve o termo anterior, sem elevá-lo a um expoente maior do que 1, como,

por exemplo:

1) progressões aritméticas: an = an−1 + r;

2) progressões geométricas: an = an−1q;

3) fatorial: an = nan−1;

4) potências com expoente natural: an = aan−1.

Note que, para denir uma sequência desse modo, não basta dar a recorrên-

cia, mas é preciso dizer qual é o seu primeiro termo. Isto é óbvio nos casos de

PAs e PGs. No caso (3), obtemos o fatorial se tomarmos a1 = 1. Se tomarmos

a1 = 2, por exemplo, obtemos a sequência:

a1 = 2, a2 = 4, a3 = 12, a4 = 48, . . . ,

que não representa o fatorial. Temos também que (4) somente dene as po-

tências de a se tomarmos a1 = a.

Na presente unidade, vamos nos dedicar, essencialmente, a determinar fór-

mulas fechadas para algumas recorrências lineares de primeira ordem, onde, por

fórmula fechada, entendemos uma expressão an = φ(n) para an como função

de n. Quando determinamos uma fórmula fechada para uma recorrência, di-

zemos que ela foi resolvida. Aprenderemos como resolver recorrências do tipo

an+1 = can+ f(n), onde f é uma função com domínio o conjunto dos naturais

e c é uma constante.

2

Unidade 7Recorrências Lineares de Primeira Ordem

7.2 Sequências Denidas Recursivamente

Muitas sequências são denidas recursivamente (isto é, por recorrência),

ou seja, por intermédio de uma regra que permite calcular qualquer termo em

função do(s) antecessor(es) imediato(s).

Exemplo 1A sequência (xn) dos números naturais ímpares 1, 3, 5, 7, . . . pode ser de-

nida por xn+1 = xn + 2 (n > 1), com x1 = 1.

Exemplo 2Qualquer progressão aritmética (xn) de razão r e primeiro termo a pode

ser denida por xn+1 = xn + r (n > 1), com x1 = a.

Exemplo 3Qualquer progressão geométrica (xn) de razão q e primeiro termo a pode

ser denida por xn+1 = q · xn (n > 1), com x1 = a.

Exemplo 4A sequência (Fn), dita de Fibonacci, cujos termos são 1, 1, 2, 3, 5, . . . e na

qual cada termo é a soma dos dois imediatamente anteriores, é denida por

Fn+2 = Fn+1 + Fn (n > 0), com F0 = F1 = 1.

É fácil ver que uma recorrência, por si só, não dene a sequência. Por exemplo,

a recorrência do Exemplo 1, xn+1 = xn + 2, é satisfeita não apenas pela

sequência dos números ímpares, mas por todas as progressões aritméticas de

razão 2. Para que a sequência que perfeitamente determinada é necessário

também o conhecimento do(s) primeiro(s) termo(s).

Observe que, nos Exemplos 1, 2 e 3 temos recorrências de primeira ordem,

isto é, nas quais cada termo é expresso em função do antecessor imediato, e

que, no Exemplo 4, temos uma recorrência de segunda ordem, ou seja, na qual

cada termo é expresso em função dos dois antecessores imediatos.

+ Para Saber Mais - Dois Exemplos Mais Sosticados - Clique para ler

3

Unidade 7 Exercícios Recomendados

7.3 Exercícios Recomendados

1. Determine x5 na sequência denida por xn+2 = 2xn+1 + xn, x0 = x1 = 1.

2. Seja xn o número máximo de regiões em que n retas podem dividir o plano.

Caracterize xn recursivamente.

Sugestão: Lembre-se da pizza de Steiner.

3. Prove que uma recorrência de primeira ordem, xn+1 = f(xn), com uma

condição inicial x1 = a, tem sempre uma e uma só solução.

4. Prove que uma recorrência de segunda ordem xn+2 = f(xn+1, xn), com

condições iniciais x1 = a e x2 = b, tem sempre solução única.

5. Determine xn, dada a sequência:

a) xn+1 = 2xn e x1 = 3; b) xn+1 = xn + 3 e x1 = 2.

7.4 Exercícios Suplementares

1. Seja xn o número máximo de regiões em que n círculos podem dividir o

plano. Caracterize xn recursivamente.

2. Determine o número de permutações caóticas de 5 elementos.

3. Prove que o número de permutações caóticas de n elementos é

Dn = n!n∑

k=0

(−1)n

k!.

4

Unidade 7Recorrências Lineares de Primeira Ordem

7.5 Recorrências Lineares de Primeira Ordem

Uma recorrência de primeira ordem expressa xn+1 em função de xn. Ela é

dita linear se (e somente se) essa função for do primeiro grau.

Exemplo 5As recorrências xn+1 = 2xn−n2 e xn+1 = nxn são lineares e a recorrência

xn+1 = x2n não é linear. As duas últimas são ditas homogêneas, por não

possuirem termo independente de xn.

Não há grandes diculdades na resolução de uma recorrência linear homo-

gênea de primeira ordem, conforme mostram os exemplos a seguir.

Exemplo 6Resolva a recorrência xn+1 = nxn, x1 = 1.

Solução. Temos

x2 = 1x1

x3 = 2x2

x4 = 3x3

. . . . . . . . .

xn = (n− 1)xn−1

Daí, multiplicando, obtemos xn = (n − 1)!x1. Como x1 = 1, temos xn =

(n− 1)!.

Exemplo 7Resolva a recorrência xn+1 = 2xn.

Solução. Temos

x2 = 2x1

x3 = 2x2

x4 = 2x3

. . . . . . . . .

xn = 2xn−1

Daí, multiplicando, obtemos xn = 2n−1x1. é claro que como não foi prescrito

o valor de x1, há uma innidade de soluções para a recorrência, xn = C · 2n−1,

onde C é uma constante arbitrária.

5

Unidade 7 Recorrências Lineares de Primeira Ordem

As recorrências lineares não-homogêneas de primeira ordem que mais facil-

mente se resolvem são as da forma xn+1 = xn + f(n).

Com efeito, temos

x2 = x1 + f(1)

x3 = x2 + f(2)

x4 = x3 + f(3)

. . . . . . . . .

xn = xn−1 + f(n− 1)

Somando, obtemos xn = x1 +n−1∑k=1

f(k).

Exemplo 8 Resolva a recorrência xn+1 = xn + 2n, x1 = 1.

Solução. Temos

x2 = x1 + 2

x3 = x2 + 22

x4 = x3 + 23

. . . . . . . . .

xn = xn−1 + 2n−1

Somando, resulta

xn = x1 + (2 + 22 + 23 + · · ·+ 2n−1)

= 1 + 2 + 22 + 23 + · · ·+ 2n−1

= 12n − 1

2− 1= 2n − 1.

6

Unidade 7Recorrências Lineares de Primeira Ordem

Exemplo 9Resolva xn+1 = xn + n, x1 = 0.

Solução. Temos

x2 = x1 + 1

x3 = x2 + 2

x4 = x3 + 3

. . . . . . . . .

xn = xn−1 + (n− 1).

Somando, resulta

xn = x1 + 1 + 2 + 3 + · · ·+ (n− 1)

= 1 + 2 + 3 + · · ·+ (n− 1)

=n(n− 1)

2.

O teorema a seguir mostra que qualquer recorrência linear não-homogênea

de primeira ordem pode ser transformada em uma da forma xn+1 = xn+ f(n).

Teorema 1Solução de

Recorrências Lineares

de Primeira Ordem

Se an é uma solução não-nula da recorrência xn+1 = g(n)xn, então a

substituição xn = anyn transforma a recorrência xn+1 = g(n)xn + h(n) em

yn+1 = yn + h(n)[g(n) · an]−1.

DemonstraçãoA substituição xn = anyn transforma

xn+1 = g(n)xn + h(n) em an+1yn+1 = g(n)anyn + h(n).

Mas, an+1 = g(n)an, pois an é solução de xn+1 = g(n)xn. Portanto, a equação

se transforma em

g(n)anyn+1 = g(n)anyn + h(n),

ou seja, yn+1 = yn + h(n)[g(n) · an]−1.

7

Unidade 7 Recorrências Lineares de Primeira Ordem

Exemplo 10 Resolva xn+1 = 2xn + 1, x1 = 2.

Solução. Uma solução não-nula de xn+1 = 2xn é, por exemplo, xn = 2n−1,

conforme vimos no Exemplo 7. Fazendo a substituição xn = 2n−1yn, obtemos

2nyn+1 = 2nyn + 1, ou seja, yn+1 = yn + 2−n. Daí se tem

y2 = y1 + 2−1

y3 = y2 + 2−2

y4 = y3 + 2−3

. . . . . . . . .

yn = yn−1 + 2−(n−1).

Somando, resulta

yn = y1 + 2−1 + 2−2 + 2−3 + · · ·+ 2−(n−1)

= y1 + 2−1 (2−1)n−1 − 1

2−1 − 1

= y1 − 21−n + 1.

Como xn = 2n−1yn e x1 = 2, temos y1 = 2 e yn = 3 − 21−n. Daí, xn =

3 · 2n−1 − 1.

Exemplo 11 Resolva xn+1 = 3xn + 3n, x1 = 2.

Solução. Uma solução não-nula de xn+1 = 3xn é, por exemplo, xn = 3n−1

(ou qualquer outra progressão geométrica de razão 3). Façamos a substituição

xn = 3n−1yn. Obtemos 3nyn+1 = 3nyn + 3n, ou seja, yn+1 = yn + 1. Daí,

yn é uma progressão aritmética de razão 1. Logo, yn = y1 + (n − 1)1. Como

xn = 3n−1yn e x1 = 2, temos y1 = 2 e yn = n+ 1. Daí, xn = (n+ 1)3n−1.

8

Unidade 7Recorrências Lineares de Primeira Ordem

7.6 Exercícios Recomendados

1. Resolva a recorrência do Exercício Recomendado 1, da Seção 3 (pizza de

Steiner).

2. Quantas são as sequências de n termos, todos pertencentes a 0, 1, quepossuem em número ímpar de termos iguais a 0?

3. Quantas são as sequências de n termos, todos pertencentes a 0, 1, 2,que possuem em número ímpar de termos iguais a 0?

4. Sheila e Helena disputam uma série de partidas. Cada partida é iniciada

por quem venceu a partida anterior. Em cada partida, quem iniciou tem

probabilidade 0,6 de ganhá-la e probabilidade 0,4 de perdê-la. Se Helena

iniciou a primeira partida, qual é a probabilidade de Sheila ganhar a n-

ésima partida?

5. Resolva as seguintes recorrências:

a) xn+1 = (n+ 1)xn + n, x1 = 1;

b) (n+ 1)xn+1 + nxn = 2n− 3, x1 = 1;

c) xn+1 − nxn = (n+ 1)!, x1 = 1.

7.7 Exercícios Suplementares

1. Um círculo foi dividido em n (n > 2) setores. De quantos modos podemos

colorí-los, cada setor com uma só cor, se dispomos de k (k > 2) cores

diferentes e setores adjacentes não devem ter a mesma cor?

2. A torcida do Fluminense tem hoje p0 membros. A taxa anual de natalidade

é i, a mortalidade é j e, além disso, todo ano um número xo de R

torcedores desiste de vez. Se i > j, determine o número de torcedores

daqui a n anos. A torcida está condenada a extinção?

3. Ache o número máximo de regiões em que n círculos podem dividir o

plano, ou seja resolva a recorrência do Exercício Suplementar 1 da Seção

4.

9

Unidade 7 Textos Complementares

7.8 Textos Complementares

Para Saber Mais Dois Exemplos Mais Sosticados

Apresentaremos aqui dois exemplos mais sosticados.

Exemplo 1. Quantas são as sequências de 10 termos, pertencentes a 0, 1, 2,que não possuem dois termos consecutivos iguais a 0?

Solução. Chamando xn o número de sequências com n termos, o valor de

xn+2 será a soma das seguintes quantidades:

i) O número de sequências de n+2 termos que começam por 1 e não possuem

dois zeros consecutivos. Isso é precisamente igual a xn+1, pois se o primeiro

termo é 1, para formar a sequência basta determinar os termos a partir do

primeiro, o que pode ser feito de xn+1 modos.

ii) O número de sequências de n+2 termos que começam por 2 e não possuem

dois zeros consecutivos. Analogamente, isso é igual a xn+1.

iii) O número de sequências de n+2 termos que começam por 0 e não possuem

dois zeros consecutivos. Se o primeiro termo é zero, temos dois modos de

escolher o segundo termo (1 ou 2) e, escolhido o segundo termo, temos xnmodos de escolher os demais. Há, pois, 2xn sequências começadas em 0.

Logo, xn+2 = 2xn+1 + 2xn. É fácil ver que x1 = 3 e que x2 = 8. Daí

obtemos x3 = 2x2 + 2x1 = 22, x4 = 60, . . . , x10 = 24 960.

Exemplo 2. Seja Dn o número de permutações caóticas de 1, 2, . . . , n, isto é,

o número de permutações simples de 1, 2, . . . , n, nas quais nenhum elemento

ocupa o seu lugar primitivo. Mostre que Dn+2 = (n+1)(Dn+1+Dn), se n > 1.

Solução. Calculemos Dn+2, o número de permutações simples de 1, 2, . . . , n+

2 nas quais nenhum elemento ocupa o seu lugar primitivo. As permutações

podem ser divididas em dois grupos: aquelas nas quais o 1 ocupa o lugar do

número que ocupa o primeiro lugar e aquelas nas quais isso não ocorre.

Para formar uma permutação do primeiro grupo, devemos escolher o número

que trocará de lugar com o 1, o que pode ser feito de n + 1 modos, e, em

seguida, devemos arrumar os demais n elementos nos restantes n lugares, sem

que nenhum desses elementos ocupe o seu lugar primitivo, o que pode ser feito

de Dn modos. Há, portanto, (n+ 1) ·Dn permutações no primeiro grupo.

10

Unidade 7Recorrências Lineares de Primeira Ordem

Para formar uma permutação do segundo grupo, temos de escolher o lugar

que será ocupado pelo número 1 (chamemos esse lugar de k), o que pode

ser feito de n + 1 modos, e, em seguida devemos arrumar os restantes n + 1

elementos dos demais n + 1 lugares, sem que o elemento k ocupe o primeiro

lugar e sem que nenhum dos demais elementos ocupe o seu lugar primitivo, o

que pode ser feito de Dn+1 modos. Há, portanto, (n+ 1) ·Dn+1 permutações

no segundo grupo.

Consequentemente, Dn+2 = (n+1)(Dn+1+Dn), como queríamos demons-

trar.

11

Unidade 7 Textos Complementares

12

8

1

Recorrências Lineares deSegunda Ordem

Sumário

8.1 Introdução . . . . . . . . . . . . . . . . . . . . . . . 2

8.2 A equação Característica . . . . . . . . . . . . . . . 3

8.3 Recorrências de Segunda Ordem . . . . . . . . . . . 4

8.4 Exercícios recomendados . . . . . . . . . . . . . . . 11

Unidade 8 Introdução

8.1 Introdução

Um exemplo de recorrência linear de segunda ordem é a recorrência que

dene a sequência de Fibonacci: xn = xn−1 + xn−2.

Mais geralmente, uma recorrência linear de segunda ordem é uma recorrência

do tipo

f(n)xn + g(n)xn−1 + h(n)xn−2 + k(n) = 0,

onde f, g, h e k são funções cujos domínios são o conjunto dos números naturais

e f(n) nunca se anula. Quando k = 0, a recorrência é dita homogênea. Para

que uma recorrência do tipo acima nos dena uma sequência, é preciso estipular

os valores dos seus dois termos iniciais.

Nesta unidade, apresentaremos uma técnica para resolver recorrências line-

ares homogêneas de segunda ordem, com coecientes constantes. Essa técnica

consiste em encontrar progressões geométricas da forma rn que resolvem a re-

corrência e cujas razões r são raízes de uma equação algébrica do segundo grau

chamada equação característica da recorrência. O termo geral da sequência é

então obtido como uma combinação linear dessas progressões com coecientes

determinados graças aos valores dos termos iniciais x1 e x2.

Essa técnica pode ser plenamente justicada usando Álgebra Linear (cf. [7])

e é a mesma utilizada na resolução de equações diferenciais lineares homogê-

neas com coecientes constantes, onde as PGs são substituídas por funções

exponenciais.

Tal como na teoria das equações diferenciais, as soluções de uma equa-

ção com coecientes constante se obtêm somando uma solução particular da

equação dada às soluções da equação homogênea associada.

Para saber (bem) mais sobre sequências recorrentes, recomendamos a lei-

tura do artigo Sequências Recorrentes, de Carlos Gustavo Moreira, que o leitor

encontrará anexado ao material dessa semana e que não será cobrado nas ava-

liações, por ser de caráter complementar.

2

Unidade 8Recorrências Lineares de Segunda Ordem

8.2 A equação Característica

Inicialmente, trataremos das recorrências lineares de segunda ordem homo-

gêneas com coecientes constantes, isto é, recorrências da forma

xn+2 + pxn+1 + qxn = 0.

Suporemos sempre q 6= 0, pois se q = 0, a recorrência seria, na realidade, uma

recorrência de primeira ordem.

A cada recorrência linear de segunda ordem homogênea, com coecientes

constantes, da forma acima, associaremos uma equação do segundo grau, r2 +

pr + q = 0, chamada equação característica. A nossa suposição preliminar de

que q 6= 0 implica que 0 não é raiz da equação característica.

Exemplo 1A recorrência xn+2 = xn+1+xn tem equação característica r2 = r+1. As

raízes da equação característica são

r1 =1 +√5

2e r2 =

1−√5

2.

O teorema a seguir mostra que se as raízes da equação característica são

r1 e r2, então qualquer sequência da forma an = C1rn1 + C2r

n2 é solução da

recorrência, quaisquer que sejam os valores das constantes C1 e C2.

Teorema 1Se as raízes de r2 + pr + q = 0 são r1 e r2, então an = C1rn1 + C2r

n2 é

solução da recorrência xn+2+ pxn+1+ qxn = 0, quaisquer que sejam os valores

das constantes C1 e C2.

DemonstraçãoSubstituindo an = C1rn1 + C2r

n2 na recorrência xn+2 + pxn+1 + qxn = 0,

obtemos, agrupando convenientemente os termos,

C1rn1 (r

21 + pr1 + q) + C2r

n2 (r

22 + pr2 + q)

= C1rn10 + C2r

n20 = 0.

3

Unidade 8 Recorrências de Segunda Ordem

Exemplo 2 A equação xn+2 + 3xn+1 − 4xn = 0 tem r2 + 3r − 4 = 0 como equação

característica. As raízes da equação característica são 1 e −4. De acordo com

o Teorema 1, todas as sequências da forma an = C11n+C2(−4)n são soluções

da recorrência.

8.3 Recorrências de Segunda Ordem

O teorema a seguir mostra que, se r1 6= r2, todas as soluções da recorrência

têm a forma apontada no Teorema 1.

Teorema 2 Se as raízes de r2 + pr + q = 0 são r1 e r2, com r1 6= r2, então todas as

soluções da recorrência xn+2+pxn+1+qxn = 0 são da forma an = C1rn1+C2r

n2 ,

C1 e C2 constantes.

Demonstração Seja yn uma solução qualquer de xn+2 + pxn+1 + qxn = 0. Determinemos

constantes C1 e C2 que sejam soluções do sistemas de equaçõesC1r1 + C2r2 = y1

C1r21 + C2r

22 = y2

isto é,

C1 =r22y1 − r2y2r1r2(r2 − r1)

e C2 =r1y2 − r21y1r1r2(r2 − r1)

.

Isso é possível pois r1 6= r2 e r1 6= 0 e r2 6= 0.

Armamos que yn = C1rn1 + C2r

n2 para todo n natural, o que provará o

teorema. Com efeito, seja zn = yn − C1rn1 − C2r

n2 . Mostraremos que zn = 0

para todo n. Temos

zn+2+pzn+1+qzn = (yn+2+pyn+1+qyn)−C1rn1 (r

21+pr1+q)−C2r

n2 (r

22+pr2+q).

O primeiro parêntese é igual a zero porque yn é solução de xn+2+pxn+1+qxn =

0; os dois últimos parênteses são iguais a zero porque r1 e r2 são raízes de

r2 + pr + q = 0. Então zn+2 + pzn+1 + qzn = 0.

Além disso, como C1r1+C2r2 = y1 e C1r21+C2r

22 = y2, temos z1 = z2 = 0.

Mas, se zn+2 + pzn+1 + qzn = 0 e z1 = z2 = 0, então zn = 0 para todo n.

4

Unidade 8Recorrências Lineares de Segunda Ordem

Exemplo 3Vamos determinar as soluções da recorrência

xn+2 + 3xn+1 − 4xn = 0.

A equação característica r2 + 3r − 4 = 0, tem raízes 1 e −4. De acordo

com os Teoremas 1 e 2, as soluções da recorrência são as sequências da forma

an = C11n+C2(−4)n, isto é, an = C1+C2(−4)n, onde C1 e C2 são constantes

arbitrárias.

Exemplo 4Fibonacci revisitado. Determinemos o número de Fibonacci Fn denido por

Fn+2 = Fn+1 + Fn, com F0 = F1 = 1.

A equação característica é r2 = r + 1 e as suas raízes são dadas por

r1 =1 +√5

2e r2 =

1−√5

2.

Então,

Fn = C1

(1 +√5

2

)n

+ C2

(1−√5

2

)n

.

Para determinar C1 e C2, basta usar F0 = F1 = 1.

Obtemos o sistema C1 + C2 = 1

C11+√5

2+ C2

1−√5

2= 1

Logo,

Fn =

√5 + 1

2√5

(1 +√5

2

)n

+

√5− 1

2√5

(1−√5

2

)n

,

isto é,

Fn =1√5

(1 +√5

2

)n+1

− 1√5

(1−√5

2

)n+1

.

Se as raízes da equação característica forem complexas, a solução an =

C1rn1 +C2r

n2 , C1 e C2 constantes arbitrárias pode ser escrita de modo a evitar

cálculos com complexos. Pondo as raízes na forma trigonométrica, teremos:

r1 = ρ(cos θ + i sen θ), r2 = ρ(cos θ − i sen θ)

5

Unidade 8 Recorrências de Segunda Ordem

rn1 = ρn(cosnθ + i sennθ), rn2 = ρn(cosnθ − i sennθ).

Logo,

C1rn1 + C2r

n2 = ρn[(C1 + C2) cosnθ + i(C1 − C2) sennθ].

É claro que C ′1 = C1 +C2 e C ′2 = i(C1−C2) são novas constantes e a solução

pode ser escrita

an = ρn[C ′1 cosnθ + C ′2 sennθ].

Exemplo 5 A recorrência xn+2+xn+1+xn = 0 tem equação característica r2+r+1 = 0,

cujas raízes são

r1 =1 + i

√3

2e r2 =

1− i√3

2,

que são complexas de módulo ρ = 1 e argumento principal θ = ±π3.

A solução é

xn = ρn[C1 cosnθ + C2 sennθ] = C1 cosnπ

3+ C2 sen

3.

O que aconteceria se as raízes da equação característica fossem iguais? Os

teoremas a seguir respondem essa pergunta.

Teorema 3 Se as raízes de r2 + pr + q = 0 são iguais, r1 = r2 = r, então, an =

C1rn+C2nr

n é solução da recorrência xn+2+ pxn+1+ qxn = 0, quaisquer que

sejam os valores das constantes C1 e C2.

Demonstração Se as raízes são iguais, então r = −p2. Substituindo an = C1r

n + C2nrn

na recorrência

xn+2 + pxn+1 + qxn = 0

obtemos, agrupando convenientemente os termos,

C1rn(r2 + pr + q) + C2nr

n(r2 + pr + q) + C2rnr(2r + p)

= C1rn0 + C2nr

n0 + C2rnr0 = 0.

6

Unidade 8Recorrências Lineares de Segunda Ordem

Teorema 4Se as raízes de r2 + pr + q = 0 são iguais, r1 = r2 = r, então todas as

soluções da recorrência xn+2 + pxn+1 + qxn = 0 são da forma C1rn + C2nr

n,

C1 e C2 constantes.

DemonstraçãoSeja yn uma solução qualquer de xn+2 + pxn+1 + qxn = 0. Determine

constantes C1 e C2 que sejam soluções do sistema de equações.C1r + C2r = y1

C1r2 + 2C2r

2 = y2

,

isto é,

C1 = 2y1r− y2r2

e C2 =y2 − ry1

r2.

Isso é possível pois r 6= 0.

Armamos que yn = C1rn + C2nr

n para todo n natural, o que provará o

teorema. Com efeito, seja zn = yn − C1rn − C2nr

n. Mostraremos que zn = 0

para todo n. Temos

zn+2 + pzn+1 + qzn = (yn+2 + pyn+1 + qyn)−− C1r

n(r2 + pr + q)− C2nrn(r2 + pr + q)− C2r

nr(2r + p).

O primeiro parêntese é igual a zero porque yn é solução de xn+2 + pxn+1 +

qxn = 0; o segundo e o terceiro parênteses são iguais a zero porque r é raiz

de r2 + pr + q = 0; o quarto é igual a zero porque 2r + p = 0 já que, quando

r1 = r2 = r, tem-se r = −p2. Então zn+2 + pzn+1 + qzn = 0

Além disso, como C1r+C2r = y1 e C1r2+2C2r

2 = y2, temos z1 = z2 = 0.

Mas, se zn+2 + pzn+1 + qzn = 0 e z1 = z2 = 0 então zn = 0 para todo n.

Exemplo 6A recorrência xn+2 − 4xn+1 + 4xn = 0 tem equação característica r2 −4r + 4 = 0. As raízes são r1 = r2 = 2 e a solução da recorrência é xn =

C12n + C2n2

n.

O teorema a seguir mostra um processo para resolver algumas recorrências

não-homogêneas.

7

Unidade 8 Recorrências de Segunda Ordem

Teorema 5 Se an é uma solução da equação

xn+2 + pxn+1 + qxn = f(n),

então a substituição xn = an + yn transforma a equação em

yn+2 + pyn+1 + qyn = 0.

Demonstração Substituindo xn por an + yn na equação, obtemos

(an+2 + pan+1 + qan) + (yn+2 + pyn+1 + qyn) = f(n).

Mas an+2+pan+1+ qan = f(n) pois an é a solução da equação original. Logo,

a equação se transformou em

yn+2 + pyn+1 + qyn = 0.

De acordo com o Teorema 5, a solução de uma recorrência não-homogênea

é constituída de duas parcelas: uma solução qualquer da não-homogênea e a

solução homogênea. A solução da homogênea, sabemos achar. Uma solução

da não-homogênea, procuraremos por tentativas.

Exemplo 7 A recorrência xn+2 − 6xn+1 + 8xn = n + 3n tem equação característica

r2 − 6r + 8 = 0, cujas raízes são r1 = 2 e r2 = 4. Portanto, a solução da

homogênea, isto é, de xn+2− 6xn+1+8xn = 0 é hn = C1+C24n. Tentaremos

agora descobrir uma solução particular, tn, da recorrência

xn+2 − 6xn+1 + 8xn = n+ 3n.

Ora, se substituirmos tn em xn+2−6xn+1+8xn devemos encontrar n+3n. Que

tipo de função deve ser tn? é bastante razoável imaginar que tn seja a soma de

um polinômio do primeiro grau com uma exponencial de base 3. Tentaremos

tn = An+B + C3n. Substituindo em

xn+2 − 6xn+1 + 8xn = n+ 3n,

8

Unidade 8Recorrências Lineares de Segunda Ordem

obtemos 3An+3B−4A−C3n = n+3n. tn terá solução se 3A = 1, 3B−4A = 0

e −C = 1. Logo,

A =1

3, B =

4

9e C = −1.

Daí,

tn =1

3n+

4

9− 3n.

Exemplo 8A recorrência xn+2 − 6xn+1 + 8xn = 1 + 2n tem equação característica

r2−6r+8 = 0, cujas raízes são r1 = 2 e r2 = 4. Portanto, a solução da equação

homogênea, isto é, de xn+2−6xn+1+8xn = 0 é hn = C12n+C24

n. Tentaremos

agora descobrir uma solução particular, tn da recorrência xn+2−6xn+1+8xn =

1 + 2n. Ora, se substituirmos tn em xn+2 − 6xn+1 + 8xn devemos encontrar

1 + 2n. Que tipo de função deve ser tn? é bastante razoável imaginar que

tn seja a soma de um polinômio constante com uma exponencial de base 2.

Tentaremos tn = A+B2n. Substituindo em

xn+2 − 6xn+1 + 8xn = 1 + 2n,

obtemos 3A = 1 + 2n. Essa igualdade é impossível. A recorrência não admite

solução da forma tn = A+B2n.

Parando para pensar no que aconteceu, vericamos que era óbvio que a

nossa tentativa não podia dar certo. O espírito da nossa tentativa era tentar

uma constante A para que obtivéssemos uma constante que igualaríamos a

1 e tentar B2n para gerar uma exponencial que pudéssemos igualar a 2n. É

claro que o termo B2n não poderia cumprir o seu papel. B2n é solução da

homogênea (é a solução da homogênea que é obtida pondo C1 = B e C2 = 0)

e, substituído da equação, daria zero e não uma exponencial que pudéssemos

igualar a 2n.

Vamos corrigir a nossa tentativa para tn = A + Bn2n. Sempre que na

nossa tentativa em algum bloco não cumprir o seu papel, fazemos a correção

aumentando o grau, isto é, multiplicando o bloco por n. Agora, substituindo

obtemos 3A−B4B2n = 1 + 2n.

Se 3A = 1 e −4B = 1, isto é,

A =1

3e B = −1

4,

9

Unidade 8 Recorrências de Segunda Ordem

temos a solução

tn =1

3− n2n

4.

A solução da recorrência é a soma de hn com tn. Portanto,

xn = C12n + C24

n +1

3− n2n

4.

10

Unidade 8Recorrências Lineares de Segunda Ordem

8.4 Exercícios recomendados

1. Resolva as recorrências a seguir:

a) xn+2 + 5xn+1 + 6xn = 0.

b) xn+2 + 6xn+1 + 9xn = 0.

c) xn+2 + 2xn+1 + 2xn = 0.

d) xn+2 − 5xn+1 + 6xn = n.

e) xn+2 − 5xn+1 + 6xn = 1 + 3 · 4n.

f) xn+2 − 5xn+1 + 6xn = 2n.

g) xn+2 − 5xn+1 + 6xn = n+ 3n.

h) xn+2 − 6xn+1 + 9xn = n− 3n.

i) xn+2 + xn = 1.

j) xn+2 − 6xn+1 + 9xn = 1 + n3n.

2. Resolva as recorrências a seguir:

a) xn+2 + 5xn+1 + 6xn = 0; x0 = 3; x1 = −6.

b)xn+2 + xn+1 − 6xn = 6− 8n; x0 = 1; x1 = 4.

c)xn+2 − 4xn+1 + 4xn = 2n+3; x0 = 3; x1 = 6.

3. Quantas são as sequências de n termos, todos pertencentes a 0, 1, 2,que não possuem dois termos consecutivos iguais a 0?

4. Determine o número de modos de cobrir um tabuleiro 2×n com dominós

2× 1 iguais.

11

Unidade 8 Exercícios recomendados

5. Uma planta é tal que cada uma de suas sementes produz, um ano após

ter sido plantada, 21 novas sementes e, a partir daí, 44 novas sementes

a cada ano. Se plantarmos hoje uma semente e se, toda vez que uma

semente for produzida ela for imediatamente plantada, quantas sementes

serão produzidas daqui a n anos?

6. O salário de Carmelino no mês n é Sn = a+ bn. Sua renda mensal é for-

mada pelo salário e pelos juros de suas aplicações nanceiras. Ele poupa

anualmente 1/p de sua renda e investe sua poupança a juros mensais de

taxa i. Determine a renda de Carmelino no mês n.

7. Cinco times de igual força disputarão todo ano um torneio. Uma taça

será ganha pelo primeiro time que vencer três vezes consecutivas. Qual

a probabilidade da taça não ser ganha nos n primeiros torneios?

8. Em um jogo, em cada etapa Olavo, pode fazer 1 ou 2 pontos. De quantos

modos ele pode totalizar n pontos?

9. Mostre que

2√5 + 1

2√5

(1−√5)n +

2√5− 1

2√5

(1 +√5)n

é, para todo natural n, um número inteiro.

10. Mostre que a parte inteira de (1 +√3)2n+1 é sempre par.

12

Referências Bibliográcas

[1] Carmo, Manfredo P.; Morgado, Augusto C., Wagner, Eduardo & Pitom-

beira, João Bosco. Trigonometria e Números Complexos. Rio de Janeiro:

SBM, Coleção Professor de Matemática.

[2] Eves, Howard. An Introduction to the History of Mathematics. New York:

Holt, Rinehart and Winston, 1964.

[3] Figueiredo, Djairo G. Análise I Rio de Janeiro: LTC, 1996.

[4] Figueiredo, Djairo G. Números Irracionais e Transcedentes Rio de Janeiro:

SBM, Coleção Iniciação Cientíca.

[5] Halmos, Paul. Naive Set Theory. New York: Springer, 1974.

[6] Hefez, Abramo e Fernandez, Cecília de Souza. Introdução à Álgebra Linear.

Rio de Janeiro: SBM, Coleção PROFMAT, 2012.

[7] Fernandes, C. S. Hefez, A. Introdução à Álgebra Linear. SBM, Coleção

PROFMAT. 2

[8] Lima, Elon Lages. Coordenadas no Espaço. Rio de Janeiro: SBM, Coleção

Professor de Matemática.

[9] Lima, Elon Lages. Curso de Análise, Vol. 1. Rio de Janeiro: SBM, Projeto

Euclides, 1976.

[10] Lima, Elon Lages. Logaritmos. Rio de Janeiro: SBM, Coleção Professor de

Matemática.

[11] Lima, Elon Lages. Meu Professor de Matemática e Outras Histórias. Rio

de Janeiro: SBM, Coleção Professor de Matemática.

bibitemelon-analisereal Lima, Elon Lages. Análise Real, Vol. 1. Rio de Janeiro:

IMPA, Coleção Matemática Universitária.

13

9

1

Matemática Financeira

Sumário

9.1 Introdução . . . . . . . . . . . . . . . . . . . . . . . 2

9.2 Juros Compostos . . . . . . . . . . . . . . . . . . . . 2

9.3 A Fórmula das Taxas Equivalentes . . . . . . . . . . 7

9.4 Exercícios Recomendados . . . . . . . . . . . . . . . 9

Unidade 9 Introdução

9.1 Introdução

Nesta unidade e na próxima, serão apresentados rudimentos de Matemática

Financeira, cuja inspiração vem da vida real. Esse conhecimento é fundamental

em sociedades de consumo, como a nossa, e deve fazer parte da bagagem

cultural de todo cidadão que nelas vive para que saiba defender minimamente

os seus interesses.

Quotidianamente, estamos frente a problemas práticos, tais como se deve-

mos ou não parcelar uma compra e, se for o caso, em quantas parcelas? Se

devemos ou não antecipar o pagamento de uma dívida, usando o décimo ter-

ceiro salário? Esses são desaos que, se resolvidos corretamente, nos auxiliam

a tomar decisões que podem proporcionar uma boa economia.

A ferramenta matemática básica que é utilizada nesse tipo de questões são

as progressões geométricas, bastando, para resolvê-las, modelar corretamente

cada problema.

O assunto principal de que tratamos é o cálculo de juros em diversas si-

tuações decorrentes da operação de empréstimo, seja em aplicações (quando

emprestamos), seja em compras a crédito (quando tomamos emprestado).

Esta unidade repousa sobre um resultado (teorema) fundamental que nos

diz como se transforma um capital inicial quando aplicado por um período de

tempo, sendo submetido a um regime de juros compostos.

9.2 Juros Compostos

Uma das importantes aplicações de progressões geométricas é a Matemá-

tica Financeira. A operação básica da matemática nanceira é a operação de

empréstimo.

Alguém que dispõe de um capital C (chamado de principal ), empresta-o

a outrem por um certo período de tempo, e após esse período, recebe o seu

capital C e volta, acrescido de uma remuneração J pelo empréstimo. Essa

remuneração é chamada de juro. A soma C+J é chamada de montante e será

representada por M . A razão i =J

Cque é a taxa de crescimento do capital,

será sempre referida ao período da operação e chamada de taxa de juros.

2

Unidade 9Matemática Financeira

Exemplo 1Lúcia tomou um empréstimo de R$ 100,00. Dois meses após, pagou R$

140,00. Os juros pagos por Lúcia são de R$ 40,00 e a taxa de juros é de40

100= 0, 40 = 40% ao bimestre. O principal, que é a dívida inicial de Lúcia,

é igual a R$ 100,00; o montante, que é a dívida na época do pagamento, é de

R$ 140,00.

Exemplo 2Manuel tomou um empréstimo de 100 reais, a juros de taxa 10% ao mês.

Após um mês, a dívida de Manuel será acrescida de 0, 10 × 100 reais de juros

(pois J = iC), passando a 110 reais. Se Manuel e seu credor concordarem em

adiar a liquidação da dívida por mais um mês, mantida a mesma taxa de juros,

o empréstimo será quitado, dois meses depois de contraído, por 121 reais, pois

os juros relativos ao segundo mês serão de 0, 10× 110 reais = 11 reais. Esses

juros assim calculados são chamados de juros compostos. Mais precisamente, no

regime de juros compostos, os juros em cada período são calculados, conforme

é natural, sobre a dívida do início desse período.

As pessoas menos educadas matematicamente têm tendência a achar que

juros de 10% ao mês dão em dois meses juros de 20%. Note que juros de 10%

ao mês dão em dois meses de juros de 21%.

Teorema 1No regime de juros compostos de taxa i, um principal C0 transforma-se,

depois de n períodos de tempo, em um montante Cn = C0(1 + i)n.

DemonstraçãoBasta observar que os valores do capital crescem a uma taxa constante i

e, portanto, formam uma progressão geométrica de razão 1 + i.

Exemplo 3Pedro investe 150 reais a juros de 12% ao mês. Qual será o montante de

Pedro três meses depois?

Solução. C3 = C0(1 + i)3 = 150(1 + 0, 12)3 = 210, 74 reais.

É importante perceber que o valor de uma quantia depende da época à qual

ela está referida. Se eu consigo fazer com que meu dinheiro renda 10% ao mês,

3

Unidade 9 Juros Compostos

é-me indiferente pagar agora R$ 100,00 ou pagar R$ 110,00 daqui a um mês.

é mais vantajoso pagar R$ 105,00 daqui a um mês do que pagar R$ 100,00

agora. é mais vantajoso pagar R$ 100,00 agora do que pagar R$ 120,00 daqui

a um mês.

No fundo, só há um único problema de Matemática Financeira: deslocar

quantias no tempo.

Outro modo de ler o Teorema 1, Cn = C0(1+ i)n, é que uma quantia, hoje

igual a C0, transformar-se-á, depois de n períodos de tempo, em uma quantia

igual a C0(1 + i)n. Isto é, uma quantia, cujo valor atual é A, equivalerá no

futuro, depois de n períodos de tempo, a F = A(1 + i)n.

Essa é a fórmula fundamental da equivalência de capitais: Para obter o valor

futuro, basta multiplicar o atual por (1 + i)n. Para obter o valor atual, basta

dividir o futuro por (1 + i)n.

O exemplo a seguir é, pode-se dizer, um resumo de todos os problemas de

Matemática Financeira.

Exemplo 4 Pedro tomou um emprétimo de 300 reais, a juros de 15% ao mês. Dois

meses após, Pedro pagou 150 reais e, um mês após esse pagamento, Pedro

liquidou seu débito. Qual o valor desse último pagamento?

Solução. Os esquemas de pagamento abaixo são equivalentes. Logo, 300 reais,

na data 0, têm o mesmo valor de 150 reais dois meses após, mais um paga-

mento igual a P , na data 3.

Figura 9.1: Esquemas de pagamento

Igualando os valores, na mesma época (0, por exemplo), dos pagamentos nos

dois esquemas, obtemos

300 =150

(1 + 0, 15)2=

p

(1 + 0, 15)3.

4

Unidade 9Matemática Financeira

daí, P = 283, 76. O último pagamento foi de R$ 283,76.

Exemplo 5Pedro tem duas opções de pagamento na compra de um televisor:

i) três prestações mensais de R$ 160,00 cada;

ii) sete prestações mensais de R$ 70,00 cada.

Em ambos os casos, a primeira prestação é paga no ato da compra. Se o

dinheiro vale 2% ao mês para Pedro, qual a melhor opção que Pedro possui?

Solução. Para comparar, determinaremos o valor dos dois conjuntos de paga-

mentos na mesma época, por exemplo na época 2. Os esquemas de pagamentos

são:

Figura 9.2: Esquemas de pagamento

Para comparar, determinaremos o valor dos dois conjuntos de pagamentos na

mesma época. Por exemplo, na época 2, temos,

a = 60(1 + 0, 02)2 + 160(1 + 0, 02) + 160 = 489, 66

b = 70(1 + 0, 02)2 + 70(1 + 0, 02) + 70 +70

1 + 0, 02

+70

(1 + 0, 02)2+

70

(1 + 0, 02)3+

70

(1 + 0, 02)4= 480, 77.

Pedro deve preferir o pagamento em seis prestações.

é um absurdo que muitas pessoas razoavelmente instruídas achem que o

primeiro esquema é melhor pois o total pago é de R$ 480,00 ao passo que no

segundo esquema o total pago é de R$ 490,00.

Para xar, faremos mais alguns exemplos.

5

Unidade 9 Juros Compostos

Exemplo 6 Pedro tem três opções de pagamento na compra de vestuário.

i) à vista, com 30% de desconto.

ii) em duas pretações mensais iguais, sem desconto, vencendo a primeira um

mês após a compra.

iii) em três prestações mensais iguais, sem desconto, vencendo a primeira no

ato da compra.

Qual a melhor opção para Pedro, se o dinheiro vale, para ele, 25% ao mês?

Solução. Fixando o preço do bem em 30, temos os três esquemas abaixo

Figura 9.3: Esquemas de pagamento

Comparando os valores, por exemplo, na época 0, obtemos:

a = 21

b =15

1 + 0, 25+

15

(1 + 0, 25)2= 21.6

c = 10 +10

1 + 0, 25+

10

(1 + 0, 25)2= 24, 4.

A melhor alternativa é a primeira e a pior é a em três prestações.

Exemplo 7 Uma loja oferece duas opções de pagamento:

i) à vista, com 30% de desconto.

ii) em duas prestações mensais iguais, sem desconto, a primeira prestação sendo

paga no ato da compra.

Qual a taxa mensal dos juros embutidos nas vendas a prazo?

6

Unidade 9Matemática Financeira

Solução. Fixando o valor do bem em 100, temos os esquemas de pagamentos

abaixo:

Figura 9.4: Esquemas de pagamento

Igualando os valores, por exemplo, na época 0 (a data usada nessas comparações

é chamada de data focal), obtemos 70 = 50 +50

1 + i. Daí, i = 1, 5 = 150%. A

loja cobra 150% ao mês nas vendas a prazo.

Exemplo 8Investindo seu capital a juros mensais de 8%, em quanto tempo você

dobrará o seu capital inicial?

Solução. Temos C0(1 + 0, 08)n = 2C0. Daí,

1, 08n = 2 e n =log 2

log 1, 08∼= 9

Em aproximadamente nove meses você dobrará o seu capital inicial.

9.3 A Fórmula das Taxas Equivalentes

Um importante resultado que já foi obtido na Unidade 6 e será repetido é a

Fórmula das taxas equivalentes. Se a taxa de juros relativamente a um

determinado período de tempo é igual a i, a taxa de juros relativamente a n

períodos de tempo é I tal que 1 + I = (1 + i)n.

Exemplo 9A taxa anual de juros equivalente a 12% ao mês é I tal que 1 + I =

(1 + 0, 12)12. Daí, I ∼= 2, 90 = 290% ao ano.

7

Unidade 9 A Fórmula das Taxas Equivalentes

Um erro muito comum é achar que juros de 12% ao mês equivalem a juros

anuais de 12× 12% = 144% ao ano. Taxas como 12% ao mês e 144% ao ano

são chamadas de taxas proporcionais, pois a razão entre elas é igual à razão

dos períodos aos quais elas se referem.

Taxas proporcionais não são equivalentes. Um (péssimo) hábito em Ma-

temática Financeira é o de anunciar taxas proporcionais como se fossem equi-

valentes. Uma frase como 144% ao ano, com capitalização mensal signica

que a taxa usada na operação não é a taxa de 144% anunciada e sim a taxa

mensal que lhe é proporcional.

Portanto, a tradução da expressão 144% ao ano, com capitalização mensal

é 12% ao mês. As pessoas menos educadas matematicamente podem pensar

que os juros sejam realmente de 144% ao ano, mas isso não é verdade. Como

vimos no Exemplo 9, os juros são de 290% ao ano.

A taxa de 144% ao ano é chamada de taxa nominal e a taxa de 290% ao

ano é chamada de taxa efetiva.

Exemplo 10 24% ao ano com capitalização semestral signica 12% ao semestre; 1%

ao mês com capitalização trimestral signica 3% ao trimestre e 6% ao ano

com capitalização mensal signica 0,5% ao mês.

Exemplo 11 Verônica investe seu dinheiro a juros de 6% ao ano com capitalização

mensal. Qual a taxa anual de juros à qual está investido o capital de Verônica?

Solução. O dinheiro de Verônica está investido a juros de taxa i = 0, 5% ao mês.

A taxa anual equivalente a I tal que 1+I = (1+i)12. Daí, I = 0, 0617 = 6, 17%

ao ano. A taxa de 6% ao ano é nominal e a taxa de 6,17% ao ano é efetiva.

Exemplo 12 A taxa efetiva semestral correspondente a 24% ao semestre com capitali-

zação mensal é I tal que 1 + I = (1 + 0, 04)6. Daí, I = 26, 53% ao semestre.

8

Unidade 9Matemática Financeira

9.4 Exercícios Recomendados

1. Investindo R$ 450,00 você retira, após 3 meses, R$ 600,00. A que taxa

mensal de juros rendeu seu investimento?

2. Determine as taxas mensais equivalentes a 100% ao ano e a 39% ao trimes-

tre.

3. Determine as taxas anuais equivalentes a 6% ao mês e a 12% ao trimestre.

4. Determine as taxas efetivas anuais equivalente a:

a) 30% ao ano, com capitalização mensal.

b) 30% ao ano, com capitalização trimestral.

c) i ao ano, capitalizados k vezes ao ano.

5. Qual o limite, quando k tende para innito, da resposta ao item c) do

problema anterior? Neste caso diz-se que os juros estão sendo capitalizados

continuamente e i é chamado de taxa instantânea de juros.

6. Use a resposta do problema anterior para dar uma denição nanceira do

número e.

7. Determine

a) a taxa efetiva trimestral equivalente a 12% ao trimestre com capitalização

contínua.

b) a taxa instantânea anual equivalente à taxa efetiva anual de 60%.

c) a taxa instantânea semestral equivalente à taxa efetiva anual de 60%.

8. A Mesbla, em vários natais, ofereceu a seus clientes duas alternativas de

pagamento:

a) pagamento de uma só vez, um mês após a compra.

b) pagamento em três prestações mensais iguais, vencendo a primeira no ato

da compra.

Se você fosse cliente da Mesbla, qual seria a sua opção?

9. O Foto Studio Sonora convidou, em dezembro de 1992, os seus clientes

a liquidarem suas prestações mensais vincendas, oferecendo-lhes em troca um

desconto. O desconto seria dado aos que pagassem, de uma só vez, todas as

prestações a vencer em mais de 30 dias, e seria de 30%, 40% ou 50%, conforme

9

Unidade 9 Exercícios Recomendados

fossem pagas uma, duas ou três prestações. Supondo que o dinheiro valia 27%

ao mês, a oferta era vantajosa?

10. Lúcia comprou um exaustor, pagando R$ 180,00, um mês após a compra

e R$ 200,00, dois meses após a compra. Se os juros são de 25% sobre o saldo

devedor, qual é o preço à vista?

11. Uma geladeira custa R$ 1 000,00 à vista e pode ser paga em três prestações

mensais iguais. Se são cobrados juros de 6% ao mês sobre o saldo devedor,

determine o valor da prestação, supondo que a primeira prestação é paga:

a) no ato da compra;

b) um mês após a compra;

c) dois meses após a compra.

12. Ângela tomou um empréstimo de R$ 400,00, por dez meses. Os juros

foram de 3% ao mês durante os quatro primeiros meses, de 5% ao mês durante

os cinco meses seguintes e de 9% ao mês no último mês. Calcule:

a) a taxa média de juros.

b) o montante pago.

10

Referências Bibliográcas

[1] Carmo, Manfredo P.; Morgado, Augusto C., Wagner, Eduardo & Pitom-

beira, João Bosco. Trigonometria e Números Complexos. Rio de Janeiro:

SBM, Coleção Professor de Matemática.

[2] Eves, Howard. An Introduction to the History of Mathematics. New York:

Holt, Rinehart and Winston, 1964.

[3] Figueiredo, Djairo G. Análise I Rio de Janeiro: LTC, 1996.

[4] Figueiredo, Djairo G. Números Irracionais e Transcedentes Rio de Janeiro:

SBM, Coleção Iniciação Cientíca.

[5] Halmos, Paul. Naive Set Theory. New York: Springer, 1974.

[6] Hefez, Abramo e Fernandez, Cecília de Souza. Introdução à Álgebra Linear.

Rio de Janeiro: SBM, Coleção PROFMAT, 2012.

[7] Fernandes, C. S. Hefez, A. Introdução à Álgebra Linear. SBM, Coleção

PROFMAT. 2

[8] Lima, Elon Lages. Coordenadas no Espaço. Rio de Janeiro: SBM, Coleção

Professor de Matemática.

[9] Lima, Elon Lages. Curso de Análise, Vol. 1. Rio de Janeiro: SBM, Projeto

Euclides, 1976.

[10] Lima, Elon Lages. Logaritmos. Rio de Janeiro: SBM, Coleção Professor de

Matemática.

[11] Lima, Elon Lages. Meu Professor de Matemática e Outras Histórias. Rio

de Janeiro: SBM, Coleção Professor de Matemática.

bibitemelon-analisereal Lima, Elon Lages. Análise Real, Vol. 1. Rio de Janeiro:

IMPA, Coleção Matemática Universitária.

11

10

1

Matemática Financeira(continuação)

Sumário

10.1 Introdução . . . . . . . . . . . . . . . . . . . . . . . 2

10.2 Renda Perpétua . . . . . . . . . . . . . . . . . . . . 2

10.3 Sistemas de Amortização . . . . . . . . . . . . . . . 6

10.4 Exercícios Recomendados . . . . . . . . . . . . . . . 12

10.5 Exercícios Suplementares . . . . . . . . . . . . . . . 13

Unidade 10 Introdução

10.1 Introdução

Nesta unidade, continuaremos o estudo de Matemática Financeira iniciado

na Unidade 9 e que se encerrará na próxima unidade. Os principais resultados

dessa unidade analisam essencialmente três tipos de empréstimos, geralmente

de longo prazo como, por exemplo, nanciamentos da casa própria ou de bens

duráveis.

O primeiro tipo de empréstimo, refere-se à situação em que a taxa de juros

é pré-xada e o valor da prestação também. O Teorema 2 fornece uma fórmula

que permite saber quanto da dívida foi pago após n pagamentos (amortização

da dívida).

Cada parcela paga de um empréstimo consiste de duas partes: uma se refere

ao pagamento dos juros e a outra se refere ao abatimento do principal da dívida,

chamada de amortização.

O segundo tipo de empréstimo estudado é o Sistema de Amortização Cons-

tante (SAC), em que a parte da prestação que visa amortizar a dívida é cons-

tante. O Teorema 4 permite calcular a cada mês o valor da prestação especi-

cando o valor da amortização (constante), o valor da parcela relativa aos juros

(variável) e, nalmente, o estado atual da dívida (no caso da pessoa querer

quitar a dívida, por exemplo).

O terceiro tipo de empréstimo é o Sistema Francês ou Tabela Price, em que

as prestações e a taxa de juros são constantes. O Teorema 5 fornece fórmulas

para calcular, mês a mês, o valor da prestação (constante), a parcela relativa à

amortização do principal, a parcela relativa aos juros pagos e o estado atual da

dívida.

Os cálculos nanceiros podem se complicar bastante em presença de forte

inação, como foi o caso no Brasil alguns anos atrás.

10.2 Renda Perpétua

Um conjunto de quantias (chamadas usualmente de pagamentos ou termos),

referidas a épocas diversas, é chamada de série, ou de anuidade (apesar no

nome, nada a ver com ano) ou, ainda, renda. Se esses pagamentos forem iguais

e igualmente espaçados no tempo, a série é dita uniforme.

2

Unidade 10Matemática Financeira (continuação)

Teorema 2O valor de uma série uniforme de n pagamentos iguais a P , um tempo antes

do primeiro pagamento, é, sendo i a taxa de juros, igual a A = P1− (1 + i)−n

i.

Demonstração

Figura 10.1: Série uniforme

O valor da série na época 0 é

A =P

1 + i+

P

(1 + i)2+

P

(1 + i)3+ · · ·+ P

(1 + i)n,

que é a soma de n termos de uma progressão geométrica. temos

A =P

1 + i

1−(

11+i

)n

1− 11+i

= P1− (1 + i)−n

i.

O corolário seguinte trata do valor de uma renda perpétua. Rendas perpé-

tuas aparecem em locações. Com efeito, quando se aluga um bem, cede-se a

posse do mesmo em troca de um aluguel, digamos, mensal. Então, o conjunto

dos aluguéis constitui uma renda perpétua ou perpetuidade.

Corolário 3O valor de uma perpetuidade de termos iguais a P , um tempo antes do

primeiro pagamento, é, sendo i a taxa de juros, igual aP

i.

DemonstraçãoBasta fazer n tender para innito no Teorema 2.

3

Unidade 10 Renda Perpétua

Exemplo 1 Um bem, cujo preço é R$ 120,00, é vendido em 8 prestações mensais iguais,

a primeira sendo paga um mês após a compra. Se os juros são de 8% ao mês,

determine o valor das prestações.

Solução. Um pequeno comentário: essas prestações são ditas postecipadas,

pois a primeira prestação só é paga um tempo depois da compra.

Figura 10.2:

Igualando os valores na época 0 (essa é a escolha natural da data de comparação:

um tempo antes do primeiro termo da série), obtemos:

120 = P1− (1 + 0, 08)−8

0, 08

P = 1200, 08

1− 0, 08−8= 20, 88.

As prestações são de R$ 20,88.

Exemplo 2 Um bem, cujo preço à vista é R$ 120,00, é vendido em 6 prestações mensais

iguais, antecipadas (isto é, a primeira é paga no ato da compra). Se os juros

são de 10% ao mês, determine o valor das prestações.

4

Unidade 10Matemática Financeira (continuação)

Figura 10.3: Comparando séries

Igualando os valores na época −1 (essa escolha, que pode parecer exótica,

é muito conveniente pois dispomos de uma fórmula que calcula diretamente o

valor da série nessa época), obtemos:

120

1 + 0, 1= P

1− (1 + 0, 1)−6

0, 1

P ∼= 25, 05.

Exemplo 3Se o dinheiro vale 1% ao mês, por quanto deve ser alugado um imóvel que

vale 40 mil reais?

Solução. Quando você aluga um imóvel, você cede a posse do imóvel em troca

de uma renda perpétua cujos termos são iguais ao valor do aluguel. Então,

o valor do imóvel deve ser igual ao valor do conjunto de aluguéis. Temos, de

acordo com o Corolário 3,

40 =P

i=

P

0, 01= 40× 0, 01 = 0, 4 mil reais.

Exemplo 4Helena tem duas alternativas para obter uma copiadora:

a) Alugá-la por 35 ao ano. Nesse caso, o locador se responsabiliza pelas despesas

de manutenção.

b) Comprá-la por 150. Nesse caso, já que a vida econômica da copiadora é de

5 anos, Helena venderá a copiadora após 5 anos. O valor residual da copiadora

5

Unidade 10 Sistemas de Amortização

após 5 anos é de 20. As despesas de manutenção são de responsabilidade de

Helena e são de 5 por ano, nos dois primeiros anos e de 8 por ano, nos anos

seguintes. Se o dinheiro vale 7% ao ano, qual a melhor opção?

Solução. Vamos tomar receitas como positivas e despesas como negativas.

Na segunda alternativa, o uxo de caixa de Helena será:

Figura 10.4: Alternativa (a)

Vamos determinar o uxo uniforme equivalente.

Figura 10.5: Alternativa (b)

Igualando os valores na época 0, obtemos

−150− 5

1, 07− 5

1, 072− 8

1, 073− 8

1, 074+

12

1, 075= P

1− 1, 07−5

0, 07.

Daí, P = −39, 78. Comprar a copiadora é equivalente a ter um custo anual

de 39,78. Como o aluguel corresponde a um custo anual de 35,00, a melhor

alternativa para Helena é alugar.

10.3 Sistemas de Amortização

Quando um banco empresta dinheiro (crédito pessoal ou desconto de dupli-

catas), o tomador do empréstimo emite uma nota promissória, que é um papel

6

Unidade 10Matemática Financeira (continuação)

no qual o tomador se compromete a pagar ao banco, em uma data xada, uma

certa quantia, que é chamada de valor de face da promissória.

O banco então desconta a promissória para o cliente, isto é, recebe a pro-

missória de valor de face F e entrega ao cliente uma quantia A (menor que F ,

naturalmente). A diferença F − A é chamada de desconto.

Os bancos efetuam o desconto de acordo com a fórmula A = F (1− d . t),

onde d é uma taxa xada pelo banco e chamada de taxa de desconto bancário

(ou taxa de desconto simples por fora) e t é o prazo da operação, medido na

unidade de tempo a que se refere a taxa.

Exemplo 5Pedro desconta uma promissória de valor 100, com vencimento em 60 dias,

em um banco cuja taxa de desconto é de 12% ao mês.

a) Quanto Pedro receberá?

b) Qual a taxa mensal de juros que Pedro está pagando?

Solução. Ora, A = F (1− dt) = 100(1− 0, 12 . 2) = 76.

Logo, Pedro receberá agora 76, para pagar 100 em 60 dias.

Se i é a taxa mensal de juros à qual cresce a dívida de Pedro, temos 100 =

76(1 + i)2. Daí, i = 0, 1471 = 14, 71%.

Observe que anunciar a taxa de desconto e não a taxa de juros é um modo sutil

de fazer crer aos mais ingênuos estarem eles pagando juros menores que os que

realmente lhes estão sendo cobrados.

Quando se paga parceladamente um débito, cada pagamento efetuado tem

dupla nalidade. Uma parte do pagamento quita os juros e outra parte amortiza

(abate) a dívida.

Exemplo 6Pedro tomou um empréstimo de 100, a juros mensais de taxa 10%. Quitou-

-o em três meses, pagando a cada mês os juros devidos e amortizando 30% da

dívida no primeiro mês e 30% e 40% nos dois meses seguintes.

Na planilha abaixo, Ak, Jk, Pk e Dk são, respectivamente, a parcela de

amortização, a parcela de juros, a prestação e o estado da dívida (isto é, o valor

da dívida após o pagamento da prestação) na época k.

7

Unidade 10 Sistemas de Amortização

k Pk Ak Jk Dk

0 − − − 100

1 40 30 10 70

2 37 30 7 40

3 44 40 4 −

Para facilitar a compreensão, olhe cada linha na ordem Ak, Dk, Jk e Pk.

Os sistemas usuais de amortização são o sistema de amortização constante

(SAC) e o sistema francês de amortização, também chamado de Tabela Price

(Richard Price foi um economista inglês). O sistema francês é caracterizado

por prestações constantes.

Exemplo 7 Uma dívida de 100 é paga, com juros de 15% ao mês, em 5 meses, pelo

SAC. Faça a planilha de amortização.

Solução. Como as amortizações são iguais, cada amortização será de1

5da

dívida inicial.

A planilha é, portanto:

k Pk Ak Jk Dk

0 − − − 100

1 35 20 15 80

2 32 20 12 60

3 29 20 9 40

4 26 20 6 20

5 23 20 3 −

Para facilitar a compreensão, olhe cada linha na ordem Ak, Dk, Jk e Pk.

Teorema 4 No SAC, sendo n o número de pagamentos e i a taxa de juros, temos

Ak =D0

n, Dk =

n− k

nD0 , Jk = iDk−1, Pk = Ak + Jk.

Demonstração Se a dívida D0 é amortizada em n quotas iguais, cada quota é igual a

Ak =D0

n.

8

Unidade 10Matemática Financeira (continuação)

O estado da dívida, após k amortizações, é

Dk = D0 − kD0

n=

n− k

nD0.

As duas últimas fórmulas são óbvias.

Exemplo 8Uma dívida de 150 é paga, em 4 meses, pelo sistema francês, com juros de

8% ao mês. Faça a planilha de amortização.

No sistema francês, as prestações são constantes. Pelo Teorema 4, cada

prestação vale

P = D0i

1− (1 + n)−n= 150

0, 08

1− 1, 08−4= 45, 29.

k Pk Ak Jk Dk

0 − − − 150, 00

1 45, 29 33, 29 12, 00 116, 71

2 45, 29 35, 95 9, 34 80, 76

3 45, 29 38, 83 6, 46 41, 93

4 45, 29 41, 93 3, 35 −

Para mais fácil compreensão, olhe cada linha na ordem Pk, Jk, Ak e Dk.

Teorema 5No sistema francês de amortização, sendo n o número de pagamentos e i

a taxa de juros, temos

Pk = D0i

1− (1 + i)−n,

Dk = D01− (1 + i)−(n−k)

1− (1 + i)−n,

Jk = iDk−1, A = Pk − Jk.

9

Unidade 10 Sistemas de Amortização

Demonstração A primeira fórmula é simplesmente o Teorema 4 e as duas últimas fórmulas

são óbvias. Quanto à segunda fórmula, observe que Dk é a dívida que será

liquidada, postecipadamente, por n−k pagamentos sucessivos a Pk. Portanto,

novamente pelo Teorema 4, temos

Dk = Pk1− (1 + i)−(n−k)

i.

Substituindo o valor de Pk, obteremos a segunda fórmula.

Exemplo 9 Em um mês cuja inação foi de 25%, Paulo Jorge investiu seu capital a

juros de 30% ao mês. Evidentemente, isso não signica que Paulo Jorge tenha

aumentado seu poder de compra em 30%, pois, embora a quantidade de reais de

Paulo Jorge tenha crescido 30%, o valor do real sofreu uma redução. Dizemos

nesse caso que 30% ao mês é a taxa nominal de juros mensais de Paulo Jorge.

Suponhamos que, no início do referido mês, o capital C de Paulo Jorge

pudesse comprar x artigos de preço unitário igual a p. No m do mês, o capital

passou a ser 1, 3C e o preço unitário passou a ser 1, 25p. Logo, Paulo Jorge

poderá agora comprar1, 3C

1, 25p= 1, 04x artigos.

O poder de compra de Paulo Jorge aumentou de 4% nesse mês.

Essa taxa de 4% ao mês, à qual cresceu o poder de compra de Paulo Jorge,

é chamada de taxa real de juros.

Exemplo 10 Em algumas situações (prazos pequenos, juros de mora) são usados juros

simples e não juros compostos. No regime de juros simples, os juros em cada

época são calculados sobre o principal e não sobre o montante da época anterior.

Por exemplo, um principal igual a 100, a juros simples de 10% ao mês evolui

de acordo com a tabela abaixo:

n 0 1 2 3 4 . . .

Cn 100 110 120 130 140 . . .

Não há diculdade em calcular juros simples pois a taxa incide sempre sobre o

capital inicial. No nosso exemplo, os juros são sempre de 10% de 100, ou seja,

10.

10

Unidade 10Matemática Financeira (continuação)

É claro então que, Cn = C0 + niC0, o que faz com que os valores de Cn

formem uma progressão aritmética.

Olhando para os grácos de evolução de um mesmo principal C0 a juros de

taxa i, a juros simples e a juros compostos, observamos que o montante a juros

compostos é superior ao montante a juros simples, exceto se o prazo for menor

que 1. É por isso que juros simples só são utilizados em cobranças de juros em

prazos inferiores ao prazo ao qual se refere a taxa de juros combinada.

Figura 10.6: Comparando juros

11

Unidade 10 Exercícios Recomendados

10.4 Exercícios Recomendados

1. Um televisor, cujo preço à vista é de R$ 400,00, é vendido em dez presta-

ções mensais iguais. Se são pagos juros de 6% ao mês sobre o saldo devedor,

determine o valor das prestações, supondo a primeira prestação paga:

a) no ato da compra.

b) um mês após a compra.

c) dois meses após a compra.

2. Se a taxa corrente de juros é de 0,6% ao mês, por quanto se aluga um

imóvel cujo preço a vista é R$ 50 000,00, supondo:

a) o aluguel mensal pago vencido?

b) o aluguel mensal pago adiantadamente?

3. Supondo juros de 0,5% ao mês, quanto você deve investir mensalmente,

durante 30 anos, para obter ao m desse prazo, por 30 anos, uma renda mensal

de R$ 100,00?

4. Supondo juros de 0,5% ao mês, quanto você deve investir mensalmente,

durante 35 anos, para obter, ao m desse prazo, uma renda perpétua de R$

100,00.

5. Faça as planilhas de amortização de uma dívida de R$ 3 000,00, em 8

pagamentos mensais, com juros de 10% ao mês:

a) pela tabela Price.

b) pelo SAC.

6. Leigh investiu 30% do seu capital a juros de 10% ao mês e os 70% restantes

a 18% ao mês. Qual a taxa média de juros obtidas?

7. Laura quer comprar um violão em uma loja que oferece um desconto de

30% nas compras à vista ou pagamento em três prestações mensais, sem juros

e sem desconto. Determine a taxa mensal de juros embutida nas vendas a

prazo, supondo o primeiro pagamento:

a) no ato da compra.

b) um mês após a compra.

c) dois meses após a compra.

8. Regina tem duas opções de pagamento:

12

Unidade 10Matemática Financeira (continuação)

a) à vista, com x% de desconto.

b) em duas prestações mensais iguais, sem juros, vencendo a primeira um mês

após a compra.

Se o dinheiro vale 5% ao mês, para que valores de x ela preferirá a segunda

alternativa?

9. Um banco efetua descontos à taxa de 6% ao mês. Qual a taxa mensal de

juros cobrada pelo banco nas operações:

a) de um mês?

b) de dois meses?

c) de três meses?

10. Um banco efetua descontos à taxa de 6% ao mês, mas exige que 20% do

valor efetivamente liberado sejam aplicados no próprio banco, a juros de 2% ao

mês. Essa é a chamada reciprocidade. Qual a taxa mensal de juros paga pelos

tomadores de empréstimo por dois meses?

10.5 Exercícios Suplementares

1. No cálculo de juros, considera-se sempre o ano comercial de 360 dias, ou

seja, 12 meses de 30 dias. Essa é a chamada regra dos banqueiros. Os juros

assim calculados são chamados de ordinários, ao passo que os juros calculados

com o ano de 365 (ou 366) dias são chamados de exatos e não são usados em

lugar nenhum.

a) Mostre que, dados o principal e a taxa anual, os juros ordinários produzidos

em t dias são maiores que os exatos.

b) Para um principal de R$ 1 000,00 e juros de 12% ao ano, determine os juros

simples, ordinários e exatos, produzidos em 16 dias.

c) Refaça o item (b) para juros compostos.

2. Uma conta de R$ 700,00 vencia no dia 25 de outubro de 1996 e foi paga

em 5 de novembro de 1996. Quais os juros pagos, se os juros de mora são de

12% ao mês?

3. Determine a melhor e a pior alternativa para tomar um empréstimo por três

meses:

13

Unidade 10 Exercícios Suplementares

a) juros simples de 16% ao mês.

b) juros compostos de 15% ao mês.

c) desconto bancário com taxa de desconto de 12% ao mês.

4. Henrique vai emprestar dinheiro a Mário, por quatro meses e pretende receber

juros compostos de 12% ao mês. Como Mário só pretende pagar juros simples,

qual a taxa mensal de juros simples que Henrique deve cobrar?

5. Quando uma operação é pactuada por um número inteiro de períodos de

tempo, há três modos de calcular os juros relativos a frações de períodos:

a) Só são pagos juros nos períodos inteiros de tempo.

b) São pagos juros compostos durante todo o período. Essa é a chamada

convenção exponencial.

c) São pagos juros compostos nos períodos inteiros e juros simples nas frações

de períodos de tempo. Essa é a chamada convenção linear.

Evidentemente o processo (a) se aplica quando os bancos pagam e, o pro-

cesso (c), quando recebem.

Em 5 de janeiro de 1996 foi feito um investimento de 300 reais, a juros de

15% ao mês. Determine, pelos três processos, o montante em 12 de abril de

1996.

6. Considere a amortização de uma dívida de R$ 35 000,00, em 180 meses,

com juros de 1% ao mês, pelo sistema francês. Determine:

a) o valor da centésima prestação.

b) o estado da dívida nessa época.

7. Refaça o problema anterior pelo SAC.

8. Considere a amortização de uma dívida em 150 meses, com juros de 1% ao

mês, pelo sistema francês.

a) De quanto se reduzirá a prestação, dobrando-se o prazo?

b) Que fração da dívida já terá sido amortizada na época do 75o pagamento?

9. Considere a amortização de uma dívida em 150 meses, com juros de 1% ao

mês, pelo SAC.

a) De quanto se reduzirá a prestação inicial, dobrando-se o prazo?

b) Que fração da dívida já terá sido amortizada na época do 75o pagamento?

14

Unidade 10Matemática Financeira (continuação)

10. Uma lanterna de Gol, original, custa R$ 280,00 e tem vida útil de 5 anos.

Uma lanterna alternativa custa R$ 70,00 e tem vida útil de 1 ano. Gilmar

precisa trocar a lanterna de seu Gol. Considere que o dinheiro vale 12% ao ano,

que lanterna ele deve preferir?

11. Um equipamento pode ser alugado por R$ 75,00 mensais ou comprado

por R$ 2 000,00. A vida útil do equipamento é de 30 meses e o valor residual

ao m desse período é de R$ 300,00. Se o equipamento for comprado, há um

custo mensal de R$ 5,00 de manutenção. Considere o valor do dinheiro de 1%

ao mês, qual deve ser a decisão: comprar ou alugar?

12. As cadernetas de poupança renderam 1 416% em um ano cuja inação foi

de 1 109%. Qual a rentabilidade real?

15

Referências Bibliográcas

[1] Carmo, Manfredo P.; Morgado, Augusto C., Wagner, Eduardo & Pitom-

beira, João Bosco. Trigonometria e Números Complexos. Rio de Janeiro:

SBM, Coleção Professor de Matemática.

[2] Eves, Howard. An Introduction to the History of Mathematics. New York:

Holt, Rinehart and Winston, 1964.

[3] Figueiredo, Djairo G. Análise I Rio de Janeiro: LTC, 1996.

[4] Figueiredo, Djairo G. Números Irracionais e Transcedentes Rio de Janeiro:

SBM, Coleção Iniciação Cientíca.

[5] Halmos, Paul. Naive Set Theory. New York: Springer, 1974.

[6] Hefez, Abramo e Fernandez, Cecília de Souza. Introdução à Álgebra Linear.

Rio de Janeiro: SBM, Coleção PROFMAT, 2012.

[7] Fernandes, C. S. Hefez, A. Introdução à Álgebra Linear. SBM, Coleção

PROFMAT. 2

[8] Lima, Elon Lages. Coordenadas no Espaço. Rio de Janeiro: SBM, Coleção

Professor de Matemática.

[9] Lima, Elon Lages. Curso de Análise, Vol. 1. Rio de Janeiro: SBM, Projeto

Euclides, 1976.

[10] Lima, Elon Lages. Logaritmos. Rio de Janeiro: SBM, Coleção Professor de

Matemática.

[11] Lima, Elon Lages. Meu Professor de Matemática e Outras Histórias. Rio

de Janeiro: SBM, Coleção Professor de Matemática.

bibitemelon-analisereal Lima, Elon Lages. Análise Real, Vol. 1. Rio de Janeiro:

IMPA, Coleção Matemática Universitária.

16

11

1

Combinatória I

Sumário

11.1 Introdução . . . . . . . . . . . . . . . . . . . . . . . 2

11.2 Princípios Básicos . . . . . . . . . . . . . . . . . . . 2

Unidade 11 Introdução

11.1 Introdução

Combinatória é um vasto e importante campo da matemática que engloba

temas como a Combinatória Enumerativa, Combinatória Algébrica, Combina-

tória Extrema, Teoria de Grafos e muito mais. As suas aplicações são inúmeras

e vão desde Probabilidade e Estatística e Teoria dos Jogos até campos tão

abstratos quanto a Computação Teórica.

A combinatória foi responsável pela introdução de novos métodos em matemá-

tica e requereu o desenvolvimento de um modo próprio de raciocínio. Para se

ter sucesso no seu estudo, é preciso adquirir certas atitudes e formas de pensar.

No nosso curso, veremos apenas rudimentos de Combinatória Enumerativa, que

é essencialmente a arte da contagem. Contar é uma atividade básica e saber

fazê-lo corretamente é importante e de grande utilidade prática.

No Ensino Médio, a parte da matemática que se ocupa de contagem chama-se

Análise Combinatória e geralmente ela é considerada uma matéria difícil. Ali

se aprendem fórmulas para arranjos, combinações, com repetição ou sem repe-

tição, permutações, permutações circulares, caóticas, etc., mas não se aprende

o essencial, que é raciocinar!

Ao invés de apresentar um formulário e pedir para que seja decorado, o que

se propõe aqui é focar em alguns princípios e técnicas básicas e desenvolver

um raciocínio combinatório próprio que permitirá resolver uma grande gama de

problemas.

Esta unidade baseia-se no Princípio Fundamental da Contagem que diz simples-

mente que, se temos x modos de escolher um objeto e y modos de escolher

outro, temos x×y modos de escolher os dois objetos. Esse princípio é utilizado

nas mais variadas situações.

11.2 Princípios Básicos

O princípio fundamental da contagem diz que se há x modos de tomar uma

decisão D1 e, tomada a decisão D1, há y modos de tomar a decisão D2, então

o número de modos de tomar sucessivamente as decisões D1 e D2 é xy.

2

Unidade 11Combinatória I

Exemplo 1Com 5 homens e 5 mulheres, de quantos modos de pode formar um casal?

Solução. Formar um casal equivale a tomar as decisões:

D1: Escolha do homem (5 modos).

D2: Escolha da mulher (5 modos).

Há 5× 5 = 25 modos de formar casal.

Exemplo 2Uma bandeira é formada por 7 listras que devem ser coloridas usando apenas

as cores verde, azul e cinza. Se cada listra deve ter apenas uma cor e não se

pode usar cores iguais em listras adjacentes, de quantos modos se pode colorir

a bandeira?

Solução. Colorir a bandeira equivale a escolher a cor de cada listra. Há 3 modos

de escolher a cor da primeira listra e, a partir daí, 2 modos de escolher a cor de

cada uma das outras 6 listras. A resposta é 3× 26 = 192.

Exemplo 3Quantos são os números de três dígitos distintos?

Solução. O primeiro dígito pode ser escolhido de 9 modos, pois ele não pode

ser igual a 0. O segundo dígito pode ser escolhido de 9 modos, pois não pode

ser igual ao primeiro dígito. O terceiro dígito pode ser escolhido de 8 modos,

pois não pode ser igual nem ao primeiro nem ao segundo dígito.

A resposta é 9× 9× 8 = 648.

Você deve ter percebido nesses exemplos qual é a estratégia para resolver

problemas de Combinatória:

1) Postura. Devemos sempre nos colocar no papel da pessoa que deve fazer a

ação solicitada pelo problema e ver que decisões devemos tomar. No Exemplo

3, nós nos colocamos no papel da pessoa que deveria escrever o número de

três dígitos; no Exemplo 2, nós nos colocamos no papel da pessoa que deveria

colorir a bandeira; no Exemplo 1, nós nos colocamos no papel da pessoa que

deveria formar o casal.

2) Divisão. Devemos, sempre que possível, dividir as decisões a serem tomadas

em decisões mais simples. Formar um casal foi dividido em escolher o homem

e escolher a mulher; colorir a bandeira foi dividido em colorir cada listra; formar

um número de três dígitos foi dividido em escolher cada um dos três dígitos.

3

Unidade 11 Princípios Básicos

Vamos voltar ao exemplo anterior − Quantos são os números de três dígitos

distintos? − para ver como algumas pessoas conseguem, por erros de estratégia,

tornar complicadas as coisas mais simples.

Começando a escolha dos dígitos pelo último dígito, há 10 modos de escolher

o último dígito. Em seguida, há 9 modos de escolher o dígito central, pois não

podemos repetir o dígito já usado. Agora temos um impasse: de quantos modos

podemos escolher o primeiro dígito: A resposta é depende. Se não tivermos

usado o 0, haverá 7 modos de escolher o primeiro dígito, pois não poderemos

usar nem o 0 nem os dois dígitos já usados nas demais casas; se já tivermos

usado o 0, haverá 8 modos de escolher o primeiro dígito.

Um passo importante na estratégia para resolver problemas de Combinatória é:

3) Não adiar diculdades. Pequenas diculdades adiadas costumam se trans-

formar em imensas diculdades. Se uma das decisões a serem tomadas for mais

restrita que as demais, essa é a decisão que deve ser tomada em primeiro lugar.

No Exemplo 3, a escolha do primeiro dígito era uma decisão mais restrita do

que as outras, pois o primeiro dígito não pode ser igual a 0. Essa é portanto a

decisão que deve ser tomada em primeiro lugar e, conforme acabamos de ver,

postergá-la só serve para causar problemas.

Exemplo 4 O código Morse usa duas letras, ponto e traço, e as palavras têm de 1 a 4

letras. Quantas são as palavras do código Morse?

Solução. Há 2 palavras de uma letra. Há 2×2 = 4 palavras de duas letras, pois

há dois modos de escolher a primeira letra e dois modos de escolher a segunda

letra; analogamente, há 2×2×2 = 8 palavras de três letras e 2×2×2×2 = 16

palavras de 4 letras. O número total de palavras é 2 + 4 + 8 + 16 = 30.

Exemplo 5 Quantos divisores inteiros e positivos possui o número 360? Quantos divi-

sores são pares? Quantos são ímpares? Quantos são quadrados perfeitos?

Solução. a) 360 = 23 × 32 × 5. Os divisores inteiros e positivos de 360 são os

números da forma 2α × 3β × 5γ, com

α ∈ 0, 1, 2, 3 , β ∈ 0, 1, 2 e γ ∈ 0, 1.

Há 4× 3 = 24 maneiras de escolher os expoentes α, β e γ. Há 24 divisores.

4

Unidade 11Combinatória I

b) Para o divisor ser par, α não pode ser 0. Há 3× 3× 2 = 18 divisores pares.

c) Para o divisor ser ímpar, α dever ser 0. Há 1× 3× 2 = 6 divisores ímpares.

Claro que poderíamos ter achado essa resposta subtraindo (a)-(b).

d) Para o divisor ser quadrado perfeito, os expoentes α, β e γ devem ser pares.

Há 2× 2× 1 = 4 divisores que são quadrados perfeitos.

Exemplo 6Quantos são os números pares de três dígitos distintos?

Solução. Há 5 modos de escolher o último dígito. Note que começamos pelo

último dígito, que é o mais restrito; o último dígito só pode ser 0, 2, 4, 6 ou 8.

Em seguida, vamos ao primeiro dígito. De quantos modos se pode escolher

o primeiro dígito? A resposta é depende: se não tivermos usado o 0, haverá 8

modos de escolher o primeiro dígito, pois não poderemos usar nem o 0 nem o

dígito usado na última casa; se tivermos usado o 0, haverá 9 modos de escolher

o primeiro dígito, pois apenas o 0 não poderá ser usado na primeira casa.

Esse tipo de impasse é comum na resolução de problemas e há dois métodos

de vencê-lo.

O primeiro método consiste em voltar atrás e contar separadamente. Con-

taremos separadamente os números que terminam em 0 e os que não terminam

em 0.

Para os que terminam em 0, há 9 modos de escolher o primeiro dígito e 8

modos de escolher o dígito central. Há 1× 9× 8 = 72 números que terminam

em 0.

Para os que não terminam em 0, há 4 modos de escolher o último dígito,

8 modos de escolher o primeiro e 8 modos de escolher o dígito central. Há

4× 8× 8 = 256 números que não terminam em 0.

A resposta é 72 + 256 = 328.

O segundo método consiste em ignorar uma das repetições do problema,

o que nos fará contar em demasia. Depois descontaremos o que houver sido

contado indevidamente.

Primeiramente fazemos de conta que o 0 pode ser usado na primeira casa

do número. Procedendo assim, há 5 modos de escolher o último dígito (só

pode ser 0, 2, 4, 6 ou 8), 9 modos de escolher o primeiro dígito (não podemos

repetir o dígito usado na última casa; note que estamos permitindo o uso do 0

5

Unidade 11 Princípios Básicos

na primeira casa) e 8 modos de escolher o dígito central. Há 5× 9× 8 = 360

números, aí inclusos os que começam por 0.

Agora vamos determinar quantos desses números começam por zero; são

esses os números que foram contados indevidamente. Há 1 modo de escolher o

primeiro dígito (tem que ser 0), 4 modos de escolher o último dígito (só pode

ser 2, 4, 6 ou 8 − lembre-se que os dígitos são distintos) e 8 modos de escolher

o dígito central (não podemos repetir os dígitos já usados). Há 1× 4× 8 = 32

números começados por 0.

A resposta é: 360− 32 = 328.

É claro que este problema poderia ter sido resolvido com um truque. Para

determinar quantos são os números pares de três dígitos distintos, poderíamos

fazer os números de três dígitos distintos menos os números ímpares de três

dígitos distintos.

Para os números de três dígitos distintos, há 9 modos de escolher o primeiro

dígito, 9 modos de escolher o segundo e 8 modos de escolher o último. Há

9× 9× 8 = 648 números de três dígitos distintos.

Para os números ímpares de três dígitos distintos, há 5 modos de escolher

o último dígito, 8 modos de escolher o primeiro e 8 modos de escolher o dígito

central. Há 5× 8× 8 = 320 números ímpares de três dígitos distintos.

A resposta é: 648− 320 = 328.

6

Unidade 11Combinatória I

Exercícios Recomendados

1. Quantos são os gabaritos possíveis de um teste de 10 questões de múltipla-

escolha, com 5 alternativas por questão?

2. Quantos subconjuntos possui um conjunto que tem n elementos?

3. De quantos modos 3 pessoas podem se sentar em 5 cadeiras em la?

4. De quantos modos 5 homens e 5 mulheres podem se sentar em 5 bancos

de 2 lugares, se em cada banco deve haver um homem e uma mulher?

5. De quantos modos podemos colocar 2 reis diferentes em casas não-

adjacentes de um tabuleiro 8× 8? E se os reis fossem iguais?

6. De quantos modos podemos colocar 8 torres iguais em um tabuleiro 8×8,de modo que não haja duas torres na mesma linha ou na mesma coluna?

E se as torres fossem diferentes?

7. De um baralho comum de 52 cartas, sacam-se sucessivamente e sem

reposição duas cartas. De quantos modos isso pode ser feito se a primeira

carta deve ser de copas e a segunda não deve ser um rei?

8. O conjunto A possui 4 elementos e, o conjunto B, 7 elementos. Quantas

funções f : A→ B existem? Quantas delas são injetoras?

9. a) De quantos modos o número 720 pode ser decomposto em um produto

de dois inteiros positivos? Aqui consideramos, naturalmente, 8×90 como

sendo o mesmo que 90× 8.

b) E o número 144?

7

12

1

Combinatória II

Continuação

Sumário

12.1 Introdução . . . . . . . . . . . . . . . . . . . . . . . 2

12.2 Permutações e Combinações . . . . . . . . . . . . . 2

Unidade 12 Introdução

12.1 Introdução

Nesta unidade, são estudadas as permutações e as combinações, desenvolvendo

modos especícos de contagem. Não há fórmulas a decorar, mas procedimentos

de contagem a compreender.

A unidade termina com uma lista de 10 problemas; resolva quantos puder,

redigindo as suas soluções.

12.2 Permutações e Combinações

Há alguns (poucos) problemas de Combinatória que, embora sejam aplica-

ções do princípio básico, aparecem com muita frequência. Para esses problemas,

vale a pena saber de cor as suas respostas. O primeiro desses problemas é o:

Problema das permutações simples

De quantos modos podemos ordenar em la n objetos distintos?

A escolha do objeto que ocupará o primeiro lugar pode ser feita de n modos;

a escolha do objeto que ocupará o segundo lugar pode ser feita de n−1 modos;

a escolha do objeto que ocupará o terceiro lugar pode ser feita de n−2 modos,

etc...; a escolha do objeto que ocupará o último lugar pode ser feita de 1 modo.

A resposta é n(n− 1)(n− 2) · · · 1 = n!.

Cada ordem que se dá aos obejtos é chamada de uma permutação simples

dos objetos. Assim, por exemplo, as permutações simples das letras a, b e c

são (abc), (acb), (bac), (bca), (cab) e (cba).

Portanto, o número de permutações simples de n objetos distintos é Pn =

n!.

Exemplo 1 Quantos são os anagramas da palavra calor? Quantos começam com

consoantes?

Solução. cada anagrama corresponde a uma ordem de colocação dessas 5 letras.

O número de anagramas é P5 = 5! = 120.

Para formar um anagrama começado por consoante devemos primeiramente

escolher a consoante (3 modos) e, depois, arrumar as quatro letras restantes em

2

Unidade 12Combinatória II Continuação

seguida à consoante (4! = 24 modos). Há 3× 24 = 72 anagramas começados

por consoante.

Exemplo 2De quantos modos podemos arrumar em la 5 livros diferentes de Mate-

mática, 3 livros diferentes de Estatística e 2 livros diferentes de Física, de modo

que livros de uma mesma matéria permaneçam juntos?

Solução. Podemos escolher a ordem das matérias de 3! modos. Feito isso,

há 5! modos de colocar os livros de Matemática nos lugares que lhe foram

destinados, 3! modos para os de Estatística e 2! modos para os de Física.

A resposta é 3!5!3!2! = 6× 120× 6× 2 = 8 640.

Exemplo 3Quantos são os anagramas da palavra BOTAFOGO?

Solução. Se as letras fossem diferentes a resposta seria 8!. Como as três letras

O são iguais, quando as trocamos entre si obtemos o mesmo anagrama e não

um anagrama distinto, o que aconteceria se fossem diferentes. Isso faz com que

na nossa contagem de 8! tenhamos contado o mesmo anagrama várias vezes,

3! vezes precisamente, pois há 3! modos de trocar as letras O entre si.

A resposta é8!

3!= 6 720.

De modo geral, o número de permutações de n objetos, dos quais α são

iguais a A, β são iguais a B, γ são iguais a C, etc, é Pα,β,γ,...n =

n!

α!β!γ! . . ..

Exemplo 4De quantos modos podemos dividir 8 objetos em um grupo de 5 objetos e

um de 3 objetos?

Solução. Um processo de fazer a divisão é colocar os objetos em la; os 5

primeiros formam o grupo de 5 e os 3 últimos formam o grupo de 3.

Há 8! modos de colocar os objetos em la.

Entretanto, note que las como abcde | fgh e badce | ghf são las diferentes

e geram a mesma divisão de grupos. Cada divisão em grupos foi contada uma

vez para cada ordem dos objetos dentro de cada grupo. Há 5!3! modos de

arrumar os objetos em cada grupo. Cada divisão em grupos foi contada 5!3!

vezes.

3

Unidade 12 Permutações e Combinações

A resposta é8!

5!3!= 56.

O segundo problema importante é o:

Problema das combinações simples

De quantos modos podemos selecionar p objetos distintos entre n objetos

distintos dados?

Cada seleção de p objetos é chamada de uma combinação simples de classe p

dos n objetos. Assim, por exemplo, as combinações simples de classe 3 dos obje-

tos a, b, c, d e são a, b, c, a, b, d, a, b, e, a, c, d, a, c, e, a, d, e,b, c, d,b, c, e, b, d, e e c, d, e. Representamos o número de combinações simples

de classe p de n elementos por Cpn ou

(np

). Assim, C3

5 =(53

)= 10.

Para resolver o problema das combinações simples basta notar que selecionar

p entre os n objetos equivale a dividir os n objetos em um grupo de p objetos,

que são selecionados, e um grupo de n−p objetos, que são os não-selecionados.

Esse é o problema de Exemplo 4 e a resposta é

Cpn =

n!

p!(n− p)!.

Exemplo 5 Com 5 homens e 4 mulheres, quantas comissões de 5 pessoas, com exata-

mente 3 homens, podem ser formadas?

Solução. Para formar a comissão devemos escolher 3 dos homens e 2 das

mulheres. Há C35 · C2

4 = 10× 6 = 60 comissões.

Exemplo 6 Com 5 homens e 4 mulheres, quantas comissões de 5 pessoas, com pelo

menos 3 homens, podem ser formadas?

Solução. Há comissões com: 3 homens e 2 mulheres, 4 homens e 1 mulher, 5

homens. A resposta é

C25 · C2

4 + C45 · C1

4 + C55 = 10× 6 + 5× 4 + 1 = 81.

Exemplo 7 Tem-se 5 pontos sobre uma reta R e 8 pontos sobre uma reta R′ paralela

a R. Quantos triângulos e quantos quadriláteros convexos com vértices nesses

4

Unidade 12Combinatória II Continuação

pontos existem?

Solução. Para formar um triângulo ou você toma um ponto em R e dois pontos

em R′, ou toma um ponto em R′ e dois pontos em R. O número de triângulos

é 5 · C28 + 8 · C2

5 = 140 + 80 = 220.

Também se poderia pensar em tomar 3 dos 13 pontos e excluir dessa con-

tagem as escolhas de pontos colineares, o que daria

C313 − C3

8 − C35 = 286− 56− 10 = 220.

Para formar um quadrilátero convexo, devemos tomar dois pontos em R e

dois pontos em R′, o que pode ser feito de C35 · C2

8 = 10 · 28 = 280 modos.

A seguir damos mais uma série de exemplos para xar o método de resolução.

Exemplo 8De quantos modos 5 crianças podem formar uma roda de ciranda?

Figura 12.1:

Solução. À primeira vista parece que para formar uma roda com as cinco

crianças basta escolher uma ordem para elas, o que poderia ser feito de 5! =

120 modos. Entretanto, as rodas ABCDE e EABCD são iguais, pois na roda

o que importa é a posição relativa das crianças entre si e a roda ABCDE

pode ser virada na roda EABCD. Como cada roda pode ser virada de cinco

modos, a nossa contagem de 120 rodas contou cada roda 5 vezes e a resposta

é 120/5 = 24.

De modo geral, o número de modos de colocar n objetos em círculo, de modo

que disposições que possam coincidir por rotação sejam consideradas iguais, isto

é, o número de permutações circulares de n objetos é (PC)n =n!

n= (n− 1)!.

5

Unidade 12 Permutações e Combinações

O exemplo a seguir mostra um tipo de racíocinio que, apesar de inesperado,

pode ser muito eciente.

Exemplo 9 Quantos são os anagramas da palavra BULGARO que não possuem duas

vogais adjacentes?

Solução. Vamos primeiramente arrumar as consoantes e, depois, vamos entre-

mear as vogais. O número de modos de arrumar em la as consoantes B, L,

G, R é P4 = 4! = 24. Arrumadas as consoantes, por exemplo na ordem BLGR,

devemos colocar as vogais U, A, O nos 5 espaços da gura. Como não podemos

colocar duas vogais no mesmo espaço, três dos espaços serão ocupados, cada

um com uma vogal e dois dos espaços carão vazios. Temos C35 = 10 modos

de escolher os três espaços que serão ocupados e P3 = 3! = 6 modos de colocar

as vogais nos espaços escolhidos.

B L G R

A resposta é 24× 10× 6 = 1440.

Exemplo 10 Quantas são as soluções inteiras e não-negativas da equação x1 + x2 +

· · ·+ xn = p?

Solução. A resposta deste problema é representada por CRpn.

Para determinar o valor de CRpn, vamos representar cada solução da equação

por uma la de sinais + e | . Por exemplo, para a equação x + y + z = 5, as

soluções (2,2,1) e (5,0,0) seriam representadas por ++ |++|+ e +++++||,respectivamente. Nossa representação, as barras são usadas para separar as

incógnitas e a quantidade de sinais + indica o valor de cada incógnita.

Para a equação x1 + x2 + · · · + xn = p, cada solução seria representada

por uma la com n − 1 barras (as barras são para separar as incógnitas; para

separar n incógnitas, usamos n− 1 barras) e p sinais +. Ora, para formar uma

la com n− 1 barras e p sinais +, basta escolher dos n+ p− 1 lugares da la

os p lugares onde serão colocados os sinais +, o que pode ser feito de Cpn+p−1

modos. Portanto, CRpn = Cp

n+p−1.

Exemplo 11 De quantos modos podemos comprar 3 sorvetes em um bar que os oferece

em 6 sabores distintos?

6

Unidade 12Combinatória II Continuação

Solução. A resposta não é C36 = 20. C3

6 seria o número de modos de comprar

três sorvetes diferentes.

Chamando de xk o número de sorvetes do k-ésimo sabor que vamos comprar,

devemos determinar valores inteiros e não-negativos para xk, k = 1, 2, 3, 4, 5, 6,

tais que x1+x2+ · · ·+x6 = 3. Isso pode ser feito de CR36 = C3

8 = 56 modos.

7

Unidade 12 Permutações e Combinações

Exercícios Recomendados

1. Quantos são os anagramas da palavra CAPITULO.

a) possíveis?

b) que começam e terminam por vogal?

c) que têm as vogais e as consoantes intercaladas?

d) que têm as letras c, a, p juntas nessa ordem?

e) que têm as letras c, a, p juntas em qualquer ordem?

f) que têm a letra p em primeiro lugar e a letra a em segundo?

g) que têm a letra p em primeiro lugar ou a letra a em segundo?

h) que têm p em primeiro lugar ou a em segundo ou c em terceiro?

i) nos quais a letra a é uma das letras à esquerda de p e a letra c é

uma das letras à direita de p?

2. Se A é um conjunto de n elementos, quantas são as funções f : A→ A

bijetoras?

3. De quantos modos é possível colocar 8 pessoas em la de modo que duas

dessas pessoas, Vera e Paulo, não quem juntas?

4. De quantos modos é possível colocar 8 pessoas em la de modo que duas

dessas pessoas, Vera e Paulo, não quem juntas e duas outras, Helena e

Pedro, permaneçam juntas?

5. Quantas são as permutações simples dos números

1, 2, 3, . . . , 10,

nas quais o elemento que ocupa o lugar de ordem k, da esquerda para a

direita, é sempre maior que k − 3?

6. De quantos modos é possível dividir 15 atletas em três times de 5 atletas,

denominados Esporte, Tupi e Minas?

7. De quantos modos é possível dividir 15 atletas em três times de 5 atletas?

8

Unidade 12Combinatória II Continuação

8. De quantos modos é possível dividir 20 objetos em 4 grupos de 3 ou 2

grupos de 4?

9. Um campeonato é disputados por 12 clubes em rodadas de 6 jogos cada.

De quantos modos é possível selecionar os jogos da primeira rodada?

10. Permutam-se de todas as formas possíveis os algarismos 1, 2, 4, 6, 7 e

escrevem-se os números assim formados em ordem crescente. Determine:

a) que lugar ocupa 62 417.

b) que número que ocupa o 66o lugar.

c) qual o 166o algarismo escrito.

d) a soma dos números assim formados.

9

13

1

Combinatória III

Continuação

Sumário

13.1 Introdução . . . . . . . . . . . . . . . . . . . . . . . 2

13.2 O Triângulo Aritmético . . . . . . . . . . . . . . . . 4

13.3 O Binômio de Newton . . . . . . . . . . . . . . . . . 5

Unidade 13 Introdução

13.1 Introdução

A unidade se inicia com o triângulo de Tartaglia-Pascal, que é uma tabela deformato triangular (não limitada) de números naturais, fácil de construir e quepermite obter de modo imediato os coecientes do desenvolvimento de (a+b)n.

1

1 1

1 2 1

1 3 3 1

1 4 6 4 1

1 5 10 10 5 1

1 6 15 20 15 6 1

· · · · · · · · · · · · · · ·

Esse triângulo foi descoberto pelo matemático chinês Yang Hui (1238-1298) e

suas propriedades aritméticas foram estudadas pelo matemático francês Blaise

Pascal (1623-1662). Este escreveu o livro Traité du Triangle Arithmétique,

publicado em 1654, razão pela qual o triângulo leva o seu nome. Pascal, junto

com Fermat, foi o criador da Análise Combinatória (assunto das Unidades 11-

16) e da Teoria de Probabilidades, que estudaremos nas Unidades 17-20.

Dentre as propriedades notáveis do triangulo de Pascal, destacam-se a simetria

axial com relação ao eixo vertical central e a relação de Stifel(n− 1

m− 1

)+

(n− 1

m

)=

(n

m

),

onde (n

m

)=

n!

m!(n−m)!,

número esse também denotado por Cmn .

Essa relação é a base da construção do triângulo, pois permite determinar os

elementos de uma linha conhecendo os elementos da linha anterior. Destacam-

se também o Teorema das Linhas o Teorema das Colunas, dentre muitas outras

propriedades.

A seguir, é apresentado o Binômio de Newton, ou seja, a fórmula que fornece o

desenvolvimento de (a+b)n de um modo diferente do que foi feito na Unidade 4.

2

Unidade 13Combinatória III Continuação

Aqui se utilizam argumentos combinatórios, ao invés dos argumentos algébricos

que foram utilizados lá.

O Binômio de Newton era conhecido muito antes de Newton, mas leva o seu

nome porque ele teve a formidável idéia de usar esse desenvolvimento com ex-

poentes racionais para fazer uma generalização inesperada do clássico Teorema

da Função Implícita para equações polinomiais f(X, Y ) = 0, onde f(0, 0) = 0,

em condições onde não se aplica o teorema clássico, ou seja, quando

∂f

∂x(0, 0) = 0 e

∂f

∂y(0, 0) = 0.

Como são denidos tais desenvolvimentos? Bem, formalmente, podemos

denir, para n racional e m natural os coecientes binomiais como de costume(n

m

)=n(n− 1) · · · (n−m+ 1)

m!.

Note que, se n for inteiro (xado), então(nm

)se anula para m ≥ n + 1, o

que não é o caso se n for um número racional α que não é natural. Nessasituação, o coecientes binomiais nunca se anulam. Portanto, podemos escre-ver formalmente, como Newton fez, o desenvolvimento em série innita

(1+X)α = 1+αX+α(α− 1)

2X2+· · ·+α(α− 1) · · · (α−m+ 1)

m!Xm+· · · . (13.1)

Essa série foi responsável pelo famoso paradoxo do binômio, que intrigou os

matemáticos até ser denitivamente esclarecido por Gauss. Esse paradoxo se

obtém, por exemplo, fazendo em (1) a substituição X = −2 e α = −1, obtendo

−1 = 1 + 2 + 22 + · · ·

A razão do surgimento desse paradoxo, como explicado por Gauss, consiste em

tratar somas innitas como se fossem nitas. A igualdade só vale se a série da

direita for convergente, o que só ocorre quando |X| < 1, e isso não é o caso

quando X = −2. Por aí pode-se ter mais uma comprovação da genialidade de

Gauss, que introduziu a noção de convergência para séries, iniciando o ramo

da Análise Matemática. Gauss fez o estudo completo da série hipergeométrica,

que contém, como casos particulares, várias séries conhecidas. Infelizmente, a

matemática de Gauss é muito pouco abordada no Ensino Médio.

Para nalizar, resolva a lista de problemas propostos e leia a seção Sobre o

Ensino de Combinatória.

3

Unidade 13 O Triângulo Aritmético

13.2 O Triângulo Aritmético

Chamamos de triângulo aritmético de Tartaglia1-Pascal2 ao quadro abaixo,

formado com os diversos valores de Cpn.

C00

C01 C1

1

C02 C1

2 C22

C03 C1

3 C23 C3

3

C04 C1

4 C24 C3

4 C44

C05 C1

5 C25 C3

5 C45 C5

5

1

1 1

1 2 1

1 3 3 1

1 4 6 4 1

1 5 10 10 5 1

Observe que, enumerando as linhas e colunas a partir de zero, Cpn aparece na

linha n e coluna p.

A propriedade que permite construir rapidamente o triângulo é a relação de

Stifel3, que diz que somando dois elementos lado a lado no triângulo obtém-se

o elemento situado embaixo do da direita. Assim, a próxima linha do triângulo

seria

1, 1 + 5 = 6, 5 + 10 = 15, 10 + 10 = 20, 10 + 5 = 15, 5 + 1 = 6, 1.

Proposição 1Relação de Stifel

Cpn + Cp+1

n = Cp+1n+1.

Demonstração Considere um conjunto A de n+1 elementos, um dos quais é x. O número

de subconjuntos de A com p+1 elementos é Cp+1n+1. Esse número é igual à soma

do número de subconjuntos nos quais x não gura, Cp+1n , com o número de

subconjuntos nos quais x gura, Cpn.

Outra relação importante é dada pelo:

1Tartaglia, Nicolo Fontana (1500-1557), matemático italiano.2Pascal, Blaise (1623-1662), matemático, lósofo e físico francês.3Stifel, Michael (1487?-1567), algebrista alemão.

4

Unidade 13Combinatória III Continuação

Teorema 2Teorema das Linhas

C0n + C1

n + C2n + · · ·+ Cn

n = 2n.

DemonstraçãoBasta observar que os dois membros são iguais ao número de subconjuntos

de um conjunto com n elementos.

Exemplo 1Um palácio tem 7 portas. De quantos modos pode ser aberto o palácio?

Solução. Há C17 modos de abrir o palácio abrindo uma só porta, C2

7 modos de

abrir o palácio abrindo duas portas, etc. A resposta é

C11 + C2

7 + · · ·+ C77 = 27 − C0

7 = 128− 1 = 127.

Finalmente, a relação que declara que, em cada linha, elementos equidis-

tantes dos extremos são iguais.

Proposição 3Combinações

Complementares

Cpn = Cn−p

n .

DemonstraçãoBasta observar que o número de modos de escolher, entre n objetos, p

objetos para usar é igual ao de escolher n− p objetos para não usar.

13.3 O Binômio de Newton

A fórmula do binômio de Newton4 é a fórmula que dá o desenvolvimento

de (x+ a)n. Para obtê-la basta multiplicar

(x+ a) · (x+ a) · · · · · (x+ a).

O termo genérico do produto é obtido tomando em p dos fatores, p = 0, 1, 2, . . . , n,

a segunda parcela e tomando nos restantes n − p fatores a primeira parcela.

4Newton, Isaac (1642-1727), matemático e físico inglês.

5

Unidade 13 O Binômio de Newton

Como isso pode ser feito de Cpn modos, o termo genérico do produto é Cp

napxn−p

e

(x+ a)n =n∑p=0

Cpna

pxn−p

= C0na

0xn + C1na

1xn−1 + C2na

2xn−2 + · · ·+ Cnna

nx0.

Exemplo 2 Determine o coeciente de x3 no desenvolvimento de(x4 − 1

x

)7

.

Solução. O termo genérico do desenvolvimento é

Cp7

(−1x

)p(x4)7−p = Cp

7 (−1)px28−5p.

O termo em x3 é obtido se 28− 5p = 3, ou seja, se p = 5.

O termo procurado é C57(−1)5x3 = −21x3 . O coeciente é −21.

Exemplo 3 Determine o termo máximo do desenvolvimento de(1 +

1

3

)50

.

Solução. O termo genérico do desenvolvimento é

tp = Cpna

pxn−p = Cp50

(1

3

)p.

Vamos descobrir para que valores de p os termos crescem. Para isso, calcu-

lamos

tp − tp−1 = Cp50

(1

3

)p− Cp−1

50

(1

3

)p−1

=50!

p!(50− p)!3p− 50!

(p− 1)!(51− p)!3p−1

=50!

(p− 1)!(50− p)!3p−1

(1

3p− 1

51− p

)=

50!

(p− 1)!(50− p)!3p−1

(51− 4p

3p(51− p)

).

6

Unidade 13Combinatória III Continuação

Temos tp−tp−1 positivo, isto é, tp > tp−1 quando 51−4p > 0 e temos tp < tp−1

quando 51− 4p < 0.

Portanto, tp > tp−1 quando p 6 12 e tp < tp−1 quando p > 13. Logo,

t0 < t1 < · · · < t11 < t12 > t13 > t14 > · · · > t50. O termo máximo é

t12 =C12

50

312.

+ Na Sala de Aula - Sobre o Ensino de Combinatória - Clique para ler

7

Unidade 13 O Binômio de Newton

Exercícios Recomendados

1. Com 7 vitaminas diferentes, quantos coquetéis de duas ou mais vitaminas

podemos formar?

2. Determine p para que seja máximo:

a) Cp10 b) Cp

21

3. Determine o termo independente de x no desenvolvimento de(x3 − 1

x2

)10

.

4. Determine o coeciente de xn no desenvolvimento de (1− x)2 · (x+2)n.

5. Determine o valor da soma C0n + 3C1

n + 32C2n + · · ·+ 3nCn

n .

6. Se (1 + x+ x2)n = A0 +A1x+A2x2 + · · ·+A2nx

2n, determine o valor

de:

a) A0 + A1 + A2 + · · ·+ A2n

b) A0 + A2 + A4 + · · ·+ A2n.

7. Determine o termo máximo do desenvolvimento de(1 +

1

2

)100

.

8. Prove que 10150 > 9950 + 10050.

8

Unidade 13Combinatória III Continuação

Na Sala de AulaSobre o Ensino de Combinatória

1. Não faça fórmulas demais ou casos particulares demais. Isso obscurece

as ideias gerais e torna as coisas mais complicadas. Quem troca o princípio

básico da contagem por fórmulas de arranjos, permutações e combinações tem

diculdade de resolver até mesmo o nosso segundo exemplo (o das bandeiras).

2. Aprenda e faça com que os alunos aprendam com os erros. É importante,

diante de uma solução errada, analisar porque ela está errada.

3. Você quer mostrar que é o bom ou quer que seus alunos aprendam? Se você

prefere a segunda alternativa, resista à tentação de em cada problema buscar

solução mais elegante. O que deve ser procurado é um método que permita

resolver muitos problemas e não um truque que resolva maravilhosamente um

problema. Sendo mais especíco: no exemplo 6, da seção de princípios básicos,

foram apresentados dois métodos e um truque. Não se deve mostrar o truque

antes de mostrar os métodos. A beleza de alguns truques só pode ser apreciada

por quem tem domínio dos métodos. Combinatória não é difícil; impossível é

aprender alguma coisa apenas com truques em vez de métodos.

4. Não dê preferência a raciocínios destrutivos, raciocínios do tipo contar a

mais e depois descontar o que não servia e foi contado indevidamente. Os

raciocínios que resolvem a maior parte dos problemas de Combinatória são

essencialmente construtivos. Embora em certos casos seja melhor usar um

raciocínio destrutivo, seus alunos só se sentirão seguros quando dominarem os

raciocínios construtivos. Por exemplo, no exemplo 7 da parte de combinações, a

primeira solução apresentada é melhor do que a segunda para educar o raciocínio

do aluno.

5. Um processo seguro de tornar as coisas complicadas é começar assim: esse

é um problema de arranjos ou de combinações? Como se resolveriam, por

exemplo, os problemas dos exemplos 2, 3 e 5 da Unidade 11 e os problemas

propostos números 1, 5, 8 e 10 da próxima unidade? Aliás, para que servem

arranjos?

9

14

1

Problemas de

Combinatória

Lista I

Unidade 14

Problemas

Lista 1

1. Em um corredor há 900 armários, numerados de 1 a 900, inicialmente

todos fechados. 900 pessoas, numeradas de 1 a 900, atravessam o corre-

dor. A pessoa de número k reverte o estado de todos os armários cujos

números são múltiplos de k. Por exemplo, a pessoa de número 4 mexe

nos armários de números 4, 8, 12, . . . , abrindo os que encontra fechados

e fechando os que encontra abertos. Ao nal, quais armários carão

abertos?

2. Dispomos de 5 cores distintas. De quantos modos podemos colorir os

quatro quadrantes de um círculo, cada quadrante com uma só cor, se

quadrantes cuja fronteira é uma linha não podem receber a mesma cor?

3. De quantos modos podemos formar uma palavra de 5 letras de um alfa-

beto de 26 letras, se a letra A deve gurar na palavra mas não pode ser

a primeira letra da palavra? E se a palavra devesse ter letras distintas?

4. As placas dos veículos são formadas por três letras (de um alfabeto de

26) seguidas por 4 algarismos. Quantas placas poderão ser formadas?

5. Um vagão do metrô tem 10 bancos individuais, sendo 5 de frente e 5

de costas. De 10 passageiros, 4 preferem sentar de frente, 3 preferem

sentar de costas e os demais não têm preferência. De quantos modos eles

podem se sentar, respeitadas as preferências?

6. Escrevem-se os inteiros de 1 até 2 222. Quantas vezes o algarismo 0 é

escrito?

7. Quantos são os inteiros positivos de 4 dígitos nos quais o algarismo 5

gura?

8. Em uma banca há 5 exemplares iguais da Veja, 6 exemplares iguais da

Época e 4 exemplares iguais da Isto é. Quantas coleções não-vazias

de revistas dessa banca podem ser formadas?

2

Unidade 14Problemas de Combinatória Lista I

9. Uma turma tem aulas as segundas, quartas e sextas, de 13h às 14h e de

14h às 15h. As matérias são Matemática, Física e Química, cada uma

com duas aulas semanais, em dias diferentes. De quantos modos pode

ser feito o horário dessa turma?

10. O problema do Exemplo 1 da Unidade 11 − Com 5 homens e 5 mulheres,

de quantos modos se pode formar um casal?− foi resolvido por um aluno

do modo a seguir: A primeira pessoa do casal pode ser escolhida de 10

modos, pois ela pode ser homem ou mulher. Escolhida a primeira pessoa,

a segunda pessoa só poderá ser escolhida de 5 modos, pois deve ser de

sexo diferente da primeira pessoa. Há portanto 10 × 5 = 50 modos de

formar um casal. Onde está o erro?

11. Escrevem-se números de 5 dígitos, inclusive os começados em 0, em car-

tões. Como 0, 1 e 8 não se alteram de cabeça para baixo e como 6,

de cabeça para baixo, se transforma em 9 e vice-versa, um mesmo car-

tão pode representar dois números (por exemplo, 06198 e 86190). Qual

é o número mínimo de cartões para representar todos os números de 5

dígitos?

12. Qual a soma dos divisores positivos de 360?

3

15

1

Problemas de

Combinatória

Lista II

Unidade 15

Problemas

Lista 2

1. De quantos modos é possível colocar r rapazes e m moças em la de

modo que as moças permaneçam juntas?

2. Quantos dados diferentes é possível formar gravando números de 1 a 6

sobre as faces de um cubo?

a) Suponha uma face de cada cor.

b) Suponha faces iguais.

c) Suponha que as faces são iguais e que a soma dos pontos de faces

opostas deva ser igual a 7.

3. Resolva o problema anterior, no caso b), para os outros 4 poliedros regu-

lares.

4. Determine n para quen∑

k=1

k! seja um quadrado perfeito.

5. Quantos são os anagramas da palavra ESTRELADA?

6. O conjunto A possui n elementos. Quantos são os seus subconjuntos

com p elementos?

7. Uma faculdade realiza seu vestibular em dois dias de provas, com 4 maté-

rias em cada dia. Este ano a divisão foi: Matemática, Português, Biologia

e Inglês no primeiro dia e Geograa, História, Física e Química no segundo

dia. De quantos modos pode ser feito o calendário de provas?

8. Qual é o erro na solução do problema abaixo?

Com 5 homens e 4 mulheres, quantas comissões de 5 pessoas, com pelo

menos 3 homens, podem ser formadas?

Solução: Primeiramente vamos escolher 3 homens para a comissão, o

que pode ser feito de C35 = 10 modos. Agora devemos escolher mais

duas pessoas para a comissão, homens ou mulheres, entre as 6 pessoas

restantes, o que pode ser feito de C26 = 15. A resposta é 10×15 = 150.

2

Unidade 15Problemas de Combinatória Lista II

9. Quantas diagonais possui:

a) um octaedro regular?

b) um icosaedro regular?

c) um dodecaedro regular?

d) um cubo?

e) um prisma hexagonal regular?

10. Sejam Im = 1, 2, . . . ,m e In = 1, 2, . . . , n, com m 6 n. Quantas

são as funções f : Im → In estritamente crescentes?

11. Quantos são os números naturais de 7 dígitos nos quais o dígito 4 gura

exatamente 3 vezes e o dígito 8 exatamente 2 vezes?

12. Quantos são os subconjuntos de a1, a2, . . . , an, com p elementos, nos

quais:

a) a1 gura;

b) a1 não gura;

c) a1 e a2 guram;

d) pelo menos um dos elementos a1, a2 gura;

e) exatamente um dos elementos a1 e a2 gura.

13. De um baralho de pôquer (7, 8, 9, 10, valete, dama, rei e ás, cada um

desses grupos aparecendo em 4 naipes: copas, ouros, paus, espadas),

sacam-se simultaneamente 5 cartas.

a) Quantas são as extrações possíveis?

Quantas são as extrações nas quais se forma:

b) um par (duas cartas em um mesmo grupo e as outras três em três

outros grupos diferentes)?

c) dois pares (duas cartas em um grupo, duas em outro grupo e uma

em um terceiro grupo)?

d) uma trinca (três cartas em um grupo e as outras duas em dois outros

grupos diferentes)?

3

Unidade 15

e) um four (quatro cartas em um grupo e uma em outro grupo)?

f) um full hand (três cartas em um grupo e duas em outro grupo)?

g) uma sequência (5 cartas de grupos consecutivos, não sendo todas

do mesmo naipe)?

h) um ush (5 cartas do mesmo naipe, não sendo elas de 5 grupos

consecutivos)?

i) um straight ush (5 cartas de grupos consecutivos, todas do

mesmo naipe)?

j ) um royal straight ush (10, valete, dama, rei e ás de um mesmo

naipe)?

14. O conjunto A possui p elementos e o conjunto B possui n elementos.

Determine o número de funções f : A→ B sobrejetoras para:

a) p = n; b) p = n+ 1; c) p = n+ 2.

15. Considere um conjunto C de 20 pontos do espaço que tem um subcon-

junto C1 formado por 8 pontos coplanares. Sabe-se que toda vez que 4

pontos de C são coplanares, então eles são pontos de C1. Quantos são

os planos que contêm pelo menos três pontos de C?

4

16

1

Problemas de

Combinatória

Lista III

Unidade 16

Problemas

Lista 3

1. Uma la de cadeiras no cinema tem 10 poltronas. De quantos modos 3

casais podem se sentar nessas poltronas de modo que nenhum marido se

sente separado de sua mulher?

2. Quantos são os anagramas da palavra PARAGUAIO que não possuem

consoantes adjacentes?

3. De quantos modos podemos selecionar p elementos, sem selecionar dois

números consecutivos, no conjunto 1, 2, . . . , n?

4. Onze cientistas trabalham num projeto sigiloso. Por questões de segu-

rança, os planos são guardados em um cofre protegido por muitos cade-

ados de modo que só é possível abri-los todos se houver pelo menos 5

cientistas presentes.

a) Qual é o número mínimo possível de cadeados?

b) Na situação do item a), quantas chaves cada cientista deve ter?

5. Depois de ter dado um curso, um professor resolve se despedir de seus 7

alunos oferecendo, durante 7 dias consecutivos, 7 jantares para 3 alunos

cada. De quantos modos ele pode fazer os convites se ele não deseja que

um mesmo par de alunos compareça a mais de um jantar?

6. Formam-se as combinações simples de classe 5 dos elementos a1, a2, . . . ,

a12, as quais são escritas com os elementos em ordem crescente de índices.

Quantas são as combinações nas quais o elemento a8 ocupa o 3o lugar?

7. De quantos modos é possível colocar em la h homens e m mulheres,

todos de alturas diferentes, de modo que os homens entre si e as mulheres

entre si quem em ordem crescente de alturas?

8. Em uma escola, x professores se distribuem em 8 bancas examinadoras

de modo que cada professor participa de exatamente duas bancas e cada

duas bancas têm exatamente um professor em comum.

2

Unidade 16Problemas de Combinatória Lista III

a) Calcule x.

b) Determine quantos professores há em cada banca.

9. A partir de um conjunto de a atletas formam-se t times de k atletas

cada. Todos os atletas participam de um mesmo número de times e cada

par de atletas ca junto no mesmo time um mesmo número de vezes.

Determine:

a) de quantos times cada atleta participa;

b) em quantos times cada par de atletas ca junto.

10. De quantos modos podemos formar uma mesa de buraco com 4 jogado-

res?

11. De quantos modos podemos formar uma roda de ciranda com 5 meninos

e 5 meninas de modo que pessoas de mesmo sexo não quem juntas?

12. De quantos modos podemos formar uma roda de ciranda com 6 crianças,

de modo que duas delas, Vera e Isadora, não quem juntas?

13. Quantas são as soluções inteiras e positivas de x+ y + z = 7?

14. Quantas são as soluções inteiras e não-negativas de x+ y + z 6 6?

15. Uma indústria fabrica 5 tipos de balas que são vendidas em caixas de

20 balas, de um só tipo ou sortidas. Quantos tipos de caixas podem ser

montados?

3

17

1

Probabilidade

Sumário

17.1 Introdução . . . . . . . . . . . . . . . . . . . . . . . 2

17.2 Conceitos Básicos . . . . . . . . . . . . . . . . . . . 2

Unidade 17 Introdução

17.1 Introdução

Iniciamos, nesta unidade, o estudo de Probabilidade, cuja parte mais ele-

mentar é uma das aplicações da Combinatória. A Teoria de Probabilidade, como

diz o nome, é o estudo de fenômenos que envolvem a incerteza e se originou

como instrumento para modelar jogos de azar, como cartas e dados.

Probabilidade é a base para a Estatística, ciência utilizada nas mais diversas ati-

vidades humanas, sendo fundamental em várias áreas, como Ciências Humanas,

Ciências da Saúde, Economia e Finanças, Ecologia e Teoria dos Jogos, entre

muitos outros. Do ponto de vista teórico, atualmente, a Teoria de Probabili-

dade é utilizada como ferramenta em algumas áreas da Física e, cada vez mais,

em áreas da própria Matemática. Por esse motivo, o ensino de Probabilidade

no Ensino Médio é importante e atual.

Esse assunto é muito vasto, mas aqui só trataremos de alguns conceitos básicos

e suas aplicações. Denem-se o conjunto espaço amostral e a noção de proba-

bilidade como sendo uma função numérica com domínio no conjunto das partes

desse espaço. Os subconjuntos do espaço amostral são os chamados eventos.

As propriedades básicas da função probabilidade são dadas no Teorema 3, que

bastarão para resolver os problemas dessa unidade.

17.2 Conceitos Básicos

Experiências que repetidas sob as mesmas condições produzem geralmente

resultados diferentes são chamadas de aleatórias. Por exemplo, retira-se uma

carta de um baralho e verica-se se ela é ou não um curinga; compra-se uma

lâmpada e verica-se se ela queima ou não antes de 100h de uso; joga-se um

dado até se obter um seis e conta-se o número de lançamentos.

Definição 1Espaço Amostral

Chamaremos de espaço amostral o conjunto de todos os resultados possíveis

de uma experiência aleatória. Representaremos o espaço amostral por S e

só vamos considerar aqui o caso de S ser nito ou innito enumerável. Os

subconjuntos de S serão chamados de eventos. Diremos que um evento ocorre

quando o resultado da experiência pertence ao evento.

2

Unidade 17Probabilidade

Exemplo 1Lança-se uma moeda e observa-se a face que cai voltada para cima. O

espaço amostral é S = cara, coroa e há 4 eventos:

∅, A = cara, B = coroa e S.

∅ é um evento que não ocorre nunca e é chamado de evento impossível.

O evento A ocorre se e somente se o lançamento resulta em cara. S ocorre

sempre e é chamado de evento certo.

Exemplo 2Lança-se um dado e observa-se a face que cai voltada para cima. O espaço

amostral é S = 1, 2, 3, 4, 5, 6 e há 64 eventos. Alguns desses eventos são:

∅, que não ocorre nunca; S, que ocorre sempre; A = 2, 4, 6, que ocorre se e

somente se o resultado do lançamento for par. Se o resultado do lançamento

for seis, ocorrem os eventos 6, 5, 6, 2, 4, 6, etc.

Exemplo 3Se A e B são eventos em um mesmo espaço amostral S, A∪B é o evento

que ocorre se e somente se ocorre o evento A ou ocorre o evento B, isto é,

ocorre pelo menos um dos eventos A e B; A ∩ B é o evento que ocorre se e

somente se ocorrem ambos os eventos A e B; A \ B é o evento que ocorre

se e somente se ocorre o evento A mas não ocorre o evento B; A, chamado

de evento oposto a A, é o evento que ocorre se e somente se o evento A não

ocorre.

Associaremos a cada evento um número, que chamaremos de probabilidade

do evento e que traduzirá nossa conança na capacidade do evento ocorrer.

Definição 2Probabilidade

Uma probabilidade é uma função que associa a cada evento A um número

P (A) de forma que:

i) Para todo evento A, 0 6 P (A) 6 1.

ii) P (S) = 1

iii) Se A e B são eventos mutuamente excludentes, isto é, eventos que não

podem ocorrer simultaneamente (isto é, A ∩B = ∅) então

P (A ∪B) = P (A) + P (B).

3

Unidade 17 Conceitos Básicos

Exemplo 4 Lança-se uma moeda e observa-se a face que cai voltada para cima. O

espaço amostral é S = cara, coroa e há 4 eventos: ∅ , A = cara, B =

coroa, S. Uma probabilidade que pode ser denida é

P1(∅) = 0, P1(A) = P1cara = 0, 5, P1(B) = P1coroa = 0, 5 P1(S) = 1.

Verique que as três condições da denição de probabilidade são satisfeitas.

Outra probabilidade que pode ser denida é

P2(∅) = 0, P2(A) = P2cara = 0, 3, P2(B) = P2coroa = 0, 7 P2(S) = 1.

Verique que as três condições da denição de probabilidade são satisfeitas.

É claro que se desejamos que a probabilidade traduza nossa conança na

capacidade do evento ocorrer, P1 constitui um modelo adequado quando acre-

ditamos ser o resultado cara tão provável quanto o resultado coroa. P2, por sua

vez seria mais adequado se tivéssemos lançado a moeda um número grande de

vezes e obtido o resultado cara em 30% dos lançamentos.

Encerrando o exemplo, um breve comentário a respeito de notação. Deve-

ríamos ter escrito P (cara) e não Pcara. Entretanto, quando não houver

risco de confusão daremos preferência à notação mais simples.

Os modelos probabilísticos que usamos mais frequentemente são exatamente

os apresentados no exemplo anterior.

Um é o modelo equiprobabilístico. Se temos n elementos no espaço amostral

e queremos que todos os eventos unitários tenham a mesma probabilidade,

devemos atribuir a cada evento unitário a probabilidade1

n. Não poderia ser de

outra forma pois se S = x1, x2, . . . , xn e P (x1) = P (x2) = · · · = P (xn) = k,

temos, por iii),

1 = P (S) = Px1, x2, . . . , xn = P (x1 ∪ x2 ∪ · · · ∪ xn)= P (x1) + P (x2) + · · ·+ P (xn)

= k + k + · · ·+ k = nk e k =1

n.

Analogamente, é fácil ver que, nesse modelo, se um evento X é formado

por j elementos então P (X) =j

n. Ou seja, a probabilidade de um evento é a

razão entre o número de casos favoráveis ao evento e o número total de casos

4

Unidade 17Probabilidade

possíveis. Foi esse o modelo adotado por vários matemáticos como Cardano 1,

Pascal e Laplace2 entre outros, no estudo dos jogos de azar.

Outro é o modelo frequencial. Se repetimos a experiência n vezes e o evento

A ocorreu em j dessas experiências, adotamos para P (A) a frequência relativa

do evento A, isto é, o número de vezes que o evento A ocorreu dividido pelo

número total de repetições da experiência, ou seja, P (A) =j

n.

O teorema a seguir contém as propriedades das probabilidades.

Teorema 3Propriedades da

probabilidade

Se A e B são eventos, então:

i) P (A) = 1− P (A).

ii) P (∅) = 0.

iii) P (A \B) = P (A)− P (A ∩B).

iv) P (A ∪B) = P (A) + P (B)− P (A ∩B).

v) Se A ⊃ B então P (A) > P (B).

Demonstraçãoi) 1 = P (S) = P (A ∪ A) = P (A) + P (A). Daí, P (A) = 1− P (A).

ii) P (S) = P (S∪∅) = P (S)+P (∅), pois S e ∅ são mutuamente excludentes.

Daí, P (∅) = 0.

iii) P (A) = P [(A \B)∪ (A∩B)] = P (A \B)+P (A∩B) pois A \B e A∩Bsão mutuamente excludentes. Daí, P (A \B) = P (A)− P (A ∩B).

iv) P (A ∪ B) = P [(A \ B) ∪ B] = P (A \ B) + P (B) pois A \ B e B

são mutuamente excludentes. Como P (A \ B) = P (A) − P (A ∩ B), resulta

P (A ∪B) = P (A) + P (B)− P (A ∩B).

v) Como P (A \ B) = P (A) − P (A ∩ B), se A ⊂ B resulta P (A \ B) =

P (A)− P (B). Como P (A \B) > 0, temos P (A) > P (B).

Exemplo 5Em um grupo de r pessoas, qual é a probabilidade de haver pelo menos

duas pessoas que façam aniversário no mesmo dia?

Solução. Vamos determinar a probabilidade disso não acontecer. O número de

1Cardano, Jerônimo (1501-1576), matemático italiano.2Laplace, Pierre Simon (1749-1827), matemático francês.

5

Unidade 17 Conceitos Básicos

casos possíveis para os aniversários das r pessoas é 365r. O número de casos

favoráveis a que todas aniversariem em dias diferentes é 365 × 364 × · · · ×(366 − r), havendo r fatores nesse produto. Portanto, a probabilidade de não

haver pelo menos duas pessoas que façam aniversário no mesmo dia é de

365× 364× · · · × (366− r)

365r

e a de haver pelo menos duas pessoas que tenham o mesmo dia de aniversário

é de

1− 365× 364× · · · × (366− r)

365r.

A tabela abaixo dá, para alguns valores de r, a probabilidade de haver

coincidência de aniversários.

r Probabilidade

5 0, 03

10 0, 12

15 0, 25

20 0, 41

23 0, 51

25 0, 57

30 0, 71

40 0, 89

45 0, 94

50 0, 97

O resultado é surpreendente. Em um grupo de 23 pessoas, é mais provável

haver duas pessoas com o mesmo aniversário do que todas aniversariarem em

dias diferentes.

Exemplo 6 Em uma loteria de N números há um só prêmio. Salvador compra n

(1 < n < N) bilhetes para uma só extração e Sílvio compra n bilhetes, um para

cada uma de n extrações. Qual dos dois jogadores tem mais chance de ganhar

algum prêmio?

Solução. A probabilidade de Salvador ganhar algum prêmio é

n

N.

6

Unidade 17Probabilidade

A probabilidade de Sílvio não ganhar nenhum prêmio é

(N − 1)n

Nn.

Logo, a probabilidade de Sílvio ganhar algum prêmio é

1− (N − 1)n

Nn.

Armamos que Salvador tem mais chance de ser premiado, isto é, armamos

quen

N> 1− (N − 1)n

Nn,

ou, equivalentemente, armamos que

(N − 1)n

Nn> 1− n

N.

A prova dessa armação faz-se por indução.

Para n = 2 temos

(N − 1)n

Nn=

(N − 1)2

N2= 1− 2

N+

1

N2> 1− 2

N= 1− n

N.

Se(N − 1)n

Nn> 1− n

N

multiplicando porN − 1

N

obtemos(N − 1)n+1

Nn+1> 1− n

N− 1

N+

n

N2> 1− n+ 1

N.

7

Unidade 17 Conceitos Básicos

Exercícios Recomendados

1. Lançam-se dois dados não-tendenciosos. Qual a probabilidade da soma

dos pontos ser igual a 7 ?

2. 24 times são divididos em dois grupos de 12 times cada. Qual é a proba-

bilidade de dois desses times carem no mesmo grupo?

3. Mostre que

P (A ∪B ∪ C) = P (A) + P (B) + P (C)−P (A ∩B)− P (A ∩ C)− P (B ∩ C) + P (A ∩B ∩ C).

4. Se P (A) =2

3e P (B) =

4

9, mostre que:

a) P (A ∪B) >2

3;

b)2

96 P (A ∩B) 6

5

9;

c)1

96 P (A ∩B) 6

4

9.

5. Cinco dados são jogados simultaneamente. Determine a probabilidade de

se obter:

a) um par;

b) dois pares;

c) uma trinca;

d) uma quadra;

e) uma quina;

f) uma sequência;

g) um full hand, isto é, uma trinca e um par.

6. Um polígono regular de 2n+1 lados está inscrito em um círculo. Escolhem-

se três dos seus vértices, formando um triângulo. Determine a probabili-

dade do centro do círculo ser interior ao triângulo.

8

Unidade 17Probabilidade

7. Doze pessoas são divididas em três grupos de 4. Qual é a probabilidade

de duas determinadas dessas pessoas carem no mesmo grupo?

8. Em um grupo de 4 pessoas, qual é a probabilidade de haver alguma

coincidência de signos zodiacais?

9. Em um armário há 5 pares de sapatos. Escolhem-se 4 pés de sapatos.

Qual é a probabilidade de se formar exatamente um par de sapatos?

9

18

1

ProbabilidadeCondicional

Sumário

18.1 Introdução . . . . . . . . . . . . . . . . . . . . . . . 2

18.2 Probabilidade Condicional . . . . . . . . . . . . . . . 2

Unidade 18 Introdução

18.1 Introdução

Nessa unidade, é apresentada mais uma técnica básica importante em pro-

babilidades, a chamada Probabilidade Condicional. Usa-se essa técnica quando

se quer calcular a probabilidade de um evento, na presença de uma informação

privilegiada. Mais precisamente, é uma maneira de calcular a probabilidade

de ocorrer um evento B, sabendo que ocorreu o evento A, ambos do mesmo

espaço amostral.

Por exemplo, numa turma de 60 alunos, 30 só estudam inglês, 20 só estudam

espanhol e 10 estudam ambas as línguas. Suponhamos que um sorteio é re-

alizado, com apenas um vencedor. A probabilidade de um aluno que estuda

ambas as línguas ser sorteado é igual a

número de alunos que estudam ambas as línguasnúmero total de alunos

=10

60=

1

6.

Agora, suponhamos que o sorteio é realizado, e alguém nos sopra que o sor-

teado estuda inglês. Isto certamente vai inuir no nosso modo de calcular a

probabilidade do vencedor ser bilíngue, pois agora o espaço amostral se reduz

aos 40 alunos que estudam inglês, dos quais 10 também estudam espanhol;

logo, a probabilidade passa a ser10

40=

1

4.

O resultado é tão simples quanto mostrado no exemplo acima, mas, se bem

aplicado, resolve problemas incríveis!

18.2 Probabilidade Condicional

Exemplo 1 Consideremos a experiência que consiste em jogar um dado não-viciado e

observar a face de cima. Consideremos o evento B = o resultado é par.Temos P (B) =

3

6= 0, 5. Essa é a probabilidade de B a priori, isto é,

antes que a experiência se realize. Suponhamos que, realizada a experiên-

cia, alguém nos informe que o resultado não foi o número 1, isto é, que

A = o resultado é diferente de 1 ocorreu.Nossa opinião sobre a ocorrência de B se modica com essa informação pois

passamos a ter apenas 5 casos possíveis, dos quais 3 são favoráveis à ocorrência

2

Unidade 18Probabilidade Condicional

de B. Essa opinião é quanticada com a introdução de uma probabilidade a

posteriori, ou probabilidade de B na certeza de A,

P (B|A) = 3

5= 0, 6.

Note que os casos possíveis não são mais todos os elementos do espaço

amostral S e sim os elementos de A e que os casos favoráveis à ocorrência de

B não são mais todos os elementos de B e sim os elementos de A∩B pois só

os elementos que pertencem a A podem ocorrer.

Exemplo 2A tabela abaixo dá a distribuição dos alunos de uma turma, por sexo e por

carreira pretendida.

masculino feminino total

cientíca 15 5 20

humanística 3 7 10

total 18 12 30

Escolhe-se ao acaso um aluno. Sejam M, F, C e H os eventos, o aluno

selecionado é do sexo masculino, é do sexo feminino, pretende uma carreira

cientíca e pretende uma carreira humanística, respectivamente. Temos

P (H) =10

30=

1

3;

P (H|M) =3

18=

1

6;

P (H|F ) =7

12;

P (F |H) =7

10.

Definição 1Probabilidade

Condicional

Dados dois eventos A e B, com P (A) 6= 0, a probabilidade condicional de

B na certeza de A é o número

P (B|A) = P (A ∩B)

P (A).

3

Unidade 18 Probabilidade Condicional

Na realidade, poucas vezes usaremos a fórmula acima para calcular uma

probabilidade condicional. Usá-la-emos, isto sim, para o cálculo de P (A ∩ B);

P (A ∩B) = P (A) · P (B|A).

Exemplo 3 Uma urna contém 4 bolas brancas e 6 bolas pretas. Sacam-se, sucessiva-

mente e sem reposição, duas bolas dessa urna. Determine a probabilidade de

ambas serem brancas.

Solução. Sejam B1 = a primeira bola é branca e B2 = a segunda bola é

branca. Temos

P (B1 ∩B2) = P (B1) · P (B2|B1) =4

10· 39=

2

15.

Note que foi bastante simples o cálculo de P (B2|B1). Realmente, na certeza de

que a primeira bola foi branca, é fácil calcular a probabilidade da segunda bola

ser branca, pois, para a segunda extração, a urna está com 3 bolas brancas e 6

pretas. De modo mais geral, é fácil calcular probabilidades condicionais quando

as coisas estão na ordem certa, isto é, é fácil calcular probabilidades de coisas

futuras na certeza de coisas passadas.

Exemplo 4 Uma urna contém 4 bolas brancas e 6 bolas pretas. Sacam-se, sucessiva-

mente e sem reposição, duas bolas dessa urna. Determine a probabilidade da

primeira bola ser branca, sabendo que a segunda bola é branca.

Solução. Sejam B1 = a primeira bola é branca e B2 = a segunda bola é

branca. Queremos P (B1|B2). Note que essa é uma probabilidade do passado

na certeza do futuro. Aqui usamos a fórmula da denição de probabilidade

condicional.

P (B1|B2) =P (B1 ∩B2)

P (B2)·

P (B1 ∩B2) foi calculada no exemplo anterior e vale2

15.

O cálculo de P (B2) não é imediato pois não sabemos como está a urna no

momento da segunda extração. Para calcular P (B2), consideramos todas as

possibilidades quanto à primeira bola. Para a segunda bola ser branca, ou a

segunda é branca e a primeira foi branca, ou a segunda é branca e a primeira

4

Unidade 18Probabilidade Condicional

foi preta. Isto é,

P (B2) = P [(B1 ∩B2) ∪ (P1 ∩B2)]

= P (B1 ∩B2) + (P1 ∩B2)

=2

15+ P (P1) · P (B2|P1)

=2

15+

6

10· 49

=2

Logo,

P (B1|B2) =P (B1 ∩B2)

P (B2)=

2

15÷ 2

5=

1

3.

Uma maneira eciente de lidar com experiências que possuem vários está-

gios é o uso das árvores de probabilidade.

Figura 18.1: Árvore de probabilidade

Nesses diagramas colocamos as probabilidades condicionais da extremidade

de cada galho na certeza da origem do galho. Para determinar uma proba-

bilidade usando esse diagrama, basta percorrer todos os caminhos que levam

ao evento cuja probabilidade é procurada, multiplicando as probabilidades em

cada caminho e somando os produtos ao longo dos vários caminhos. Assim,

por exemplo,

P (B1 ∩B2) =4

10· 39=

2

15;

5

Unidade 18 Probabilidade Condicional

P (B2) =4

10· 39+

6

10· 49=

2

Exemplo 5 Escolhe-se uma entre três moedas. Duas dessas moedas são não-viciadas

e a outra tem duas caras. A moeda selecionada é lançada e é obtida uma cara.

Qual é a probabilidade de ter sido selecionada a moeda de duas caras?

Figura 18.2: Moeda de duas caras

P (V |C) =P (V ∩ C)

P (C)·

P (V ∩ C) =1

3· 1 =

1

P (C) =1

3· 1 + 2

3· 12=

2

P (V |C) =1

3÷ 2

3=

1

O exemplo a seguir mostra um dos mais poderosos métodos de estimação

em Estatística, o método da máxima verossimilhança.

Exemplo 6 Em certa cidade, os táxis são numerados de 1 a N . Para estimar o número

N de táxis da cidade, um turista anotou os números de todos os táxis que pegou:

47, 12, 33 e 25. Determine a probabilidade do turista ter tomado os táxis que

têm esses números e determine o valor de N para o qual essa probabilidade é

6

Unidade 18Probabilidade Condicional

máxima.

Solução. Sejam A = o primeiro táxi tem número 47, B=o segundo táxi

tem número 12, etc. A probabilidade pedida é

P (A ∩B ∩ C ∩D)

= P (A) · P (B|A) · P [C|(A ∩B)] · P [D|(A ∩B ∩ C)]

=1

N· 1N· 1N· 1N· = 1

N4·

Essa probabilidade de ocorrer o que efetivamente ocorreu é chamada de veros-

similhança. No caso, ela é máxima quando N é mínimo. Ora, como N > 47,

o valor de N que torna máxima a verossimilhança é 47.

A estimativa de máxima verossimilhança de N é 47.

Exemplo 7Algumas pesquisas estatísticas podem causar constrangimentos aos entre-

vistados com perguntas do tipo você usa drogas? e correm o risco de não

obter respostas sinceras ou não obter respostas de espécie alguma. Para es-

timar a proporção p de usuários de drogas em certa comunidade, pede-se ao

entrevistado que, longe das vistas do entrevistador, jogue uma moeda: se o

resultado for cara, responda a você usa drogas? e, se o resultado for coroa,

responda a sua idade é um número par?. Assim, caso o entrevistado diga

sim, o entrevistador não saberá se ele é um usuário de drogas ou se apenas tem

idade par.

Se s é a probabilidade de um entrevistado responder sim, s é facilmente

estimado pela proporção de respostas sim obtidas nas entrevistas. A relação

entre s e p pode ser determinada pela árvore abaixo.

Figura 18.3: Método indireto de entrevista

7

Unidade 18 Probabilidade Condicional

s = P (sim) = 0, 5p+ 0, 5 · 0, 5.

Daí, p = 2s− 0, 5.

Por exemplo, se 30% dos entrevistados respondem sim, você pode estimar

em 10% a proporção de usuários de drogas.

O exemplo a seguir é um interessante exemplo de probabilidade geométrica.

Quando selecionamos um ponto ao acaso em uma parte do plano é extrema-

mente razoável supor que a probabilidade do ponto selecionado pertencer a uma

certa região seja proporcional à área dessa região.

Exemplo 8 Selecionam-se ao acaso dois pontos em um segmento de tamanho 1,

dividindo-o em três partes. Determine a probabilidade de que se possa for-

mar um triângulo com essas três partes.

Solução. Sejam x ∈ [0, 1] e y ∈ [0, 1] os pontos escolhidos, x 6 y.

Figura 18.4:

Escolher x e y pertencentes a [0, 1], com x 6 y, equivale a escolher um

ponto (x, y) no triângulo T da gura abaixo.

Figura 18.5: Como escolher os pontos x e y

8

Unidade 18Probabilidade Condicional

Para que exista um triângulo de lados x, y − x e 1 − y devemos ter x <

y − x + 1− y e y − x < x + 1− y e 1− y < x + y − x, o que dá x < 0, 5 e

y < x+0, 5 e y > 0, 5. Em suma, o triângulo existirá se e somente se o ponto

(x, y) for selecionado na parte sombreada do triângulo T .

Sendo A o evento as três partes formam um triângulo e sendo S o evento

certo, temos que P (A) é proporcional à área da parte sombreada e P (S) = 1

é proporcional à área de T . Logo,

P (A) =P (A)

P (S)=

área sombreadaárea de T

=1

Exemplo 9A e B lançam sucessivamente um par de dados até que um deles obtenha

soma de pontos 7, caso em que a disputa termina e o vencedor é o jogador que

obteve soma 7. Se A é o primeiro a jogar, qual é a probabilidade de A ser o

vencedor?

Solução. A probabilidade de obter soma 7 é

6

36=

1

6

e a de não ser soma 7 é

1− 1

6=

5

Para A ganhar, ou A ganha na primeira mão, ou na segunda, ou na terceira,

etc. A probabilidade de A ganhar na primeira mão é1

6. Para A ganhar na

segunda mão, A não pode obter soma 7 na primeira mão e B não pode obter

soma 7 na primeira mão e A deve obter soma 7 na segunda mão, o que ocorre

com probabilidade (5

6

)2

· 16·

Para A ganhar na terceira mão, A não pode obter soma 7 nas duas primeiras

mãos e B não pode obter soma 7 nas duas primeiras mãos e A deve obter soma

7 na terceira mão, o que ocorre com probabilidade(5

6

)2

· 16,

9

Unidade 18 Probabilidade Condicional

etc.

A probabilidade de A ganhar é

1

6+

(5

6

)2

· 16+

(5

6

)4

· 16+ · · · =

16

1−(

56

)2 =6

11.

Uma solução mais elegante pode ser obtida ignorando as mãos sem vence-

dores. A probabilidade de A ganhar uma mão é de1

6; de B ganhar uma mão

é de5

6· 16=

5

36,

pois, para B ganhar, A não pode obter soma 7 e B deve obter soma 7; a de

ninguém ganhar é de5

6· 56=

25

36,

pois, para que ninguém ganhe, A não pode obter soma 7 e B não pode obter

soma 7.

A probabilidade A ganhar é a probabilidade A ganhar em uma mão em que

houve vencedor, isto é,

P (A|A ∪B) =P [A ∩ (A ∪B)]

P (A ∪B)=

P (A)

P (A ∪B)=

16

1− 2536

=6

11.

Como, analogamente,

P (A|A ∪B) =P (B)

P (A ∪B),

observe que a razão entre P (A|A∪B) e P (B|A∪B) é igual à razão entre P (A)

e P (B), pois P (A ∪ B) é simplicado. Esse é o princípio de preservação das

chances relativas. Em um jogo em que pode haver empates, e é repetido até

que alguém vença, a razão entre as probabilidades de vitória dos dois jogadores

é igual à razão de suas probabilidades de vitória em uma única partida.

Conhecendo o princípio, poderíamos ter resolvido o problema do modo se-

guinte:

Em uma mão, as probabilidades de vitória de A e de B são respectivamente

de1

6e de

5

36.

10

Unidade 18Probabilidade Condicional

A razão dessas probabilidades é de6

5. A razão das probabilidades de vitória de

A e de B no jogo é também de6

5e, como um dos dois ganha o jogo, a soma

dessas probabilidades é 1. Então, essas probabilidades são iguais a6

11e

5

11,

respectivamente.

11

Unidade 18 Probabilidade Condicional

Exercícios Recomendados

1. Joga-se um dado não-viciado duas vezes. Determine a probabilidade con-

dicional de obter 3 na primeira jogada, sabendo que a soma dos resultados

foi 7.

2. Um estudante resolve um teste de múltipla escolha de 10 questões, com

5 alternativas por questão. Ele sabe 60% da matéria do teste. Quando

ele sabe uma questão, ele acerta, e, quando não sabe, escolhe a resposta

ao acaso. Se ele acerta uma questão, qual é a probabilidade de que tenha

sido por acaso?

3. Por denição, dois eventos A e B são independentes, quando ocorre

P (A ∩ B) = P (A) · P (B). Três eventos A, B e C são independentes,

por denição, quando P (A∩B) = P (A)·P (B), P (B∩C) = P (B)·P (C),

P (A∩C) = P (A) ·P (C) e P (A∩B∩C) = P (A) ·P (B) ·P (C). Jogue

um dado duas vezes. Considere os eventos A = o resultado do primeiro

lançamento é par, B = o resultado do segundo lançamento é par e C

= a soma dos resultados é par.

a) A e B são independentes?

b) A e C são independentes?

c) B e C são independentes?

d) A, B e C são independentes?

4. Determine a probabilidade de obter ao menos

a) um seis em 4 lançamentos de um dado;

b) um duplo seis em 24 lançamentos de um par de dados.

5. Um exame de laboratório tem eciência de 95% para detectar uma doença

quando ela de fato existe. Entretanto o teste aponta um resultado falso-

positivo para 1% das pessoas sadias testadas. Se 0, 5% da população tem

a doença, qual é a probabilidade de uma pessoa ter a doença, dado que

o seu exame foi positivo?

6. Quantas vezes, no mínimo, se deve lançar um dado para que a probabili-

dade de obter algum seis seja superior a 0,9?

12

Unidade 18Probabilidade Condicional

7. Em uma cidade com n+ 1 habitantes, uma pessoa conta um boato para

outra pessoa, a qual, por sua vez, conta o boato para uma terceira pessoa,

e assim por diante. Evidentemente ninguém é distraído a ponto de contar

o boato para quem lhe havia contado o boato. Determine a probabilidade

do boato ser contado k vezes:

a) sem retornar ao inventor do boato.

b) sem repetir nenhuma pessoa.

8. Em uma cidade, as pessoas falam a verdade com probabilidade1

3. Supo-

nha que A faz uma armação e que D diz que C diz que B diz que A

falou a verdade. Qual a probabilidade de A ter falado a verdade?

9. Um prisioneiro possui 50 bolas brancas, 50 bolas pretas e duas urnas

iguais. O prisioneiro deve colocar do modo que preferir as bolas nas urnas,

desde que nenhuma urna que vazia. As urnas serão embaralhadas e o

prisioneiro deverá, de olhos fechados, escolher uma urna e, nesta urna,

escolher uma bola. Se a bola for branca ele será libertado e, se for

preta, será condenado. Como deve agir o prisioneiro para maximizar a

probabilidade de ser libertado?

13

19

1

Problemas de

Probabilidade

Lista I

Unidade 19

Problemas

Lista 1

1. Distribuindo ao acaso 5 sorvetes de creme e 5 de chocolate a 10 pessoas,

das quais 3 preferem creme, 2 preferem chocolate e as demais não têm

preferência, qual é a probabilidade de todas saírem satisfeitas?

2. Escolhem-se ao acaso duas peças de um dominó comum. Qual é a pro-

babilidade delas possuírem um número comum?

3. No jogo da quina concorrem 80 dezenas e são sorteadas 5 dezenas. Clara

apostou em 8 dezenas. Qual a probabilidade de Clara acertar:

a) 3 dezenas?

b) 4 dezenas?

c) 5 dezenas?

4. Em uma roda são colocadas n pessoas. Qual é a probabilidade de duas

dessas pessoas carem juntas?

5. Uma pessoa tem um molho de n chaves, das quais apenas uma abre

a porta. Se ela vai experimentando as chaves até acertar, determine a

probabilidade dela só acertar na tentativa de ordem k, supondo:

a) que a cada tentativa frustrada ela toma a sábia providência de descartar

a chave que não serviu.

b) supondo que ela não age como no item a).

6. Há 8 carros estacionados em 12 vagas em la. Determine a probabilidade:

a) das vagas vazias serem consecutivas.

b) de não haver duas vagas vazias adjacentes.

7. Se P (A) = 0, 4, P (B) = 0, 5, P (C) = P (A ∩B) = 0, 3, P (A ∩ C) = 0

e P (B ∩ C) = 0, 1, determine:

a) P (A ∪B ∪ C).

b) P [A− (B ∪ C)].

2

Unidade 19Problemas de Probabilidade Lista I

c) P [A ∩ (B ∪ C)].

d) P [(A ∩B) ∪ C].

8. Em certa escola a probabilidade de um aluno ser torcedor do Flamengo

é 0,60, de assistir novela é 0,70 e de gostar de praia é 0,80. Entre

que valores está compreendida a probabilidade de um aluno dessa escola,

simultaneamente: a) assistir novela e gostar de praia. b) torcer pelo

Flamengo.

9. Laura e Telma retiram cada uma um bilhete numerado de uma urna que

contém bilhetes numerados de 1 a 100. Determine a probabilidade do

número de Laura ser maior que o de Telma, supondo a extração:

a) sem reposição.

b) com reposição.

10. Em uma gaveta há 10 pilhas, das quais duas estão descarregadas. Testando-

se as pilhas uma a uma até serem identicadas as duas descarregadas,

determine a probabilidade de serem feitos:

a) cinco testes.

b) mais de cinco testes.

c) menos de cinco testes.

3

20

1

Problemas de

Probabilidade

Lista II

Unidade 20

Problemas

Lista 2

1. 2n jogadores de igual habilidade disputam um torneio. Eles são divididos

em grupos de 2, ao acaso, e jogadores de um mesmo grupo jogam entre

si. Os perdedores são eliminados e os vencedores são divididos novamente

em grupos de 2 e assim por diante até restar apenas um jogador que é

proclamado campeão. Qual é a probabilidade de dois jogadores A e B

se enfrentarem durante o torneio. Qual é a probabilidade do jogador A

jogar exatamente k partidas?

2. Em um torneio como o descrito no exercício anterior, os 16 jogadores têm

habilidades diferentes e não há surpresas nos resultados (se A é melhor

que B, A vence B).

a) Qual é a probabilidade do segundo melhor jogador ser vice-campeão

do torneio?

b) Qual é a probabilidade do quarto melhor jogador ser vice-campeão

do torneio?

c) Qual é o número máximo de partidas que o décimo melhor jogador

consegue disputar? Qual é a probabilidade dele disputar esse número

máximo de partidas?

3. Em um programa da televisão italiana, os candidatos devem escolher uma

entre três portas. Atrás de uma dessas portas há um prêmio e atrás de

cada uma das outras duas portas há um bode. Escolhida uma porta

pelo candidato, o apresentador, que sabe onde estão os bodes, abre uma

das outras portas, atrás da qual se encontra um bode, e pergunta ao

candidato se ele quer car com a porta que escolheu ou se prefere trocá-

la pela outra porta que ainda está fechada. Admitindo que, quando o

candidato escolhe a porta em que está o prêmio, o apresentador escolha

ao acaso uma porta para abrir, você acha que o candidato deve trocar,

não deve trocar ou que tanto faz?

4. Qual é a probabilidade de serem obtidas exatamente 5 caras em 10 lan-

çamentos de uma moeda não-tendenciosa?

2

Unidade 20Problemas de Probabilidade Lista II

5. Uma urna contém 4 bolas brancas e 6 bolas pretas. Sacam-se sucessiva-

mente bolas dessa urna de acordo com o seguinte processo: cada vez que

uma bola é sacada, ela é devolvida à urna e são acrescentadas mais duas

bolas da mesma cor que ela. Determine a probabilidade de:

a) a segunda bola sacada ser branca.

b) a primeira bola sacada ter sido branca na certeza de que a segunda

bola sacada foi preta.

6. Um juiz de futebol meio trapalhão tem no bolso um cartão amarelo, um

cartão vermelho e um cartão com uma face amarela e uma face vermelha.

Depois de uma jogada violenta, o juiz mostra um cartão, retirado do bolso

ao acaso, para um atleta. Se a face que o jogador vê é amarela, qual é a

probabilidade da face voltada para o juiz ser vermelha?

7. A e B disputam uma série de partidas. Ganha um prêmio quem primeiro

completar 10 vitórias. A é mais habilidoso do que B, sendo de 0,6 a

probabilidade de A ganhar uma partida e de 0,4 a probabilidade de B

ganhar uma partida. No momento o placar está 7×4 a favor de B. Qual

é a probabilidade de A ganhar o prêmio?

8. Três jogadores, A, B e C, disputam um torneio. Os três têm probabili-

dades iguais de ganhar o torneio; têm também probabilidades iguais de

tirarem o segundo lugar e têm probabilidades iguais de tirarem o último

lugar. É necessariamente verdadeiro que cada uma das seis ordens possí-

veis de classicação dos três jogadores tem probabilidade1

6de ocorrer?

Justique.

9. Selecionam-se ao acaso dois pontos em uma circunferência. Qual a pro-

babilidade da corda determinada por esses pontos ter comprimento maior

do que o lado do triângulo equilátero inscrito na circunferência?

10. Seleciona-se ao acaso um ponto X em um diâmetro AB de uma circunfe-

rência. Qual a probabilidade da corda que contém X e é perpendicular a

AB ter comprimento maior do que o lado do triângulo equilátero inscrito

na circunferência?

3

Unidade 20

11. Cristina e Maria, que não são pessoas muito pontuais, marcaram um

encontro às 16 horas. Se cada uma delas chegará ao encontro em um

instante qualquer entre 16 e 17 horas e se dispõe a esperar no máximo

10 minutos pela outra, qual é a probabilidade delas se encontrarem?

4

21

1

Médias e Princípio dasGavetas I

Sumário

21.1 Introdução . . . . . . . . . . . . . . . . . . . . . . . 2

21.2 Médias . . . . . . . . . . . . . . . . . . . . . . . . . 2

Unidade 21 Introdução

21.1 Introdução

Nessa unidade, são apresentadas algumas noções de médias, como as médias

aritméticas, geométricas, harmônicas, quadráticas e ponderadas.

Em seguida, é apresentado, como aplicação de cálculo de médias, o importante

Princípio das Gavetas de Dirichlet que possui inúmeras aplicações e que se

enuncia como se segue:

Se n+ 1 ou mais objetos são colocados em n ou menos gavetas, então pelo

menos uma gaveta recebe mais de um objeto.

Esse princípio, também chamado de Princípio da Casa dos Pombos (consegue

imaginar por quê?), tem inúmeras aplicações, algumas das quais surpreendentes.

Você achará algumas delas na lista de exercícios propostos no nal da unidade.

21.2 Médias

Uma ideia bastante importante é a ideia de média. Uma média de uma lista de

números é um valor que pode substituir todos os elementos da lista sem alterar

uma certa característica da lista. Se essa característica é a soma dos elementos

da lista, obtemos a mais simples de todas as médias, a média aritmética. A

média aritmética (simples) da lista de n números x1, x2, . . . , xn é um valor x

tal que x1 +x2 + · · ·+xn = x+ x+ · · ·+ x = nx. Portanto, temos a seguinte

denição:

Definição 1Média Aritmética

A média aritmética (simples) da lista de n números x1, x2, . . . , xn é denida

por

x =x1 + x2 + · · ·+ xn

n.

Por exemplo, a média aritmética dos números 3, 36 e 54 é

3 + 36 + 54

3= 31.

Se a característica a ser considerada for o produto dos elementos da lista,

obteremos a média geométrica. A média geométrica (simples) dos n números

2

Unidade 21Médias e Princípio das Gavetas I

positivos x1, x2, . . . , xn é um valor positivo g tal que x1x2 · · ·xn = g · g · · · g =

gn. Portanto, temos a seguinte denição:

Definição 2Média Geométrica

A média geométrica (simples) dos n números positivos x1, x2, . . . , xn é

denida por

g = G(x1, x2, . . . , xn) = n√x1x2 . . . xn.

Observe que só denimos a média geométrica para números positivos. Assim

evitamos a possibilidade da média não existir (por exemplo, qual seria a média

geométrica entre 2 e −2?).

Por exemplo, a média geométrica dos números 3, 36 e 54 é 3√

3 · 36 · 54 =

18.

Se a característica for a soma dos inversos dos elementos da lista, obteremos

a média harmônica. A média harmônica (simples) dos n números positivos

x1, x2, . . . , xn é um valor h tal que

1

x1+

1

x2+ · · ·+ 1

xn=

1

h+

1

h+ · · ·+ 1

h=n

h.

Portanto, temos a seguinte denição:

Definição 3Média Harmônica

A média harmônica (simples) dos n números positivos x1, x2, . . . , xn é

denida por

h =n

1x1

+ 1x2

+ · · ·+ 1xn

.

A média harmônica é, pois, o inverso da média aritmética dos inversos dos

números.

Por exemplo, a média harmônica dos números 3, 36 e 54 é

313

+ 136

+ 154

=3

36+3+2108

=3× 108

41=

324

41∼= 7, 9.

Observe que só denimos a média harmônica para números positivos. Assim

evitamos a possibilidade da média não existir (por exemplo, qual seria a média

harmônica entre 2 e −2?).

3

Unidade 21 Médias

Exemplo 1 Uma empresa produziu, durante o primeiro trimestre do ano passado, 500,

200 e 200 unidades, em janeiro, fevereiro e março, respectivamente. Qual foi a

produção média mensal nesse trimestre?

Comentário. Resista à tentação de tirar rapidamente a média aritmética e

ponto nal. Você sempre corre o risco de um aluno perguntar porque não

podia ter tirado a média geométrica.

Solução. Que média é essa que queremos? Queremos uma média M tal que,

se a produção mensal fosse sempre igual a M , a produção trimestral seria a

mesma. A produção trimestral foi de 500 + 200 + 200. Se em todos os meses

a produção fosse igual a M , a produção trimestral seria igual a 3M . Logo,

3M = 500 + 200 + 200 e

M =500 + 200 + 200

3= 300.

A média desejada era a média aritmética.

Resposta: 300.

Exemplo 2 Uma empresa aumentou sua produção durante o primeiro bimestre do ano

passado. Em janeiro e em fevereiro, as taxas de aumento foram de 21% e 8%,

respectivamente. Qual foi a taxa média de aumento mensal nesse bimestre?

Comentário. A resposta não é (21% + 8%)÷ 2 = 14, 5%.

Solução. Que média queremos? Queremos uma taxa média i tal que, se em

todos os meses a taxa de aumento fosse igual a i, o aumento bimestral seria o

mesmo. O aumento bimestral foi de 30,68%, conforme mostra o esquema

100 7−→ 100 · 1, 21 7−→ 100 · 1, 21 · 1, 08 = 130, 68.

Se em todos os meses tivéssemos um aumento de taxa i, teríamos

100 7−→ 100(1 + i) 7−→ 100(1 + i)2.

Então,

100(1 + i)2 = 100 · 1, 21 · 1, 08

(1 + i)2 = 1, 21 · 1, 08

1 + i =√

1, 21 · 1, 08 ∼= 1, 1432

i ∼= 0, 1432 = 14, 32%.

4

Unidade 21Médias e Princípio das Gavetas I

A média procurada era uma média geométrica. Mais precisamente: a taxa

média, aumentada de uma unidade, é a média geométrica das taxas mensais

aumentadas de uma unidade.

Exemplo 3Um concurso anual distribui igualmente entre os vencedores um prêmio

total de R$ 1 800,00. Nos últimos três anos houve 2, 1 e 3 premiados, respec-

tivamente. Qual foi o prêmio médio desses ganhadores?

Comentário. Embora o número médio de ganhadores tenha sido igual a 2, o

prêmio médio não foi de R$ 1800, 00 ÷2 = R$ 900, 00.

Solução. Queremos uma média tal que, se todos os prêmios fossem iguais

a essa média, o total distribuído seria o mesmo. Essa é precisamente a média

aritmética. Os prêmios foram de 1800÷2 = 900, 1800÷1 = 1800 e 1800÷3 =

600. O prêmio médio foi de (900 + 1800 + 600)÷ 3 = 1100 reais.

Observe que a média aritmética dos rateios é igual a

1800× 12

+ 1800× 11

+ 1800× 13

3

= 1800×12

+ 11

+ 13

3

= 1800÷ 312

+ 11

+ 13

e que3

12

+ 11

+ 13

é a média harmônica dos números de ganhadores.

O rateio médio é o rateio que corresponderia a uma quantidade de ganha-

dores igual à média harmônica dos números de ganhadores.

Outra média importante é a média quadrática.

Definição 4Média quadrática

A média quadrática dos números x1, x2, . . . , xn é denida por

q =

√x21 + x22 + · · ·+ x2n

n,

isto é, a média quadrática é a raiz quadrada da média aritmética dos quadrados

dos números.

5

Unidade 21 Médias

Por exemplo, a média quadrática dos números 1 e 7 é√12 + 72

2= 5.

Exemplo 4 A qualidade de uma aproximação é medida pelo seu erro, que é a diferença

entre o valor da aproximação e o valor real da grandeza. Por exemplo, 4 é uma

aproximação de 3,8 com erro de 0,2 (também se diz uma aproximação de 3,8

por excesso, com erro de 0,2) e 5,5 é uma aproximação de 5,7 com erro de

−0, 2 (ou uma aproximação de 5,7 por falta, com erro de 0,2). Evidentemente,

quanto mais próximo de zero estiver o erro, tanto melhor será a aproximação.

Assim, por exemplo, 39 é uma aproximação de 40 (erro igual a −1) que é melhor

do que a aproximação 42 (erro igual a 2).

Mede-se a qualidade de uma lista de aproximações pela média quadrática

dos seus erros. Também se usa o erro médio quadrático, que é o quadrado

dessa média quadrática, ou seja, é a média aritmética dos quadrados dos erros.

Abaixo temos duas listas de aproximações do número 4:

S1 : 3; 4, 5; 3, 6 S2 : 3, 2; 4, 8

Os erros médios quadráticos são respectivamente iguais a

12 + 0, 52 + 0, 42

3= 0, 47 e

0, 82 + 0, 82

2= 0, 64.

S1 é uma lista de aproximações de 4 que é melhor do que S2.

Uma importante propriedade da média aritmética é:

Se a média aritmética dos números x1, x2, . . . , xn é igual a x, pelo menos

um dos números x1, x2, . . . , xn é maior que ou igual a x.

Com efeito, se fosse x1 < x, x2 < x, · · · , xn < x, teríamos

x1 + x2 + · · ·+ xn < nx,x1 + x2 + · · ·+ xn

n< x, x < x,

o que é absurdo.

Exemplo 5 Mostre que, em um grupo de 50 pessoas, há sempre pelo menos 5 que

nasceram no mesmo mês.

Solução. O número médio de pessoas por mês é 50÷ 12 = 4, 1 . . .. Logo, em

6

Unidade 21Médias e Princípio das Gavetas I

algum mês o número de nascidos nesse mês (que é um inteiro) é maior que ou

igual a 4,1..., ou seja, é maior que ou igual a 5.

Uma consequência imediata do exemplo 5 é o Princípio das Gavetas de

Dirichlet1:

Proposição 5Princípio das Gavetas

Se n+1 ou mais objetos são colocados em n ou menos gavetas, então pelo

menos uma gaveta recebe mais de um objeto.

DemonstraçãoProva. O número médio de objetos por gaveta é maior que ou igual an+ 1

n, que é maior que 1. Logo, em alguma gaveta haverá um número de

objetos maior que 1.

Exemplo 6Mostre que todo inteiro positivo n tem um múltiplo que se escreve apenas

com os algarismos 0 e 1.

Solução. Considere os n + 1 primeiros números da sequência 1, 11, 111,... .

Divida-os por n e considere os restos dessas divisões. Esses restos só podem

ser iguais 0, 1, 2, . . . , n− 1.

Pensando nos números como objetos e nos restos como gavetas, temos mais

objetos do que gavetas. O Princípio das Gavetas assegura que alguma gaveta

receberá mais de um objeto, isto é, há dois números na sequência que dão o

mesmo resto quando divididos por n, digamos 11 ... 1 (p algarismos) e 11 ...

1 (q algarismos), p < q. A diferença desses números é um múltiplo de n e se

escreve 11 . . . 10 . . . 0, com p algarismos 0 e q − p algarismos 1.

Exemplo 7Cinco pontos são tomados sobre a superfície de um quadrado de lado 2.

Mostre que há dois desses pontos tais que a distância entre eles é menor que

ou igual a√

2.

Solução. Divida o quadrado de lado 2 em quatro quadrados de lado 1, ligando

os pontos médios dos lados opostos. Pensando nos pontos como objetos e nos

quadrados como gavetas, temos mais objetos do que gavetas. O Princípio das

1Peter Gustav Lejeune Dirichlet (1805-1859), matemático alemão.

7

Unidade 21 Médias

Gavetas assegura que alguma gaveta receberá mais de um objeto, isto é, haverá

dois pontos no mesmo quadrado de lado 1. A distância entre esses pontos é no

máximo igual ao comprimento da diagonal do quadrado, que é√

2.

Exemplo 8 Um enxadrista, durante 11 semanas, joga pelos menos uma partida por dia

mas não joga mais de 12 partidas por semana. Mostre que é possível achar

um conjunto de dias consecutivos durante os quais ele jogou exatamente 20

partidas.

Solução. Em 11 semanas temos 77 dias. Chamemos de Sk, k = 1, 2, . . . , 77, o

número de partidas jogadas desde o primeiro até o k-ésimo dia, inclusive. Como

ele joga pelo menos uma partida por dia, temos 1 6 S1 < S2 < · · · < S77.

Além disso, S77 6 132 pois ele não joga mais de 12 partidas por semana.

Denindo S0 = 0, a quantidade de partidas jogadas do dia p ao dia q,

inclusive, é igual a Sq − Sp−1. Queremos mostrar que é possível determinar p e

q de modo que Sq − Sp−1 = 20.

Considere os 154 números

S1, S2, . . . , S77, S1 + 20, S2 + 20, . . . , S77 + 20.

Eles pertencem a 1, 2, . . . , 152. O Princípio das Gavetas assegura que

dois desses números são iguais. Como S1 < S2 < · · · < S77, os números iguais

devem estar em metades diferentes dessa lista de 154 números. Então existem

m e n tais que Sm = Sn +20. O enxadrista joga 20 partidas entre os dias n+1

e m, inclusive.

Finalmente, denimos médias ponderadas.

Definição 6Média Ponderada

A média aritmética ponderada dos números x1, x2, . . . , xn com pesos res-

pectivamente iguais a p1, p2, . . . , pn é denida por

p1x1 + p2x2 + · · ·+ pnxnp1 + p2 + · · ·+ pn

.

Embora a ideia primitiva seja que a média aritmética ponderada é uma média

aritmética simples de uma lista de números dos quais p1 são iguais a x1, p2 são

8

Unidade 21Médias e Princípio das Gavetas I

iguais a x2,. . ., pn são iguais a xn, não há problema em considerar pesos não

inteiros.

Aliás, é bastante útil trabalhar com pesos relativos e considerar a média arit-

mética ponderada dos números x1, x2, . . . , xn, com pesos iguais a p1, p2, . . . , pn,

respectivamente, como sendo

p1p1 + p2 + · · ·+ pn

x1

+p2

p1 + p2 + · · ·+ pnx2

+ · · ·+ pnp1 + p2 + · · ·+ pn

xn.

Assim, uma média aritmética ponderada dos números x1, x2, . . . , xn é uma

expressão da forma λ1x1 + λ2x2 + · · ·+ λnxn, onde

λ1 + λ2 + · · ·+ λn = 1.

Exemplo 9Em um grupo de pessoas, 70% das pessoas são adultos e 30% são crianças.

O peso médio dos adultos é 70kg e o peso médio das crianças é de 40kg. Qual

o peso médio do grupo?

Solução. É a média aritmética ponderada dos dois subgrupos, com pesos

relativos de 0,7 e 0,3. A resposta éé 0, 7× 70 + 0, 3× 40 = 61kg.

9

Unidade 21 Médias

Exercícios recomendados

1. Um carro percorre metade de certa distância d com velocidade v1 e per-

corre a outra metade com velocidade v2. Qual a sua velocidade média?

2. Um carro tem velocidade v1 durante metade do tempo t de percurso e tem

velocidade v2 durante a outra metade do tempo. Qual a sua velocidade

média?

3. A população de um país cresceu 44% em uma década e cresceu 21%

na década seguinte. Qual é, aproximadamente, a taxa média decenal de

crescimento nesses 20 anos?

4. No problema anterior, qual a taxa média anual de crescimento nesses 20

anos?

5. A valorização mensal das ações de certa empresa nos quatro primeiros

meses do ano foi de +25%, +25%, −25% e −25%. Qual a valorização

total e qual a valorização média mensal nesse quadrimestre?

6. Em uma cela há três túneis. Um conduz à liberdade em 3 horas; outro,

em 5 horas, e o último conduz ao ponto de partida depois de 9 horas.

Qual o tempo médio que os prisioneiros que descobrem os túneis gastam

para escapar?

7. Suponha que, no problema anterior, os prisioneiros que entram pelo ter-

ceiro túnel, quando voltam ao ponto de partida, não se lembram de qual

foi o túnel em que entraram e, portanto, escolhem para a próxima tenta-

tiva um entre os três túneis.

8. Prove que a média aritmética x de uma lista de números satisfaz m 6 x 6

M , onde m e M são respectivamente o menor e o maior dos números.

9. Prove que a média geométrica g de uma lista de n números positivos

satisfaz m 6 g 6 M , onde m e M . são respectivamente o menor e o

maior dos números.

10

Unidade 21Médias e Princípio das Gavetas I

10. Prove que a média harmônica h de uma lista de n números positivos

satisfaz m 6 h 6 M , onde m e M . são respectivamente o menor e o

maior dos números.

11. Em um concurso, havia apenas provas de Português e Matemática. O

resultado do concurso está no quadro abaixo.

Candidato Port. Mat. Classicação

João 5 7 2

Pedro 6 4 1

José 2 5 4

Paulo 4 1 3

João achou que havia erro na classicação porque zera mais pontos

que Pedro e classicara-se atrás dele. Houve necessariamente erro na

classicação?

12. Pneus novos duram 40 000 km, quando usados nas rodas dianteiras, e

duram 60 000 km, quando usados nas rodas traseiras.

a) Com 4 pneus novos e fazendo um rodízio adequado entre eles, quan-

tos quilômetros um carro pode rodar? Como?

b) E com 5 pneus novos? Como?

c) A resposta do item a) é uma média entre 40 000 km e 60 000 km.

Qual?

13. A média aritmética de 50 números é 40. Se dois desses números, 125 e 75,

forem suprimidos, qual será a média aritmética dos números restantes?

14. Qual a característica conservada pela média quadrática?

15. Prove que a média quadrática q de uma lista de n números positivos

satisfaz m 6 q 6 M , onde m e M são respectivamente o menor e o

maior dos números.

11

Unidade 21 Médias

Exercícios Suplementares

1. Prove que, para dois números positivos x1 e x2, suas médias aritmética

A, geométrica G, harmônica H e quadrática Q, satisfazem H 6 G 6

A 6 Q. Prove também que duas quaisquer dessas médias são iguais se e

somente se x1 = x2.

2. Qual seria o problema de se medir a qualidade de uma lista de aproxima-

ções pela média aritmética dos erros?

3. Para determinar uma grandeza desconhecida x, foram feitas várias medi-

ções. Os resultados obtidos foram x1, x2, . . . , xn. Determine a estimativa

de x para a qual o erro médio quadrático é mínimo.

4. Para determinar uma grandeza desconhecida x, foram feitas várias me-

dições. Os resultados obtidos foram x1, x2, . . . , xn tais que x1 6 x2 6

· · · 6 xn. Determine a estimativa de x para a qual a média dos valores

absolutos dos erros é mínima.

5. Eduardo observou que o consumo de energia elétrica em sua casa estava

aumentando muito. Fez então um gráco do consumo anual, em kWh, nos

últimos 5 anos, tomando 1991 como ano 0. Os valores obtidos encontram-

se no quadro abaixo e Eduardo achou que o gráco parecia-se com uma

reta.

ANO (x) 0 1 2 3 4

CONSUMO (y) 820 1000 1200 1350 1550

É fácil ver que os pontos encontrados não são colineares, mas pode-se

notar no gráco que é possível traçar retas que passem bem perto dos

cinco pontos. Mostrando o gráco a seus amigos Augusto e Sérgio, eles

sugeriram as retas y = 170x + 850 e y = 180x + 800, respectivamente,

como as retas que mais se aproximariam dos pontos.

a) Mostre que os pontos realmente não são colineares.

b) Calcule os erros médios quadráticos e determine qual das duas retas

mais se aproxima dos pontos.

12

Unidade 21Médias e Princípio das Gavetas I

c) Entre todas as retas do plano, qual é a que mais se aproxima dos

pontos?

6. Mostre que em qualquer conjunto de 8 inteiros há sempre dois deles cuja

diferença é um múltiplo de 7.

7. Em uma festa há 20 crianças sentadas em torno de uma mesa circular.

Um garçom coloca diante de cada criança, sem perguntar qual a sua

preferência, uma taça de sorvete. Alguns desses sorvetes são de creme e

os outros são de ocos. 10 das crianças preferem creme e 10 preferem

ocos. Mostre que, sem mexer nas crianças e fazendo apenas uma rotação

da mesa, é possível fazer com que pelo menos 10 crianças tenham suas

preferências respeitadas.

8. Mostre que em toda reunião de n pessoas há sempre duas pessoas com

o mesmo número de conhecidos.

9. Mostre que existe um múltiplo de 1997 cujos dígitos são todos iguais a

1.

10. Qual é o número mínimo de pessoas que deve haver em um grupo para

que possamos garantir que nele há pelo menos 7 pessoas nascidas no

mesmo mês?

11. São dados, no plano, cinco pontos de coordenadas inteiras. Mostre que,

entre os dez segmentos determinados por esses pontos, pelo menos um

tem como ponto médio um ponto de coordenadas inteiras.

12. Prove que se Nk + 1 objetos são colocados em N gavetas, pelo menos

uma gaveta recebe mais de k objetos.

13. 40100 candidatos estão fazendo uma prova de 20 questões de múltipla

escolha, com 5 alternativas por questão. Suponha que nenhum candidato

deixe de responder a nenhuma questão. Considere a armação: Pelo

menos k candidatos responderão de modo idêntico às 4 primeiras questões

da prova. Determine o maior valor de k para o qual a armação é

certamente verdadeira.

13

Unidade 21 Médias

14. 40100 candidatos estão fazendo uma prova de 20 questões de múltipla

escolha, com 5 alternativas por questão. Suponha que nenhum candidato

deixe de responder a nenhuma questão. Considere a armação: Pelo

menos 4 candidatos responderão de modo idêntico às k primeiras questões

da prova. Determine o maior valor de k para o qual a armação é

certamente verdadeira.

15. Os pontos de uma reta são coloridos com 11 cores. Mostre que é possível

achar dois pontos com a mesma cor tal que a distância entre eles é um

número inteiro.

16. Em um campeonato cada dois times jogam entre si uma única vez. Mostre

que em qualquer momento há sempre dois times que disputaram o mesmo

número de partidas.

17. Sete pontos são selecionados dentro de um retângulo 3 × 4. Prove que

há dois desses pontos tais que a distância entre eles é no máximo igual a√5.

18. Selecionam-se oito números distintos no conjunto 1, 2, . . . , 15. Mostre

que há pelo menos três pares de números selecionados com a mesma

diferença entre o maior e o menor número do par.

19. Sejam x1 e x2 números reais, x1 < x2.

a) Mostre que os números reais x tais que x1 < x < x2 podem ser

escritos na forma x = λ1x1 +λ2x2 com λ1 +λ2 = 1, λ1 e λ2 positi-

vos, isto é, são médias aritméticas ponderadas, com pesos positivos,

de x1 e x2. Essa representação é única?

b) Mostre que os números reais x da forma x = λ1x1 + λ2x2 com

λ1 + λ2 = 1, λ1 e λ2 positivos, pertencem a (x1, x2).

c) Onde estão os pontos x = λ1x1 +λ2x2, com λ1 +λ2 = 1 e λ1 > 1?

d) E com λ1 + λ2 = 1 e λ1 < 0?

20. Sejam x1, x2, . . . , xn números reais, x1 < x2 < · · · < xn, n > 2.

14

Unidade 21Médias e Princípio das Gavetas I

a) Mostre que os números reais x tais que x1 < x < xn podem ser

escritos na forma x = λ1x1 + λ2x2 + · · ·+ λnxn com

λ1 + λ2 + · · ·+ λn = 1, λ1, λ2, . . . , λn > 0.

Essa representação é única?

b) Mostre que os números reais x da forma

x = λ1x1 + λ2x2 + · · ·+ λnxn, n > 2

com λ1 + λ2 + · · · + λn = 1, λ1, λ2, . . . , λn positivos, pertencem

a (x1, xn).

21. Em um grupo de pessoas há 30 homens e 10 mulheres. Os homens têm

altura média de 1,75m e, as mulheres, de 1,67m. Qual a altura média do

grupo?

15

22

1

Médias e Princípio dasGavetas II

Sumário

22.1 A Desigualdade das Médias . . . . . . . . . . . . . . 2

Unidade 22 A Desigualdade das Médias

Nessa unidade, faz-se a comparação entre as várias médias, resultando numa

desigualdade fundamental entre a média aritmética, a média geométrica e a

média harmônica.

22.1 A Desigualdade das Médias

A desigualdade das médias arma que a média aritmética de n números positivos

é maior que ou igual à sua média geométrica e só é igual se os números forem

todos iguais. Isto é, se x1, x2, . . . , xn são números positivos

x1 + x2 + · · ·+ xn

n> n√x1x2 · · ·xn.

Além disso,x1 + x2 + · · ·+ xn

n= n√x1x2 · · ·xn

se e somente se x1 = x2 = · · · = xn.

Várias e interessantes demonstrações dessa desigualdade são encontradas

em Meu Professor de Matemática de Elon Lages Lima. Aqui faremos apenas

um esboço da demonstração que foi feita por Cauchy1.

Provaremos primeiramente a desigualdade no caso n = 2. Sendo A(x1, x2)

a média aritmética dos números positivos x1 e x2 e sendo G(x1, x2) sua média

geométrica, temos

A(x1, x2)−G(x1, x2) =x1 + x2

2−√x1x2

=x1 + x2 − 2

√x1x2

2

=(√x1 −

√x2)

2

2> 0

e A(x1, x2)−G(x1, x2) só é igual a 0 quando x1 = x2, o que prova a desigual-

dade no caso n = 2.

Para prová-la no caso n = 4, aplicamos o resultado anterior aos números

x1 + x2

2e

x3 + x4

2,

obtendox1+x2

2+ x3+x4

2

2>

√(x1 + x2

2

)(x3 + x4

2

),

1Cauchy, Louis (1789-1857), matemático francês.

2

Unidade 22Médias e Princípio das Gavetas II

ou seja,

x1 + x2 + x3 + x4

4>

√(x1 + x2

2

)(x3 + x4

2

),

a igualdade só sendo obtida quando

x1 + x2

2e

x3 + x4

2

forem iguais. Aplicando agora duas vezes a desigualdade no caso n = 2, pri-

meiramente para x1 e x2, e posteriormente para x3 e x4, obtemos√x1 + x2

2

x3 + x4

2>√√

x1x2

√x3x4 = 4

√x1x2x3x4,

a igualdade sendo obtida apenas quando x1 = x2 e x3 = x4.

Portanto,x1 + x2 + x3 + x4

4> 4√x1x2x3x4,

a igualdade só sendo obtida quando x1 = x2 e x3 = x4 e

x1 + x2

2=

x3 + x4

2,

isto é, quando x1 = x2 = x3 = x4.

É claro que, repetindo esse argumento, provaríamos a desigualdade das

médias para 8, 16, 32,... números positivos.

Esse argumento permite provar, por indução, a desigualdade para n = 2k

números positivos.

Provaremos agora a desigualdade para três números positivos.

Sejam x1, x2 e x3 números positivos e sejam A sua média aritmética e G

sua média geométrica. É claro que

x1 + x2 + x3 + A

4=

3A+ A

4= A.

Aplicando a desigualdade das médias no caso n = 4 aos números x1, x2, x3 e

A, obtemos

A =x1 + x2 + x3 + A

4> 4√

x1x2x3A.

A4 > x1x2x3A, A3 > x1x2x3, A > 3√x1x2x3 = G a igualdade só se vericando

quando x1 = x2 = x3 = A, isto é, quando x1 = x2 = x3. Se desejássemos

3

Unidade 22 A Desigualdade das Médias

provar a desigualdade para cinco números positivos x1, x2, x3, x4 e x5, aplica-

ríamos a desigualdade aos 8 números x1, x2, x3, x4 e x5, A, A e A, onde A é

a média aritmética dos números x1, x2, x3, x4 e x5.

O mesmo raciocínio pode mostrar que, se a desigualdade é verdadeira para

n = k, então ela é também verdadeira para todo n < k.

Exemplo 1 Mostre que, entre todos os retângulos de perímetro 2p, o quadrado é o de

maior área.

Solução. Se os lados do retângulo são x e y, temos x+ y = p, isto é, a média

aritmética x e y é igual ap

2. A área do retângulo é A = xy. Temos

√A =

√xy 6

x+ y

2=

p

2.

Portanto,

A 6p2

4

e a igualdade só é obtida quando x = y. Portanto, o retângulo de maior área

é o quadrado de área p2/4.

Exemplo 2 Mostre que, entre todos os retângulos de área A, o quadrado é o de menor

perímetro.

Solução. Se os lados do retângulo são x e y, temos xy = A, isto é, a média

geométrica de x e y é igual a√A. O perímetro do retângulo é 2(x+y). Temos

2(x+ y) = 4x+ y

2> 4√xy = 4

√A.

Portanto, 2(x+ y) > 4√A e a igualdade só é obtida quando x = y. Portanto,

o retângulo de menor perímetro é o quadrado de perímetro 4√A.

A desigualdade das médias pode ser generalizada como segue:

Se x1, x2, . . . , xn são números positivos e Q, A, G e H são suas médias

quadrática, aritmética, geométrica e harmônica, respectivamente, então Q >

A > G > H. Além disso, duas quaisquer dessas médias são iguais se e somente

se x1 = x2 = · · · = xn.

4

Unidade 22Médias e Princípio das Gavetas II

Exercícios Recomendados

1. Prove que o produto de dois números de soma constante é máximo quando

esses números são iguais.

2. Prove que a soma de dois números positivos de produto constante é

mínima quando esses números são iguais.

3. Prove que a média harmônica de n números positivos

x1, x2, . . . , xn

é sempre menor que ou igual a sua média geométrica e só é igual quando

todos os números são iguais.

4. Prove que a média quadrática de n números positivos

x1, x2, . . . , xn

é sempre maior que ou igual a sua média aritmética e só é igual quando

todos os números são iguais.

5. Prove que se a1, a2, . . . , an são números positivos e

b1, b2, . . . , bn

é uma reordenação de a1, a2, . . . , an entãob1a1

+b2a2

+ · · ·+ bnan

> n.

6. Prove que x2 + y2 + z2 > xy + yz + zx, para quaisquer x, y e z reais.

7. Prove que se a1, a2, a3 são positivos, então

a1 + a2 + a33

>

√a1a2 + a1a3 + a2a3

3> 3√a1a2a3.

8. Mostre que se a equação x3 − ax2 + bx − c = 0, na qual a, b e c são

números positivos possuir três raízes reais então a6 > 27b3 > 729c2.

9. Um mágico se apresenta usando um paletó cintilante e uma calça colorida

e não repete em suas apresentações o mesmo conjunto de calça e paletó.

Para poder se apresentar em 500 espetáculos, qual o menor número de

peças de roupa que pode ter seu guarda-roupa?

5

Unidade 22 A Desigualdade das Médias

10. Prove que entre todos os triângulos de perímetro constante, o equilátero

é o de maior área.

11. a) Prove que, se x é positivo, então x+1

x> 2.

b) Qual o valor mínimo dex+4

x, x positivo?

12. Prove que a sequência de termo geral an =

(1 +

1

n

)n

é estritamente

crescente, isto é, prove que, para todo n inteiro e positivo

(1 +

1

n

)n

<(1 +

1

n+ 1

)n+1

.

Exercícios Suplementares

1. Prove que, se x, y e z são positivos, então

1

x+

1

y+

1

z>

9

x+ y + z.

2. Prove que, se x, y e z são positivos, então

(√x+√y +√z)2 > 9 3

√xyz.

3. Se x, y e z são números positivos tais que 1 6 xy+ yz + zx 6 3, qual é

o conjunto de valores de xyz? E de x+ y + z?

4. Se x, y e z são números positivos tais que xy + yz + zx 6 3, qual é o

conjunto de valores de xyz? E de x+ y + z?

5. Se x, y e z são números positivos tais que xy + yz + zx 6 1, qual é o

conjunto de valores de xyz? E de x+ y + z?

6. Se x, y e z são números positivos tais que 1 6 x + y + z 6 3, qual é o

conjunto de valores de xyz? E de xy + yz + zx?

7. Se x, y e z são números positivos tais que 1 6 xyz 6 3, qual é o conjunto

de valores de xy + yz + zx? E de x+ y + z?

6

Unidade 22Médias e Princípio das Gavetas II

8. Se x, y e z são números positivos tais que xyz = 8, qual é o conjunto de

valores de xy + yz + zx? E de x+ y + z?

9. Prove que, se a desigualdade das médias é válida para m números posi-

tivos, m > 2, então ela é válida também para m− 1 números positivos.

7

Exercıcios de MA 12Sugestoes e Respostas

Unidades 5 e 6

Unidade 5 - Exercıcios Recomendados

1. O aumento de um triangulo causa o aumento de 2 palitos. Logo, onumero de palitos constitui uma progressao aritmetica de razao 2. an =a1 + (n− 1)r = 3 + (n− 1)2 = 2n+ 1.

4. A soma de todos os elementos da matriz e 1 + 2 + · · ·+ n2 =(n2 + 1)n2

2.

Como a soma de todos os elementos e igual a n vezes a constante magica,esta vale

C =1

n· (n2 + 1)n2

2=n(n2 + 1)

2.

5. Considerando a menor e a maior das medias que podem ser obtidas,

1 + 2 + · · ·+ (n− 1)

n− 16 16, 1 6

2 + 3 + · · ·+ n

n− 1.

n

2≤ 16, 1 ≤ n+ 2

2

30, 2 ≤ n ≤ 32, 2

n so pode valer 31 ou 32.

7. A soma pedida e a soma de uma progressao aritmetica de razao 1, com

1

primeiro termo igual a 10n−1 e ultimo termo igual a 10n − 1.

S =(10n−1 + 10n − 1)(10n − 10n−1)

2=

102n − 102n−2 − 10n + 10n+1

2

= 12[102n + 10n−1]− 1

2[102n−2 + 10n]

= 12

n−1︷ ︸︸ ︷1000 . . . 00

n︷ ︸︸ ︷1000 . . . 00−1

n−4︷ ︸︸ ︷1000 . . . 001

n+1︷ ︸︸ ︷1000 . . . 00

= 12· 98

n−3︷ ︸︸ ︷999 . . . 99

n︷ ︸︸ ︷1000 . . . 00 = 494

n−3︷ ︸︸ ︷999 . . . 99 55

n−1︷ ︸︸ ︷000 . . . 00 .

8. Quem disser 55 ganha o jogo, pois nao permite ao adversario alcancar63 e, escolhendo o complemento para 8 do numero escolhido pelo adversario,alcancara 63.

Analogamente, as posicoes ganhadoras sao 63, 55, 47, 39, 31, 23, 15,7. O primeiro jogador tem a estrategia ganhadora: comecar dizendo 7 e, apartir daı, escolher sempre o complemento para 8 do numero escolhido peloadversario.

9. O Botafogo joga 23 partidas, o Santos joga (sem contar a partida contrao Botafogo, ja contada) 22 vezes etc. A resposta e

23 + 22 + 21 + · · ·+ 1 + 0 =(23 + 0) · 24

12= 276.

10. Veja a “pizza de Steiner” na Unidade 4.

11a. Um ano nao-bissexto tem 52 semanas e 1 dia; um ano bissexto tem 52semanas e 2 dias. Logo, o ano x+ 1 comeca um dia da semana adiantado emrelacao ao ano x, se x nao e bissexto, e dois dias adiantado, se x e bissexto.

De 1997 a 2500 sao multiplos de 4 os anos 2000, 2004, 2008, . . ., 2496,num total de 125 anos. Mas 2100, 2200, 2300 nao sao bissextos por seremmultiplos de 100, mas nao de 400. Ha, portanto, 122 anos bissextos.

11b. Se o ano de 1997 comecou numa quarta-feira, o ano de 2500 comecara(2500 -1997)+122=625 dias de semana depois. Como 625 = 7 × 89 + 2, oano de 2500 comecara numa sexta-feira.

11c. Em cada bloco de 400 anos ha 100 anos que sao multiplos de 4 e, destes,3 nao sao bissextos por serem multiplos de 100, mas nao de 400. A respostae 97

400= 0, 2425.

2

14a.∑

3k = 12

∑nk=1 3k(3− 1) = 1

2

∑nk=1 ∆3k =

3n+1 − 3

4.

14b.∑n

k=1 kk! =∑n

k=1[(k + 1)k!− k!] =∑nk=1[(k + 1)!− k!] =

∑nk=1 ∆k! = (n+ 1)!− 1.

14c.n∑k=1

1

k(k + 1)=

n∑k=1

(1

k− 1

k + 1

)= −

n∑k=1

∆1

k=

−(

1

n+ 1− 1

)=

n

n+ 1.

Unidade 5 - Exercıcios Suplementares

1. A soma dos angulos internos de um pentagono convexo e 540o.

2. −x− (3− x) =√

9− x− (−x).

3. Do inteiro a (inclusive) ao inteiro b (inclusive), ha b− a+ 1 inteiros.

4. Faca um diagrama para os conjuntos X = x ∈ Z : 100 6 x 6 500,A = x ∈ X : x e divisıvel por 2, B = x ∈ X : x e divisıvel por 3e A = x ∈ X : x e divisıvel por 5. Queremos determinar o numero deelementos do complementar de A ∪B ∪ C em relacao ao universo X.

5. Se para passar do 32 para o 227 e para o 942 avancamos respectivamente

p e q termos, temos 227 − 32 + pr e 942 = 32 + qr. Daı,p

q=

195

910. Como p

e q sao inteiros positivos, e facil descobrir todos os valores possıveis para p e

q; basta descobrir todas as fracoes que sao iguais a195

910.

6. Se 100 = (a+1)+(a+2)+ · · ·+(a+n), com n > 1, 100 =(2a+ n+ 1)n

2.

Daı se conclui que (2a+n+ 1)n = 200 e tanto nquanto 2a+n+ 1 devem serdivisores de 200. Para evitar muitas contas, note tambem que sempre umdos numeros n e 2a+ n+ 1 e ımpar.

7. Uma solucao muito bonita pode ser obtida pensando nos pontos riscadoscomo vertices de um polıgono. Uma solucao “normal” pode ser obtida ob-servando que o ultimo numero riscado na primeira volta e 991, o primeiroriscado na segunda volta e 6, etc...

3

10. Faca a soma de todas as fracoes e subtraia a soma das redutıveis, quesao as que tem numeradores multiplos de 2 ou 3. Um diagrama de conjuntosajuda.

11. Voce pode substituir 1000! = 1× 2× 3× 4× 5× · · · × 1000 por 7× 14×21× · · · × 994 = 7142(1× 2× 3× · · · × 142).

12a. Parta de (k+1)4 = k4 +4k3 +6k2 +4k+1 e proceda como no Exemplo13.

12b.n∑k=1

(2k − 1)(3k + 1) =n∑k=1

(6k2 − k − 1).

13a. bxc = k, k > 0, se e somente se k 6 x < k + 1. b√xc = k, k > 0 se,

e somente se, k2 6 x < k2 + 2k + 1. Ha portanto 2k + 1 inteiros positivos x

para os quais b√xc = k. A soma pedida e

n−1∑k=1

(2k + 1)k.

13d. se x e um inteiro positivo, √x = k, k 6 0, se e somente se k − 1

2<

√x < k +

1

2, ou seja, k2 − k +

1

4< x < k2 + k +

1

4, ou ainda k2 − k + 1 6

x 6 k2 + k. Ha 2k inteiros positivos x tais que √x = k.

15. O primeiro elemento da 31a linha e precedido por 1+2+ · · ·+30 termos.

18. Em algum momento o segundo jogador recebera a soma maior do queou igual a 49.

19. Considere a bobina formada por cırculos cujos raios formam uma pro-gressao aritmetica cuja razao e a espessura do papel.

20a. Se nao considerassemos as linhas suprimidas, terıamos a progressaoaritmetica dos numeros naturais e o primeiro elemento da linha k seria pre-

cedido por 1 + 2 + · · ·+ (k − 1) =k(k − 1)

2naturais, sendo, portanto, igual

a 1 +k(k − 1)

2=k2 − k + 2

2.

Com a supressao, o que passa a ser a linha k e a antiga linha 2k−1. Logo,

o primeiro elemento da linha k e(2k − 1)2 − (2k − 1) + 2

2= 2k2 − 3k + 2.

20b. Como na linha k ha 2k− 1 elementos, o elemento central e 2k2 − 3k +2 + (k − 1) = 2k2 − 2k + 1.

4

20c. Como os elementos da linha k formam uma progressao aritmetica, bastamultiplicar o termo medio pela quantidade de termos.A resposta e (2k2 − 2k + 1)(2k − 1) = 4k3 − 6k2 + 4k − 1.

20d. A soma pedida e

k∑n=1

(4n3 − 6n2 + 4n− 1) =k∑

n=1

[n4 − (n− 1)4] = k4 − 04 = k4.

21. Basta mostrar que an e an+1 sao polinomios de grau p cujos termos demaior grau sao identicos e cujos termos de grau p− 1 sao diferentes.

22. Se F (k) = apkp + ap−1k

p−1 + · · ·+ a1k + a0, entao

n∑k=1

F (k) = ap

n∑k=1

kp + ap−1

n∑k=1

kp−1 + · · ·+ a1

n∑k=1

k +n∑k=1

a0.

23. Os termos da primeira progressao sao da forma 2 + 3t, 0 6 t 6 110 e osda segunda sao da forma 7 + 5s, 0 6 s 6 30. Devemos ter 2 + 3t = 7 + 5s.Daı, 3t = 5(1 + s) e t deve ser multiplo de 5. Se t = 5k, s = 3k − 1. Aslimitacoes 0 6 t 6 110 e 0 6 s 6 30 dao origem a uma limitacao para k.

24. Procure primeiramente entender porque os calendarios de 1985 e 1991sao iguais. Em segundo lugar, note que, como ha mais anos nao-bissextosdo que bissextos, provavelmente a colecao ficara completa quando Benjamimtiver todos os calendarios de anos bissextos.

Unidade 6 - Exercıcios Recomendados

5. O valor, em mil reais, do carro com n anos de uso forma a progressaogeometrica na qual a0 = 18 e a4 = 12. Determine a1.

6. Sejam a, aq, aq2 os numeros.a+ aq + aq2 = 19a2 + a2q2 + a2q4 = 133.Daı, a(1 + q + q2) = 19a2(1 + q2 + q4) = 133

Dividindo, a(1− q + q2) =133

19= 7

5

Daı,1 + q + q2

1− q + q2=

19

7

q =3

2ou q =

2

3.

7. Se 2p− 1 e primo,os divisores de 2p−1(2p− 1) sao da forma 2α× (2p− 1)β,com α ∈ 0, 1, . . . , p − 1 e β ∈ 0, 1. Para calcular a soma dos divisores,some separadamente os divisores que tem β = 0 e os que tem β = 1.

8. A k-esima parcela da soma e 1 + 10 + 102 + · · ·+ 10k−1. A soma e igual a

n∑k=1

10k − 1

9=

1

9

n∑k=1

10k − n

9=

10

9· 10n − 1

9− n

9=

10n+1 − 10− 9n

81.

9. Em cada operacao, a quantidade de vinho diminui de1

p.

10a. 5 + 2 · 4

9· 5 + 2

(4

9

)2

· 5 + · · · = 5 +2 · 4

9· 5

1− 49

= 13 metros.

10b. O tempo que uma bola gasta, partindo do repouso, para cair de uma

altura h e√

2h/g. Como as alturas (em metros) das quedas sao 5,4

95,(

4

9

)2

5, . . ., supondo g = 10m/s2, os tempos de queda (em segundos) serao

1,2

3,

(2

3

)2

, . . ..

O tempo total de queda e 1 +2

3+

(2

3

)2

+ · · · = 1

1− 23

= 3 segundos.

A este tempo devemos adicionar o tempo gasto pela bola nas subidas,que e o mesmo, a excecao do 1s da queda inicial.

A resposta e 5s, aproximadamente.

11. lim an = 300 + 0, 3× 200 + 0, 32 × 300 + 0, 33 × 200 + . . ..

14. A2 = 5A.

Unidade 6 - Exercıcios Suplementares

8. Comece pela progressao aritmetica x−r, x, x+r. A progressao geometrica

sera x−r+1, x, x+r. Temos (x−r+1)+x+(x+r) = 19 ex

x− r + 1=x+ r

x.

6

9. Os numeros sao x− 6, x, x+ 6, x− 6 ex+ 6

x=x− 6

x+ 6.

10. O numero e 9 + 8 · 10 + 8 · 102 + · · ·+ 8 · 10n + 4 · 10n + · · ·+ 4 · 102n−1.

11. Cada operacao dobra o numero de folhas. Use 210 = 1024 ∼= 103.

14b. Sao duas progressoes geometricas.

14c. Sendo S a soma pedida, calculeS

2e subtraia.

14c. Sendo S a soma pedida, calcule xS e subtraia.

14e. Sao tres progressoes geometricas.

15. Os triangulos sao semelhantes e a razao de semelhanca de cada um parao anterior e sempre a mesma.

16. A abscissa do ponto assintotico e 2− 1 +1

2− 1

4+ . . .

19a. A expressao e igual a x12 . x

14 . x

18 . . .

20c. O que acontece com os pontos de abscissas1

3,

1

32,

1

33etc?

21. Tomando 1600 anos como unidade de tempo, a massa existente noinstante t e M(t) = M(0)0, 5t.

22. a = 1 + 10 + 102 + · · ·+ 10n−1 e b = 10n + 5.

23. Pn+1 =4

3e An+1 +

√3

12

(4

9

)n.

26a. Somacao por partes com ak+1 = k2 e ∆bk =

(1

2

)k26b. Somacao por partes com ak+1 = k e ∆bk = 2k.

7

Exercıcios Resolvidos de MA 12

Unidades 7 e 8

Unidade 7

Secao 4

Exercıcios Recomendados

1.

x2 = 2x1 + x0 = 2.1 + 1 = 3

x3 = 2x2 + x1 = 2.3 + 1 = 7

x4 = 2x3 + x2 = 2.7 + 3 = 17

x5 = 2x4 + x3 = 2.17 + 7 = 41

2. O numero maximo de regioes e determinado quando, para cada n, areta n+1 intersecta as n ja existentes. Neste caso, a nova reta subdividen+1 regioes, criando assim n+1 novas regioes. Logo, o numero maximode regioes xn determinado por n retas satisfaz xn+1 = xn+(n+1), paran = 0, 1, 2, . . . , com x0 = 1.

3. Por inducao sobre n. O valor de x1 esta bem definido, ja que x1 = a.Suponhamos agora que xn esteja bem definido. Entao, como xn+1 =f(xn), o valor de xn+1 tambem esta bem definido para todo natural n.

4. Por inducao sobre n. Os valores de x1 e x2 estao bem definidos, jaque x1 = a e x2 = b. Suponhamos agora que xn e xn+1 estejam bemdefinidos. Entao, como xn+2 = f(xn, xn+1), o valor de xn+2 tambemesta bem definido. Logo, pelo Principio da Inducao Finita, o valor dexn esta bem definido para todos natural n.

5. (a) A razao xn+1/xn entre dois termos consecutivos e constante e iguala 2. Logo, a sequencia e uma progressao geometrica de razao 2. Comoo primeiro termo e x1 = 3, o termo geral e dado por xn = 3.2n−1. (b) A

1

diferenca xn+1 − xn entre dois termos consecutivos e constante e iguala 3. Logo, a sequencia e uma progressao aritmetica de razao 3. Comoo primeiro termo e x1 = 2, o termo geral e dado por xn = 2 + 3n− 1 =3n + 1.

Secao 4

Exercıcios Suplementares

1. O circulo n + 1 e subdividido em no maximo 2n arcos pelos n ja exis-tentes. Cada um destes arcos subdivide uma regiao existente, deter-minando assim 2n regioes. Logo, o numero maximo xn de regioes de-terminadas por n cırculos satisfaz a recursao xn+1 = xn + 2n, comx1 = 2.

2. Pelo Exemplo 2 em Para Saber Mais, o numero Dn de permutacoescaoticas satisfaz a recursao Dn+2 = (n+1)(Dn+1+Dn), para n ≥ 1. Poroutro lado, D1 = 0 (ja que 1 necessariamente ocupa o seu proprio lugar)e D2 = 1 (corresponde a permutacao (2,1)). Daı, D3 = 2(D2+D1) = 2,D4 = 3(D3 + D2) = 9 e D5 = D4 + D3 = 44.

3. Podemos proceder por Inducao Finita. E imediato verificar que D1 =0 = 1!( 1

0!− 1

1!) e D2 = 1 = 2!(( 1

0!− 1

1!) + 1

2!). Suponhamos agora que a

expressao do numero de permutacoes caoticas esteja correta para n e

2

n + 1. Temos:

Dn+1 =(n + 1)(Dn+1 −Dn)

=(n + 1)

(n!

n∑k=0

(−1)k

k!+ (n + 1)!

n+1∑k=0

(−1)k

k!

)

=

((n + 1)((n! + (n + 1)!)

n∑k=0

(−1)k

k!

)+ (n + 1)!

(−1)n+1

(n + 1)!

=

((n + 1)(1 + n + 1)n!

n∑k=0

(−1)k

k!

)+ (n + 1)(−1)n+1

=

((n + 1)(n + 2)n!

n∑k=0

(−1)k

k!

)+ ((n + 2)− 1)(−1)n+1

=(n + 2)!

(n∑k=0

(−1)k

k!

)+

(n + 2)!

(n + 1)!(−1)n+1 +

(n + 2)!

(n + 2)!(−1)n+2

=(n + 2)!n+2∑k=0

(−1)k

k!

Secao 6

Exercıcios Recomendados

1. Do problema 2 da secao anterior, o numero maximo xn de regioes emque n retas podem dividir o plano satisfaz a recorrencia xn+1 = xn +n + 1, com x0 = 1. Daı:

x0 = 1

x1 = x0 + 1

x2 = x1 + 2

· · ·xn = xn−1 + n

Somando, resulta:

xn = 1 + 1 + · · ·+ n = 1 +n(n + 1)

2

3

2. Seja xn o numero de sequencias de n termos 0 ou 1 com quantidadeımpar de termos iguais a 0. O numero de sequencias de n + 1 termos0 ou 1 com numero ımpar de termos iguais a 0 e igual ao numero desequencias comecadas com 1, seguindo de uma sequencia de n ter-mos com numero ımpar de zeros somado ao numero de sequenciascomecadas com 0, seguido de uma sequencia de n termos com umnumero par de zeros. Portanto, xn+1 = xn + (2n − xn) = 2n (paraa segunda parcela, note que 2n e o numero total de sequencias forma-das por 0 ou 1). Logo, xn = 2n−1, para todo n.

3. Sequencias de n+ 1 termos 0, 1 ou 2 com um numero ımpar de termosiguais a 0 podem ser de dois tipos: as que comecam com 1 ou 2, seguidopor uma sequencia de n termos com numero ımpar de zeros e as quecomecam com 0, seguido por uma sequencia de n termos com numeropar de zeros. Daı, temos a recorrencia xn+1 = 2xn + (3n− xn), ou seja,xn+1 = xn + 3n, com x1 = 1. Termos:

x1 = 1

x2 = x1 + 31

· · ·xn = xn−1 + 3n−1

Somando, resulta

xn = 1 + 3 + · · ·+ 3n−1 =3n − 1

2.

4. Para Sheila ganhar a (n + 1)-esima partida, ou ela ganha a n-esimapartida (com probabilidade xn) e ganha a seguinte (com probabilidadecondicional 0,6) ou perde a n-esima (com probabilidade 1−xn) e ganhaa seguinte (com probabilidade condicional 0,4). Logo, a probabilidadexn+1 de vitoria na (n + 1)-esima partida e dada por xn+1 = 0, 6xn +0, 4(1− xn), ou seja, xn+1 = 0, 2xn + 0, 4, com x1 = 0, 4. Para resolvera recorrencia comecamos com uma solucao nao nula de xn+1 = 0, 2xn;por exemplo, an = (0, 2)n−1. Fazendo a substituicao xn = (0, 2)n−1yn,temos (0, 2)nyn+1 = (0, 2)nyn + 0, 4, ou seja, yn+1 = yn + 0,4

(0,2)n, com

4

y1 = x1/a1 = 0, 4. Temos:

y1 = 0, 4

y2 = y1 +0, 4

0, 2

y3 = y2 +0, 4

(0, 2)2

· · ·

yn = yn−1 +0, 4

(0, 2)n−1

Somando, vem

yn = 0, 4 +0, 4

0, 2+ · · ·+ 0, 4

(0, 2)n−1= 0, 4

1− (0, 2)n

0, 8(0, 2)n−1

Finalmente

xn = (0, 2)n−1yn =1− (0, 2)n

2.

5. a) ma solucao da equacao homogenea xn+1 = (n + 1)xn e an = n!.Fazendo a substituicao xn = anyn, temos

(n + 1)!yn+1 = (n + 1)n!yn + n,

ou seja,

yn+1 = yn +n

(n + 1)!,

com y1 = x1!1!

= 1.

Daı:

y1 = 1

y2 = y1 +1

2!· · ·

yn = yn−1 +n− 1

n!

5

Somando:

yn = 1 +1

2!+ · · ·+ n− 1

n!.

Mas n−1n!

= n−1n!

= 1(n−1)! −

1n!

. Logo

yn = 1 + (1

1!− 1

2!) + · · ·+

(1

(n− 2)!− 1

(n− 1)!

)+

(1

(n− 1)!− 1

n!

)= 1 + 1− 1

n!= 2− 1

n!

Finalmentexn = n!yn = 2n!− 1.

b) Uma solucao da equacao homogenea (n + 1)xn+1 = −nxn e an =(−1)nn

. Fazendo a substituicao xn = anyn, temos

(−1)n+1yn+1 = (−1)n+1yn + 2n− 3,

ou seja,yn+1 = yn + (−1)n+1(2n− 3),

com y1 = x1a1

= −1.

Assim, temos:

y1 = −1

y2 = y1 + (2− 3)

· · ·yn = yn−1 + (−1)n(2(n− 1)− 3)

Somando:

yn = −1− 1− 1 + 3− 5 + 7− 9 + · · ·+ (−1)n(2(n− 1)− 3)

Quando n e par,

yn = −2 + (−1 + 3) + (−5 + 7) + (−(2(n− 2)− 3)) + (2(n− 1)− 3)

= −2 + (n− 2) = n− 4

e xn = anyn = nn

= 1− 4n.

6

Quando n e ımpar, yn = (n−5)− (2(n−1)−3) = −n e xn = anyn = 1.

As duas expressoes podem ser colocadas em uma unica, escrevendo,por exemplo: xn = 1− 2+2(−1)n

n.

c) Uma solucao da equacao homogenea xn+1 = nxn e an = (n − 1)!.Fazendo a substituicao xn = anyn, temos n!yn+1 = n!yn + (n + 1)!, ouseja, yn+1 = yn + (n + 1), com y1 = x1/a1 = 1.

Assim

y1 = 1

y2 = y1 + 2

· · ·yn = yn−1 + n

Somando,

yn = 1 + 2 + · · ·+ n =n(n + 1)

2

e xn = anyn = (n+1)!2

.

Secao 6

Exercıcios Suplementares

1. Seja xn o numero de coloracoes para n setores e consideremos o pro-blema de colorir n+ 1 setores. O primeiro setor pode ser colorido de kmodos e cada setor subsequente pode ser colorido de k − 1 modos, jaque nao pode receber a mesma cor do anterior, resultando em k(k−1)n

coloracoes. Mas este resultado inclui os casos em que o ultimo setorrecebe a mesma cor do primeiro, o que e proibido. Os casos contadosindevidamente correspondem as coloracoes que sao validas, exceto pelofato de dois setores adjacentes terem a mesma cor. Considerando es-tes dois setores como um unico, estas coloracoes a serem descontadascorrespondem a coloracoes validas em um disco dividido em n setores.Portanto, xn+1 = k(k − 1)n − xn, com x2 = k(k − 1). Uma solucao

7

da equacao homogenea xn+1 = −xn e an = (−1)n−1. Fazendo a subs-tituicao xn = anyn, vem (−1)nyn+1 = (−1)nyn + k(k − 1)n, ou sejayn+1 = yn + (−1)nk(k − 1)n, com y2 = x2/a2 = −k(k − 1).

Assim

y2 = −k(k − 1)

y3 = y2 + k(k − 1)2

· · ·yn = yn−1 + (−1)n−1k(k − 1)n−1

Daı,

yn = k(k − 1)− k(k − 1)2 + · · ·+ (−1)n−1k(k − 1)n−1,

que e a soma dos termos de uma P.G. de razao −k(k − 1). Logo,

yn = −k(k − 1)1− (−1)n−1(k − 1)n−1

1 + (k − 1)= −(k − 1) + (−1)n(k − 1)n.

Finalmente,

xn = anyn =−(k − 1) + (−1)n(k − 1)n

(−1)n= (−1)n(k − 1) + (k − 1)n.

2. O numero de torcedores no ano n + 1 e pn+1 = pn(1 + i − j) − R.Uma solucao da recorrencia homogenea pn+1 = pn(1 + i − j) e an =(1 + i − j)n−1. Substituindo pn = anyn e fazendo r = 1 + i = j, vemrnyn+1 = rnyn − R, ou seja, yn+1 = yn − R

rn, com y0 = p0/a0 = p0r.

Assim

y0 = p0r

y1 = y0 −R

y2 = y1 −R

r· · ·

yn = yn−1 −R

rn−1

8

Somando

yn = p0r −R

(1 +

1

r+ · · ·+ 1

rn−1

)= p0 −R

rn − 1

rn−1(r − 1).

Finalmente,

pn = anyn = p0rn −R

rn − 1

r − 1=

(p0 −

R

r − 1

)rn − R

r − 1

=

(p0 −

R

i− j

)(1 + i− j)n +

R

i− j.

A torcida se extingue quando o coeficiente(p0 − R

i−j

)de rn e negativo,

ou seja quando R > p0(i− j).

Unidade 8

Exercıcios Recomendados

1. (a) As raızes da equacao caracterıstica r2 + 5r + 6 = 0 sao r1 = −2 er2 = −3. Logo, a solucao geral e xn = C1(−2)n + C2(−3)n, ondeC1 e C2 sao constantes arbitrarias.

(b) A equacao caracterıstica r2 + 6r + 9 = 0 tem duas raızes iguais a-3. Logo, a solucao geral e xn = C1(−3)n + C2n(−3)n, onde C1 eC2 sao constantes arbitrarias.

(c) As raızes da equacao caracterıstica r2 +4r+4 = 0 sao os numeroscomplexos r1 = −1 + i e r2 = −1 − i. Logo, a solucao gerale xn = C1(−1 + i)n + C2(−1 − i)n, onde C1 e C2 sao constantesarbitrarias. Usando o fato de que as raızes sao numeros complexosde modulo

√2 e argumento ±3π

4, esta solucao pode ser escrita na

forma xn = C1 cos 3nπ4

+ rsen 3nπ4

.

(d) Como as raızes da equacao caracterıstica r2 − 5r + 6 = 0 saor1 = 2 e r2 = 3, a solucao geral da equacao homogenea e xn =C12

n + C23n. Por outro lado, tentando uma solucao particular

9

da forma xn = An + B, obtemos 2An + 2B − 3A = n, que severifica quando A = 1

2e B = 3

4. Portanto, xn = 1

2n + 3

4e uma

solucao particular e, em consequencia, a solucao geral da equacaonao-homogenea e xn = 1

2n + 3

4+ C12

n + C23n.

(e) A solucao geral da homogenea e a mesma do exercıcio anterior.Tentando uma solucao particular da forma xn = An + B + C4n,obtemos 2An + 2B − 3A + 2C4n = 1 + 3.4n, que se verifica paraA = 0, B = 1/2 e C = 3/2, mostrando que 1

2+ 3

24n e uma solucao

particular. Logo, a solucao geral da equacao nao-homogenea exn = 1

2+ 3

24n + C12

n + C23n.

(f) A solucao geral da homogenea e a mesma dos exercıcios anteriores.Como xn = 2n e solucao da homogenea, na busca por uma solucaoparticular temos xn = An2n, o que leva a A(n + 2)2n+2 − 5A(n +1)2n+1 + 6An2n. Daı, obtemos A = −1

2, que fornece a solucao

particular xn = −12n2n = −n2n−1. A solucao geral da equacao

nao homogenea e xn = −12n2n = −n2n−1 + C12

n + C23n.

(g) A solucao geral da homogenea e a mesma dos exercıcios anteriores.Como xn = 3n e solucao da homogenea e o termo nao homogeneoe n + 3n, vamos a buscar uma solucao particular da forma xn =An + B + Cn3n. Substituindo na equacao, obtemos

A(n + 2) + B + C(n + 2)3n + 2− 5A(n + 1)− 5B

−5C(n + 1)3n+1 + 6An + 6B + Cn3n = n + 3n.

Simplificando, vem 2An−3A+2B+5C3n = n+3n, que e satisfeitapara A = 1/2, B = 3/4 e C = 1/5. Logo, uma solucao particulare xn = 1

2n + 3

4+ 1

53n e a solucao geral da equacao nao homogenea

e xn = 12n + 3

4+ 1

53n + C12

n + C23n.

(h) A equacao caracterıstica r2−6r+9 = 0 tem duas raızes iguais a 3.Logo, a solucao geral da equacao homogenea e xn = C13

n+C2n3n.Como o termo nao homogeneo e n− 3n, mas tanto 3n quanto n3n

sao solucoes da homogenea, tentamos uma solucao particular daforma An + B + Cn23n. Substituindo na equacao, obtemos

A(n + 2) + B + C(n + 2)23n+2 − 6A(n + 1)

−6B − 6C(n + 1)23n+1 + 9An + 9B + 9Cn23n = n− 3n.

10

Simplificando, vem 4An−4A+4B+18C3n = n−3n, que e satisfeitapor A = 1/4, B = 1/4 e C = −1/18, que leva a solucao particularxn = n

4+ 1

4− n33n

4e a solucao geral xn = n

4+ 1

4− n33n

4+C13

n+C2n3n.

(i) As solucoes da equacao caracterıstica r2 + 1 = 0 sao r1 = i er2 = −i. A solucao geral da equacao homogenea e xn = C1i

n +C2(−i)n = C1 cos nπ

2+C2sen nπ

2. Tentando uma solucao particular

da forma xn = A, obtemos 2A = 1, ou seja, xn = A = 1/2.A solucao geral da equacao nao homogenea e, portanto, xn =12

+ C1 cos nπ2

+ C2sen nπ2

.

(j) Como no ıtem h), a solucao da homogenea e xn = C13n + C2n3n.

Tentando, como em h), uma solucao particular da forma An +B + Cn23n, chegamos a 4An − 4A + 4B + 18C3n = 1 + n3n.Observando que e necessario elevar o grau do termo que multiplica3n; por outro lado, nao e preciso incluir na solucao particularum termo em n. Devemos tentar, assim, uma solucao particularna forma xn = A + (Bn2 + Cn3)3n. Substituindo na equacao esimplificando obtemos 4A + 54Bn3n + (54B + 18C)3n = 1 + 3n,que e satisfeita para A = 1/4, B = 1/54 e C = −1/18, levandoa solucao particular xn = 1

4+ n23n

54− n33n

18e a solucao geral xn =

14

+ n23n

54− n33n

18+ C13

n + C23n.

2. (a) No Exercıcio 1(a), encontramos a solucao geral xn = C1(−2)n +C2(−3)n. Substituindo n = 0 e n = 1 na solucao, encontramos:

C1 + C2 =3

−2C1 − 3C2 =− 6

Resolvendo, encontramos C1 = 0 e C2 = 3, levando a solucaoxn = 3(−2)n.

(b) As solucoes da equacao caracterıstica r2 + r − 6 = 0 sao r1 = 2e r2 = −3, conduzindo a solucao geral xn = C12

n + C2(−3)n

para a solucao homogenea. Tentando uma solucao particular daforma xn = An + B, encontramos que a equacao e satisfeita paraA = 2 e B = 0. Assim, xn = 2n e uma solucao particular exn = 2n + C12

n + C2(−3)n e a solucao geral da equacao naohomogenea.

11

Substituindo n = 0 e n = 1 nesta solucao, obtemos:

C1 + C2 =1

2C1 − 3C2 =2

Resolvendo, encontramos C1 = 0 e C2 = 1. Portanto, a solucao exn = 2n + 2n

(c) A equacao caracterıstica r2 − 4r + 4 = 0 tem duas raızes iguais a2, conduzindo a solucao geral xn = C12

n+C2n2n para a parte ho-mogenea. Tentando uma solucao particular da forma xn = An22n,verificamos que xn = n22n e uma solucao particular e que, assim,xn = n22n + C12

n + C2n2n e a solucao geral da equacao nao ho-mogenea.

Substituindo n = 0 e n = 1 encontramos:

C1 =3

2C1 + 2C2 =4

Resolvendo, encontramos C1 = 3 e C2 = −1. Logo, a solucao daequacao e xn = 32n − n2n + n22n

3. Seja xn o numero de sequencias formadas por n termos iguais a 0, 1ou 2 sem dois zeros repetidos. As sequencias de n + 2 termos que naotem dois termos consecutivos podem comecar por 0, 1 ou 2 . As quecomecam por 0 tem o proximo elemento igual a 1 ou 2 e, a seguir,uma sequencia de n termos sem zeros repetidos. Logo, ha 2xn taissequencias. As que comecam por 1 ou 2 tem, a seguir uma sequenciade n + 1 termos sem zeros repetidos. Logo, ha 2xn+1 sequencias destetipo. Assim, xn satisfaz a recorrencia xn+2 = 2xn + 2xn+1, ou seja,xn+2− 2xn+1− 2xn = 0, com x1 = 3 e x2 = 8 (todas as 3 sequencias decomprimento 1 cumprem o requisito e todas as 32 = 9 de comprimento2, exceto a 00, tambem cumprem a condicao).

As raızes da equacao caracterıstica r2 − 2r − 2 = 0 sao r1 = 1 +√

3 er2 = 1−

√3, levando a solucao geral xn = C1(1 +

√3)n + C2(1 +

√3)n

para a recorrencia. Substituindo n = 1 e n = 2, obtemos

(1 +√

3)C1 + (1−√

3)C2 = 3

(3 + 2√

3)C1 + (3− 2√

3)C2 = 8

12

Resolvendo o sistema, encontramos C1 = 3+2√3

6e C2 = 3−2

√3

6. Logo o

numero de sequencias por n termos iguais a 0, 1 ou 2 sem dois zerosrepetidos e xn = 3−2

√3

6(1 +

√3)n + 3+2

√3

6(1−

√3)n.

4. Considere um tabuleiro com 2 linhas e n+ 2 colunas. Para preencher ocanto esquerdo do tabuleiro, ha duas alternativas: colocar um domino“em pe”, restando tabuleiro com 2 linhas e n+1 colunas a preencher, oucolocar dois dominos “deitado” restando um tabuleiro com 2 linhas e ncolunas. Logo, o numero xn de modos de preencher um tabuleiro 2×ncom dominos 2× 1 satisfaz a recorrencia xn+2 = xn +xn+1, com x1 = 1e x− 2 = 2. Esta e exatamente a sequencia de Fibonacci estudada no

Exemplo 4. Logo, temos xn = Fn = 1√5

(1+√5

2

)n+1

− 1√5

(1−√5

2

)n+1

.

5. No ano n + 2 sao geradas 21 sementes para cada semente gerada noano n+ 1 e 44 sementes para cada semente gerada nos anos anteriores.Logo, se xn denota o numero de sementes geradas no ano n, temos

xn+2 = 21xn+1 + 44(xn + xn−1 + · · ·+ x1 + x0),

com x1 = 1 e x2 = 44 + 21.21 = 485. Para transformar esta recorrenciaem uma recorrencia linear de segunda ordem, escrevemos a expressaopara xn+1:

xn+1 = 21xn + 44(xn−1 + xn−2 + · · ·+ x1 + x0),

Subtraindo as duas expressoes, obtemos:

xn+2 = 22xn+1 + 23xn,

ou seja,xn+2 − 22xn+1 − 23xn = 0

A equacao caracterıstica r2−22r−23 = 0 tem raızes r1 = 23 e r2 = −1,levando a solucao geral xn = C123n + C2(−1)n para a recorrencia.Usando as condicoes iniciais, obtemos

23C1 − C2 =21

529C1 + C2 =485

Resolvendo, encontramos C1 = 11/12 e C2 = 1/12. A solucao darecorrencia e, assim, xn = 11

1223n + 1

12(−1)n.

13

6. A renda xn no mes n e igual ao salario Sn mais o rendimento sobremontante yn−1 das aplicacoes financeiras no mes anterior. Ou seja, xn =Sn + iyn−1. Por outro lado, o montante das aplicacoes financeiras nomes n e igual ao do mes anterior, acrescido do valor poupado no mes n.Ou seja, yn = yn−1 + 1

pxn. Da primeira equacao, tiramos yn−1 = xn−Sn

i

e yn = xn+1−Sn+1

i. Substituindo estas expressoes na segunda equacao,

vem xn+1−Sn+1

i= xn−Sn

i+ 1

pxn, ou seja, xn+1 −

(1 + i

p

). Fazendo a

substituicao xn = anzn, encontramos kn−1zn = kn−1zn−1 + b, ou seja,zn = zn−1 + b

kn−1 . Assim, como x0 = a, temos z0 = a/k−1 = ak e:

z0 = ak

z1 = z0 + b

· · ·

zn = zn−1 +b

kn−1

Somando, vem:

zn = ak + b1− kn

kn−1(1− k)

e

xn = anzn = akn + b1− kn

1− k

Substituindo k =(

1 + ip

), temos, finalmente

xn =

(a +

pb

i

)(1 +

i

p

)n− pb

i.

7. Seja pn a probabilidade de que a taca nao seja ganha nos primeirosn torneios. Qualquer time pode ganhar o primeiro torneio. Vamosexprimir pn+2 em funcao de pn e pn+1 usando probabilidade condicional.Se o segundo torneio for ganho por um time diferente do que ganhouo primeiro (o que ocorre com probabilidade 4

5), tudo se passa como se

a serie de torneios estivesse comecando no segundo torneio. Ou seja, aprobabilidade condicional de que a taca nao seja ganha ate o torneion + 2 e igual a pn+1. Se o segundo torneio for ganho pelo mesmo timedo primeiro, mas no terceiro nao (ocorre com probabilidade 1

5.45), tudo

14

se passa como se a serie de torneios comecasse no terceiro jogo e aprobabilidade condicional de que a taca nao seja ganha ate o torneion + 2 torneio e igual a pn. Finalmente, se os tres primeiros torneiosforem ganhos pelo mesmo time, a taca e ganha na terceira realizacao e,portanto, a probabilidade condicional de que ela nao tenha sido ganhaate o torneio n + 2 e igual a zero, para todo n ≥ 1. Assim, temospn+2 = 4

5pn+1 + 1

5.45pn, com p1 = p2 = 1 (ja que a taca certamente

nao e ganha nas duas primeiras realizacoes). A equacao caracterıstica

r2 − 45r − 1

5.45

= 0 tem raızes r1 = 2+2√2

5e r2 = 2−2

√2

5. Logo, a solucao

geral da recorrencia e pn = C1

(2+2√2

5

)n+ C2

(2−2√2

5

)n.

Substituindo n = 1 e n = 2, temos:(2 + 2

√2

5

)C1 +

(2− 2

√2

5

)C2 = 1(

12 + 8√

2

25

)C1 +

(12− 8

√2

25

)C2 = 1

Resolvendo, encontramos C1 = 10+5√2

16e C2 = 10−5

√2

16, levando a solucao

pn = 10+5√2

16

(2+2√2

5

)n+ 10−5

√2

16

(2−2√2

5

)n8. Seja xn o numero de modos de obter 1 ou 2 pontos no primeiro jogo.

No primeiro caso, ele tem que obter n+1 pontos nos jogos seguintes; nosegundo caso, ele tem que obter n pontos a seguir. Logo, xn+2 = xn +xn+1, com x1 = 1 e x2 = 2. Esta e a recorrencia que define a sequencia

de Fibonacci Fn. Logo, xn = Fn = 1√5

(1+√5

2

)n+1

− 1√5

(1−√5

2

)n+1

, para

todo n.

9. Os numeros r1 = (1−√

5) e r2 = (1+√

5) sao as raızes da equacao r2−2r−4 = 0. Portanto, a sequencia xn = 2

√5+1

2√5

(1−√

5)n+ 2√5−1

2√5

(1+√

5)n

satisfaz a recorrencia xn+2 = 2xn+1 + 4xn. Alem disso, substituindo osvalores n = 0 e n = 1, obtemos x0 = 2 e x1 = 1. Assim, x0 e x1

sao naturais e a recorrencia nos garante que, se xn e xn+1 sao naturais,entao xn+2 tambem e. Portanto, pelo Principio de Inducao Finita, xne natural para todo n.

15

10. Para cada n natural, o numero xn = (1 +√

3)2n+1 + (1 −√

3)2n+1 eum inteiro par, pois os termos que contem

√3 se cancelam e os demais

(que sao inteiros) aparecem duas vezes. Alternativamente, poderıamosargumentar como no exercıcio 10: (1 +

√3)2 e (1−

√3)2 sao raızes da

equacao r2 − 8r − 4 = 0. Logo, xn satisfaz a equacao de recorrenciaxn+2 = 8xn+1 + 4xn. Como x0 = 2 e x1 = 20, decorre por inducao quexn e inteiro par para todo n.

Por outro lado, −1 < 1 −√

3 < 0, ja que√

3 ≈ 1, 73. Logo, −1 <(1−√

3)2n+1 < 0, pata todo n. Somando (1+√

3)2n+1 a todos os termosdesta desigualdade, obtemos (1 +

√3)2n+1 − 1 < (1 +

√3)2n+1 + (1 −√

3)2n+1 < (1+√

3)2n+1, ou seja, (1+√

3)2n+1−1 < xn < (1+√

3)2n+1, oque mostra que o numero inteiro par xn e a parte inteira de (1+

√3)2n+1.

16

Exercıcios Resolvidos de MA 12

Unidades 9 e 10

Unidade 9

Exercıcios Recomendados

1. 600 = 450(1 + i)3

i =(600450

)1/3 − 1 = 10, 06%

2. (a) 1 + I = (1 + i)12

1 + 1 = (1 + i)12

i = 21/12 − 1 = 5, 95%

(b) 1 + I = (1 + i)3

1 + 0, 39 = (1 + i)3

i = 1, 391/3 − 1 = 11, 60%

3. (a) 1 + I = (1 + i)12

I = 1, 0612 − 1 = 101, 22%

(b) 1 + I = (1 + i)4

I = 1, 124 − 1 = 57, 37%

4. (a) A taxa e de 30%/12 = 2, 5% ao mes.

1 + I = (1 + i)12

I = 1, 02512 − 1 = 34, 49%

(b) A taxa e de 30%/4 = 7, 5% ao trimestre.

1 + I = (1 + i)4

I = 1, 0754 − 1 = 33, 55%.

(c) A taxa relativa ao perıodo de capitalizacao e i/k.

1 + I = (1 + ik)k

I = (1 + ik)k − 1

1

5. lim(1 + ik)k − 1 = ei − 1

6. O numero e e o valor de montante gerado em um ano por um principaligual a 1, a juros de 100% ao ano, capitalizados continuamente.

7. (a) eδ − 1 = e0,12 − 1 = 12, 75%

(b) ln(1 + i) = ln 1, 6 = 47, 00%

(c) Aproveitando o ıtem anterior, 47, 00%/2 = 23, 50%.

8. Seja 0 a data de compra. Seja 3 o preco do artigo. Usemos a data 1como data focal.

Na alternativa (a), paga-se A = 3.

Na alternativa (b), paga-se B = 11+i

+ 1 + (1 + i)

B − A = i2

1+i> 0

Logo, como B > A, a alternativa (a) e preferıvel.

9. (a) Supondo uma prestacao vincenda de 100 e tomando a data atualcomo focal:

aceitando: pago, na data 0, 70.

nao aceitando: pago, na data 1, 100, o que equivale a pagar, nadata 0,100

1+0,27= 78, 74

A proposta e vantajosa.

(b) Supondo duas prestacoes vincendas de 100 cada uma e tomandoa data atual como focal:

aceitando: pago, na data 0, 120.

nao aceitando: pago, na data 1, 100, e na data 2, 100, o queequivale a pagar na data 0,100

1+0,27+ 100

(1+0,27)2= 140, 74

A proposta e vantajosa.

(c) Supondo tres prestacoes vincendas de 100 cada uma e tomando adata atual como focal:

aceitando: pago, na data 0, 150.

nao aceitando: pago, na data 1, 100, na data 2, 100, e, na data 3,100, o que equivale a pagar, na data 0,

2

1001+0.27

+ 100(1+0.27)2

+ 100(1+0.27)3

= 189, 56

A proposta e vantajosa

10. 1801+0.25

+ 200(1+0.25)2

= 272

O preco a vista e R$ 272,00.

11. (a) Tomando a data focal um mes antes da compra,P

1,06= 10001−1,06−3

0,06

P = 352, 93

(b) Tomando a data focal no ato da compra,

P = 10001−1,06−3

0,06

P = 374, 11

(c) Tomando a data focal um mes depois da compra,

P.1, 06 = 10001−1,06−3

0,06

P = 396, 56

12. O montante pago foi 400.1, 034.1, 055.1, 09 = 626, 30.

A taxa media de juros e calculada por 626, 30 = 400.(1+i)10, i = 4, 59%ao mes.

Unidade 10

Exercıcios Recomendados

1. (a) 4001,06

= P 1−1,06−10

0,06

P = 51, 27

(b) 400 = P 1−1,06−10

0,06

P = 54, 35

(c) 400.1, 06 = P 1−1,06−10

0,06

P = 57, 61

2. (a) P = Ai = 50000.0, 006 = 300, 00

3

(b) A1+i

= Pc

P = 500001,006

.0, 006 = 298, 21

3. O montante que voce deve acumular e 100 = 1−1,005−360

0,005= 16.679, 16

Para isso, P 1−1,005−360

0,005.1, 005360 = 16.779, 16 e P = 16, 60

4. O montante que voce deve acumular e 1000,005

= 20.000

Para isso, P 1−1,005−420

0,005.005420 = 20.000 e P = 14, 04

5. A prestacao pela tabela Price e P = 3.000 0,11−1,1−8 = 562, 33

A amortizacao pelo SAC e 3.000/8 = 375

TABELA PRICEEPOCA PRESTACAO JUROS AMORTIZACAO ESTADO DA DIVIDA

0 - - - 3 000,001 562,33 300,00 262,33 2737,672 562,33 273,77 288,56 2.449,113 562,33 244,91 317,42 2.131,694 562,33 213,17 349,16 1.782,535 562,33 178,25 384,08 1.398,456 562,33 139,84 422,49 975,967 562,33 97,60 464,73 511,238 562,33 51,12 511,23 -

SACEPOCA PRESTACAO JUROS AMORTIZACAO ESTADO DA DIVIDA

0 - - - 3 000,001 675,00 300,00 375,00 2.625,002 637,50 262,50 375,00 2.250,003 600,00 225,00 375,00 1.875,004 562,50 187,50 375,00 1.500,005 525,00 150.00 375,00 1,125,006 487,50 112,50 375,00 750,007 450,00 75,00 375,00 375,008 412,00 37,50 375,00 -

6. O montante e 0, 31.1, 1t + 0, 7.1, 18t

A taxa media de juros e calculada por 0, 3.1, 1t + 0, 1.1, 18t = 1.(1 +i)t i = (0, 3.1, 1t + 0, 1.1, 18t)1/t − 1, onde t e o numero de meses do

4

investimento. Se t = 1, a taxa e 15, 60%; se t = 2, e 15,66%; se t→∞,a taxa e 18%.

7. (a) Usando a data da compra como data focal e considerando umpreco igual a 30,

21 = 10 + 101+i

+ 10(1+i)2

Resolvendo, i = 51, 08%.

(b) Tomando para data focal a data da compra e considerado umpreco igual a 30,

21 = 101+i

+ 10(1+i)2

+ 10(1+i)3

Resolvendo, i = 20, 20%

(c) Tomando para data focal a data da compra e considerado umpreco igual a 30,

21 = 10(1+i)2

+ 10(1+i)3

+ 10(1+i)4

Resolvendo, i = 12, 81%.

8. Arbitrando o preco em 100 e usando a data da compra como data focal,

100− x > 501,05

+ 501,052

x < 7, 03%

9. (a) 100 = 94(1 + i)

i = 6, 38%

(b) 100 = 88(1 + i)2

i = 6, 60%

(c) 100 = 72(1 + i)3

i = 6, 84%

10. Sem reciprocidade, receberia 88 para pagar 100 em dois meses. Comreciprocidade, recebe 0, 8.88 = 70, 4 para pagar 100 − 17, 6.1, 022 =81, 69.

81, 69 = 70, 4.(1 + i)2

i = 7, 72%

5

Exercıcios Suplementares

1. (a) i/365 < i/360

(b) 1000.0, 12.16/360 = 5, 33

1000.0, 12.16/365 = 5, 26

(c) O montante e 1000.(1 + 0, 12)16/360 = 1005, 05 e os juros sao deR$ 5,05; O montante e 1000.(1 + 0, 12)16/365 = 1004, 98 e os jurossao de R$ 4,98.

2. 700.0, 12.11/30 = 30, 80.

3. (a) 48% ao trimestre.

(b) I = 1, 153 − 1 = 52, 09% ao trimestre.

(c) 100 = 64.(1 + I); I = 56, 25% ao trimestre.

A melhor e (a) e a pior e (c).

4. 1, 124−1 = 57, 35% sao os juros quadrimestrais. Deve cobrar 57, 35%/4 =14, 33% ao mes.

5. (a) 300.1, 153 = 456, 26

(b) 300.1, 153+ 830 = 437, 59

(c) 456, 26 + 456, 26.0, 15.8/30 = 474, 51

6. (a) A prestacao e 35.000 0,011−1,01−180 = 420, 06

(b) 420, 061−1,01−180

0,01= 23.056, 28

7. (a) A amortizacao e 35.000/180 = 194, 44

A dıvida na epoca da 99a¯ prestacao e 81.194, 44 = 15750.

Os juros da centesima prestacao sao 157,50 e a centesima prestacaoe igual a 194,44+157,50=351,94

(b) O estado da dıvida e 80.194, 44 = 15.555, 56

8. (a) Supondo a dıvida igual a 100, a prestacao para 150 meses e

P150 = 100 0,011−1,01−150 = 1, 29 e a prestacao para 300 meses e

P300 = 100 0,011−1,01−300 = 1, 05

A reducao e de 0, 24/1, 29 = 18%, aproximadamente

6

9. (a) A dıvida igual a 300, a prestacao para 150 meses e

P150 = 300150

+ 3 = 5

A prestacao para 300 meses e P300 = 300300

+ 3 = 4

A reducao e de 20%.

(b) 50%.

10. A original custa, por ano, 280 0,121,12(1−1,12−5)

= 69, 35.

Como a alternativa implica em um custo anual de 70,00, e melhorcomprar a original.

11. O custo de compara e 2.000− 3001,0130

= 1.777, 42

Isso equivale a um custo mensal 1.777, 42 0,011−1,01−30 = 68, 87 mais a ma-

nutencao, dando um custo mensal total de 73,87.

E melhor comprar.

12. .

dinheiro 100 1516preco 1 12,09

Poder de compra 100 1516/12, 09 ∼= 125

A rentabilidade real foi de 25%.

7

Exercıcios Resolvidos de MA 12

Unidades 11 e 12

Unidade 11

Exercıcios Recomendados

1. A primeira pergunta pode ser respondida de 5 modos; a segunda, de 5modos, etc.

A resposta e 5 × 5 × · · · × 5 = 510 = 9.765.625.

2. Para formar um subconjunto, deve-se decidir, para cada elemento doconjunto, se ele pertencera ou nao ao subconjunto. Ha 2 modos dedecidir o que fazer com o primeiro elemento do conjunto, 2 modos como segundo, etc.

A resposta e 2 × 2 × · · · × 2 = 2n

Outra Solucao:

Quando se acrescenta um elemento a um conjunto, os subconjuntos donovo conjunto sao os subconjuntos dobra. Entao, se An e o numero desubconjuntos de um conjunto com n elementos, (An) e uma progressaogeometrica de razao 2. Logo, An = A0.2

n = 2n pois o conjunto vaziopossui um unico subconjunto.

3. A primeira pessoa tem 5 escolhas; a segunda, 4; a terceira, 3. A respostae 5 × 4 × 3 = 60.

4. Os bancos em que os homens se sentam podem ser escolhidos de 5×4×3×2×1 = 120 modos, o mesmo ocorrendo com os bancos das mulheres.Em cada banco, os casais podem se sentar de 2 modos diferentes. Aresposta e 1202 × 25 = 460.800.

5. As 64 casas do tabuleiro dividem-se, naturalmente, em tres grupos:

i) as 4 casas dos vertices;

1

ii) as 24 casas da borda do tabuleiro, mas que nao sao vertices;

iii) as restantes 36 casas, que sao interiores ao tabuleiro.

Vamos separar a nossa contagem conforma o tipo de casa ocupada pelorei negro:

i) ha 4 possıveis para o rei negro e 60 para o rei branco;

ii) ha 24 possıveis para o rei negro e 58 para o rei branco;

iii) ha 36 possıveis para o rei negro e 55 para o rei branco.

A resposta e 4 × 60 × 24 × 58 + 36 × 55 = 3612.

Se os reis sao iguais, a resposta passa a ser a metade da resposta ante-rior, pois, trocando a posicao dos reis, agora obtem-se a mesma confi-guracao.

6. Havera uma torre em cada linha e em cada coluna. A posicao da torreda primeira linha pode ser escolhida de 8 modos; a da segunda linha,de 7, etc.

A resposta e 8 × 7 × 6 × 5 × 4 × 3 × 2 × 1 = 40.320.

Se as torres fossem diferentes, para cada uma das escolhas de posicao,terıamos que escolher uma das torres. A resposta seria, portanto, 8 ×8×7×7×6×6×5×5×4×4×3×3×2×2×1×1 = (8!)2 = 1.625.702.400.

7. Se a primeira carta e o rei de copas, a segunda pode ser escolhida de 48modos (pode ser qualquer carta, exceto os 4 reis). Se a primeira carta ede copas mas nao e o rei, ela pode ser escolhida de 12 modos. Neste caso,a segunda carta pode ser escolhida de 47 modos (nao pode ser a primeiraescolhida, nem nenhum dos 4 reis). A resposta e 48 + 12 × 47 = 612.

8. (a) Para construir uma funcao, devemos, para cada elemento de A,escolher sua imagem em B. Ha 47 modos de escolher a imagemdo primeiro elemento de A, 7 modos de escolher a imagem dosegundo elemento, etc.

A resposta e 7 × 7 × 7 × 7 = 2.401.

(b) Para a funcao ser injetora, elementos diferentes devem ter imagensdiferentes. Ha 7 modos de escolher a imagem do primeiro elementode A, 6 modos de escolher a imagem do segundo elemento, etc.

A resposta e 7 × 6 × 5 × 4 = 840.

2

9. (a) Como 720 = 24× 32× 51, 720 possui 5× 3× 2 = 30 divisores. Aospares, estes divisores formam produtos iguais a 720. Logo, ha 15modos de escrever 720 como produtos de divisores.

(b) Como 144 = 24 × 32, 144 possui 3 × 3 = 9 divisores. Com eles,podem ser formados 4 pares de divisores cujo produto e 144 e,alem disso, pode ser formado 12 × 12. Assim, ha 5 modos deescrever 144 como um produto de divisores.

Unidade 12

Exercıcios Recomendados

1. (a) O numero total de anagramas e 8! = 40.320.

(b) Ha 4 modos de escolher a vogal que sera a primeira letra do ana-grama e 3 modos de selecionar a vogal que sera a ultima letra doanagrama. Depois disso, ha 6! modos de arrumar as demais letrasentre a primeira e a ultima.

A resposta e 4 × 3 × 6! = 4 × 3 × 720 = 8.640.

(c) As vogais e consoantes podem aparecer na ordem CV CV CV CVou na ordem VC VC VC VC. No primeiro caso, devemos colocar as4 vogais nos 4 lugares de ordem par (4! modos) e as 4 consoantesnos 4 lugares de ordem ımpar (4! modos).

Ha 4! × 4! = 24 × 24 = 576 anagramas do primeiro tipo. Analo-gamente, ha 576 anagramas do segundo tipo.

A resposta e 576 + 576 = 1.152.

(d) Tudo se passa como se CAP fosse uma unica letra. Devemos,portanto, arrumar em fila 6 objetos: CAP,I,T,U,L,O.

A resposta e 6! = 720.

(e) Primeiramente, devemos escolher a ordem em que as letras C, A,P aparecerao. Ha 3! modos. Depois, devemos arrimar em a fila 6objetos: o bloco de letras C, A, P e as 5 letras I, T, U, L, O. Ha6! modos.

A resposta e 3! × 6! = 3 × 720 = 4320.

3

(f) Basta arrumar em fila, depois de PA, as restantes 6 letras.

A resposta e 6! = 720.

(g) Ha 7! anagramas com a letra P em primeiro lugar e ha 7! anagra-mas com a letra A em segundo lugar. Ha tambem 6! anagramascom a letra P em primeiro lugar e A em segundo lugar. Ao somar-mos 7! com 7!, encontramos o numero de anagramas com P emprimeiro lugar ou A em segundo lugar, mas contamos duas vezesos anagramas que tem P em primeiro lugar e A em segundo lugar.A resposta e 7! + 7! − 6! = 5040 + 5040 − 720 = 9.360.

(h) Ha 7! anagramas com a letra P em primeiro lugar, 7! anagramascom a letra A em segundo e 7! anagramas com a letra C em ter-ceiro. Ha tambem 6! anagramas com P em primeiro lugar e A emsegundo lugar, 6! anagramas com P em primeiro e C em terceiroe 6! anagramas com A em segundo e C em terceiro. Finalmente,ha 5! anagramas com P em primeiro lugar, A em segundo e C emterceiro.

Ao somarmos 7! com 7! com 7!, encontramos o numero de ana-gramas que tem P em primeiro lugar ou A em segundo ou C emterceiro, mas contamos alguns anagramas varias vezes.

Contamos duas vezes os anagramas que tem P em primeiro lugare A em segundo; o mesmo se deu com os que tem P em primeiroe C em terceiro e com os que tem A em segundo e C em terceiro.Descontando essas contagens indevidas, chegamos a 7! + 7! + 7!−6! − 6! − 6! = 3.7! − 3.6!.

Entretanto, anagramas com P em primeiro lugar e A em segundo eC em terceiro foram, inicialmente, contados tres vezes e, posterior-mente, descontados tres vezes, o que significa que nao estao sendocontados. Incluindo-os na contagem, obtemos a resposta correta,que e 3.(7!) − 3.(6!) + 5! = 3.(5.040) − 3.(720) + 120 = 13.080.

(i) Como ha 6 ordens possıveis para as letras C, A e P, os anagramaspedidos sao exatamente 1/6 do total, ou seja, 8!/6 = 6.720.

Outra solucao:

Basta escolher as 3 posicoes a serem ocupadas pelas letras P, A,C, o que pode ser feito de C3

8 = 56 modos e distribuir 5 letrasrestantes nas demais posicoes, o que pode ser feito de 5! = 120modos. O total de anagramas e 56 × 120 = 6.720.

4

2. O valor de f(a1) pode ser escolhido de n modos; o valor de f(a2), den− 1 modos; . . . ; o de f(an), de 1 modo.

A resposta e n(n− 1) · · · 1 = n!.

3. O numero total de modos de sentar 8 pessoas em 8 cadeiras e o numerode modos de arrumar 8 pessoas em fila, 8!. O numero de modos dearrumar 8 pessoas em fila de modo que duas dessas pessoas, Vera ePaulo, fiquem juntas e 2.7!, pois, para formar uma tal fila, devemosinicialmente decidir em que ordem se colocarao Vera e Paulo e, emseguida formar uma fila de 7 objetos: o bloco formado por Vera ePaulo; as demais 6 pessoas.

A resposta e 8! − 2.7! = 40.320 − 10.080 = 30.240.

4. Como visto no problema anterior, o numero de filas nas quais duaspessoas (neste caso Helena e Pedro) ficam juntas e 2.7! = 10.080. Onumero de filas onde Helena e Pedro e tambem Vera e Paulo ficamjuntos e obtido de modo analogo: agora sao dois blocos de duas pes-soas, cada um podendo ser arrumado de dois modos distintos e mais4 pessoas. Portanto, o numero de tais filas e 2.2.6! = 2.880. Logo, onumero de filas em que Helena e Pedro ficam juntos, mas Vera e Paulonao, e 10, 080 − 2.880 = 7.200.

5. O elemento da permutacao que ocupa o 10o¯ lugar deve ser maior que 7.

Pode ser escolhido de 3 modos. O elemento da 9a¯ posicao deve ser maior

que 6; haveria 4 possibilidades, mas uma delas ja foi usada na escolhado elemento que ocupa a 10a

¯ posicao. Pode ser escolhido de 3 modos.Prosseguindo com esse raciocınio, vemos que a cada nova casa abranda-se a restricao, criando uma possibilidade a mais, mas ao mesmo tempodiminui-se uma possibilidade, pois uma delas foi usada na etapa. Ouseja, ha 3 possibilidades para cada casa ate a 3a

¯ casa. O elementoda 3a

¯ posicao deve ser maior que 3 − 3 = 0; havera 10 possibilidades,mas 7 delas ja foram usadas nas etapas anteriores. Pode ser escolhidode 10 − 7 = 3 modos. O elemento da 2a

¯ posicao deve ser maior que2−3 = −1; haveria 10 possibilidades mas 8 ja foram usadas nas etapasanteriores. Pode ser escolhido de 10 − 8 = 2 modos. Finalmente, oelemento de posicao 1 deve ser maior que 1 − 3 = −2; haveria 10possibilidades, mas 9 delas ja foram usadas nas etapas anteriores. Podeser escolhido de 10 − 9 = 1 modo.

5

A resposta e 38.2.1 = 13.122.

6. Ha C515 modos de formar o Esporte; depois disso, C5

10 modos de formaro Tupi; finalmente, 1 unico modo de formar o Minas.

A resposta e C515 × C5

10 × 1 = 756.756.

7. O numero de possibilidades e igual ao numero obtido no problemaanterior dividido por 3! = 6, ja que permutando os nomes dos times asubdivisao continua a mesma. A resposta e 756.745/6 = 126.126.

8. Escolha, sucessivamente, 3 pessoas para formar os 4 grupos de 3; istopode ser feito, sucessivamente, de C3

20, C317, C

314 e C3

11 modos. A seguir,com as 8 pessoas restantes for os 2 grupos restantes, o que pode serfeito de C4

8 e C44 modos, respectivamente. Fazendo isso, contamos cada

divisao 4!.2! vezes, porque, quando formamos os mesmos grupos de 3 eos mesmos grupos de 4 em outra ordem, contamos como se fosse outradivisao em grupos.

A resposta eC3

20.C317.C

314.C

31 .C

48 .C

44

4!.3!=

20!

(3!)4(4!)24!2!= 67.897.830.000.

Outra solucao:

Forme uma fila com 20 pessoas. Isso automaticamente as divide em4 grupos de 3 e 2 grupo de 4: as 3 primeiras formam um grupo, as 3seguintes formam outro etc.. Ha 20! modos de formar a fila. Entretanto,uma mesma divisao em grupos correspondentes a varias filas diferentes,o que faz com que, no resultado 20!, cada divisao tenha sido contadavarias vezes. Devemos corrigir nossa contagem dividendo o resultadopelo numero de vezes que cada divisao foi contada. Trocando a ordemdos elementos em cada grupo, o que pode ser feito de 3!.3!.3!.3!.4!.4!modos, ou a ordem dos grupos, o que pode ser feito de 4!.2! modos,a divisao em grupos nao se altera, mas a fila sim. Cada divisao foi,

assim, contada (6!)3.2!.3! vezes e a resposta e20!

(3!)4(4!)24!2!

9. Os adversarios em cada jogo podem ser escolhidos, sucessivamente, deC2

12, C210, C

28 , C2

6 , C24 e C2

2 modos. No entanto, assim contamos cadapossıvel rodada 6! vezes, ja que contamos diferentes ordens dos jogos

6

como se fossem rodadas diferentes. A resposta eC2

12.C210.C

28 .C

26 .C

24 .C

22

6!=

12!

26.6!= 10.395

Outra solucao:

Colocados os 12 times em fila automaticamente formamos os 6 jogosda rodada. No entanto, a mesma rodada e contada varias vezes; osadversarios em cada jogo podem ser ordenados de 2 modos, enquantoos jogos podem ser ordenados de 6! modos. A resposta e, portanto,12!

26.6!.

10. (a) Para determinar o lugar ocupado pelo numero 62.417, devemoscontar quantos numeros estao antes dele. Antes dele estao osnumeros comecados por:

i) 1 (4!=24 numeros)

ii) 2(4!=24 numeros)

iii) 4 (4!=24 numeros)

iv) 61 (3!=6 numeros)

v) 621 (4!=2 numeros)

Ha 24 + 24 + 24 + 6 + 2 = 80 numeros antes do 62.417.

A resposta e 81o¯

(b) Como ha 4! = 24 numeros comecados por 1, e 4! = 24 numeroscomecados por 2 e 3! = 6 numeros comecados por 41, e 3! = 6numeros comecados por 42, e 3! = 6 numeros comecados por 46,o 66o

¯ numero escrito e o ultimo dos numeros comecados por 46,ou seja, 46.721.

A resposta e 46721.

(c) Como ha 5 algarismos em cada numero, o 166o¯ algarismo escrito

e o primeiro algarismo do 36o¯ numero.

Como ha 4! = 24 numeros comecados por 1, e 3! = 6 numeroscomecados por 21, e 3! = 6 numeros comecados por 24, o 36o

¯numero escrito e o ultimo dos numeros comecados por 26. Logo,seu primeiro algarismo e 2.

(d) Nas casas das unidades desses numeros, aparecem apenas os alga-rismos 1, 2, 4, 6, 7, cada um deles 4! = 24 vezes. A soma das unida-des desses numeros e, portanto, (1+2+4+6+7).24 = 480 unidades,

7

ou seja, 480. A soma das dezenas e, analogamente, igual a 480 de-zenas, ou seja, 4.800. A soma das centenas e igual a 480 centenas,ou seja, 48.000. A soma das unidades de milhar e igual a 480 uni-dades de milhar, ou seja, 480.000. Finalmente, a soma das dezenasde milhar e igual a 480 dezenas de milhar, ou seja, 4.800.000. Aresposta e 480 + 4.800 + 48.000 + 480.000 + 4.800.000 = 5.333.280.

Outra Solucao:

Ha 5! = 120 parcelas na soma. Podemos agrupa-las em 60 pares,juntando a cada numero o que dele se obtem trocando o 1 com o7, trocando o 2 com o 6, e conservando a posicao do 4. Em cadapar, a soma vale 88.888.

A resposta e 88.888 × 60 = 5.333.280

8

Exercıcios Resolvidos de MA 12

Unidades 13 e 14

Unidade 13

Exercıcios Recomendados

1. C27 + C3

7 + · · ·+ C77 = 27 − C0

7 − C17 = 128− 1− 7 = 120.

2. (a) Cp10 e elemento da linha 10. Em qualquer linha, o elemento maximo

e o do meio. A resposta e p = 5.

(b) Cp21 e elemento da linha 21. Em qualquer linha, o elemento maximo

e o do meio, A resposta e p = 10 ou p = 11.

3. Tp+1 = Cp10

((−1)x2

)p(x3)10−p = Cp

10(−1)px30−5p independera de x para

30− 5p = 0, ou seja, p = 6.

A resposta e T7 = C610(−1)6x0 = 210.

4. (1− x2) = 1− 2x+ x2 e (1 + x)n = xn + nxn−1 + n(n−1)2

xn−2 + · · · . Ostermos em xn no produto sao:

1.xn = xn; −2x.xn−1 = −2nxn; x2 · n(n−1)2

xn−2 = n(n−1)2

xn.

A resposta e 1− 2n+ n(n−1)2

= n2−5n+2)2

.

5. A soma pedida en∑k=0

Ckn3k = (1 + 3)n = 4n.

6. (a) Seja P (x) = (1 + x + x2)n. Entao, A0 + A1 + A2 + · · · + An =P (1) = 3n.

(b) Sejam S0 = A0+A2+· · ·+A2n e S1 = A1+A3+· · ·+A2n−1. Entao

P (1) = S0 + S1 e P (−1) = S0 − S1. Lodo, S0 =P (1) + P (−1)

2=

3n + 1

2.

1

7. Tp+1 = Cp120

(12

)p=

120!

p!(120− p)!·

1

2pe

Tp = Cp−1120

(12

)p−1=

120!

(p− 1)!(121− p)!·

1

2p−1;

Tp+1

Tp=

121− p2p

; Tp+1 > Tp se p ≤ 40 e Tp+1 < Tp se p ≥ 41

Daı T1 < T2 < · · · < T40 < T41 > T42 > · · · > T121.

O termo maximo e T41 =C40

120

240.

8. Sejam a = 10150 e b = 10050 + 9950. Temos:

a =(100 + 1)50 = C05010050 + C1

5010049 + C25010048

+ · · ·+ C4950100 + C50

50

b =10050 + (100− 1)50 = C05010050 − C1

5010049 + C25010048

− · · · − C4950100 + C50

50 + 10050

Logo,

a− b = 2C15010049 + 2C3

5010047 + · · ·+ 2C4950100− 10050

= 2C15010049 + · · ·+ 2C49

50100 > 0.

Portanto, a > b.

Unidade 14

Exercıcios Recomendados

1. Um armario ficara aberto se ele for mexido um numero ımpar de vezes.Por outro lado, o armario de ordem k e mexido pelas pessoas cujosnumeros sao divisores de k. Logo, estarao abertos os armarios cujosnumeros possuem um numero ımpar de divisores. Isto ocorre com os

2

numero cujos expoentes sao todos pares na decomposicao em fatoresprimos, ou seja, sao quadrados perfeitos. Assim, permaneceram abertosos armarios cujos numeros sao quadrados perfeitos, ou seja, os numeros12, 22, . . . , 302.

2. Separemos o caso em que o primeiro e o terceiro quadrantes tem coresiguais do caso em que eles tem cores diferentes.

No caso de cores iguais, ha 5 modos de escolher a cor unica para o pri-meiro e o terceiro quadrante, 4 modos de escolher a cor para o segundoquadrante e 4 modos de escolher a cor para o quarto quadrante. Ha,portanto, 5× 4× 4 = 80 modos de colorir o mapa usando cores iguaisno primeiro e no terceiro quadrantes.

No caso de cores diferentes, ha 5 modos de escolher a cor para o primeiroquadrante, 4 modos de escolher a cor para o terceiro quadrante, 3 modosde escolher a cor para o segundo quadrante e 3 modos de escolher a corpara o quarto quadrante. Ha 5 × 4 × 3 × 3 = 180 modos de colorir omapa usando cores iguais no primeiro e no terceiro quadrantes.

No total, temos, portanto, 80 + 180 = 260 modos de colorir a figura.

3. (a) Ha 265 = 11.881.376 palavras de 5 letras. Delas, devemos subtrairas palavras que comecam por A, 1× 264 = 456.976, e aquelas nasquais a letra A nao figura, 255 = 9.765.625.

A resposta e 11.881.376− 456.976− 9.765.625 = 1.658.775.

(b) O numero total de palavras de 5 letras distintas e 26×25×24×23×22 = 7.893.600. Delas devemos subtrair as palavras que comecampor A, 1× 25× 24× 23× 22 = 303.600 e aquelas nas quais a letraA nao figura, 25× 24× 23× 22× 21 = 6.375.600.

A resposta e 7.893.600− 303.600− 6.375.600 = 1.214.400.

Outra Solucao:

Ha 4 posicoes para colocar a letra A; depois disso, as quatro casasvazias podem ser preenchidas de 25, 24, 23 e 22 modos.

A resposta e 4× 25× 24× 23× 22 = 1.214.400.

4. Cada letra pode ser escolhida de 26 modos, enquanto cada algarismopode ser escolhido de 10 modos. Logo, o numero total de placas e263 × 104 = 175.760.000.

3

5. O numero de modos de acomodar os passageiros que pretendem sentarde frente e 5 × 4 × 3 × 2 = 120; o numero de modos de acomodar ospassageiros que pretendem sentar de costas e 5× 4× 3 = 60; o numerode modos de acomodar os demais passageiros e 3× 2× 1 = 6.

A resposta e 120× 60× 6 = 43.200.

6. Vamos primeiramente determinar quantos zeros sao escritos na casadas unidades, depois na das dezenas, etc.

Ha 222 numeros que tem o como algarismo das unidades, pois antes dozero podem ser colocados os inteiros de 1 (inclusive) a 222 (inclusive).

Ha 22 × 10 = 220 numeros que tem 0 como algarismo nas dezenas,pois antes do zero podem ser colocados os inteiros de 1 (inclusive) a 22(inclusive) e depois do zero, os inteiros 0 (inclusive) a 9 (inclusive).

Ha 2 × 100 = 200 numeros que tem 0 como algarismo das centenas,pois antes do zero podem ser colocados os inteiros de 1 (inclusive) a 2(inclusive) e depois do zero os inteiros de 0 (inclusive) a 99 (inclusive).

A resposta e 222 + 220 + 200 = 642.

7. E mais simples contar, primeiramente, os numeros onde o algarismo5 nao aparece. O primeiro digito pode ser escolhido de 8 modos (naopode ser igual a 0 nem igual a 5) e cada um dos demais tres dıgitospode ser selecionado de 9 modos (deve ser diferente de 5). Logo, ha8×93 = 5.832 numeros de a algarismos em que nao aparece o algarismo5.

A quantidade de numero de 4 dıgitos, com ou sem o digito 5, e 9×103 =9.000 (pois ha 9 modos de selecionar o primeiro digito, que deve serdiferente de 0, e 10 modos de selecionar cada um dos demais 4 dıgitos).Logo, ha 9.000 − 5.832 = 3.168 numeros de 4 algarismos em que o 5nao aparece.

8. Devemos decidir quantos exemplares de cada revista devem ser postosna colecao. Ha 6 possibilidades para a “Veja” (0, 1, 2, 3, 4, ou 5exemplares), 7 para a “Manchete” e 5 para a “Isto e”. O numero decolecoes e 6× 7× 5 = 210, e o numero de colecoes nao-vazias e 209.

9. Em cada dia, duas das materia sao ensinadas e uma folga. Ha 3 possi-bilidades para escolher a materia que folga na segunda, 2 para escolher

4

a que folga na quarta e 1 para escolher a que folga na sexta. Portanto,ha 6 modos para escolher as materias de cada dia. Para escolher oshorarios, ha 2 possibilidades em cada dia. Logo, o numero total dehorarios e 6× 8 = 48.

10. Foi feita uma distincao artificial ao se considerar cada casal ordenadode dois modos diferentes: comecando pela mulher ou pelo homem. Poresta razao, o resultado encontrado foi igual ao dobro do correto.

11. Ha tres tipos de cartoes: os que virados de cabeca para baixo naorepresentam numeros, como, por exemplo, 41.809; os que virados decabeca para baixo representam o mesmo numero, como, por exemplo,86.198; os que virados de cabeca para baixo representam numeros di-ferentes, como, por exemplo, 66.810. Os cartoes do ultimo tipo saoos que permitem economia porque um mesmo cartao serve para re-presentar dois numeros. Ha 5 × 5 × 5 × 5 × 5 = 3.125 cartoes quevirados de cabeca para baixo representam numero, iguais ou diferen-tes, pois tais cartoes devem ter como algarismos apenas 0, 1, 8, 6 ou9. Destes, 5 × 5 × 3 = 75 sao do segundo tipo, pois um tal cartaodeve ter as casas das extremidades preenchidas por 00, 11, 88, 69 ou96, a segunda e a quarta casa preenchidas por 00, 11, 88, 69 ou 96, e acasa central preenchida por 0, 1 ou 8. Portanto, os cartoes do terceirotipo sao em numero de 3.125 − 75 = 3.050. Podem ser economizados3.050/2 = 1.525 cartoes. O numero mınimo de cartoes que se necessitae 100.000− 1.525 = 98.475.

12. A decomposicao de 360 em fatores primos e 720 = 23.32.5. Os divisoresinteiros e positivos de 720 sao numeros da forma 2α.3β.5γ, com α ∈0, 1, 2, 3, β ∈ 0, 1, 2 e γ ∈ 0, 1. A soma dos divisores e S =∑

2α.3β.5γ, com α ∈ 0, 1, 2, 3, β ∈ 0, 1, 2 e γ ∈ 0, 1. Paracalcular essa soma, dividimos as parcelas em dois grupos, conforme sejaγ = 0 ou γ = 1. S =

∑(2α.3β.50) +

∑(2α.3β.51) = 6

∑(2α.3β) porque

a segunda soma e igual ao quıntuplo da primeira. Agora, dividimosas parcelas em grupos, conforme seja β = 0, β = 1 ou β = 2. S =6[∑

(2α.30) +∑

(2α.31) +∑

(2α.32)] = 6[∑

2α + 3∑

2α + 9∑

2α] =6[13

∑2α] = 78

∑2α = 78[20 + 21 + 22 + 23] = 78× 15 = 1.170.

5

Exercıcios Resolvidos de MA 12

Unidades 15 e 16

Unidade 15

Exercıcios Recomendados

1. Devemos inicialmente escolher a ordem em que as moca ficarao juntas,o que pode ser feito de m! maneiras. Em seguida, devemos arrumarem fila r + 1 objetos, os r rapazes e o bloco das mocas, o que pode serfeito de (r + 1)! modos.

A resposta e m!(r + 1)!.

2. (a) A face a receber o numero 1 pode ser escolhida de 6 modos, a donumero 2 de 5 modos, e assim por diante. O numero de possibili-dades e 6.5.4.3.2.1 = 720.

(b) Inicialmente, fazemos a conta que as faces tem cores diferentes.Contamos, pelo problema anterior, 720 dados. Como as facessao indistinguıveis, o mesmo dado foi contado varias vezes. Porexemplo, pense em um dado que tenha o 6 na face de baixo (facepreta) e o 1 na face de cima (face branca). Ele e, certamente,diferente de um dado que tenha o 1 na face de baixo (face preta) e o6 na face de cima (face branca). Mas sendo as faces indistinguıveis,o dado que tem 6 na face de baixo e o 1 na face de cima e igualao dado que tem o 1 na face de baixo e o 6 na face de cima; estee, simplesmente, aquele de cabeca para baixo. Esse mesmo dadoaparece outra vez com o 1 na face da frente e o 6 na face de tras,com o 1 na face da esquerda e o 6 na face da direita, etc. Em suma,o mesmo dado foi contado tantas vezes quantas sao as posicoes decoloca-lo.

O numero de posicoes de colocar um cubo e 6 × 4 = 24, pois ha6 modos de escolher a face de baixo e 4 de escolher, nessa face, olado que fica de frente.

1

A resposta e720

24= 30.

Outra solucao:

Todo dado pode ser imaginado com a face 1 embaixo. Realmente,se o 1 nao estiver embaixo, e possıvel rodar o dado de modo queo 1 va para baixo. Fixando o 1 embaixo, devemos escolher quemocupara a face oposta a face do 1. Isso pode ser feito de 5 modos.Digamos que tenha sido escolhido o 6. Com o 1 fixo embaixo eo 6 fixo em cima, devemos colocar os numeros 2, 3, 4 e 5 nasfaces laterais. O 2 sempre pode ser imaginado na face da frente.Com efeito, se o 2 nao estiver na face da frente, uma convenienterotacao coloca-lo-a na face da frente, sem tirar o 1 da face de baixonem o 6 da face de cima. Fixamos o 2 na frente, o 1 embaixo e o6 em cima, devemos escolher quem ocupara a face oposta a facedo 2. Isso pode ser feito de 3 modos. Digamos que tenha sidoescolhido o 4. Agora, devemos colocar o 3 e o 5 nas faces dadireita e da esquerda. Note que qualquer movimento com o dadoou retirara o 1 de baixo, ou o 6 de cima, ou o 2 da frente, ou o4 de tras, Portanto, ha 2 modos de preencher as faces direita eesquerda com os numeros 3 e 5.

A resposta e 5× 3× 2 = 30.

(c) Um dado com faces de cores diferentes pode, agora, ser numeradode apenas 6.4.2 = 48 modos, ja que temos 6 faces a escolher para onumero 1 (isto determina a face do numero 6), a para o numero 2(o que determina a face do 5) e 2 para o numero 3 (que determinaa do 4). Mas como as faces sao iguais, cada dado e contado, comono ıtem anterior, 24 vezes. Logo ha apenas 48/24 = 2 dadosdistintos.

Outra Solucao:

Como antes, podemos fixar o dado com o numero 1 embaixo.Agora, no entanto, isto tambem fixa o numero 6 na frente de cima.Agora, o numero 2 pode ser fixado na face da frente (e, portanto,o numero 5 na de tras). Assim, tudo que temos a escolher e se aface lateral da direita e 3 ou o 4. Temos, portanto, apenas duaspossibilidades.

3. (a) O numero de posicoes para um tetraedro e 4 × 3 = 12, pois ha 4modos de escolher a face de apoio e 4 de escolher, nessa face, o

2

lado que fica de frente.

A resposta e4!

12= 2.

(b) O numero de posicoes para um octaedro regular e 6×4 = 24, poisha e modos de escolher o vertice de apoio e 4 de escolher, dentreas arestas que incidem nesse vertice, a que fica de frente.

A resposta e8!

24= 1.680.

(c) O numero de posicoes para um dodecaedro regular e 12× 5 = 60,pois ha 12 modos de escolher a face de apoio e 5 de escolher, nessaface, o lado que fica de frente.

A resposta e12!

60= 7.938.360.

(d) O numero de posicoes para um icosaedro regular e 20 × 3 = 60,pois ha 20 modos de escolher a face de apoio e 3 de escolher, nessaface, o lado que fica de frente.

A resposta e20!

60= 40.548.366.802.944.000 ∼= 4.1016.

4. Temos 1! = 1, que e um quadrado perfeito, 1! + 2! = 1 + 2 = 3, quenao e quadrado perfeito, 1! + 2! + 3! = 1 + 2 + 6 = 9, que novamente equadrado perfeito, 1! + 2! + 3! + 4! = 1 + 2 + 6 + 24 = 33, que nao equadrado perfeito. Todos os fatorais seguintes, a partir de 5! terminamcom zero, ja que sao multiplos de 5 e 2. Logo, todas as somas da forma∑n

1 k! para n ≥ 5 terminam com o algarismo 3 e nao sao, portanto,quadrados perfeitos. As unicas solucoes sao n = 1 e n = 3.

5. Em ESTRELADA as letras A e E aparecem 2 vezes cada e as letras S,T, R, L e D aparecem uma vez cada uma, havendo, portanto, 9 letrasna palavra.

Para formar um anagrama, devemos escolher 2 das 9 posicoes paracolocar as letras A, o que pode ser feito de C2

9 modos, 2 das 7 posicoesrestantes para colocar as letras E, o que pode ser feito de C2

7 modos, earrumar as letras S, T, R, L e D nas 5 posicoes restantes, o que podeser feito de 5! modos. A resposta e C2

9 .C27 .5! = 90.720.

Outra solucao:

O numero de anagramas e P 2,2,1,1,1,1,19 =

9!

2!2!1!1!1!1!1!= 90.720.

3

6. Formar uma solucao com p elementos significa escolher p dos n elemen-tos. A resposta e Cp

n.

7. Basta escolher as provas do primeiro dia, o que pode ser feito de C48 = 70

modos.

8. O processo de contagem apresentado conta determinadas comissoesmais de uma vez. Isto ocorre porque um homem que participe dacomissao pode ser inserido de dois modos diferentes: como um dos 3homens escolhidos dos inicialmente, ou como uma das duas pessoasescolhidas posteriormente. O pior e que nao e possıvel “corrigir” a con-tagem dividendo pelo numero de vezes que cada comissao e contada:as comissoes com 3 homens sao contadas apenas uma vez, as que tem4 homens sao contadas 4 vezes, enquanto a que contem 5 homens econtada 10 vezes.

A solucao correta e dada no Exemplo 6, contando separadamente ascomissoes com 3, 4 e 5 homens: C3

5 .C24 + C4

5 .C14 + C5

5 = 81.

9. Os segmentos que unem dos vertices de um poliedro ou sao arestas ousao diagonais de faces ou diagonais do poliedro.

(a) O octaedro regular e um poliedro formado por 8 faces triangularese que tem 6 vertices e 12 arestas. Ha C2

6 = 15 segmentos que unemdois vertices do poliedro, pois 12 dos quais sao arestas e 0 dos quaise diagonal de face.

A resposta e 15− 12− 0 = 3.

(b) O icisaedro regular e um poliedro formado por 20 faces triangu-lares e que tem 12 vertices e 30 arestas. Ha C2

12 = 66 segmentosque unem dois vertices do poliedro, 30 dos quais sao arestas e 0dos quais e diagonal de face.

A resposta e 66− 30− 0 = 36.

(c) O dodecaedro regular e um poliedro formado por 12 faces pentago-nais e que tem 20 vertices e 30 arestas. Ha C2

20 = 190 segmentosque unem dois vertices do poliedro, 30 dos quais sao arestas e

125(5− 3)

2= 60 dos quais sao diagonais de faces.

A resposta e 190− 30− 60 = 100.

4

(d) O cubo e um poliedro formado por 6 faces quadradas e que tem 8vertices e 12 arestas. Ha C2

8 = 28 segmentos que unem os vertices

do poliedro, 12 dos quais sao arestas e 64(4− 3)

212 dos quais sao

diagonais de faces.

A resposta e 28− 12− 12 = 4.

Outra solucao:

Cada diagonal de um prisma n-agonal une um vertice da base “decima” a um vertice da base “de baixo”. O vertice da base “decima” pode ser selecionado de n modos; depois disso, o da base“de baixo” pode ser selecionado de n − 3 modos, pois um dosvertices da base “de baixo”, se selecionado daria origem a umaaresta e os dois vertices adjacentes nesta base dariam origem adiagonais das faces laterais. O numero de diagonais de um prisman-agonal e portanto, n(n− 3).

Como o cubo e um prisma quadrangular, a resposta e 4(4−3) = 4.

(e) O prisma hexagonal e um poliedro formado por 6 faces quadran-gulares e 2 faces hexagonais e que tem 12 vertices e 18 arestas. HaC2

12 = 66 segmentos que unem os vertices do poliedro, 18 dos quais

sao arestas e 64(4− 3)

2+ 2

6(6− 3)

2= 30 dois quais sao diagonais

de faces.

A resposta e 66− 18− 30 = 18.

Outra solucao:

O numero de diagonais de um prisma n-agonal e, como visto emd), n(n − 3). Portanto, o numero de diagonais de um prismahexagonal e 6(6− 3) = 18.

10. Uma funcao estritamente crescente e necessariamente injetiva (se f(a) =f(b), nao pode ser a < b, pois, neste caso, f(a) < f(b), o que e absurdo;do mesmo modo, nao pode ser a > b, pois, neste caso, f(a) > f(b),o que e absurdo; logo, a = b), Logo, seu conjunto de valores teraexatamente m elementos. Para construir uma tal funcao, devemos, ini-cialmente selecionar o conjunto de valores, o que pode ser feito de Cm

n

modos.

Selecionando o conjunto de valores, a funcao esta determinada porquef(1) deve ser igual ao menor elemento do conjunto de valores, f(2)

5

deve ser igual ao segundo menor elemento do conjunto de valores, etc.

A resposta e, portanto, Cmn .

11. Vamos esquecer que a primeira casa do numero nao pode ser igual azero. Isso fara com que contemos a mais e, depois, descontaremos oque foi contado indevidamente.

Ha C37 modos de escolher as casas que serao ocupadas pelo digito 4;

depois disso, ha C24 modos de selecionar as casas que serao ocupadas

pelo digito 8; finalmente, as duas casas restantes podem ser preenchidasde 8× 8 modos (nao podemos usar nessas casas os dıgitos 4 e 8).

A “resposta” seria C37 × C2

4 × 8× 8 = 35× 6× 64 = 13.440.

Devemos subtrair os numeros comecados por zero. Se o numero comecapor 0, ha C3

6 modos de escolher as casas que serao ocupadas pelo digito4; depois disso, ha C2

3 modos de selecionar as casas que serao ocupadaspelo digito 8; finalmente, a casa restante pode ser preenchida de 8modos (nao podemos usar nessa casa os dıgitos 4 e 8). Ha C3

6×C23×8 =

20× 3× 8 = 480 numeros comecados por 0.

A resposta e 13.440− 480 = 12.960

Outra solucao:

Vamos a contar separadamente:

i) numeros que comecam com 4; ii) numeros que comecam com 8; iii)numeros que nao comecam nem com 4 nem com 8.

i) Ha 1 modo de preencher a primeira casa; depois disso, ha C26

modos de escolher as outras duas casas do numero que tambemserao preenchidas com o algarismo 4; depois disso, ha C2

4 modosde escolher as duas casas que serao ocupadas pelo algarismo 8;finalmente, as duas casas restantes podem ser preenchidas de 8×8modos (nao podemos usar nessas casas os dıgitos 4 e 8).

Ha 1 × C26 × C2

4 × 8 × 8 = 1 × 15 × 6 × 64 = 5.760 numeros dotipo i).

ii) Ha 1 modo de preencher a primeira casa; depois disso, ha 6 modosde escolher a outra casa do numero que tambem sera preenchidacom o algarismo 8; depois disso, ha C3

5 modos de escolher as tres

6

casa que serao ocupadas pelo algarismo 4; finalmente, as duas ca-sas restantes podem ser preenchidas de 8×8 modos (nao podemosusar nessas casas os dıgitos 4 e 8).

Ha 1× 6× C35 × 8× 8 = 6× 10× 64 = 3840 numeros do tipo ii).

iii) Ha 7 modos de preencher a primeira casa (nao podemos usar nem4, nem 8, nem 0); depois disso, ha C3

6 modos de escolher as trescasas do numero que serao preenchidas com o algarismo 4; depoisdisso. ha C2

3 modos de escolher as duas casas que serao ocupadaspelo algarismo 8; finalmente, a casa restante pode ser preenchidade 8 modos (nao podemos usar nessas casa os dıgitos 4 e 8).

Ha 7×C36 ×C2

3 × 8 = 7× 20× 3× 8 = 3.360 numeros de tipo iii).

A resposta e 5.760 + 3.840 + 3.360 = 12.960.

12. (a) Para formar o subconjunto devemos escolher os p − 1 outros ele-mentos do subconjunto dentre os n − 1 outros elemento do con-junto.

A resposta e Cp−1n−1.

(b) Para formar o subconjunto devemos escolher os p elementos dosubconjunto dentre os n− 1 outros elementos do conjunto.

A resposta e Cpn−1.

Outra solucao:

Ha Cpn p-subconjuntos e o elemento a1 figura em Cp−1

n−1 deles. Logo,

ha Cpn − Cp−1

n−1 subconjuntos nos quais o elemento a1 nao figura.

A resposta e Cpn − Cp−1

n−1.

Observacao: As duas solucoes apresentadas mostram que Cpn −

Cp−1n−1 = Cp

n−1. Essa e a famosa Relacao de Stifel.

(c) Para formar o subconjunto devemos escolher os p − 2 outros ele-mentos do subconjunto dentre os n − 2 outros elementos do con-junto.

A resposta e Cp−2n−2.

(d) O total de p-subconjuntos e Cpn. Para formar um subconjunto em

que nem a1 nem a2 figurem devemos escolher os p elementos dosubconjuntos dentre os n − 2 outros elementos do conjunto. Ha,portanto, Cp

n−2 subconjuntos nos quais nem a1 nem a2 figuram.

7

Logo, o numero de subconjuntos nos quais pelo menos um dessesdois elementos figura e Cp

n − Cpn−2.

Outra solucao:

Ha Cp−1n−1 p-subconjuntos nos quais o elemento a1 figura e ha Cp−1

n−1

subconjuntos nos quais o elemento a2 figura. Ha, tambem, Cp−2n−2

p-subconjuntos nos quais os elementos a1 e a2 figuram ambos. Aosomarmos Cp−1

n−1 + Cp−1n−1 = 2Cp−1

n−1 obtemos o numero de subcon-juntos nos quais pelo menos um dos elementos a1 e a2 figura, mascontamos duas vezes aqueles que a1 e a2 figuram ambos.

A resposta e, portanto, 2Cp−1n−1 − Cp−2

n−2.

Outra solucao:

Ha, como mostrado em c), Cp−2n−2 p-subconjuntos em que os ele-

mento a1 e a2 figuram ambos.

Ha Cp−1n−2 p-subconjuntos em que o elemento a1 figura e o elemento

a2 nao figura, pois, para formar um tal subconjunto, basta escolheros outros p−1 elementos do subconjunto dentre os n−2 elementosdo conjunto que sao diferentes de a1 e de a2.

Ha analogamente, Cp−1n−2 p-subconjuntos em que o elemento a2

figura e o elemento a1 nao figura. Portanto, o numero de p-subconjuntos em que figura pelo menos um desses dois elementose 2Cp−1

n−2 + Cp−2n−2.

(e) Como visto na solucao anterior, a resposta e 2Cp−1n−2.

Outra solucao:

Ha, como visto em d). 2Cp−1n−1 − Cp−2

n−2 p-subconjuntos nos quaispelo menos um dos elementos a1 e a2 figura. Ha, como vistoem c), Cp−2

n−2 p-subconjuntos em que os elementos a1 e a2 figuramambos.

A resposta e, portanto, 2Cp−1n−1 − Cp−2

n−2 − Cp−2n−2 = 2Cp−1

n−1 − 2Cp−2n−2.

Outra solucao:

Ha como visto em d), 2Cp−1n−2 +Cp−2

n−2 p-subconjuntos nos quais pelomenos um dos elementos a1 e a2 figura. Ha, como visto em c),Cp−2

n−2 p-subconjuntos em que os elementos a1 e a2 figuram ambos.

A resposta e, portanto, 2Cp−1n−2 + Cp−2

n−2 − Cp−2n−2 = 2Cp−1

n−2.

13. (a) Como ha 32 cartas, a resposta e C232 = 201.376.

8

(b) Ha 8 modos de escolher o grupo do par propriamente dito (porexemplo, valete), C2

4 modos de escolher os naipes das duas cartasdo par (por exemplo, copas e paus), C3

7 modos de escolher osgrupos das outras tres cartas (por exemplo, 10, 8 e rei) e 4×4×4 =43 modos de escolher os naipes dessas tres cartas.

A resposta e 8× C24 × C3

7 × 43 = 107.520.

(c) Ha C28 modos de escolher os grupos das cartas que formarao os dois

pares, (C24)2 modos de escolher seus naipes, 6 modos de escolher

o grupo da outra carta e 4 modos de escolher seu naipe.

A resposta e C28 × (C2

4)2 × 6× 4 = 24.192.

Observacao: Um erro muito comum e o exposto a seguir.

Ha 8 modos de escolher o grupo do primeiro par, C24 modos de

escolher os naipes do primeiro par, 7 modos de escolher o grupodo segundo par, C2

4 modos de escolher os naipes do segundo par,6 modos de escolher o grupo da outra carta e 4 modos de escolherseu naipe. A resposta ERRADA seria 8 × C2

4 × 7 × C24 × 6 ×

4 = 48.384. A explicacao do ERRO e simples: Ao fazermos ainexistente distincao entre primeiro par e segundo par, contamospares de valetes e reis como diferentes de pares de reis e valetes.

A resposta ERRADA pode ser corrigida dividindo-a por 2.

(d) Ha 8 modos de escolher o grupo da trinca, C34 modos de escolher

os naipes das cartas da trinca, C27 modos de escolher os grupos

das outras duas cartas e 4 × 4 = 42 modos de escolher os naipesdessas duas cartas.

A resposta e 8× C34 × C2

7 × 42 = 10.752

(e) Ha 8 modos de escolher o grupo do “four”, 1 modo de escolher osnaipes das quatro cartas do “four”, 7 modos de escolher o grupoda outra carta e 4 modos de escolher o naipe dessa carta.

A resposta e 8× 1× 7× 4 = 224.

(f) Ha 8 modos de escolher o grupo da trinca, C34 modos de escolher

os naipes das cartas da trinca, 7 modos de escolher o grupo dopar e C2

4 modos de escolher os naipes das cartas do par.

A resposta e 8× C34 × 7× C2

4 = 1.344

(g) Ha apenas 4 tipos de sequencias: 7. 8. 9. 10, valete, dama, rei;10, valete, dama, rei, as. Escolhido o tipo da sequencia, haveria

9

4× 4× 4× 4× 4× 4 = 45 modos de escolher os naipes das cartasdas sequencias, mas 4 desses modos nao sao permitidos: todas deouros, todas de paus, todas de copas e todas de espadas.

A resposta e 4× [45 − 4] = 4.080.

(h) Os grupos das cartas podem ser escolhidos de C58 − 4 modos e o

naipe unico, de 4 modos.

A resposta e (C58 − 4)× 4 = 208.

(i) Ha 4 modos de escolher os grupos de cartas e 4 modos de escolhero naipe unico.

A resposta e 4× 4 = 16.

(j) Ha 4 modos de escolher o naipe unico. A resposta e 4.

14. (a) Neste caso f e bijetiva e, se #A = #B = n, o numero de funcoesf : A → B bijetivas e n!, como foi mostrado no exercıcio 4 dasecao 2.2.

(b) Neste caso dois elementos de A terao uma mesma imagem em Be a correspondencia entre os demais n − 1 elementos de A e osdemais n− 1 elementos de B sera bijetiva.

Ha(n+12

)modos de escolher os dois elementos de A, n modos de

escolher a imagem deles em B e (n− 1)! modos de construir umacorrespondencia bijetiva entre os elementos restantes.

A resposta e(n+12

)· n · (n− 1)! =

n(n + 1)!

2.

(c) Neste caso temos as alternativas

i) Tres elementos de A tem a mesma imagem em B e a corres-pondencia entre os demais n− 1 elementos de A e os demaisn− 1 de B e bijetiva.Ha(n+23

)modos de escolher os tres elementos de A, n modos

de escolher a imagem deles em B e (n−1)! modos de construiruma correspondencia bijetiva entre os elementos restantes.

Ha(n+23

)· n · (n− 1)! =

n(n + 2)!

6funcoes desse tipo.

ii) Ha dois pares de elementos de A com imagens identicas emB e a correspondencia entre os demais n− 2 elementos de Ae os demais n− 2 elementos de B e bijetiva.

10

Ha(n2

)modos de escolher os dois elementos de B,

(n+22

)×(n2

)modos de escolher suas imagens inversas em A e (n−2)! modosde estabelecer a correspondencia entre os elementos restantes.

Ha(n2

)×(n+22

)×(n2

)× (n − 2)! =

n(n− 1)(n + 2)!

8funcoes

desse tipo.A resposta e:

n(n + 2)!

6+

n(n− 1)(n + 2)!

8=

n(3n + 1)(n + 2)!

24.

15. Chamemos de D o conjunto C − C1.

Ha quatro tipos de planos:

i) determinados por tres pontos de D;

ii) determinados por dois pontos de D e um de C1;

iii) determinados por um ponto de D e dois de C1;

iv) determinados por tres pontos de C1.

A resposta e C312 + C2

12.8 + 12.C28 + 1 = 1.085

Outra solucao:

Para determinar um plano, devemos selecionar 3 dos 20 pontos, o quepode ser feito de C3

20 = 1140 modos. Nessa contagem, o plano quecontem os 8 pontos de C1 foi contado C3

8 = 56 vezes.

A resposta e 1.140− 56 + 1 = 1.085.

Unidade 16

Exercıcios Recomendados

1. Escolhida a ordem de cada casal, o que pode ser feito de 23 modos,temos que arrumar em fila 4 espacos vazios e 3 casais, o que pode serfeito de C4

7 modos (escolha dos espacos vazios) vezes 3! (colocacao dos3 casais nos 3 lugares restantes).

A resposta e 23 × C47 × 3! = 1.680.

11

2. Primeiro, colocamos as vogais. Como a letra A aparece 3 vezes e asletras U, I e O aparecem 1 vez cada, o numero de modos de dispo-lase P 3,1,1,1

6 = 6!3!

= 7206

= 120. A seguir, colocamos as consoantes emtres dos 7 espacos antes, entre e depois das vogais. O lugar do P podeser qualquer um destes 7 espacos, o do R qualquer dos 6 restantes eo do G qualquer dos 5 restantes. O numero total de possibilidades e120.7.6.5 = 25.200.

3. Vamos a formar uma fila com os numeros 1, 2, . . . , n e assinalar com Eos p numeros escolhidas e com N os n− p nao escolhidos. A condicaopara que nao sejam escolhidos numeros consecutivos e que entre doisE haja pelo menos um N. Comecamos escrevendo os n− p E. A seguir,devemos escolher, para colocar os E, p dentre os n − p + 1 espacossituados antes, entre e depois dos N. Isto pode ser feito de Cp

n−p+1

modos.

4. Chegam 4 cientistas A, B, C, D. Com as chaves que possuem, abremalguns cadeados, mas nao todos. Existe pelo menos um cadeado queeles nao conseguem abrir. Na situacao do numero mınimo de cadeados,existe exatamente um cadeado que eles nao conseguem abrir. Batizetal cadeado de ABCD. Portanto, ABCD e p cadeado cuja chave naoesta em poder de A, nem de B, nem de C e nem de D. Qualquer outrocientista tem a chave desse cadeado, pois esse cientista e A, B, C eD formam um grupo de 5 cientistas e, portanto, nesse grupo alguempossui a chave. Como o alguem nao e nem A, nem B, nem C e nem D,deve ser o outro. Analogamente batize os demais cadeados. Verifiqueagora que a correspondencia entre cadeados e seus nomes e biunıvoca,isto e, cadeados diferentes tem nomes diferentes (isso porque estamosna situacao do numero mınimo de cadeados) e cadeados de nomes di-ferentes sao diferentes (se X esta no nome de um cadeado e nao estano nome de outro, X tem a chave deste e nao tem a chave daquele).

(a) O numero mınimo de cadeados e igual ao numero de nomes decadeados, C4

11 = 330.

(b) Cada cientista X possui as chaves dos cadeados que nao possuemX no nome, C4

10 = 210.

5. Nenhum aluno pode comparecer a mais de tres jantares. Com efeito, seA1 vai a um jantar com A2 e A3, ele so pode ir a outro jantar com outros

12

dois estudantes, digamos A4 e A5 e so poder ir a um terceiro jantar emcompanhia de outros dois, digamos A6 e A7 e nao tera companhia parair a um quarto jantar. Como ha 21 convites e sao 7 estudantes, cadaestudante tera que compadecer a exatamente 3 jantares.

Se A1 comparece a tres jantares, podemos escolher os seus companhei-ros dividindo os outros 6 estudantes em 3 grupos de 2, o que pode ser

feito deC2

6 × C24 × 1

3!= 15 modos.

Entao, os 3 jantares sao, digamos, A1A2A3, A1A4A5, A1A6A7.

A2 devera comparecer a mais dos jantares, nenhum deles em companhiade A3, e A3 tambem devera comparecer a mais dois jantares. Portanto,os 4 jantares que faltam sao:

A2−−, A2−−, A3−−, A3−−

Como A4 deve compareces a mais dois jantares (A4 nao pode compare-ces a ambos em companhia de A2 nem a ambos em companhia de A3),esses quatro jantares sao:

A2A4−, A2−−, A3A4−, A3−−;

A5 tem que comparecer ainda a dois jantares, nenhum deles em com-panhia de A4.

A2A4−, A2A5−, A3A4−, A3A5−.

Agora ha duas possibilidades:

A2A4A6, A2A5A7, A3A4A7, A3A5A6 e

A2A4A7, A2A5A6, A3A4A6, A3A5A7.

Ha portanto 15× 2 = 30 maneiras de escolher os grupos de convidado.Para distribuir os 7 grupos nos 7 dias, ha 7! alternativas.

A resposta e 7!× 30 = 151.200.

6. Os dois primeiros lugares so podem ser ocupados por elementos dea1, a2, . . . , a7 e os dois ultimos por exemplo de a9, a10, a11, a12.A resposta e C2

7 × C24 = 126.

7. Ha Cmm+h × 1 modos de escolher os lugares para os homens. Feito isso,

so ha 1 modo de formar a fila.

A resposta e Cmm+h × 1 = Cm

m+h.

13

8. (a) Cada professor fica caracterizado pelas duas bancas a que per-tence. O numero de professores e igual ao numero de modos deescolher duas das oito bancas.

A resposta e C28 = 28.

(b) O numero de professores pertencentes a uma banda e igual aonumero de modos de escolher a outra banca a que ele pertence.

A resposta e 7.

9. (a) Imagine um quadro em que cada linha e relacao dos atletas de umtime. O numero de elementos do quadro e o numero de times, t,multiplicado pelo tamanho de cada time, k, e e tambem igual aonumero de atletas, a, multiplicado pelo numero de times de que acada atleta participa, x.

Logo, ax = tk e x = tka

.

(b) No mesmo quadro, o numero de pares de atletas na mesma linhae igual ao numero de linhas, t, multiplicado pelo numero de paresde atletas em uma linha, C2

k , e e tambem igual ao numero de paresde atletas, C2

a , multiplicado pelo numero de times em que cadapar de atletas fica junto, y.

Logo, yC2a = tC2

k e y =tC2

k

C2a

= tk(k−1)a(a−1)

.

10. A resposta e o numero de permutacoes circulares de 4 elementos, ouseja, 3! = 6.

11. Ha (PC)5 = 4! modos de formar uma roda com as meninas. Depoisdisso, os 5 meninos devem ser postos nos 5 lugares entre as meninas, oque pode ser feito de 5! modos. A resposta e 4!×5! = 24×120 = 2.880.

12. E mas simples calcular o numero total de rodas e excluir aquelas emque Vera e Isadora ficam juntas. O numero total de rodas e PC6 =5! = 120. Para formar as rodas em que Vera e Isadora se colocarao naroda. Ha 2 possibilidades: Vera-Isadora e Isadora-Vera. Agora tudose passa como se Vera e Isadora fossem uma unica crianca. Assim,ha 2(PC)5 = 2.4! = 48 rodas em que Vera e Isadora ficam juntas. Aresposta e 120− 48 = 72 rodas.

13. Chamando x de 1 +a, y de 1 + b e z de 1 + c, o problema se transformaem encontrar todas as solucoes inteiras e nao-negativas de (a+1)+(b+1)+(c+1) = 7, ou seja, de a+c+c = 4. A resposta e CR4

3 = C46 = 15.

14

14. Cada solucao inteira e nao negativa da inequacao x+ y + z ≤ 6 corres-ponde a uma solucao inteira e nao negativa da equacao x+y+z+f ≤ 6.Logo, ha CR6

4 = C69 = 84 solucoes.

15. Para formar uma caixa, devemos selecionar 20 dentre os 5 tipos, valendorepeticao na escolha. Ou seja, devemos formar solucoes inteiras e naonegativas de x1+x2+x3+x4+x5 = 20, onde xi e o numero de bombonsdo tipo i. A resposta e CR20

5 = C2024 = 10.626.

15

Exercıcios Resolvidos de MA 12

Unidades 17 e 18

Unidade 17

Exercıcios Recomendados

1. Ha 6×6 = 36 resultados possıveis igualmente provaveis, em 6 dos quaisa soma vale 7. A resposta e 6

36= 1

6.

2. Basta escolher os times do primeiro grupo, o que pode ser feito de C1224

modos. Os dois times em questao ficam juntos quando ficam ambos noprimeiro grupo ou ambos no segundo grupo. Em ambos os casos, isto

pode ser feito de C1022 modos. Logo, a resposta e

2C1022

C1224

=2.22!.12!

24!10!=

11

23.

Outra solucao:

Supondo ja escolhido o grupo do primeiro time, seus 11 companheirosde grupo podem ser escolhidos de C11

23 modos. Dentre os grupos assimformados os que tambem incluem o segundo time sao C10

22 , ja que saoformados escolhendo 10 times entre os 22 restantes. Logo, a resposta eC10

22

C1123

=11

23.

3. Usaremos o fato, ja provado, de que

P (A ∪B) = P (A) + P (B)− P (A ∩B).

Temos

P (A ∪B ∪ C) =P ((A ∪B) ∪ C) = P (A ∪B) + P (C)− P ((A ∪B) ∩ C)

=P (A) + P (B)− P (A ∩B) + P (C)− P ((A ∪B) ∩ C).

Agora, (A ∪B) ∩ C = (A ∩ C) ∪ (B ∩ C) e daı

P ((A ∪B) ∩ C) =P (A ∩ C) + P (B ∩ C)− P ((A ∩ C) ∩ (B ∩ C))

=P (A ∩ C) + P (B ∩ C)− P (A ∩B ∩ C).

1

Substituindo na expressao anterior, obtemos, finalmente:

P (A ∪B ∪ C) =P (A) + P (B) + P (C)− P (A ∩B)− P (A ∩ C)

− P (B ∩ C) + P (A ∩B ∩ C).

4. (a) Como A ⊂ A ∪B, temos P (A ∪B) ≥ P (A) = 23.

(b) Como A ∩ B e B sao disjuntos e A ∩ B ∪ B = A ∪ B, temosP (A ∩ B) + P (B) = P (A ∪ B) e, portanto, P (A ∩ B) = P (A ∪B)− P (B) = P (A ∪B)− 4

9.

Mas, do ıtem anterior, 23≤ P (A∪B) ≤ 1. Daı, 2

3− 4

9≤ P (A∩B) ≤

1− 49, ou seja, 2

9≤ P (A ∩B) ≤ 5

9.

(c) Observe que P (A ∩ B) + P (A ∩ B) = P (A), ja que o conjuntoda direita e a uniao disjunta dos da esquerda. Daı, P (A ∩ B) =P (A) − P (A ∩ B) = 2

3− P (A ∩ B). Como 2

9≤ P (A ∩ B) ≤ 5

9,

temos 23− 5

9≤ P (A ∩B) ≤ 2

3− 2

9, ou seja, 1

9≤ P (A ∩B) ≤ 4

9.

5. (a) O numero de casos possıveis e 65, pois ha 6 resultados para cadaum dos 5 dados. O numero de casos favoraveis e 6 ·C2

5 · 5 · 4 · 3 =3600, pois ha 6 modos de escolher o tipo de par (pode ser de 1, de2, . . . , de 6) e ha C2

5 modos de escolher os dois dados que formaraoo par; quatro outros dados, o resultado do primeiro deles pode serescolhido de 5 modos distintos (deve ser diferente do resultado dosdois primeiros dados), o do segundo pode ser escolhido de 4 modosdistintos (deve ser diferente dos anteriores) e, o do terceiro, de 4modos diferentes.

Logo, P (A2) =3.600

65=

25

54≡ 0, 463.

(b) O numero de casos possıveis e 65, pois ha 6 resultados para cadaum dos 5 dados. O numero de casos favoraveis e C2

6 ·C25 ·C2

3 · 4 =1800, pois ha C2

6 modos de escolher os tipos de pares (podem serde 1 e 2, de 1 e 3, . . . , de 5 e 6), ha C2

5 modos de escolher osdois dados que formarao o par menor 3 C2

3 modos de escolheros dados que formarao o par maior. Para o dado restante, quedeve ter resultado diferente do dos demais dados, ha 4 resultadospossıveis.

Logo, P (A3) =180

65=

25

108≡ 0, 231.

2

Observacao:

Um erro comum e contar os casos favoraveis em dobro, racionandodo modo seguinte: Ha 6 modos de escolher o tipo do primeiro par,5 modos de escolher o tipo do segundo par, C2

5 modos de escolheros dois dados que formarao o primeiro par, C2

3 modos de escolheros dados que formarao o segundo par e ha 4 modos de escolher oresultado do dado restante. Logo, o numero de casos favoraveis e6 · 5 · C2

5 · C23 · 4 = 3.600.

E claro que o erro provem da distincao artificial entre o primeiroe o segundo par, que faz com que um par de 2 e um par de 5 sejaconsiderado diferente de um par de 5 e um par de 2.

(c) O numero de casos possıveis e 65, pois ha 6 resultados para cadaum dos 5 dados. O numero de casos favoraveis e 6·C3

5 ·5·4 = 1200,pois ha 6 modos de escolher o tipo de trinca (pode ser de 1, de2, . . . , de 6) e ha C3

5 modos de escolher os tres dados que teraoresultados iguais; quatro aos outros dados, o resultado do primeirodeles pode ser escolhido de 5 modos distintos (deve ser diferentedo resultado dos tres primeiros dados) e, o do segundo, de 4 modosdistintos (deve ser diferente dos anteriores).

Logo, P (A4) =1200

65=

25

162≡ 0, 154.

(d) O numero de casos possıveis e 65, pois ha 6 resultados para cadaum dos 5 dados. O numero de casos favoraveis e 6 · C4

5 · 5 = 150,pois ha 6 modos de escolher o tipo de quadra (pode ser de 1, de2, . . . , de 6) e ha C4

5 modos de escolher os quatro dados que teraoresultados iguais; quatro ao dado restante, seu resultado pode serde 5 modos distintos (deve ser diferente do resultado dos quatroprimeiros dados).

Logo, P (A6) =150

65=

25

1296≡ 0, 019.

(e) O numero de casos possıveis e 65 e o numero de casos favoraveise 6.

Logo, P (A7) =6

65=

1

1296≡ 0, 0008.

(f) O numero de casos possıveis e 65, pois ha 6 resultados para cadaum dos 5 dados. Ha dois tipos possıveis de sequencias: a mınima

3

(12345) e a maxima (23456). A mınima pode ser formada de5! = 120 modos distintos, pois ha 5 modos de escolher o dado cujoresultado e 1, 4 modos de escolher o dado cujo resultado e 2, etc.Ha analogamente, 120 modos de formar a sequencia maxima.

Portanto, P (A8) =240

65=

5

162≡ 0, 031.

(g) O numero de casos possıveis e 65, pois ha 6 resultados para cadaum dos 5 dados. O numero de casos favoraveis e 6 · C3

5 · 5 = 300,pois ha 6 modos de escolher o tipo de trinca (pode ser de 1, de 2,. . . , de 6) e ha C3

5 modos de escolher os tres dados que formarao atrinca; quatro aos outros dados, ha 5 modos distintos de escolhero resultado comum deles.

Logo, P (A5) =300

65=

25

684≡ 0, 039.

6. Numeramos os vertices do polıgono de 0 a 2n. Imagine 0 como o verticemais alto, os vertices de 1 a n do lado direito e os vertices de n + 1 a2n do lado esquerdo. Podemos pensar que todos os triangulos tem 0

como um dos vertices. Ha C22n =

2n(2n− 1)

2= n(2n − 1) modos de

selecionar os outros dois vertices do triangulo.

Para construir o numero de triangulos que contem o centro da circun-ferencia em seu interior, observe inicialmente que a reta que contem overtice i(1 ≤ i ≤ n) e o centro da circunferencia corta novamente opolıgono no ponto medio do segmento determinado pelos vertices i+ne i + n + 1 (vertice 2n + 1= vertice 0).

Um triangulo que contenha em seu interior o centro da circunferenciasera necessariamente formado por um vertice do lado direito e umvertice do lado esquerdo. Se o vertice do lado direito for o vertice1, o do lado esquerdo so podera ser o vertice n + 1 (1 possibilidade);se for o vertice 2, podera ser qualquer dos vertices de n + 1 a n + 2 (2possibilidades); . . . ; se for o vertice n, podera ser qualquer dos verticesde n + 1 a 2n (n possibilidades). O numero de casos favoraveis e

1 + 2 + · · ·+ n =n(n + 1)

2.

A resposta en + 1

2(2n− 1).

4

7. Imagina o resultado do sorteio como uma fila de 12 lugares: o primeirolugar corresponde a primeira pessoa sorteada area o primeiro grupo;o segundo a segunda pessoa sorteada para o segundo grupo; . . . ; oultimo, a quarta pessoa sorteada para o terceiro grupo. Colocada aprimeira pessoa, ha 11 posicoes para a segunda, em 3 das quais ela ficano mesmo grupo da primeira.

A resposta e3

11.

8. Ha 12 possibilidades para o signo de cada pessoa, para um total de 124

possibilidades. Para que nao haja coincidencia de signos, o signo daprimeira pessoa pode ser escolhido de 12 modos, o da segunda de 11,o da terceira de 10 e o da quarta de 9, para um total de 12 · 11 · 10 · 9modos. Assim, a probabilidade de que nao haja coincidencia de signos

e12 · 11 · 10 · 9

124=

55

96e a probabilidade de que nao haja coincidencias

e 1−55

96=

41

96.

9. Ha C410 modos de retirar 4 pes de sapatos. Para retirar 4 pes, havendo

nesses 4 pes exatamente 1 par de sapatos, devemos inicialmente seleci-onar 1 par (o que pode ser feito de 5 modos) e depois selecionar 2 pesvindo de pares diferentes dentre os 4 pares que ainda estao no armario.Para isso devemos escolher os pares de onde virao esses sapatos (C4

2

modos) e, em cada par escolhido, decidir se retiramos o pe direito ou ope esquerdo (22 = 4 modos).

A resposta e5 · C2

4 · 4C104

=4

7.

Unidade 18

Exercıcios Recomendados

1. Sejam X e Y os resultados do primeiro e segundo lancamentos, respec-tivamente.

P (X = 3|X + Y = 7) =P (X = 3, X + Y = 7)

P (X + Y = 7)=

1/6 · 1/6

6/36=

1

6.

5

Outra solucao:

Se a soma e 2, ha 6 casos possıveis igualmente provaveis: X = 1, Y = 6;X = 2, Y = 5; X = 3, Y = 4; X = 4, Y = 3; X = 5, Y = 2;X = 6,Y = 1. Dos seis casos, somente X = 3, Y = 3 e favoravel. A resposta

e1

6.

2. P (nao sabe | acerta) =P (nao sabe e acerta)

P ( acerta)

=P (nao sabe) · P (acerta | nao sabe)

P (sabe) · P (acerta | sabe) + P (nao sabe) · P (acerta | nao sabe)

=0, 4 · 0, 2

0, 6 · 1 + 0, 4 · 0, 2=

2

17.

3. P (A∩B) = P (A) ·P (B|A) = 1/2 · 1/2 = P (A) ·P (B); logo, A e B saoindependentes.

Observe que A∩C = A∩B e que P (C) = 12·P (A∩C) = P (A∩B) =

1/2 · 1/2 = P (A) · P (C); logo, A e C sao independentes.

Observe que B ∩C = A∩B e que P (C) = 12·P (B ∩C) = P (A∩B) =

1/2 · 1/2 = P (B) · P (C); logo, B e C sao independentes.

Como A∩B ∩C = A∩B, P (A∩B ∩C) = P (A∩B) = 1/2 · 1/2, quee diferente de P (A) · P (B) · P (C) = 1/2 · 1/2 · 1/2; logo, A, B e C naosao independentes.

4. (a) A probabilidade de nenhum seis em quatro lancamentos e(56

)4 ≡0, 4823. A probabilidade de pelo menos um seis e 1 −

(56

)4 ≡1− 0, 4823 = 0, 5177.

(b) A probabilidade de nenhum duplo seis em 24 lancamentos de um

par de dados e(3536

)24 ≡ 0, 5086. A probabilidade de pelo menos

um duplo seis e 1−(3536

)24 ≡ 1− 0, 5086 = 0, 4914.

6

5.

P (doente | positivo) =P (doente e positivo)

P (positivo)

=P (doente · positivo | doente)

P (doente) · P (positivo | doente) + P (sadio) · P (positivo | sadio)

=0, 005 · 0, 95

0, 005 · 0, 95 + 0, 995 · 0, 01=

95

294∼= 0, 3231

6. A probabilidade de nao obter nenhum seis em n lancamentos e(56

)ne

de obter pelo menos um de seis e 1−(56

)n. Devemos ter 1−

(56

)n> 0, 9,

ou seja,(56

)n< 0, 1.

Daı,

ln

(5

6

)n

< ln 0, 1,

n · ln 5

6< ln 0, 1,

n >ln 0, 1

ln 56

∼= 12, 6.

A resposta e 13.

7. (a) Cada pessoa tem n modos de escolher a quem conta o boato. Logo,o numero de modos de o boato ser contado m vezes e nm.

O numero de modos de o boato ser contado m vezes, sem retornara primeira pessoa e n(n− 1)m−1, pois o primeiro ouvinte pode serselecionado de n modos e os demais, de n− 1 modos. A resposta

e n(n−1)m−1

nm =(n−1n

)m−1.

(b) Cada pessoa tem n modos de escolher a quem conta o boato. Logo,o numero de modos de boato ser contado m vezes e nm.

Para o boato ser contado m vezes, sem repetir nenhuma pessoa,o primeiro ouvinte pode ser selecionado de n modos; o segundo,de n − 1 modos; o terceiro, de n − 2 modos; . . . ; o m-esimo,de n − (m − 1) = n − m + 1 modos. O numero de modos deo boato ser contado m vezes, sem retornar a primeira pessoa, e

n · (n− 1) · · · (n−m + 1) =n!

(n−m)!.

7

A resposta en!

(n−m)!nm.

8. Considere os eventos:

A =A falou a verdade;B =B disse que A falou a verdade;C =C disse que B disse que A falou a verdade;D =D disse que C disse que B disse que A falou a verdade.Vamos aliviar a notacao escrevendo XY para representar X ∩ Y .

Queremos calcular P (A|D) =P (AD)

P (D).

P (AD) = P (ABCD) + P (ABCD) + P (ABCD) + P (ABCD)

=1

3· 1

3· 1

3· 1

3+

1

3· 2

3· 2

3· 1

3+

1

3· 1

3· 2

3· 2

3+

1

3· 2

3· 1

3· 2

3=

13

81

P (AD) = P (ABCD) + P (ABCD) + P (ABCD) + P (ABCD)

=2

3· 2

3· 1

3· 1

3+

2

3· 1

3· 2

3· 1

3+

2

3· 2

3· 2

3· 2

3+

2

3· 1

3· 1

3· 2

3=

28

81.

P (D) = P (AD) + P (AD) =13

81+

28

81=

41

81.

A resposta e P (A|D) =P (AD)

P (D)=

13/81

41/81=

13

41.

9. Uma urna recebe uma bola branca e a outra urna recebe as demais 99bolas. Com efeito, se a 1a

¯ urna recebe k bolas das quais a sao brancas,a probabilidade de libertacao e

f(a, k) =1

2

[a

k+

50− a

100− k

]=

1

2· 50k + a(100− 2k)

k(100− k).

Observe que para k = 50 a expressao vale 12, independentemente do

valor de a.

8

Observe tambem que basta estudar agora o caso k < 50 (isto e, po-demos considerar a primeira urna como sendo a que recebeu menosbolas). Nesse caso, e claro que, fixando o valor de k, quando maior fora, maior sera f(a, k). Logo, para f(a, k) ser maximo, devemos ter a = k

e a probabilidade sera g(k) =1

2

150− 2k

100− k=

75− k

100− k= 1−

25

100− k, que

e maxima para k mınimo.

Devemos, pois, ter k = 1, o que da uma probabilidade de libertacao de7499∼= 0, 75.

9

Exercıcios Resolvidos de MA 12

Unidades 19 e 20

Unidade 19

Exercıcios Recomendados

1. Para distribuir os sorvetes, devemos escolher as pessoas que receberaosorvetes de creme (C5

10 modos) e dar sorvetes de chocolate as demais(1 modo). Para distinguir os sorvetes, respeitando as preferencias,comecamos dando sorvete de creme aos que gostam de creme e dechocolate aos que gostam de chocolate (1 modo). Em seguida, deve-mos distribuir 2 sorvetes de creme e 3 sorvetes de chocolate a 5 pessoasque nao tem preferencias; para isso, devemos escolher as 2 pessoas quereceberao sorvetes de creme (C2

5 modos) e dar sorvetes de chocolate asrestantes (1 modo).

A resposta eC2

5

C510

=5

126.

2. As pecas do domino sao formadas por dois, nao necessariamente dis-tintos, dos numeros 0, 1, 2, 3, 4, 5 e 6. Ha CR2

7 = C28 = 28 pecas e

ha C228 modos de selecionar duas pecas de um domino. Para selecionar

duas pecas com um numero comum , deve-se primeiramente selecionaro numero comum (7 possibilidades) e, depois, selecionar 2 das 7 pecasque contem esse numero comum (C2

7 possibilidades).

A resposta e7C2

7

C228

=7

18.

3. O numero de sorteios possıveis e C580.

(a) O apostador acerta 3 dezenas quando sao sorteadas das 8 dezenasem que apostou e 2 das 72 em que nao apostou. Tais sorteiospodem ser efetuados de C3

8 · C272 modos.

A resposta eC3

8 · C272

C580

(que e aproximadamente igual a1

168).

1

(b) O apostados acerta 4 dezenas quando sao sorteadas 4 das 8 dezenasem que apostou e 1 das 72 em que nao apostou. Tais sorteiospodem ser efetuados de C4

8 · C172 modos.

A resposta eC4

8 · C172

C580

(que e aproximadamente igual a1

4770).

(c) O apostados acerta 5 dezenas quando sao sorteadas 5 das 8 dezenasem que apostou. Tais sorteios podem ser efetuados de C5

8 modos.

A resposta eC5

8

C580

=1

429.286.

4. Colocada a primeira pessoa na roda, ha n− 1 posicoes para a segundapessoa, das quais 2 sao favoraveis a que ela fique junto da primeirapessoa.

A resposta e2

n− 1.

5. (a) Ha n posicoes igualmente provaveis que a chave “certa” poderiaocupar: ser a primeira a ser testada, a segunda, . . . , a ultima. Aprobabilidade de ela ocupar a k-esima posicao e 1/n.

Outra solucao:

Ha n! maneiras de ordenar as chaves a serem tentadas. Paraformar as ordenacoes que tem a chave na k-esima posicao, devemoscolocar as n − 1 chaves restantes nas n − 1 posicoes restantes, oque pode ser feito de (n − 1)! modos. Logo, a probabilidade de

que a chave certa esteja na posicao k e(n− 1)!

n!=

1

n.

(b) As primeiras k tentativas podem ser feitas de nk modos (cadachave pode ser escolhida de n modos, ja que chaves correspon-dentes a tentativas frustadas nao sao descartadas). Para que seacerte na k-esima tentativa, as primeiras k − 1 chaves devem serincorretas (portanto, podem ser escolhidas de (n−1)k−1 modos) ea de ordem k deve ser a correta (1 modo). Logo, a probabilidade

de se acertar na k-esima tentativa e(n− 1)k−1

nk.

6. (a) Ha C412 = 495 modos de selecionar as 4 vagas que nao serao ocu-

padas e 9 modos de escolher 4 vagas consecutivas (1 2 3 4, 2 3 45, . . . , 9 10 11 12).

2

A resposta e9

495=

1

55.

(b) Ha C412 = 495 modos de selecionar as 4 vagas que nao serao ocupa-

das. Para contar o numero de possibilidades em que nao ha vagasvazias adjacentes, devemos escolher 4 dos 9 espacos existentes an-tes, entre e depois dos carros para ficarem vazios, Isto pode serfeito de C4

9 modos. Logo a probabilidade de que nao haja vagas

consecutivas e126

495=

14

55.

7. P (A ∩B ∩ C) = 0, pois A ∩B ∩ C ⊂ A ∩ C e P (A ∩ C) = 0.

(a)

P (A ∪B ∪ C) =P (A) + P (B) + P (C)− P (A ∩B)

− P (A ∩ C)− P (B ∩ C) + P (A ∩B ∩ C)

=0, 4 + 0, 5 + 0, 3− 0, 3− 0− 0, 1 + 0 = 0, 8

(b)

P [A− (B ∪ C)] =P (A)− P [A ∩ (B ∪ C)]

=P (A)− P [(A ∩B) ∪ (A ∩ C)]

=P (A)− P (A ∩B)− P (A ∩ C)

+ P [(A ∩B) ∩ (A ∩ C)]

=P (A)− P (A ∩B)− P (A ∩ C)

+ P [(A ∩B ∩ C)]

=0, 4− 0, 3− 0 + 0 = 0, 1

(c)

P [(A ∩B) ∪ C] =P (A ∩B) + P (C)− P (A ∩B ∩ C)

=0, 3 + 0, 3− 0 = 0, 6

8. (a) A resposta, naturalmente, e 1/2, ja que, de todos os pares denumeros distintos de 1 a 100, em exatamente a metade o primeironumero e maior do que o segundo.

3

(b) O numero total de possıveis extracoes e 100×100 = 10.000, ja queo bilhete de cada uma das mocas pode ser escolhido de 100 modos.Em 100 destas possıveis extracoes os dois numeros sao iguais e emmetade das restantes, ou seja, em 9900/2 = 4950 delas, o primeironumero e maior do que o segundo. Logo, a probabilidade de o

numero de Laura ser maior do que o de Telma e4950

10000= 0, 495.

9. (a) Sao feitos 5 testes quando uma das quatro primeiras pilhas tes-tadas esta descarregada, o mesmo ocorrendo com a quinta a sertestada. A primeira pilha a ser testada pode ser escolhida de 10modos, a segunda de 9, e assim por diante, para um total de10 · 9 · 8 · 7 · 6 modos possıveis para escolher as 5 primeiras pilhas aserem testadas. Para formar uma sequencia de teste em que a se-gunda defeituosa e detectada na 5a tentativa, devemos escolher apilha defeituosa que aparece na 5a posicao (2 modos), a posicao daoutra defeituosa (4 modos) e, finalmente, as pilhas nao defeituosaspara as demais posicoes (8 · 7 · 6 modos). Logo, a probabilidade

pedida e2 · 4 · 8 · 7 · 610 · 9 · 8 · 7 · 6

=4

45.

(b) Sao efetuados ate 5 testes quando as pilhas defeituosas aparecemnas 5 primeiras tentativas. Como visto no ıtem anterior, ha 10 ·9 ·8 · 7 · 6 modos de se fazer esta tentativa. Para formar aquelas emque as duas defeituosas estao entre as testadas devemos escolhera posicao da primeira pilha defeituosa (5 modos), a da segunda(4 modos) e, finalmente, as pilhas nao defeituosa para as outrastentativas (8 ·7 ·6 modos). A probabilidade de que sejam feitos ate

5 testes e5 · 4 · 8 · 7 · 610 · 9 · 8 · 7 · 6

=2

9e, portanto, a probabilidade pedida

e igual a 1−2

9=

7

9.

(c) Para que sejam feitos menos de 4 testes, as duas pilhas defeituosasdevem aparecer nos primeiros 4 testes. O numero total de escolhaspara os 4 primeiros testes e 10 ·9 ·8 ·7. Para formar uma sequenciade teste em que as duas defeituosas aparecem nestas 4 tentativas,devemos escolher a posicao da primeira pilha defeituosa (4 modos)a da segunda (3 modos) e, finalmente, as pilhas nao defeituosaspara as duas outras posicoes (8·7 modos). A probabilidade pedida

4

e4 · 3 · 8 · 710 · 9 · 8 · 7

=2

15

Unidade 20

Exercıcios Recomendados

1. (a) A probabilidade de eles se enfrentarem na primeira ronda e1

2n − 1porque, posto A na tabela, ha 2n − 1 posicoes possıveis para B eem 1 delas ele enfrenta B · A probabilidade deles se enfrentarem

na segunda rodada e2

2n − 1·

(1

2

)2

=1

2n − 1·

1

2, porque posto

A na tabela, ha 2n − 1 posicoes possıveis para B e em 2 delasele pode vir a enfrentar B na segunda rodada, o que ocorre comprobabilidade 1

2· 1

2. A probabilidade de eles se enfrentarem na

terceira rodada e22

2n − 1·

(1

4

)4

=1

2n − 1·

1

22, etc.

A resposta e

1

2n − 1+

1

2n − 1· 1

2+

1

2n − 1· 1

22+ · · ·+ 1

2n − 1· 1

2n−1

=1

2n − 1·

1−(12

)n1− 1

2

=1

2n−1.

(b) Se k < n, o jogador disputa exatamente k partidas se e somentese perde a k-esima partida e ganha as k − 1 partidas anteriores.

A probabilidade de isso acontecer e(12

)k−1 · 12

= 12k

. O jogadordisputa n partidas - ou seja, chega a final - se e somente se ganhaas n − 1 partidas anteriores. A probabilidade de isso acontecer e(12

)n−1= 1

2n−1 .

A resposta e 12k

, se k < n; 12n−1 , se k = n.

2. (a) O segundo jogador de melhor resultado sera vice-campeao se esomente se nao enfrentar o melhor jogador antes da final. Posto omelhor jogador na tabela, ha 15 posicoes possıveis para o segundo

5

melhor e em 8 delas ele enfrenta o melhor jogador apenas na final.

A resposta e8

15.

(b) Posto o 4o colocado na tabela, os demais times podem ser coloca-dos de 15! modos. Para que o 4o melhor time seja vice-campeao,os 3 melhores times nao podem entrar em sua chave. As posicoesdestes times podem portanto, ser escolhidas de 8, 7 e 6 modos,respectivamente. Para distribuir os 12 times restantes, ha 12! pos-

sibilidades. Logo, a probabilidade desejada e8 · 7 · 6 · 12!

15!=

8

65.

(c) Na primeira rodada, ha 6 adversarios que o 10o time conseguederrotar. Na melhor das hipoteses, ele derrota um destes e, dosoutros 5, dois conseguem sobreviver para a proxima fase. De novo,na melhor das hipoteses o 10o enfrenta (e vence) um deles, mas ooutro sera fatalmente eliminado. Assim, na 3a rodada, o 10o timejoga e perde. Logo, ele disputa no maximo tres partidas. Istoocorre quando os tres times da sua chave para os dois primeirosjogos sao todos de habilidade inferior. O numero de modos deescolher 3 adversarios e 15·14·13. O numero de modos de escolhertres adversarios entre os 6 de nıvel inferior e 6·5·4. Logo, a resposta

e6 · 5 · 4

15 · 14 · 13=

4

91.

3. Se o candidato nao troca de porta, ele ganha o premio se e so se escolhe,originalmente, a porta certa. Logo, se ele nao troca, sua probabilidadede ganhar o primeiro premio e igual a 1

3. Em contraste, ao trocar de

porta ele ganha o premio sempre que escolheu originalmente a portaerrada, o que ocorre com probabilidade 2

3. Portanto, ele deve trocar de

porta.

4. Cada um dos 10 resultado pode ser escolhido de 2 modos. Portanto,ha 210 resultados possıveis. Para formar um resultado com 5 caras, enecessario escolher 5 dos 10 lancamentos para estas caras ocorrerem, o

que pode ser feito de C510 modos. A probabilidade pedida e

C510

210=

63

256.

6

5. (a)

P (2aB) = P (1aB, 2aB) + P (1aP, 2aB)

= P (1aB) · P (2aB|1aB) + P (1aP ) · P (2aB|1aP )

=4

10· 6

12+

6

10· 4

12=

2

5

(b)

P (1aB|2aP ) =P (1aB, 2aP )

P (2aP )

=P (1aB) · P (2aP |1aB)

P (1aB) · P (2aP |1aB) + P (1aP ) · P (2aP |1aP )

=410· 612

410· 612

+ 610· 812

=1

3.

6. P (ve vermelha | mostra amarela) = P (ve vermelha e mostra amarela)P (mostra amarela)

=1/6

1/2=

1

3.

7. O jogador A ganha o primeiro se e somente se B ganhar no maximo 2das proximos 8 partidas (caso contrario, B tera sua 10a vitoria antesde A completar sua serie de 10 vitorias).

P (B ganhar 0 partidas) = 0, 68 ∼= 0, 0168

P (B ganhar 1 partidas) = 8 · 0, 67 · 0, 4 ∼= 0, 0896

P (B ganhar 2 partidas) = C28 · 0, 66 · 0, 42 ∼= 0, 2090

A probabilidade de que A ganhe o premio e aproximadamente igual a0, 0168 + 0, 0896 + 0, 2090 = 0, 3154.

8. Sejam p1, p2, p3, p4, p5, p6 as probabilidades das possıveis ordenacoesABC, BCA, CAB, ACB, CBA, BAC. As condicoes dadas no pro-blema permitem escrever um sistema de equacoes lineares envolvendoaquelas probabilidade:

7

P1 + p4 = P2 + p6 = P3 + p5 =1

3

P3 + p6 = P1 + p5 = P2 + p4 =1

3

P2 + p5 = P3 + p4 = P1 + p6 =1

3

Resolvendo o sistema, verifica-se que ele tem uma infinidade de solucoesda forma p1 = p2 = p3 = 1

6− p4 = 1

6− p5 = 1

5− p6.

Em termos mais intuitivos, basta que as ordenacoes correspondente amesma ordem circular tenham probabilidades iguais, ou seja, devemoster:

P (ABC) = P (BCA) = P (CAB)

eP (ACB) = P (CBA) = P (BAC).

Por exemplo, se P (ABC) = P (BCA) = P (CAB) = 14

e P (ACB) =P (CBA) = P (BAC) = 1

12, os tres jogadores tem a mesma chance

de ficar em primeiro, segundo ou terceiro lugar, embora as diferen-cias ordenacoes possıveis nao tenham todas a mesma probabilidade deocorrer.

9. Dois pontos A e B de um circulo determinam uma corda maior que olado de um triangulo equilatero inscrito se e somente se o menor arcoAB e maior do que 120. Isto significa, que uma vez escolhido o pontoA, o ponto B nao deve estar no arco de 240 com ponto medio emA, como mostra a figura. Como B e escolhido ao acaso, admitimosque a probabilidade de que ele esteja em um arco seja proporcional ao

comprimento do arco. Assim, a probabilidade pedida e120

360=

1

3.

8

A

120120

120B

1

10. Uma corda perpendicular a AB tem comprimento maior do que o ladodo triangulo equilatero inscrito se e somente se corta AB entre os pontosM e N , medios de OA e OB e centro do cırculo, como mostra a figura.

Logo, a probabilidade pedida eMN

AB=

1

2.

A BM O N

1

11. Contando o tempo em minutos, a partir das 16 horas, e designando porx e y os instantes de chagada de Cristina e Maria, a regiao “possıvel”e Ω = (x, y) : 0 ≤ x ≤ 60, 0 ≤ y ≤ 60 (um quadrado de lado 60) ea regiao “favoravel” e A = (x, y) : 0 ≤ x ≤ 60, 0 ≤ y ≤ 60, |x− y| ≤10. A desigualdade |x−y| ≤ 10 e equivalente a x−10 ≤ y ≤ x+10, oque mostra que a regiao favoravel e uma “faixa” em torno da diagonaldo quadrado, como mostra a figura.

9

0 10 60

10

60

1

A probabilidade desejada e:area de A

area de Ω=

602 − 2 · 50 · 50/2

602=

11

36.

10

Exercıcios Resolvidos de MA 12

Unidades 21 e 22

Unidade 21

Exercıcios Recomendados

1. Os tempos de percurso sao t1 =d

2v1e t2 =

d

2v2. Logo, a velocidade

media e vm =d

t1 + t2=

dd

2v1+ d

2v2

=2

1v1

+ 1v2

, que e a media harmonica

das velocidades v1 e v2 na ida e na volta.

2. A distancia total percorrida e d = tv1 + tv2. Logo, a velocidade media

e vm =tv1 + tv2

2t=v1 + v2

2, que e a media aritmetica das velocidades

v1 e v2 nas duas partes do percurso.

3. Apos as duas decadas, a populacao e multiplicada por 1, 44× 1, 21. Ataxa media i de crescimento decenal e a taxa que, mantendo-se cons-tante para as duas decadas, produza o mesmo crescimento total. Logo,devemos ter (1 + i)2 = 1, 44 × 1, 21, Logo, 1 + i =

√1, 44√

1, 21 =1, 2 × 1, 1 = 1, 31. Portanto, a taxa media de crescimento decenal e0, 32 = 32%.

4. Do mesmo modo, a taxa media anual de crescimento i e tal que, apli-cada a cada ano, produza o mesmo crescimento total. Logo, devemoster (1 + i)20 = 1, 44× 1, 21 = 1, 7424. Daı, 1 + i = 20

√1, 7424 = 1, 0282

e a taxa media anual de crescimento e 1, 0282 = 2, 82%.

6. Em 1/3 dos casos, os prisioneiros levem 3 horas para escapar; em outro1/3, levam 5 horas; no 1/3 restante gastam 9 horas para retornar aoponto de partida. Esses ultimos, agora, escolhem um dos dois outrostuneis. Assim, 1/6 dos prisioneiros levam 9+3 = 12 horas para escapare o restante 1/6 levam 9 + 5 = 14 horas. O tempo medio para escapare, portanto, 1

3· 3 + 1

3· 5 + 1

6· 12 + 1

6· 14 = 7 horas.

1

7. Seja M o tempo medio para escapar. Em 1/3 dos casos, os prisionei-ros levam 3 horas para escapar; em outro 1/3, levem 5 horas; no 1/3restante gastam 9 horas para retornar ao ponto de partida, do qualprecisam, em media, de um tempo adicional M para escapar. Logo,temos: M = 1

3·3+ 1

3·5+ 1

3·(9+M). Resolvendo a equacao, encontramos

M = 8, 5 horas.

8. Cada um dos n numeros ai satisfaz m ≤ ai ≤ M e, portanto, nm ≤∑ni=1 ai ≤ nM . Dividindo por n, vem m ≤ x ≤M .

9. Cada um dos n numeros ai satisfaz m ≤ a1 ≤ M e, portanto, mn ≤∏ni=1 ai ≤Mn (ja que todos os numeros envolvidos sao positivos). Ex-

traindo a raiz n-esima, vem m ≤ g ≤M .

10. Cada um dos n numeros ai satisfaz m ≤ ai ≤ M . Como cada umdeles e positivo, tem-se 1

m≥ 1

ai≥ 1

Me, daı, n

m≥∑n

i=11ai≥ nM .

Portanto, mn≥ 1∑n

i=11ai

≥ nM . Finalmente, multiplicando por n, vem

m ≤ h ≤M .

11. Nao. Depende dos pesos atribuıdos as duas provas. Se, por exemplo,Portugues tiver peso 4 e Matematica peso 1, Joao tera 27 pontos ePedro 28.

12. (a) Se um pneu roda x mil quilometros em uma roda dianteira e ymil quilometros em uma roda traseira, a fracao do pneu que gastae x

40+ y

60. Para conseguir a rodagem maxima sem trocar pneus,

todos os pneus devem gastar totalmente ao mesmo tempo (ouseja, esta fracao deve ser igual a 1 ao final do processo para cadapneu). Portanto, cada um deles devera rodar o mesmo numerode quilometros em uma roda dianteira e uma roda traseira, ouseja x = y. Logo, deve-se ter x

40+ y

60= 1, de onde resulta x =

24. Logo, cada pneu deve rodar 24 mil quilometros em uma rodadianteira e 24 mil quilometros em uma roda traseira. Portanto,o carro pode rodar 48 mil quilometros com um jogo de 4 pneus,bastando para isto trocar os pneus traseiros pelos dianteiros aos24 mil quilometros.

(b) Do mesmo modo, cada pneu deve rodar 24 mil quilometros emuma roda traseira e 24 mil quilometros em uma roda dianteira.Mas como agora temos 5 pneus, o carro pode rodar por 5/4 de

2

48 mil quilometros, ou seja, por 60 mil km. Para tal, basta fazerum rodızio dos pneus a cada 12 mil km. Por exemplo, comecandocom ABCDE (com A e B na dianteira, C e D na traseira e E noestepe), passar para BCDEA, CDEAB, DEABC e EABCD).

(c) O resultado em a) e igual a 21

40.000+ 1

60.000

. Logo, e a media harmonica

de 40.000 e 60.000.

13. A nova media e 50·40−125−7548

= 37, 5.

14. A media quadratica conserva a soma dos quadrados dos numeros.

15. Cada um dos n numeros ai satisfaz m ≤ ai ≤ M , ou seja, m2 ≤ a2i ≤M2. Logo, nm2 ≤

∑ni=1 a

2i ≤ nM2, ou seja, m2 ≤

∑ni=1 a

2i

n≤ M2. Daı,

finalmente, m ≤ q ≤M .

Exercıcios Suplementares

1. A − G = x1+x2

2− √x1x2 =

(√x1−√x2)2

2. Portanto, A − G ≥ 0, com

igualdade se e somente se x1 = x2. Aplicando este resultado aos inver-

sos de x1 e x2, temos1x1

+ 1x2

2≥√

1x1

1x2

. Ou seja, temos 1H≥ 1

G, que e

equivalente a H ≤ G; a igualdade ocorre se e somente se 1x1

= 1x2

, ouseja, quando x1 = x2.

2. A media aritmetica dos erros e necessariamente igual a 0, ja que

n∑i=1

(ai −m) = (n∑

i=1

ai)− nm = nm− nm = 0.

3. Deseja-se encontrar x de modo que seja mınimo

f(x) =n∑

i=1

(x− xi)2 = nx2 − 2(n∑

i=1

xi)x+n∑

i=1

x2i ,

que e uma funcao quadratica de x. Esta funcao atinge seu valor mınimo

para x = − b2a

=2∑n

i=1 xi

2n= x

3

4. Deseja-se encontrar x de modo que seja mınimo

f(x) =n∑

i=1

|x− xi| = |x− x1|+ |x− x2|+ · · ·+ |x− xn|.

Ha dois casos a considerar. Quando n e ımpar, f(x) e decrescentepara n menor que o termo central x(n+1)/2 e decrescente para x maiorque este termo. Logo, atinge o seu valor mınimo neste ponto, que e amediana dos numeros x1, x2, . . . , xn. Quando n e par, f(x) e crescentepara x < xn/2, constante para xn/2 < x < xn/2+1 e crescente parax > xn/2+1. Logo, o valor mınimo de f ocorre para todo valor de xentre as observacoes centrais.

5. (a) Os pontos seriam colineares se e somente se os incrementos em yfossem os mesmos para cada incremento de 1 unidade em x, o quenao ocorre.

(b) Para a reta 1, o erro medio quadratico e:

e1 = (580−820)2+(980−1000)2+(1190−1200)2+(1360−1350)2+(1350−1550)25

= 380

Para a reta 2, o erro medio quadratico e:

e1 = (800−820)2+(980−1000)2+(1160−1200)2+(1340−1350)2+(1520−1550)25

= 680

Logo, a primeira reta produz o menor erro quadratico.

(c) Consideremos uma reta de equacao y = ax+ b. A soma dos errosquadraticos e

S =(b− 820)2 + (a+ b− 1000)2 + (2a+ b− 1200)2

+ (3a+ b− 1350)2 + (4a+ b− 1550)2

=30a2 + 20ab+ 5b2 − 27300a− 11840b+ 7337400.

Podemos escrever esta soma como:

S = 30

(a+

b

3− 455

)2

+5

3(b− 822)2 + 510

Logo, S e mınimo quando a+ b3− 455 = 0 e b− 822 = 0, ou seja,

quando b = 822 e a = 181. Portanto, a reta pedida tem equacaoy = 181x+ 822.

4

6. Existem 7 restos possıveis quando se divide um numero por 7. Assim,tomando os restos como gavetas e os numeros como objetos, pode-segarantir que alguma gavetas contera dois (ou mais) objetos. Isto e, hapelo menos dois numeros que deixam o mesmo resto quando dividimospor 7, o que e equivalente a dizer que a diferencia entre eles e ummultiplo de 7.

7. A mesma pode ser colocada em 20 posicoes diferentes. Seja ai (i =1, . . . , 20) o numero de criancas cuja preferencia e atendida com a mesana posicao i. Entao a1 + a2 + · · ·+ a20 e o numero total de preferenciasatendidas. Mas cada sorvete e colocado, sucessivamente, em frentea cada crianca. Como ha exatamente 10 criancas que preferem cadasabor, o numero total de preferencias atendidas por cada sorvete e 20,para um total de 200 preferencias atendidas. Assim, temos a1 + a2 +· · ·+ an. Como a media de a1, . . . , a20 e 10, conclui-se que pelo menosum dos numeros e maior ou igual a 10 (ou seja, ha alguma posicao emque pelo menos 10 criancas sao atendidas).

8. Em uma reuniao com n pessoas, cada pessoa pode ter de 0 a n −1 conhecidos. No entanto, e impossıvel que, ao mesmo tempo, hajauma pessoa que nao tenha conhecidos e outra que conheca todos (afinal, estas duas pessoas se conhecem ou nao?). Portanto, em qualquersituacao ha apenas n−1 valores possıveis para o numero de conhecidos,o que implica que pelo menos duas das n pessoas tem o mesmo numerode conhecidos.

9. Como no exemplo 7, considere os restos da divisao por 1997, dosnumeros 1, 11, 111, . . . . Como ha apenas 1997 restos possıveis, ne-cessariamente ha dois restos coincidentes. Tomando a diferenca dosdois numeros da forma 11. . . 1 resulta a existencia de um numero daforma 11 . . . 10 . . . 0 = 11 . . . 1×10k que e multiplo de 1997. Como 1997e 10 sao primos entre si, o numero da forma 11 . . . 1 acima e necessari-amente multiplo de 1997.

10. Deve haver 73 pessoas. Podemos distribuir ate 72 pessoas de modo quehaja exatamente 6 nascidas em cada mes. Com 73 pessoas, necessari-amente um dos meses (gavetas) contera 7 ou mais pessoas (objetos).

11. As paridades das coordenadas dos pontos de coordenadas inteiras doplano determinam 4 gavetas: par-par, par-ımpar, ımpar-par e ımpar-

5

ımpar. Dados 5 pontos, pelo menos uma das gavetas contem dois pon-tos. Ou seja, ha um par de pontos em que ambas as coordenadas tem amesma paridade, o que faz com que o ponto medio tenha coordenadasinteiras.

12. Seja ai, i = 1, . . . , N , o numero de objetos em cada gaveta, Entaoa1 + · · · + aN = Nk + 1, ou seja, a media aritmetica de a1, . . . , aN ek + 1

N. Como a media dos numeros inteiros a1, . . . , aN e maior do que

k, resulta que pelo menos um deles e maior do que k.

13. As 4 primeiras questoes da prova podem ser respondidas de 5 × 5 ×5× 5 = 625 modos (gavetas) diferentes. Como sao 40100 candidatos, onumero medio de candidatos para cada possıvel padrao de resposta e40100625

= 64, 16. Pode-se garantir, em consequencia, que uma das gavetascontem 65 objetos; ou seja, pelo menos 65 candidatos respondem demodo identico as primeiras 4 questoes. Este e o maior valor possıvelpara k, ja que e possıvel distribuir os candidatos de modo que haja nomaximo 65 por cada padrao de resposta.

14. As primeiras k questoes podem ser respondidas de 5k modos (gavetas).Para garantir que pelo menos 4 candidatos respondam a estas questoesdo mesmo modo, deve-se ter pelo menos 3×5k+1 candidatos (objetos).Portanto, deve-se ter 3×5k < 40100, o que ocorre para k ≤ 5. Portanto,o valor maximo possıvel para k e 5.

15. Considere os pontos da reta com coordenadas inteiras. Como ha so-mente 11 cores disponıveis, dois deles tem a mesma cor.

16. O numero de jogos de cada um dos n times e um numero inteiro de 0 an − 1. Mas, como no exercıcio 23, nao pode existir, simultaneamente,um time com 0 jogos e outro com n − 1 (eles ja jogaram entre si ounao?). Logo, ha apenas n − 1 gavetas para n objetos, o que garanteque dois times enfrentaram o mesmo numero de adversarios.

17. Dividamos o retangulo em 6 retangulos 1 × 2. Como ha 7 pontos, hadois que estao no mesmo retangulo. A distancia entre eles e no maximoigual a diagonal do retangulo, que mede

√5.

18. Como os numeros sao escolhidos no conjunto 1, 2, . . . , 15, os valorespossıveis para a diferenca de dois numeros sao 1, 2, . . . , 14 (ou seja,

6

ha 14 valores possıveis). Por outro lado, os oito numeros formam C28 =

28 pares. Destes, no maximo 1 resulta em uma diferenca igual a 14(quando formado por 1 e 15). Em consequencia, ha pelo menos 27 parescujas diferencas pertencem ao conjunto 1, 2, . . . , 13. Como 27/13 >2, ha uma gaveta (diferenca) contendo mais de dois objetos (pares).Portanto, ha pelo menos tres pares de numeros em que a diferencaentre o maior e o menor numero do par e a mesma.

19. (a) Seja λ1 = x−x1

x2−x1e λ2 = x2−x

x2−x1. Entao λ1 e λ2 sao numeros positivos,

ja que x − x1, x2 − x e x2 − x1 o sao. Alem disso, λ1 + λ2 =x−x1+x2−x

x2−x1= 1 e λ1x1 +λ2x2 = x2−x1x+x2x−x2

x2−x1= x. Por outro lado,

se λ1x1 +λ2x2 = x, com λ1 +λ2 = 1, entao λ1x1 + (1−λ1)x2 = x,de onde resulta λ1 = x−x1

x2−x1e λ2 = x2−x

x2−x1, mostrando que a solucao

e unica.

(b) Se x = λ1x1 + λ2x2, com λ1 + λ2 = 1, λ1 > 0 e λ2 > 0, entao xe uma media ponderada, com pesos positivos, de x1 e x2. Logo,x1 < x < x2.

(c) Temos x = x1 + (λ1− 1)x1 +λ2x2 = x1 +λ2(x2−x1). Mas λ1 > 1implica em λ2 < 0. Logo, x < x1, ou seja, x ∈ (−∞, x1).

(d) Temos x = x2 + (1 − λ2)x2 + λ1x1 = x2 − λ1(x2 − x1). Comoλ1 < 0, temos x > x2, ou seja, x ∈ (x2,+∞).

20. (a) Utilizando o exercıcio anterior, x pode ser escrito na forma x =λ1x1 + λnxn com λ1 e λn positivos e λ1 + λn = 1. Ou seja, xpode ser escrito na forma pedida com λ2 = · · · = λn−1 = 0. Arepresentacao nao e unica quando n > 2. Por exemplo, aindausando o exercıcio anterior, todo x tambem pode ser escrito naforma λixi + λjxj, onde i e j sao quaisquer ındices tais que xi ≤x ≤ xj. Se n > 2, certamente ha mais de um modo de escolher ie j.

(b) Basta observar que x e uma media ponderada de x1, x2, . . . , xn.Como x1 e o menor dos numeros e xn e o maior e os pesos saotodos positivos, resulta que x1 < x < xn.

21. A altura media e 30·1,75+10·1,6740

= 1, 73.

7

Unidade 22

Exercıcios Recomendados

1. Se os numeros tem soma constante, sua media aritmetica A e tambemconstante. Pela desigualdade das medias, o maior valor possıvel paraa media geometrica G e igual a A, o que ocorre quando os numerossao iguais. Logo, o produto dos numeros (que e o quadrado de G) emaximo quando os numeros sao iguais.

2. Se os numeros tem produto constante, sua media geometricaG e tambemconstante. Pela desigualdade das medias, o menor valor possıvel paraa media aritmetica A e igual a G, o que ocorre quando os numeros saoiguais. Logo, a soma dos numeros (que e dobro de A) e mınima quandoos numeros sao iguais.

3. Da desigualdade das medias, temos1x1

+ 1x2

+···+ 1xn

n≤ n

√1x1· 1x2· · · · · 1

xn,

ou seja, 1H≥ 1

G. Daı, decorre, H ≤ G, com igualdade somente quando

1x1

= 1x2

= · · · = 1xn

, ou seja, quando todos os numeros sao iguais.

4. Temos

n∑k=1

(xk − A)2 =n∑

k=1

(xk)2 − 2An∑

k=1

xk + nA2

= nQ2 − 2A · nA+ nA2 = n(Q2 − A2)

Como uma soma de quadrados e necessariamente nao negativa, resultaQ2 ≥ A2 ou, equivalentemente, Q ≥ A. Alem disso, so se tem igualdadequando cada termo da soma inicial e nulo, ou seja, quando xk = A, paratodo k, o que significa que todos os numeros xk sao iguais.

5. A media geometrica de b1a2, b2a2, . . . , bn

ane n

√b1b2···bna1a2···an = 1 (note que os

produtos no numerador e denominador sao iguais, ja que b1, b2, . . . , bnsao uma reordenacao de a1, a2, . . . , an). Logo, sua media aritmetica e

maior que ou igual a 1. Assimb1a1

+b2a2

+···+ bnan

n≥ 1 ou, equivalentemente,

b1a1

+ b2a2

+ · · ·+ bnan≥ n.

8

6. Aplicando a desigualdade entre as medias aritmetica e geometrica aospares (x2, y2), (y2, z2) e (z2.y2), vem

x2 + y2

2≥xy

y2 + z2

2≥yz

z2 + x2

2≥zx

Somando membro a membro vem a desigualdade pedida.

7. Aplicando a desigualdade das medias a a1a2, a2a3 e a3a1, resulta

a1a2 + a2a3 + a3a13

≤ 3

√a21a

22a

23,

que equivale a √a1a2 + a2a3 + a3a1

3≤ 3√a1a2a3.

A outra desigualdade e equivalente a(a1+a2+a3

3

)2 ≥ a1a2+a2a3+a3a13

, que,por sua vez, e equivalente a a21 + a22 + a23 ≥ a1a2 + a2a3 + a3a1, que everdadeira, pelo exercıcio anterior.

8. Se x ≤ 0, entao x3−ax2+bx−c e uma soma de numeros nao positivos naqual pelo menos −c e negativo. Logo, se a equacao x3−ax2+bx−c = 0possui tres raızes reais x1, x2, x3, elas sao necessariamente positivas.Usando as relacoes entre coeficientes e raızes, temos a = x1 + x2 + x3,b = x1x2 + x2x3 + x3x1 e c = x1x2x3. Pelo exercıcio anterior, temosa3≤√

b3≤ 3√c. Elevando a sexta potencia e multiplicando por 729,

vem a6 ≥ 27b3 ≥ 729c2.

9. Com x paletos e y calcas, podem ser formadas xy roupas diferentes. Ovalor mınimo da soma de dois numeros com produto constante ocorrequando os numeros sao iguais. Se x pudesse assumir valores nao in-teiros, o mınimo ocorreria para x =

√500 = 22, 36. Claramente, 44

roupas nao bastam, porque o produto maximo de dois numeros comsoma 44 e igual a 22×22 < 500. Mas 45 bastam, ja que 22×23 = 506.Ou seja, o magico pode usar trajes diferentes em todas as apresentacoescom 23 (ou 22) calcas e 22 (ou 23) paletos.

9

10. Pela formula de Heron, a area de um triangulo de lados a, b e c e S =12

√p(p− a)(p− b)(p− c). Para triangulos de perımetro constante, a

area maxima quando o produto (p− a)(p− b)(p− c) e maximo. Mas asoma (p− a) + (p− b) + (p− c) e igual a p, portanto constante. Logo,para que (p−a)(p−b)(p−c) seja maximo deve-se p−a = p−b = p−c,ou seja, a = b = c. Logo, dentre os triangulos de perımetro constante,o equilatero e o de maior area.

11. (a) A media geometrica de x e 1x

e constante e igual a 1. Logo, suamedia aritmetica e sempre maior que ou igual a 1, isto e, x+ 1

x≥ 2,

com igualdade quando os numeros sao iguais (ou seja, quandox = 1).

(b) A media geometrica de x e 4/x e constante e igual a√x · 4

x= 2.

Em consequencia, o valor mınimo da media aritmetica e 2 e o valormınimo de x + 4

xe 4, que ocorre quando x = 4

x, ou seja, quando

x = 2.

12. A media aritmetica dos numeros x1 = x2 = · · · = xn = 1+ 1n, xn+1 = 1 e

A = n+2n+1

= 1+ 1n+1

, enquanto a sua media geometrica eG =(1 + 1

n

) nn+1 .

A desigualdade das medias fornece, portanto, 1 + 1n+1

<(1 + 1

n

) nn+1 ,

que e equivalente a(1 + 1

n+1

)n+1<(1 + 1

n

)n(note que a desigualdade

e estrita, ja que os numeros x1, . . . , xn nao sao todos iguais).

Exercıcios Suplementares

1. Aplicando a desigualdade entre a media aritmetica e a media harmonica

aos numeros 1x, 1

ye 1

zobtemos

1x+ 1

y+ 1

z

3≥ 3

x+y+zou, equivalentemente,

1x

+ 1y

+ 1z≥ 9

x+y+z.

2. Aplicando a desigualdade das medias aos numeros√x,√y e√z, vem

√x+√y+√z

3≥ 3

√√x√y√z =

√3√xyz.

3. Aplicando a desigualdade das medias aos numeros xy, yz e xz, re-sulta xy+yz+zx

3≥ 3√

(xyz)2. Como xy + yz + zx ≤ 3, temos xyz ≤

10

√(xy+yz+zx

3

)3 ≤ 1, com igualdade no caso x = y = z = 1. Por outro

lado, com x = y = 1n

e z = n, temos xy + yz + zx = 2 + 1n2 (portanto,

1 ≤ xy + yz + zx ≤ 3) e xyz = 1/n, o que mostra que xyz pode ficararbitrariamente proximo a zero, bastando para isso tomar n suficien-temente grande. Portanto, o conjunto de valores de xyz e o intervalo(0, 1].

Pelo Problema Recomendado 7, x + y + z ≥ 3√

xy+yz+zx3

≥√

3, com

igualdade no caso x = y = z =√33

(a ultima desigualdade decorre dexy+yz+zx ≥ 1). Por outro lado, novamente tomando x = y = 1

ne z =

n, temos xy+yz+zx = 2+ 1n2 (portanto, de novo, 1 ≤ xy+yz+zx ≤ 3)

e x+ y + z = n+ 2n, o que mostra que x+ y + z pode assumir valores

arbitrariamente grandes. Logo, o conjunto de valores de x + y + z e ointervalo [

√3,+∞).

4. A desigualdade xyz ≤ 1 continua valida e xyz continua a poder assumirvalores arbitrariamente proximos a zero (basta tomar, como antes, x =y = 1

ne z = n). Logo, o conjunto de valores de xyz continua a ser o

intervalo (0, 1].

Como antes, os valores de x + y + z podem se tomar arbitrariamentegrandes. Por outro lado, como foi removida a condicao xy+yz+zx ≥ 1,eles tambem podem se tomar arbitrariamente proximos de zero (porexemplo, tome x = y = z = 1

n. Logo, o conjunto de valores de x+y+z

e o intervalo (0,+∞).

5. Como no Problema Suplementar 3, xyz pode tomar valores arbitraria-mente proximos de zero. Por outro lado, uma vez removida a restricaoxy + yz + zx ≤ 3, xyz pode tambem se tomar arbitrariamente grande(por exemplo, tome x = y = z = n, com n natural). Logo, o conjuntode valores de xyz e o intervalo (0,+∞).

A desigualdade x + y + z ≥√

3 do Problema Suplementar 3 continuaa ser valida e, alem disso, tomando x = y = 1

ne z = n consegue-se

fazer x+ y+ z arbitrariamente grande. Logo, o conjunto de valores dex+ y + z continua sendo o intervalo [

√3,+∞).

6. Pela desigualdade das medias, temos 3√xyz ≤ x+y+z

3. Como x+y+z ≤

3, vem xyz ≤(x+y+z

3

)3 ≤ 1, com igualdade quando x = y = z = 1. Por

11

outro lado, tomando x = y = 1n

e z = 1, temos x+y+z = 1+ 2n∈ [1, 3]

e xyz = 1n2 . Logo xyz pode assumir valores arbitrariamente proximos

de zero. Portanto, o conjunto de valores de xyz e o intervalo (0, 1].

Pelo Problema Recomendado 7, temos xy + yz + zx ≤ 3(x+y+z

3

)2.

Justamente com x + y + z ≤ 3, isto fornece xy + yz + zx ≤ 3, comigualdade quando x = y = z = 1. Por outro lado, tomando x = y = 1

n

e z = 1, temos x+ y+ z = 1 + 2n∈ [1, 3] e xy+ yz+ zx = 2

n+ 1

n2 . Logo,xy + yz + zx pode assumir valores arbitrariamente proximos de zero.Portanto, o conjunto de valores de xy + yz + zx e o intervalo (0, 3].

7. Como visto no Problema Suplementar 3, temos xy+yz+zx3

≥ 3√

(xyz)2.Portanto, de xyz ≥ 1 decorre xy+ yz+ zx ≥ 3, com igualdade quandox = y = z = 1. Por outro lado, tomando-se x = y = 1

ne z = n2, tem-se

xyz = 1 e xy + yz + zx = 2n+ 1n2 , que assume valores arbitrariamente

grandes para n natural. Logo, o conjunto de valores de xy + yz + zx eo intervalo [3,+∞).

Pela desigualdade das medias, x+y+z3≥ 3√xyz. Logo, de xyz ≥ 1,

resulta x+ y + z ≥ 3 3√xyz ≥ 3, com igualdade quando x = y = z = 1.

Por outro lado, tomando-se x = y = 1n

e z = n2, tem-se xyz = 1 ex+ y+ z = n2 + 2

n, que assume valores arbitrariamente grandes para n

natural. Logo, o conjunto de valores de x+ y+ z e o intervalo [3,+∞).

8. Como visto no Problema Suplementar 3, xy+yz+zx3

≥ 3√

(xyz)2. Por-tanto, de xyz = 8 decorre xy + yz + zx ≥ 12, com igualdade quandox = y = z = 2. Por outro lado, tomando x = y = 2

ne z = 2n2,

vem xyz = 8 e xy + yz + zx = 8n + 4n2, que pode assumir valoresarbitrariamente grandes para n natural. Logo, o conjunto de valoresde xy + yz + zx e o intervalo [12,+∞).

Pela desigualdade das medias, x+y+z3≥ 3√xyz. Logo, de xyz = 8,

resulta x + y + z ≥ 3 3√

8 = 6, com igualdade quando x = y = z = 2.Por outro lado, tomando-se x = y = 2

ne z = 2n2, tem-se xyz = 8 e

xy + yz + zx = 2n2 + 4n, que assume valores arbitrariamente grandes

para n natural. Logo, o conjunto de valores de x + y + z e o intervalo[6,+∞).

9. Sejam A e G as medias aritmeticas e geometrica de a1, a2, . . . , am−1.Aplicando a desigualdade das medias aos m numeros a1, a2, . . . , am−1,

12

A, temos a1+a2+···+am−1+Am

≥ m√a1a2 · · · am−1A, ou seja (m−1)A+A

m≥

m√Gm−1A. Daı, obtemos Am ≥ Gm−1A, ou seja, Am−1 ≥ Gm−1 e,

finalmente, A ≥ G. Alem disso, a igualdade ocorre somente quandoa1 = a2 = · · · = am−1 = A.

13

P1 - MA 12 - 2011

Questão 1.

Considere a sequência (an)n≥1 definida como indicado abaixo:

a1 = 1

a2 = 2 + 3

a3 = 4 + 5 + 6

a4 = 7 + 8 + 9 + 10

. . .

(0.5) (a) O termo a10 é a soma de 10 inteiros consecutivos. Qual é o menor e o qual é o maior desses inteiros?

(0.5) (b) Calcule a10.

(1.0) (c) Forneça uma expressão geral para o termo an.

UMA RESPOSTA

(a) O primeiro inteiro da soma que define an é igual ao número de inteiros utilizados nos termos a1, . . . , an−1, isto

é, 1 + 2 + . . . + n− 1 mais um, isto é, é igual a 12 (n− 1)n + 1. O último inteiro é esse número mais n− 1. Portanto,

para n = 10, o primeiro inteiro é 46 e o último é 55.

(b) a10 é a soma de uma progressão aritmética de 10 termos, sendo o primeiro igual a 46 e o último igual a 55. Então

a10 =(46 + 55) · 10

2= 101 · 5 = 505 .

(c) No caso de an, trata-se da soma de uma progressão aritmética de n termos, sendo o primeiro igual a 12 n(n− 1)+ 1

e o último igual a 12 n(n− 1) + 1 + (n− 1), ou seja, 1

2 n(n− 1) + n, como visto em (a). Então

an =

[12 n(n− 1) + 1

]+

[12 n(n− 1) + n

]2

· n =(n− 1)n2 + (n + 1)n

2=

n3 + n2

.

1

P1 - MA 12 - 2011

Questão 2.

Um comerciante, para quem o dinheiro vale 5% ao mês, oferece determinado produto por 3 prestações mensais

iguais a R$ 100,00, a primeira paga no ato da compra.

(1.0) (a) Que valor o comerciante deve cobrar por esse produto, no caso de pagamento à vista?

(1.0) (b) Se um consumidor desejar pagar o produto em três prestações mensais iguais, mas sendo a primeira paga um

mês após a compra, qual deve ser o valor das parcelas?

Utilize, se desejar, os seguintes valores para as potências de 1, 05: 1, 052 = 1, 1025; 1, 05−1 = 0, 9524; 1, 05−2 =

0, 9070.

UMA RESPOSTA

(a) Trazendo os valores da segunda e da terceira prestações para o ato da compra, e somando, obtém-se

100 +1001, 05

+100

1, 052 = 100 + 95, 24 + 90, 70 = 285, 94 .

Então o comerciante poderá cobrar 285,94 reais, de forma que, se deixar seu dinheiro valorizar 5% ao mês, poderá

dispor de 100 reais no ato da compra (tirando 100 reais dos 285,94), 100 reais ao final do primeiro mês (deixando

95,24 reais valorizarem 5% durante um mês) e 100 reais ao final do segundo mês (deixando 90,70 reais valorizarem

5% ao mês durante dois meses).

(b) Para o parcelamento desejado pelo consumidor, as parcelas se deslocam um mês adiante. Então em cada uma

das três parcelas de 100 reais devem incidir juros de 5%. Portanto, são 3 parcelas de 105 reais.

2

P1 - MA 12 - 2011

Questão 3.

Considere o conjunto dos números escritos apenas com os algarismos 1, 2 e 3, em que o algarismo 1 aparece uma

quantidade par de vezes (por exemplo, 2322 e 12123). Seja an a quantidade desses números contendo exatamente n

algarismos.

(0.4) (a) Liste todos esses números para n = 1 e n = 2, indicando os valores de a1 e a2.

(0.8) (b) Explique por que an satisfaz a equação de recorrência an+1 = (3n − an) + 2an, para n ≥ 1 (note que 3n é o

número total de números com n algarismos iguais a 1, 2 ou 3).

(0.8) (c) Resolva a equação de recorrência em (b).

UMA RESPOSTA

(a) Para n = 1 só há três números possíveis: 1, 2 e 3. Somente os dois últimos têm um número par de algarismos

iguais a 1 (neste caso, nenhum algarismo igual a 1). Então a1 = 2. Os números de 2 algarismos são: 11, 12, 13, 21,

22, 23, 31, 32, 33, num total de 9 = 32. Cinco deles têm uma quantidade par de algarismos iguais a 1, então a2 = 5.

(b) (Antes de fazer o exercício, pode-se verificar se a fórmula está correta para n = 1: 5 = a2 = (31 − a1) + 2a1 =

3 + a1 = 3 + 2 = 5.) Observa-se primeiro que a quantidade de números com n algarismos tendo uma quantidade

ímpar de algarismos iguais a 1 é 3n − an, pois o número total de sequências é 3n.

Para obter a relação de recorrência, observe que todo número de n + 1 algarismos é uma concatenação de um

número de n algarismos com um número de 1 algarismo. Para que a quantidade de algarismos iguais a 1 do

número de n + 1 algarismos seja par é preciso que: ou o número de algarismos iguais a 1 de cada um dos números

concatenados seja ímpar ou o número de algarismos iguais a 1 de cada um dos números concatenados seja par.

Então, para calcular an+1, soma-se o número de concatenações do primeiro caso (ímpar-ímpar) com o número de

concatenações do segundo caso (par-par). Isto dá

an+1 = (3n − an) · (31 − a1) + an · a1 ,

isto é, a fórmula do enunciado, já que a1 = 2.

(c) Observa-se que an+1 = an + 3n, apenas simplificando-se a expressão. Isto implica

an = a1 + 31 + 32 + . . . + 3n−1 = 1 + (1 + 3 + 32 + . . . + 3n−1) ,

em que a expressão entre parênteses é a soma dos n primeiros termos da progressão geométrica de termo inicial 1 e

razão 3, que vale3n − 13− 1

.

Portanto

an =3n + 1

2.

3

P1 - MA 12 - 2011

Questão 4.

(1.0) (a) Mostre, por indução finita, que

1 · 30 + 2 · 31 + 3 · 32 + . . . + n · 3n−1 =(2n− 1)3n + 1

4.

(1.0) (b) Seja (an)n≥1 progressão geométrica com termo inicial a1 positivo e razão r > 1, e Sn a soma dos n primeiros

termos da progressão. Prove, por indução finita, que Sn ≤ rr−1 an, para qualquer n ≥ 1.

UMA RESPOSTA

(a) A equação é verdadeira para n = 1, pois 1 · 30 = 1 e

(2 · 1− 1)31 + 14

= 1 .

Supondo válida para n, vamos mostrar que vale para n + 1, isto é, vamos mostrar que, acrescentando o termo

(n + 1) · 3n, a soma resultará em(2(n + 1)− 1)3n+1 + 1

4.

Usando a hipótese de indução,

1 · 30 + 2 · 31 + 3 · 32 + . . . + n · 3n−1 + (n + 1)3n =(2n− 1)3n + 1

4+ (n + 1)3n .

Manipulando a expressão à direita,

(2n− 1)3n + 14

+ (n + 1)3n =[2n− 1 + 4(n + 1)]3n + 1

4=

(2n + 1)3n+1 + 14

=(2(n + 1)− 1)3n+1 + 1

4,

como queríamos demonstrar.

(b) Para n = 1 a desigualdade é verdadeira: como r > 1, então rr−1 > 1; e como S1 = a1 > 0, então S1 = a1 < r

r−1 a1.

Suponha agora que a desigualdade vale para n, isto é, suponha que Sn ≤ rr−1 an é verdadeira. Vamos provar que

ela vale para n + 1, isto é, vamos provar que Sn+1 ≤ rr−1 an+1. Primeiro, escrevemos Sn+1 = Sn + an+1, pois Sn+1

é a soma dos primeiros n termos adicionada do termo n + 1. Usando a hipótese de indução, Sn+1 ≤ rr−1 an + an+1.

Como se trata de uma progressão geométrica an+1 = ran, ou seja, podemos trocar an por an+1r . Então Sn+1 ≤

rr−1 ·

an+1r + an+1, isto é, Sn+1 ≤ ( 1

r−1 + 1)an+1 = rr−1 an+1, que é o que queríamos demonstrar.

4

P1 - MA 12 - 2011

Questão 5.

Seja (xn)n≥0 sequência definida pela relação de recorrência xn+1 = 2xn + 1, com termo inicial x0 ∈ R.

(0.5) (a) Encontre x0 tal que a sequência seja constante e igual a um número real a.

(1.0) (b) Resolva a recorrência com a substituição xn = yn + a, em que a é valor encontrado em (a).

(0.5) (c) Para que valores de x0 a sequência é crescente? Justifique.

UMA RESPOSTA

(a) Basta achar a tal que 2a + 1 = a. Isto dá a = −1. Se x0 = a então x1 = 2x0 + 1 = 2a + 1 = a = x0, e, da mesma

forma, x2 = x1, x3 = x2, . . ., xn+1 = xn para qualquer n ≥ 0, ou seja, a sequência é constante.

(b) Com a substituição sugerida, xn = yn − 1. Então yn+1 − 1 = 2(yn − 1) + 1, isto é, yn+1 = 2yn, com y0 = x0 + 1.

Então yn = 2ny0 = 2n(x0 + 1) e xn = yn − 1 = −1 + 2n(x0 + 1).

(c) Se x0 + 1 > 0, isto é, x0 > −1, então 2n(x0 + 1) é crescente e xn = −1 + 2n(x0 + 1) é crescente. Se x0 + 1 < 0, isto

é x0 < −1, então xn = −1 + 2n(x0 + 1) = −1− 2n|x0 + 1| é descrescente. E se x0 = −1 então xn é constante. De

onde se conclui que xn é crescente se, e somente se, x0 ∈ (−1,+∞).

5

AV1 - MA 12 - 2012

Questão 1.

Uma venda imobiliária envolve o pagamento de 12 prestações mensais iguais a R$ 10.000,00, a primeira no

ato da venda, acrescidas de uma parcela final de R$ 100.000,00, 12 meses após a venda. Suponha que o valor

do dinheiro seja de 2% ao mês.

(a) Se o comprador preferir efetuar o pagamento da parcela final junto com a última prestação, de quanto

deverá ser o pagamento dessa parcela?

(b) Se o comprador preferir efetuar o pagamento à vista, qual deverá ser o valor desse pagamento único?

São dados alguns valores aproximados de 1, 02n:

n 1, 02n

-12 0,788

-1 0,980

12 1,268

UMA SOLUÇÃO

(a) O valor de R$100.000,00 trazido um mês para trás é igual a

100.000, 00× 11, 02

' 0, 980× 100.000, 00 = 98.000, 00 .

(b) Trazendo os valores para a data de compra, o comprador pagará

10.000, 00 +10.000, 00

1, 02+

10.000, 001, 022 + · · ·+ 10.000, 00

1, 0211 +100.000, 00

1, 0212 .

Isso é igual a

10.000, 00× (1 + 1, 02−1 + 1, 02−2 + . . . + 1, 02−11) + 100.000, 00× 1, 02−12

= 10.000, 00× 1− 1, 02−12

1− 1, 02−1 + 100.000, 00× 1, 02−12

' 10.000, 00× 1− 0, 7881− 0, 980

+ 100.000, 00× 0, 788

= 106.000, 00 + 78.800, 00 = 184.800, 00 .

Portanto, se o dinheiro vale 2% ao mês, pagar o esquema de prestações do enunciado equivale a pagar (aproxima-

damente) R$ 184.800,00 à vista.

1

AV1 - MA 12 - 2012

Questão 2.

A figura abaixo mostra uma linha poligonal que parte da origem e passa uma vez por cada ponto do plano

cujas coordenadas são números inteiros e não negativos.

(a) O conjunto dos pares de números inteiros e não negativos tem a mesma cardinalidade que os números

naturais? Por quê?

(b) Mostre que o comprimento da linha poligonal da origem até o ponto (n, n) é n2 + n, para qualquer inteiro

não negativo n.

(c) Qual é o comprimento da linha poligonal da origem até o ponto (10, 13)?

UMA SOLUÇÃO

(a) Chamemos de Z≥0 o conjunto dos inteiros não negativos. Então o conjunto dos pontos de R2 com coordenadas

inteiras e não negativas é o produto cartesiano Z2≥0 = Z≥0 ×Z≥0.

Imaginemos que a linha é percorrida com velocidade 1 a partir do instante 1 em (0, 0). A figura mostra que se

no instante k a curva está num ponto de Z2≥0 então no instante k + 1 ela estará em um outro ponto de Z2

≥0. Por

indução, estabelece-se uma função f : N→ Z2≥0 em que f (k) é o ponto de Z2

≥0 alcançado no instante k.

Como todos os pontos são atingidos, f é sobrejetiva. Como a linha não passa mais do que uma vez em cada

ponto, f é injetiva. Assim, existe uma bijeção entre N e Z2≥0, mostrando que N e Z2

≥0 têm a mesma cardinalidade.

(b) Por inspeção a afirmação é verdadeira para n = 0, pois n2 + n = 0 e realmente são 0 passos para chegar no

ponto de partida (n, n) = (0, 0). Agora suponhamos que a afirmação é válida para (n, n), isto é, que realmente são

n2 + n passos até se chegar em (n, n) (hipótese de indução). Queremos mostrar que a afirmação é válida quando

aplicada para n + 1, isto é, que são (n + 1)2 + (n + 1) passos até se chegar em (n + 1, n + 1).

De (n, n) até (n + 1, n + 1) são necessários: n passos (para encontrar um dos eixos; mais especificamente, para

encontrar a abscissa, se n é par, e para encontrar a ordenada, se n é ímpar) mais 1 passo (para avançar nesse eixo)

mais n + 1 passos (para voltar à diagonal, que é o conjunto dos pontos da forma (x, x)). Assim, são necessários

n + 1+ (n + 1) = 2n + 2 passos para ir-se de (n, n) a (n + 1, n + 1). Pela hipótese de indução, já foram n2 + n passos

para se chegar em (n, n). Portanto são (n2 + n) + (2n + 2) passos até (n + 1, n + 1). Mas

(n2 + n) + (2n + 2) = (n2 + 2n + 1) + (n + 1) = (n + 1)2 + (n + 1) ,

2

como queríamos demonstrar.

Solução alternativa 1. Para se chegar ao ponto (n, n), é preciso percorrer todos os pontos de coordenadas inteiras

do quadrado [0, n]× [0, n], exceto os situados em um dos lados. Existem (n + 1)2 pontos de coordenadas inteiras

no quadrado, dos quais n não são visitados. Logo, o comprimento da poligonal é (n + 1)2 − 1− n = n2 + n.

Solução alternativa 2. A linha poligonal da origem até o ponto (n, n) é formada por n segmentos de comprimento

1, por segmentos de comprimento 2k, para k variando de 1 a n− 1 e um segmento de comprimento n. Logo, seu

comprimento é

n + 2(1 + 2 + ... + n− 1) + n = n + 2 · (n− 1)n2

+ n = n2 + n .

(c) Primeiro, investiga-se se (10, 13) ocorre a 3 passos de distância (para mais ou para menos) de (10, 10) ou de

(13, 13), no trajeto definido pela curva. Vemos que (10, 13) está 3 unidades verticalmente acima de (10, 10) e 3

unidades horizontalmente à esquerda de (13, 13). Quando (n, n) é par, como é o caso de n = 10, a linha poligonal

prossegue na vertical para baixo, portanto no sentido contrário ao que esperaríamos se fosse encontrar (10, 13) em

3 passos. Quando (n, n) é ímpar, como é o caso de n = 13, a linha poligonal prossegue horizontalmente para a

esquerda. Neste caso, encontrará (10, 13) após 3 passos.

Portanto, como são 132 + 13 = 169+ 13 = 182 passos até (13, 13) e mais 3 passos até (10, 13), então são 185 passos

até (10, 13).

3

AV1 - MA 12 - 2012

Questão 3.

Mostre, por indução finita, que se n é um inteiro positivo então 7n − 1 é divisível por 6.

UMA SOLUÇÃO

Para n = 1, 7n − 1 = 7− 1 = 6, que é divisível por 6. Então a afirmação vale para n = 1. Suponhamos que a

afirmação seja válida para n, isto é, suponha que 7n − 1 seja múltiplo de 6. Vamos mostrar, com essa hipótese, que

7n+1 − 1 também é múltiplo de 6.

Ora, 7n+1 − 1 = 7n+1 − 7n + 7n − 1 = 7n(7− 1) + (7n − 1) = 6 · 7n + (7n − 1). O primeiro termo é múltiplo de

6, porque tem um fator 6, e o segundo também é, pela hipótese de indução. Então a soma é múltiplo de 6 e temos

demonstrado o que queríamos.

4

AV1 - MA 12 - 2012

Questão 4.

Considere a recorrência xn+2 − 4xn = 9n, com as condições iniciais x0 = x1 = 0.

(a) Encontre a solução geral da recorrência homogênea xn+2 − 4xn = 0.

(b) Determine os valores de A e B para os quais xn = A + nB é uma solução da recorrência xn+2− 4xn = 9n.

(c) Encontre a solução da recorrência original.

UMA SOLUÇÃO

(a) Se xn+2 − 4xn = 0 então xn+2 = 4xn. Então x2m = 4mx0, para todo m ≥ 0, e x2m+1 = 4mx1, para todo n ≥ 0.

Escrevendo de outra maneira, a solução é

x0, x1, 4x0, 4x1, 42x0, 42x1, 43x0, 43x1, . . .

Também pode-se dizer que xn = 2nx0, para n ≥ 0 par, e xn = 12 · 2nx1, para n ≥ 0 ímpar.

(b) Se xn = A + nB então xn+2 = A + (n + 2)B. Se, além do mais, (xn) é solução de xn+2 − 4xn = 9n, então

9n = xn+2 − 4xn = A + (n + 2)B− 4A− 4Bn = −3A + 2B− 3nB .

Para que −3A + 2B − 3nB seja igual a 9n basta que −3A + 2B = 0 (primeira equação) e que −3B = 9 (segunda

equação). Da segunda equação sai imediatamente que B = −3, e, colocando esse valor na primeira, que A = −2.

Então xn = −2− 3n é uma solução da equação não homogênea.

(c) Agora vamos combinar a solução geral da homogênea com a solução particular da não homogênea para obter

a solução de xn+2 − 4xn = 9n com x0 = x1 = 0. Seja xn = −2 − 3n a solução calculada em (b), que satisfaz

xn+2 − 4xn = 9n. Essa solução não satisfaz as condições iniciais pedidas, pois x0 = −2 e x1 = −5. Então seja

(xn) solução da homogênea satisfazendo x0 = +2 e x1 = +5. Vamos verificar que (xn) definida por xn = xn + xn

satisfaz ao mesmo tempo as condições iniciais e a relação de recorrência não homogênea.

Ora, x0 = x0 + x0 = −2 + 2 = 0 e x1 = x1 + x1 = −5 + 5 = 0. Além disso,

xn+2 − 4xn = (xn+2 − 4xn) + (xn+2 − 4xn) = 9n + 0 = 9n .

Assim, a solução do problema proposto é a sequência dada por xn = 2 · 2n − 3n− 2 = 2n+1 − 3n− 2, para n par,

xn = 52 · 2n − 3n− 2, para n ímpar.

5

AV1 - MA 12 - 2012

Questão 5.

Para todo número natural n ≥ 2, considere o número N formado por n− 1 algarismos iguais a 1, n algarismos

iguais a 2 e um algarismo igual a 5, nesta ordem.

(a) Mostre que o número N pode ser escrito na forma

A · 102n + B · 10n + C9

,

onde A, B e C são constantes independentes de n. Indique os valores de A, B e C.

(b) Mostre que N é um quadrado perfeito.

(c) Quantos algarismos tem√

N? Diga quais são esses algarismos.

UMA SOLUÇÃO

(a) Usando a expansão na base decimal, podemos escrever N como

N = 102n−1 + 102n−2 + . . . + 10n+1 + 2 · 10n + 2 · 10n−1 + . . . + 2 · 101 + 5 .

Então

N = 10n+1(1 + 10 + . . . + 10n−2) + 2 · 10 · (1 + 10 + . . . + 10n−1) + 5 .

Somando as duas PGs entre parênteses,

N = 10n+1 · 10n−1 − 110− 1

+ 20 · 10n − 110− 1

+ 5

=102n − 10n+1 + 20 · 10n − 20 + 45

9

=102n + 10 · 10n + 25

9.

Portanto A = 1, B = 10 e C = 25.

Obs: Outra forma de fazer é multiplicar N por 9 usando o algoritmo de multiplicação e ver que fica o número

10 . . . 010 . . . 025, onde o bloco de zeros mais à esquerda tem n− 2 elementos e o bloco de zero mais à direita tem

n− 1 elementos.

(b) Queremos saber se N = p2, com p ∈N. Como 102n + 10 · 10n + 25 = (10n + 5)2, então

N =

(10n + 5

3

)2.

Resta saber se 10n + 5 é divisível por 3. Mas isso é verdade, porque como 10n + 5 = 10 . . . 05, com um bloco de n− 1

zeros, a soma dos algarismos desse número é igual a 6.

6

(c) A raiz de N é o número p = 10n+53 . Como 10n + 5 = 10 . . . 05, com um bloco de n− 1 zeros, então tem n + 1

algarismos. Ao dividir por 3, passa a ter n algarismos. Então p tem n algarismos.

Para saber qual é o número, podemos escrever

p =10n − 1

3+

63

.

O termo da esquerda é 33 . . . 3 (n vezes) e o da direita é igual a 2. Então p = 3 . . . 35, onde 3 aparece repetido n− 1

vezes.

7

AV2 - MA 12 - 2011

Questão 1.

Considere os caminhos no plano iniciados no ponto (0, 0) com deslocamentos paralelos aos eixos coordenados,

sempre de uma unidade e no sentido positivo dos eixos x e y (não se descarta a possibilidade de dois movimentos

unitários seguidos na mesma direção, ver ilustração mostrando um caminho que termina em (5, 4)).

5

4

x

y

(1,0) (a) Explique por que o número de caminhos que terminam no ponto (m, n) é Cmm+n.

(1,0) (b) Quantos são os caminhos que terminam no ponto (8, 7), passam por (2, 3) mas não passam por (5, 4)?

UMA SOLUÇÃO

(a) Chamaremos de horizontais os movimentos paralelos ao eixo x e de verticais os paralelos ao eixo y. Como todos

os movimentos são positivos e unitários, são necessários m movimentos horizontais e n movimentos verticais para

se chegar em (m, n), totalizando m + n movimentos. Um caminho fica totalmente determinado se dissermos quais

desses m + n movimentos são, digamos, movimentos horizontais. Portanto, precisamos saber de quantas maneiras

podemos escolher m movimentos horizontais entre os m + n movimentos do caminho. Isso dá Cmm+n.

Evidentemente poderíamos ter determinado os caminhos dizendo quais são os n movimentos horizontais dentre

os m + n movimentos. Esse raciocínio nos levaria a Cnm+n. Mas Cn

m+n = Cmm+n.

(b) Se um caminho até (8, 7) é obrigado a passar por (2, 3) então ele é a junção de um caminho que vai de (0, 0) a

(2, 3) com um caminho que vai de (2, 3) a (8, 7). No entanto, queremos que o caminho que vai de (2, 3) a (8, 7) não

passe por (5, 4), ou seja, queremos que ele vá de (2, 3) a (8, 7) sem ser a junção de um caminho de (2, 3) a (5, 4) com

um caminho de (5, 4) a (8, 7). Isso nos indica que precisamos calcular quantos caminhos temos de (0, 0) a (2, 3),

quantos de (2, 3) a (5, 4) e quantos de (5, 4) a (8, 7).

Segundo o item anterior, há C22+3 = C2

5 maneiras de ir de (0, 0) a (2, 3). Há C33+1 = C3

4 maneiras de se ir de (2, 3) a

(5, 4), pois são necessários 3 movimentos horizontais e 1 vertical. Há C33+3 = C3

6 maneiras de se ir de (5, 4) a (8, 7),

pois são necessários 3 movimentos horizontais e 3 verticais. E há C610 maneiras de se ir de (2, 3) a (8, 7), pois são

necessários 6 movimentos horizontais e 4 verticais.

Há, portanto, C34 · C3

6 maneiras de se ir de (2, 3) a (8, 7) passando por (5, 4). Então há C610 − C3

4 · C36 maneiras de se

ir de (2, 3) a (8, 7) sem passar por (5, 4). E, por conseguinte, há

N = C25 · (C6

10 − C34 · C3

6)

1

maneiras de se ir de (0, 0) a (8, 7) passando por (2, 3) mas não passando por (5, 4).

Para termos um número, calculamos essas combinações: C25 = 5!

3! 2! = 10, C610 = 10!

4!6! = 10·9·8·74·3·2 = 210, C3

4 = 4 e

C36 = 6!

3!3! =6·5·43·2 = 20. Então

N = 10 · (210− 4 · 20) = 1300 .

2

AV2 - MA 12 - 2011

Questão 2.

Os professores de seis disciplinas (entre as quais Português e Matemática) devem escolher um dia, de segunda

a sexta, de uma única semana para a realização da prova de sua disciplina. Suponha que cada professor escolha o

seu dia de prova ao acaso, sem combinar com os demais professores.

(1,0) (a) Qual é a probabilidade de que as provas de Português e Matemática sejam realizadas no mesmo dia?

(1,0) (b) Qual é a probabilidade de que os alunos façam provas em todos os dias da semana?

UMA SOLUÇÃO

(a) Nesta questão, não é preciso olhar para as outras disciplinas. Há 5 possibilidades para o dia de prova de Por-

tuguês e 5 possibilidades para o dia de prova de Matemática. Portanto, há 25 possibilidades para o par de provas

Português e Matemática. Dessas 25, apenas 5 são ocorrências de Português e Matemática no mesmo dia (uma

ocorrência para cada dia da semana). Então a probabilidade de que essas duas provas ocorram no mesmo dia é

5/25 = 0, 2 (ou 20%).

Outra maneira de pensar: fixado o dia da prova de Matemática, há 5 possibilidades para o dia de Português, e

apenas uma delas é no mesmo dia que Matemática. Isso dá os mesmos 20% de chances.

(b) Vamos contar de quantas maneiras se distribuem 6 provas nos 5 dias da semana sem deixar um dia livre. Com

essa imposição, certamente um dia terá duas provas e os demais dias terão apenas uma. Então começamos esco-

lhendo entre as 5 possibilidades para o dia da semana que terá duas provas. Escolhido esse dia, temos que escolher

duas das seis disciplinas para preenchê-lo. Temos C26 escolhas. Escolhidas essas duas disciplinas, ainda restam 4

para distribuir nos 4 dias: são 4! escolhas. Portanto há 5 · C26 · 4! maneiras de se distribuir 6 provas em 5 dias sem

deixar um dia livre.

Agora precisamos do total de maneiras de se distribuir as 6 provas durante a semana. Cada disciplina tem 5

escolhas, então são 56 possibilidades.

Então a probabilidade de não ficar um dia livre é o quociente

5 · C26 · 4!

56 =5! · 15

56 = 4! · 354 =72 · 1610000

=115210000

= 0, 1152 ,

ou 11,52%.

3

AV2 - MA 12 - 2011

Questão 3.

Em um jogo, uma moeda honesta é jogada seguidamente. Cada vez que sai cara, o jogador ganha 1 real; cada vez

que sai coroa, o jogador ganha 2 reais. O jogo termina quando o jogador tiver acumulado 4 ou mais reais.

(0,5) (a) Qual é a probabilidade de que o jogador ganhe exatamente 4 reais?

(0,5) (b) Qual é a probabilidade de que no último lançamento saia cara?

(1,0) (c) Dado que o jogador ganhou exatamente 4 reais, qual é a probabilidade de que tenha saído cara no último

lançamento?

UMA SOLUÇÃO

Nesta questão, convém fazer primeiro a árvore das possibilidades. Indicaremos “cara” por A e “coroa” por B. Em

cada nó da árvore, indicamos a sequência obtida (linha superior), o valor acumulado pelo jogador (linha do meio)

e a probabilidade daquela sequência (linha inferior). Os nós em cinza são aqueles em que o jogo termina. Veja que

a soma das probabilidades em cada nó pintado em cinza é igual a 1, e que a probabilidade indicada em cada um é

a probabilidade de o jogo terminar com aquela sequência.

AAABAAAA

1/21/2

1/41/41/41/4

1/81/81/81/81/81/8

1/161/1654

545443

4332

21

BAA BABABBABAAABAAA

BBBAABAA

BA

(a) Para ver a probabilidade de que o jogador termine com exatamente 4 reais, basta somar as probabilidades dos

nós em cinza que têm ganho de 4 reais. São eles: AAAA (1/16), AAB (1/8), ABA (1/8), BAA (1/8) e BB (1/4). A

soma é 1116 .

(b) O jogo termina com cara em todos os nós em cinza que terminam com a letra A. Então basta somar as probabili-

dades de cada caso. São eles AAAA (1/16), ABA (1/8) e BAA (1/8), o que dá 516 .

(c) Das situações em que o jogador terminou com 4 reais, listadas em (a), que têm probabilidade de 11/16 de

ocorrer, apenas AAAA, ABA e BAA terminam com A (cara), com probabilidade de 5/16. Então a probabilidade de

se terminar com cara dado que o jogador terminou com 4 reais é 5/1611/16 = 5

11 .

4

AV2 - MA 12 - 2011

Questão 4.

Uma prova de concurso é formada por questões de múltipla escolha, com 4 alternativas por questão. Admita que

nenhum candidato deixe questões sem responder.

(1,0) (a) Qual é o número mínimo de candidatos para que seja possível garantir que pelo menos 3 deles darão exata-

mente as mesmas respostas nas 5 primeiras questões?

(1,0) (b) Qual é o valor máximo de n para o qual é possível garantir que, em um concurso com 1000 candidatos, pelo

menos 2 darão as mesmas respostas nas primeiras n questões?

UMA SOLUÇÃO

(a) O conjunto de possibilidades de respostas para as 5 primeiras questões, cada uma com 4 alternativas, é 45.

É possível distribuir as respostas de 2 · 45 = 2048 candidatos de forma que cada conjunto de respostas se repita

exatamente duas vezes, mas se houver 2 · 45 + 1 = 2049 candidatos isso não é mais possível, sempre haverá ao

menos 3 provas iguais nas cinco primeiras questões.

(b) Considerando agora as n primeiras questões, há 4n possibilidades de resposta. Para garantir que em 1000 can-

didatos pelo menos 2 respondam de forma igual a essas primeiras n questões, é necessário que 1000 ≥ 4n + 1, isto

é, 4n ≤ 999. O valor máximo de n tal que 4n ≤ 999 é 4 (pois 44 = 28 = 256 e 45 = 210 = 1024). Resposta: n = 4.

5

AV2 - MA 12 - 2011

Questão 5.

Uma caixa retangular sem tampa tem arestas medindo x, y e z (veja figura, onde as linhas tracejadas indicam

segmentos de arestas obstruídos por alguma face).

(0,5) (a) Exprima a área e o volume da caixa em função de x, y e z.

(1,0) (b) Use a desigualdade das médias para mostrar que, se o volume da caixa é igual a 32, então sua área é maior ou

igual a 48.

(0,5) (c) Determine as medidas das arestas da caixa de área mínima com volume igual a 32.

x y

z

UMA SOLUÇÃO

(a) A área da caixa é igual a xy + 2xz + 2yz e seu volume é igual a xyz.

(b) A soma xy + 2xz + 2yz é igual a 3 vezes a média aritmética simples de seus termos. Essa média é sempre maior

do que ou igual à média geométrica dos mesmos termos, isto é

13(xy + 2xz + 2yz) ≥ 3

√xy · 2xz · 2yz = 3

√4x2y2z2 .

Supondo xyz = 32 (que é dado no problema), resulta que 3√

4x2y2z2 = 3√

22 · (25)2 =3√212 = 16. Então, multipli-

cando por 3 dos dois lados, xy + 2xz + 2yz ≥ 48.

(c) A igualdade entre as médias aritmética e geométrica ocorre se, e somente se, os termos são iguais. Neste caso,

quando xy = 2xz = 2yz. Como o volume é positivo, x, y, z têm que ser positivos, em particular não nulos. Então,

da equação 2xz = 2yz tiramos y = x, e da equação xy = 2yz tiramos z = x2 . Como xyz = 32 então x · x · x

2 = 32, isto

é, x3 = 64 = 26, ou ainda x = 4. Então x = y = 4 e z = 2.

6

AV3 - MA 12 - 2011

Questão 1.

A sequência 0, 3, 7, 10, 14, 17, 21, . . . é formada a partir do número 0 somando-se alternadamente 3 ou 4 ao termo

anterior, isto é: o primeiro termo é 0, o segundo é 3 a mais que o primeiro, o terceiro é 4 a mais que o segundo, o

quarto é 3 a mais que o terceiro, o quinto é 4 a mais que o quarto e assim sucessivamente.

(0,5) (a) Qual é o centésimo termo dessa sequência?

(0,5) (b) Qual é a soma dos 100 primeiros termos dessa sequência?

(1,0) (c) Algum termo desta sequência é igual a 2000? Por quê?

UMA SOLUÇÃO

(a) Chamemos de a1, a2, a3, . . . os termos dessa sequência. A sequência dos termos com índices ímpares a1, a3, a5, . . .

é uma progressão aritmética com termo inicial 0 e passo (ou razão) 7. A sequência dos termos com índices pares

a2, a4, a6, . . . é uma progressão aritmética com termo inicial 3 e passo 7. O centésimo termo é o 50o da sequência dos

pares. Então a100 = 3 + (50− 1) · 7 = 3 + 343 = 346.

(b) Há maneiras diferentes de se fazer isso. Podemos agrupar a soma assim:

(a1 + a100) + (a2 + a99) + (a3 + a98) + . . . + (a50 + a51) .

Veja que de a1 para a2 há um acréscimo de 3 e de a99 para a100 também. Então os dois primeiros termos são iguais.

Do segundo para o terceiro há um aumento e um decréscimo de 4, logo o terceiro termo é igual ao segundo. E assim

por diante. Então todos os termos entre parênteses são iguais ao primeiro, que vale 0 + 346 = 346. Como são 50

termos, a soma dá 50 · 346 = 17300.

Outro jeito de fazer é somar separadamente as sequências com índices ímpares e pares. No segundo caso (pares),

são 50 termos da progressão aritmética de razão 7 começando em 3 e terminando em 346. A soma dessa progressão

50 · 3 + 3462

= 25 · 349 = 8725 .

No primeiro caso (ímpares), são 50 termos, mas todos 3 unidades menores do que os termos da série par. Então a

soma desses é 8725 subtraído de 50 · 3 = 150, isto é, dá 8575. Juntando as duas, ficamos com 17300.

Obs. Essa segunda soma também sairia da mesma forma como a outra, pois a PA tem primeiro termo igual a 0,

último termo igual a 343, totalizando 50 termos, logo soma

50 · 0 + 3432

= 25 · 343 = 8575 .

(c) Observe primeiro que se n é ímpar então an é múltiplo de 7, e se n é par então an − 3 é múltiplo de 7 (de fato,

valem as recíprocas, mas não precisaremos disso).

Como nem 2000 = 7 · 285+ 5 nem 1997 = 7 · 285+ 2 são múltiplos de 7, então 2000 não pode ser um an nem para

n par nem para n ímpar.

1

AV3 - MA 12 - 2011

Questão 2.

Seja Rn o número máximo de regiões determinadas no plano por n círculos.

(0,5) (a) Quais são os valores de R1 e R2?

(0,5) (b) Explique por que Rn+1 = Rn + 2n, para todo n ≥ 1.

(1,0) (c) Mostre por indução que Rn = n2 − n + 2.

UMA SOLUÇÃO

(a) Um único círculo no plano determina exatamente duas regiões (dentro e fora). Então R1 = 2. Agora colocamos

um segundo círculo no plano e olhamos para várias possibilidades: (i) se ele for idêntico ao primeiro, continuamos

com duas regiões; (ii) se um dos círculos está inteiramente contido numa das regiões delimitadas pelo outro, então

ficam delimitadas 3 regiões (mesma coisa se apenas se tangenciam); (iii) se eles se intersectam sem se tangenciarem,

ficam delimitadas 4 regiões. Esse é o máximo possível, então R2 = 4.

(b) Primeiro verifiquemos se a fórmula está compatível com a resposta anterior. Pela fórmula, deveríamos ter

R2 = R1 + 2 · 1. De fato, R1 + 2 · 1 = 2 + 2 = 4.

Agora imaginemos que n círculos já estão desenhados, definindo um certo número de regiões. Então desenha-

mos um novo círculo (diferente dos anteriores, pois neste caso a divisão de regiões permaneceria a mesma), que

intersectará os círculos anteriores em um certo número de pontos. Como o novo círculo só pode intersectar cada

um dos outros círculos em no máximo 2 pontos, ele terá no máximo 2n intersecções. Essas intersecções dividirão o

círculo em arcos de círculo, que serão no máximo 2n (e no mínimo 1, que é quando o círculo não intersecta nenhum

dos círculos já desenhados). Chamemos de k o número de arcos de círculo obtidos.

Agora suponha que numeremos esses k arcos de círculo, e vamos desenhar o n + 1-ésimo círculo arco por arco,

contando qual é o máximo acréscimo de regiões em cada etapa. O primeiro arco está inteiramente contido em uma

das regiões previamente delimitadas, e a divide em duas regiões. Isso acrescenta uma unidade na contagem de

regiões. Como o segundo arco só pode intersectar os círculos anteriores e o primeiro arco em seus extremos, ele

também está inteiramente contido em uma das regiões, incluindo as novas regiões formadas pela introdução do

primeiro arco. Ele dividirá essa região em duas, acrescentando mais uma unidade na contagem. Esse raciocínio

pode ser repetido de forma indutiva até chegarmos no k-ésimo arco. No total, serão acrescentadas k regiões à

contagem.

Como k ≤ 2n, então são acrescentadas no máximo 2n regiões à contagem, quando se passa de n círculos para

n + 1 círculos. Portanto, se n círculos não podem dividir o plano em mais do que Rn regiões, então n + 1 círculos

não poderão dividir o plano em mais do que Rn + 2n regiões. Isso define o valor de Rn+1.

Observação. A rigor, dever-se-ia mostrar que, para cada n, alguma configuração de círculos divide o plano em

Rn regiões, para se dizer que Rn é o máximo (e não apenas uma cota superior). Para tanto, em vista do que foi

feito acima, basta achar uma lista de círculos C1, C2, C3, . . . tal que, para qualquer n ≥ 1, o círculo Cn+1 intersecta

2

cada círculo C1, . . . , Cn em 2 pontos, produzindo ao todo 2n pontos de intersecção distintos entre si. Isso pode ser

realizado por

Ci = (x, y); (x− 1i)2 + y2 = 1 , i = 1, 2, 3, . . . ,

isto é, Ci é o círculo de raio 1 e centro em ( 1i , 0). Uma conta simples mostra que Cn+1 intersecta Ci nos dois pontos 1

2

(1

n + 1+

1i

), ±

√1− 1

2

(1i− 1

n + 1

)2 .

Como os valores de 1i são distintos para i = 1, 2, . . . , n, os 2n pontos de intersecção são todos distintos entre si.

(c) A fórmula vale para n = 1, pois 12 − 1 + 2 = 2 = R1. Agora, supondo que ela vale para n, isto é, supondo

Rn = n2 − n + 2 verdadeira, queremos mostrar que também vale para n + 1, isto é, queremos mostrar que Rn+1 =

(n + 1)2 − (n + 1) + 2. Ora, a relação de recorrência nos dá Rn+1 = Rn + 2n; valendo a hipótese de que Rn =

n2 − n + 1, então

Rn+1 = Rn + 2n

= (n2 − n + 2) + 2n

= n2 + n + 2

= [(n + 1)2 − 2n− 1] + n + 2

= (n + 1)2 − n− 1 + 2

= (n + 1)2 − (n + 1) + 2 .

3

AV3 - MA 12 - 2011

Questão 3. Suponha que o dinheiro valha 10% ao mês para um comerciante que vende determinado produto por

R$ 4200,00 à vista.

(1,0) (a) Se o comerciante deseja oferecer o produto para compra em duas prestações iguais, a primeira no ato da

compra, qual deve ser o valor dessas prestações?

(1,0) (b) Suponha que ele deseja oferecer o produto em 10 prestações iguais, a primeira no ato da compra. Escreva uma

expressão que permita calcular o valor da prestação.

UMA SOLUÇÃO

(a) Se x for o valor da prestação, ele quer x + x1,1 = 4200. Isso dá x(1 + 1

1,1 ) = 4200. Então x = 11×420021 = 2200.

(b) Pelo mesmo raciocínio, ele quer x tal que

x +x

1, 1+

x1, 12 + . . . +

x1, 19 = 4200 .

Ou seja,

x(

1 + 1, 1−1 + 1, 1−2 + . . . + 1, 1−9)= 4200

e

x(

1− 1, 1−10

1− 1, 1−1

)= 4200 .

Logo

x = 4200 ·1− 1

1,1

1− 11,110

=420011· 1

1− 11,110

.

4

AV3 - MA 12 - 2011

Questão 4.

Uma senha de banco é formada por 4 digítos de 0 a 9.

(1,0) (a) Quantas são as senhas em que aparecem exatamente três dígitos diferentes?

(1,0) (b) Quantas são as senhas em que não há dígitos consecutivos iguais?

UMA SOLUÇÃO

(a) Se há exatamente 3 dígitos diferentes, então há dois dígitos iguais e mais dois outros, diferentes dele e diferentes

entre si. Há 10 possibilidades para o dígito que aparece repetido. Escolhido esse dígito, precisamos de 2 dígitos

entre os 9 restantes. Temos C29 = 36 escolhas para os dígitos restantes. Portanto, na escolha dos dígitos em que o

dígito repetido está determinado, temos 360 possibilidades.

Falta agora ver de quantas maneiras diferentes eles podem ser dispostos. Primeiro escolhemos a disposição dos

dois dígitos que não se repetem. Como há C24 possibilidades de escolha de duas entre quatro posições, temos um

total de 6 disposições possíveis.

Fixada as posições dos dígitos repetidos, temos 2 maneiras de colocar os outros dois dígitos.

Então cada uma das 360 escolhas dos 3 dígitos (com o dígito que se repete determinado) pode ser arranjada de 12

maneiras distintas, o que dá um total de 360× 12 = 4320 senhas com exatamente 3 dígitos diferentes.

(b) Há 10 possibilidades para o primeiro dígito. Como o segundo só não pode ser igual ao primeiro, há 9 possibi-

lidades para o segundo (para cada escolha do primeiro). Mais uma vez, há 9 possibilidades para o terceiro (para

cada escolha dos dois primeiros) e 9 para o quarto (para cada escolha dos três primeiros). Então são 10× 93 = 7290

possibilidades.

5

AV3 - MA 12 - 2011

Questão 5.

João, ao partir para uma viagem, ficou de enviar um cartão postal para sua mãe. A probabilidade de que ele envie

o cartão é igual a 0,7. Por outro lado, a probabilidade de um cartão postal se extraviar é 0,1.

(1,0) (a) Qual é a probabilidade de que a mãe de João receba um cartão postal dele?

(1,0) (b) Se ela não receber um cartão de João, qual é a probabilidade de que ele o tenha enviado?

UMA SOLUÇÃO

(a) A probabilidade de que um cartão não extravie, dado que foi enviado, é de 1− 0, 1 = 0, 9. Portanto a probabi-

lidade de que a mãe de João receba um cartão de seu filho é igual à probabilidade de que seja enviado e não seja

extraviado (dado que foi enviado), isto é 0, 9× 0, 7 = 0, 63.

(b) A probabilidade de a mãe não receber o cartão é igual a 1− 0, 63 = 0, 37. A probabilidade de a mãe não receber

o cartão por não ter sido enviado é igual a 1− 0, 7 = 0, 3 e a probabilidade de a mãe não receber o cartão por ter

se extraviado é 0, 1× 0, 7 = 0, 07. Portanto, se for dado que ela não recebeu o cartão, a probabilidade de que ele o

tenha enviado é de 0, 07/0, 37 = 0, 7/3, 7 = 7/37.

6